You are on page 1of 173

NAECB

Product ID: NAECB

Exam

NAECB Test Review for the


National Asthma Educator
Certification Board Examination

Licensed to elliot mattison (2146499335, i.astraltears@gmail.com) of 9801 royal ln, dallas, Texas 75231. If this product is distributed to others,
elliot mattison agrees to pay Mometrix the full retail price for each unlicensed recipient. Please play fair and respect the work of our authors.
Disclaimer: The NAECB® mark is a registered trademark owned by the National Asthma Educator
Certification Board (the NAECB). Mometrix Media LLC is not associated with the NAECB, and its
products or services have not been reviewed or endorsed by the NAECB.

Licensed to elliot mattison (2146499335, i.astraltears@gmail.com) of 9801 royal ln, dallas, Texas 75231. If this product is distributed to others,
elliot mattison agrees to pay Mometrix the full retail price for each unlicensed recipient. Please play fair and respect the work of our authors.
Mometrix Scholarships
If you or someone you know is working on their undergraduate or graduate degree and is
looking for a chance to win some scholarship money, we think you’ll be excited to learn that
Mometrix Test Preparation offers some awesome scholarship opportunities.

Since hosting our first contest back in 2014, we’ve distributed more than $15,000 in
scholarship prizes to winners from multiple schools across the country. Starting with the
2017/2018 school year, we added three more scholarship contests specifically geared toward
future nurses and teachers as well as current Masters-level/graduate school students.

Each contest requires students to write an essay (the prompts are different for each contest) -
that's it! The contests generally start up in the Fall and run through the following May. For
more information, including official rules, FAQs, and how to enter visit
www.mometrix.com/scholarships.

Licensed to elliot mattison (2146499335, i.astraltears@gmail.com) of 9801 royal ln, dallas, Texas 75231. If this product is distributed to others,
elliot mattison agrees to pay Mometrix the full retail price for each unlicensed recipient. Please play fair and respect the work of our authors.
NAECB
Exam

SECRETS
Study Guide
Your Key to Exam Success

NAECB Test Review for the


National Asthma Educator
Certification Board Examination

Licensed to elliot mattison (2146499335, i.astraltears@gmail.com) of 9801 royal ln, dallas, Texas 75231. If this product is distributed to others,
elliot mattison agrees to pay Mometrix the full retail price for each unlicensed recipient. Please play fair and respect the work of our authors.
Licensed to elliot mattison (2146499335, i.astraltears@gmail.com) of 9801 royal ln, dallas, Texas 75231. If this product is distributed to others,
elliot mattison agrees to pay Mometrix the full retail price for each unlicensed recipient. Please play fair and respect the work of our authors.
Dear Future Exam Success Story:

First of all, THANK YOU for purchasing Mometrix study materials!

Second, congratulations! You are one of the few determined test-takers who are committed to doing
whatever it takes to excel on your exam. You have come to the right place. We developed these
study materials with one goal in mind: to deliver you the information you need in a format that’s
concise and easy to use.

In addition to optimizing your guide for the content of the test, we’ve outlined our recommended
steps for breaking down the preparation process into small, attainable goals so you can make sure
you stay on track.

We’ve also analyzed the entire test-taking process, identifying the most common pitfalls and
showing how you can overcome them and be ready for any curveball the test throws you.

Standardized testing is one of the biggest obstacles on your road to success, which only increases
the importance of doing well in the high-pressure, high-stakes environment of test day. Your results
on this test could have a significant impact on your future, and this guide provides the information
and practical advice to help you achieve your full potential on test day.

Your success is our success

We would love to hear from you! If you would like to share the story of your exam success or if
you have any questions or comments in regard to our products, please contact us at 800-673-8175
or support@mometrix.com.

Thanks again for your business and we wish you continued success!

Sincerely,
The Mometrix Test Preparation Team

Need more help? Check out our flashcards at: http://mometrixflashcards.com/NAECB

Copyright © 2019 by Mometrix Media LLC. All rights reserved.


Written and edited by the Mometrix Exam Secrets Test Prep Team
Printed in the United States of America

-i-

Licensed to elliot mattison (2146499335, i.astraltears@gmail.com) of 9801 royal ln, dallas, Texas 75231. If this product is distributed to others,
elliot mattison agrees to pay Mometrix the full retail price for each unlicensed recipient. Please play fair and respect the work of our authors.
- ii -

Licensed to elliot mattison (2146499335, i.astraltears@gmail.com) of 9801 royal ln, dallas, Texas 75231. If this product is distributed to others,
elliot mattison agrees to pay Mometrix the full retail price for each unlicensed recipient. Please play fair and respect the work of our authors.
TABLE OF CONTENTS

INTRODUCTION ...................................................................................................................................................... 1
SECRET KEY #1 – PLAN BIG, STUDY SMALL .................................................................................................. 2
INFORMATION ORGANIZATION ............................................................................................................................................. 2
TIME MANAGEMENT ............................................................................................................................................................... 2
STUDY ENVIRONMENT ............................................................................................................................................................ 2
SECRET KEY #2 – MAKE YOUR STUDYING COUNT ..................................................................................... 3
RETENTION ............................................................................................................................................................................... 3
MODALITY................................................................................................................................................................................. 3
SECRET KEY #3 – PRACTICE THE RIGHT WAY............................................................................................. 4
PRACTICE TEST STRATEGY .................................................................................................................................................... 5
SECRET KEY #4 – PACE YOURSELF .................................................................................................................. 6
SECRET KEY #5 – HAVE A PLAN FOR GUESSING .......................................................................................... 7
WHEN TO START THE GUESSING PROCESS ......................................................................................................................... 7
HOW TO NARROW DOWN THE CHOICES ............................................................................................................................. 8
WHICH ANSWER TO CHOOSE ................................................................................................................................................ 9
TEST-TAKING STRATEGIES ............................................................................................................................. 10
QUESTION STRATEGIES ........................................................................................................................................................ 10
ANSWER CHOICE STRATEGIES ............................................................................................................................................ 11
GENERAL STRATEGIES .......................................................................................................................................................... 12
FINAL NOTES.......................................................................................................................................................................... 13
THE ASTHMA CONDITION................................................................................................................................ 15
ASSESSMENT OF AN INDIVIDUAL WITH ASTHMA AND FAMILY ........................................................ 28
ASTHMA MANAGEMENT ................................................................................................................................... 62
ORGANIZATIONAL ISSUES ............................................................................................................................... 95
NAECB PRACTICE EXAM ................................................................................................................................... 98
ANSWERS AND EXPLANATIONS ................................................................................................................... 130
HOW TO OVERCOME TEST ANXIETY.......................................................................................................... 151
CAUSES OF TEST ANXIETY ................................................................................................................................................ 151
ELEMENTS OF TEST ANXIETY ........................................................................................................................................... 152
EFFECTS OF TEST ANXIETY .............................................................................................................................................. 152
PHYSICAL STEPS FOR BEATING TEST ANXIETY ............................................................................................................. 153
MENTAL STEPS FOR BEATING TEST ANXIETY ............................................................................................................... 154
STUDY STRATEGY ............................................................................................................................................................... 155
TEST TIPS ............................................................................................................................................................................ 157
IMPORTANT QUALIFICATION ............................................................................................................................................ 158
THANK YOU......................................................................................................................................................... 159
ADDITIONAL BONUS MATERIAL ................................................................................................................. 160

- iii -

Licensed to elliot mattison (2146499335, i.astraltears@gmail.com) of 9801 royal ln, dallas, Texas 75231. If this product is distributed to others,
elliot mattison agrees to pay Mometrix the full retail price for each unlicensed recipient. Please play fair and respect the work of our authors.
- iv -

Licensed to elliot mattison (2146499335, i.astraltears@gmail.com) of 9801 royal ln, dallas, Texas 75231. If this product is distributed to others,
elliot mattison agrees to pay Mometrix the full retail price for each unlicensed recipient. Please play fair and respect the work of our authors.
Introduction
Thank you for purchasing this resource! You have made the choice to prepare yourself for a test
that could have a huge impact on your future, and this guide is designed to help you be fully ready
for test day. Obviously, it’s important to have a solid understanding of the test material, but you also
need to be prepared for the unique environment and stressors of the test, so that you can perform
to the best of your abilities.

For this purpose, the first section that appears in this guide is the Secret Keys. We’ve devoted
countless hours to meticulously researching what works and what doesn’t, and we’ve boiled down
our findings to the five most impactful steps you can take to improve your performance on the test.
We start at the beginning with study planning and move through the preparation process, all the
way to the testing strategies that will help you get the most out of what you know when you’re
finally sitting in front of the test.

We recommend that you start preparing for your test as far in advance as possible. However, if
you’ve bought this guide as a last-minute study resource and only have a few days before your test,
we recommend that you skip over the first two Secret Keys since they address a long-term study
plan.

If you struggle with test anxiety, we strongly encourage you to check out our recommendations for
how you can overcome it. Test anxiety is a formidable foe, but it can be beaten, and we want to
make sure you have the tools you need to defeat it.

-1-
Copyright © Mometrix Media. You have been licensed one copy of this document for personal use only. Any other reproduction or redistribution is strictly prohibited. All rights reserved.

Licensed to elliot mattison (2146499335, i.astraltears@gmail.com) of 9801 royal ln, dallas, Texas 75231. If this product is distributed to others,
elliot mattison agrees to pay Mometrix the full retail price for each unlicensed recipient. Please play fair and respect the work of our authors.
Secret Key #1 – Plan Big, Study Small
There’s a lot riding on your performance. If you want to ace this test, you’re going to need to keep
your skills sharp and the material fresh in your mind. You need a plan that lets you review
everything you need to know while still fitting in your schedule. We’ll break this strategy down into
three categories.

Information Organization
Start with the information you already have: the official test outline. From this, you can make a
complete list of all the concepts you need to cover before the test. Organize these concepts into
groups that can be studied together, and create a list of any related vocabulary you need to learn so
you can brush up on any difficult terms. You’ll want to keep this vocabulary list handy once you
actually start studying since you may need to add to it along the way.

Time Management
Once you have your set of study concepts, decide how to spread them out over the time you have
left before the test. Break your study plan into small, clear goals so you have a manageable task for
each day and know exactly what you’re doing. Then just focus on one small step at a time. When
you manage your time this way, you don’t need to spend hours at a time studying. Studying a small
block of content for a short period each day helps you retain information better and avoid stressing
over how much you have left to do. You can relax knowing that you have a plan to cover everything
in time. In order for this strategy to be effective though, you have to start studying early and stick to
your schedule. Avoid the exhaustion and futility that comes from last-minute cramming!

Study Environment
The environment you study in has a big impact on your learning. Studying in a coffee shop, while
probably more enjoyable, is not likely to be as fruitful as studying in a quiet room. It’s important to
keep distractions to a minimum. You’re only planning to study for a short block of time, so make the
most of it. Don’t pause to check your phone or get up to find a snack. It’s also important to avoid
multitasking. Research has consistently shown that multitasking will make your studying
dramatically less effective. Your study area should also be comfortable and well-lit so you don’t
have the distraction of straining your eyes or sitting on an uncomfortable chair.

The time of day you study is also important. You want to be rested and alert. Don’t wait until just
before bedtime. Study when you’ll be most likely to comprehend and remember. Even better, if you
know what time of day your test will be, set that time aside for study. That way your brain will be
used to working on that subject at that specific time and you’ll have a better chance of recalling
information.

Finally, it can be helpful to team up with others who are studying for the same test. Your actual
studying should be done in as isolated an environment as possible, but the work of organizing the
information and setting up the study plan can be divided up. In between study sessions, you can
discuss with your teammates the concepts that you’re all studying and quiz each other on the
details. Just be sure that your teammates are as serious about the test as you are. If you find that
your study time is being replaced with social time, you might need to find a new team.

-2-
Copyright © Mometrix Media. You have been licensed one copy of this document for personal use only. Any other reproduction or redistribution is strictly prohibited. All rights reserved.

Licensed to elliot mattison (2146499335, i.astraltears@gmail.com) of 9801 royal ln, dallas, Texas 75231. If this product is distributed to others,
elliot mattison agrees to pay Mometrix the full retail price for each unlicensed recipient. Please play fair and respect the work of our authors.
Secret Key #2 – Make Your Studying Count
You’re devoting a lot of time and effort to preparing for this test, so you want to be absolutely
certain it will pay off. This means doing more than just reading the content and hoping you can
remember it on test day. It’s important to make every minute of study count. There are two main
areas you can focus on to make your studying count:

Retention
It doesn’t matter how much time you study if you can’t remember the material. You need to make
sure you are retaining the concepts. To check your retention of the information you’re learning, try
recalling it at later times with minimal prompting. Try carrying around flashcards and glance at one
or two from time to time or ask a friend who’s also studying for the test to quiz you.

To enhance your retention, look for ways to put the information into practice so that you can apply
it rather than simply recalling it. If you’re using the information in practical ways, it will be much
easier to remember. Similarly, it helps to solidify a concept in your mind if you’re not only reading it
to yourself but also explaining it to someone else. Ask a friend to let you teach them about a concept
you’re a little shaky on (or speak aloud to an imaginary audience if necessary). As you try to
summarize, define, give examples, and answer your friend’s questions, you’ll understand the
concepts better and they will stay with you longer. Finally, step back for a big picture view and ask
yourself how each piece of information fits with the whole subject. When you link the different
concepts together and see them working together as a whole, it’s easier to remember the individual
components.

Finally, practice showing your work on any multi-step problems, even if you’re just studying.
Writing out each step you take to solve a problem will help solidify the process in your mind, and
you’ll be more likely to remember it during the test.

Modality
Modality simply refers to the means or method by which you study. Choosing a study modality that
fits your own individual learning style is crucial. No two people learn best in exactly the same way,
so it’s important to know your strengths and use them to your advantage.

For example, if you learn best by visualization, focus on visualizing a concept in your mind and
draw an image or a diagram. Try color-coding your notes, illustrating them, or creating symbols
that will trigger your mind to recall a learned concept. If you learn best by hearing or discussing
information, find a study partner who learns the same way or read aloud to yourself. Think about
how to put the information in your own words. Imagine that you are giving a lecture on the topic
and record yourself so you can listen to it later.

For any learning style, flashcards can be helpful. Organize the information so you can take
advantage of spare moments to review. Underline key words or phrases. Use different colors for
different categories. Mnemonic devices (such as creating a short list in which every item starts with
the same letter) can also help with retention. Find what works best for you and use it to store the
information in your mind most effectively and easily.

-3-
Copyright © Mometrix Media. You have been licensed one copy of this document for personal use only. Any other reproduction or redistribution is strictly prohibited. All rights reserved.

Licensed to elliot mattison (2146499335, i.astraltears@gmail.com) of 9801 royal ln, dallas, Texas 75231. If this product is distributed to others,
elliot mattison agrees to pay Mometrix the full retail price for each unlicensed recipient. Please play fair and respect the work of our authors.
Secret Key #3 – Practice the Right Way
Your success on test day depends not only on how many hours you put into preparing, but also on
whether you prepared the right way. It’s good to check along the way to see if your studying is
paying off. One of the most effective ways to do this is by taking practice tests to evaluate your
progress. Practice tests are useful because they show exactly where you need to improve. Every
time you take a practice test, pay special attention to these three groups of questions:

 The questions you got wrong


 The questions you had to guess on, even if you guessed right
 The questions you found difficult or slow to work through

This will show you exactly what your weak areas are, and where you need to devote more study
time. Ask yourself why each of these questions gave you trouble. Was it because you didn’t
understand the material? Was it because you didn’t remember the vocabulary? Do you need more
repetitions on this type of question to build speed and confidence? Dig into those questions and
figure out how you can strengthen your weak areas as you go back to review the material.

Additionally, many practice tests have a section explaining the answer choices. It can be tempting to
read the explanation and think that you now have a good understanding of the concept. However,
an explanation likely only covers part of the question’s broader context. Even if the explanation
makes sense, go back and investigate every concept related to the question until you’re positive
you have a thorough understanding.

As you go along, keep in mind that the practice test is just that: practice. Memorizing these
questions and answers will not be very helpful on the actual test because it is unlikely to have any
of the same exact questions. If you only know the right answers to the sample questions, you won’t
be prepared for the real thing. Study the concepts until you understand them fully, and then you’ll
be able to answer any question that shows up on the test.

It’s important to wait on the practice tests until you’re ready. If you take a test on your first day of
study, you may be overwhelmed by the amount of material covered and how much you need to
learn. Work up to it gradually.

On test day, you’ll need to be prepared for answering questions, managing your time, and using the
test-taking strategies you’ve learned. It’s a lot to balance, like a mental marathon that will have a big
impact on your future. Like training for a marathon, you’ll need to start slowly and work your way
up. When test day arrives, you’ll be ready.

Start with the strategies you’ve read in the first two Secret Keys—plan your course and study in the
way that works best for you. If you have time, consider using multiple study resources to get
different approaches to the same concepts. It can be helpful to see difficult concepts from more than
one angle. Then find a good source for practice tests. Many times, the test website will suggest
potential study resources or provide sample tests.

-4-
Copyright © Mometrix Media. You have been licensed one copy of this document for personal use only. Any other reproduction or redistribution is strictly prohibited. All rights reserved.

Licensed to elliot mattison (2146499335, i.astraltears@gmail.com) of 9801 royal ln, dallas, Texas 75231. If this product is distributed to others,
elliot mattison agrees to pay Mometrix the full retail price for each unlicensed recipient. Please play fair and respect the work of our authors.
Practice Test Strategy
When you’re ready to start taking practice tests, follow this strategy:

Untimed and Open-Book Practice

Take the first test with no time constraints and with your notes and study guide handy. Take your
time and focus on applying the strategies you’ve learned.

Timed and Open-Book Practice

Take the second practice test open-book as well, but set a timer and practice pacing yourself to
finish in time.

Timed and Closed-Book Practice

Take any other practice tests as if it were test day. Set a timer and put away your study materials.
Sit at a table or desk in a quiet room, imagine yourself at the testing center, and answer questions as
quickly and accurately as possible.

Keep repeating timed and closed-book tests on a regular basis until you run out of practice tests or
it’s time for the actual test. Your mind will be ready for the schedule and stress of test day, and
you’ll be able to focus on recalling the material you’ve learned.

-5-
Copyright © Mometrix Media. You have been licensed one copy of this document for personal use only. Any other reproduction or redistribution is strictly prohibited. All rights reserved.

Licensed to elliot mattison (2146499335, i.astraltears@gmail.com) of 9801 royal ln, dallas, Texas 75231. If this product is distributed to others,
elliot mattison agrees to pay Mometrix the full retail price for each unlicensed recipient. Please play fair and respect the work of our authors.
Secret Key #4 – Pace Yourself
Once you're fully prepared for the material on the test, your biggest challenge on test day will be
managing your time. Just knowing that the clock is ticking can make you panic even if you have
plenty of time left. Work on pacing yourself so you can build confidence against the time constraints
of the exam. Pacing is a difficult skill to master, especially in a high-pressure environment, so
practice is vital.

Set time expectations for your pace based on how much time is available. For example, if a section
has 60 questions and the time limit is 30 minutes, you know you have to average 30 seconds or less
per question in order to answer them all. Although 30 seconds is the hard limit, set 25 seconds per
question as your goal, so you reserve extra time to spend on harder questions. When you budget
extra time for the harder questions, you no longer have any reason to stress when those questions
take longer to answer.

Don’t let this time expectation distract you from working through the test at a calm, steady pace,
but keep it in mind so you don’t spend too much time on any one question. Recognize that taking
extra time on one question you don’t understand may keep you from answering two that you do
understand later in the test. If your time limit for a question is up and you’re still not sure of the
answer, mark it and move on, and come back to it later if the time and the test format allow. If the
testing format doesn’t allow you to return to earlier questions, just make an educated guess; then
put it out of your mind and move on.

On the easier questions, be careful not to rush. It may seem wise to hurry through them so you have
more time for the challenging ones, but it’s not worth missing one if you know the concept and just
didn’t take the time to read the question fully. Work efficiently but make sure you understand the
question and have looked at all of the answer choices, since more than one may seem right at first.

Even if you’re paying attention to the time, you may find yourself a little behind at some point. You
should speed up to get back on track, but do so wisely. Don’t panic; just take a few seconds less on
each question until you’re caught up. Don’t guess without thinking, but do look through the answer
choices and eliminate any you know are wrong. If you can get down to two choices, it is often
worthwhile to guess from those. Once you’ve chosen an answer, move on and don’t dwell on any
that you skipped or had to hurry through. If a question was taking too long, chances are it was one
of the harder ones, so you weren’t as likely to get it right anyway.

On the other hand, if you find yourself getting ahead of schedule, it may be beneficial to slow down
a little. The more quickly you work, the more likely you are to make a careless mistake that will
affect your score. You’ve budgeted time for each question, so don’t be afraid to spend that time.
Practice an efficient but careful pace to get the most out of the time you have.

-6-
Copyright © Mometrix Media. You have been licensed one copy of this document for personal use only. Any other reproduction or redistribution is strictly prohibited. All rights reserved.

Licensed to elliot mattison (2146499335, i.astraltears@gmail.com) of 9801 royal ln, dallas, Texas 75231. If this product is distributed to others,
elliot mattison agrees to pay Mometrix the full retail price for each unlicensed recipient. Please play fair and respect the work of our authors.
Secret Key #5 – Have a Plan for Guessing
When you’re taking the test, you may find yourself stuck on a question. Some of the answer choices
seem better than others, but you don’t see the one answer choice that is obviously correct. What do
you do?

The scenario described above is very common, yet most test takers have not effectively prepared
for it. Developing and practicing a plan for guessing may be one of the single most effective uses of
your time as you get ready for the exam.

In developing your plan for guessing, there are three questions to address:

 When should you start the guessing process?


 How should you narrow down the choices?
 Which answer should you choose?

When to Start the Guessing Process


Unless your plan for guessing is to select C every time (which, despite its merits, is not what we
recommend), you need to leave yourself enough time to apply your answer elimination strategies.
Since you have a limited amount of time for each question, that means that if you’re going to give
yourself the best shot at guessing correctly, you have to decide quickly whether or not you will
guess.

Of course, the best-case scenario is that you don’t have to guess at all, so first, see if you can answer
the question based on your knowledge of the subject and basic reasoning skills. Focus on the key
words in the question and try to jog your memory of related topics. Give yourself a chance to bring
the knowledge to mind, but once you realize that you don’t have (or you can’t access) the
knowledge you need to answer the question, it’s time to start the guessing process.

It’s almost always better to start the guessing process too early than too late. It only takes a few
seconds to remember something and answer the question from knowledge. Carefully eliminating
wrong answer choices takes longer. Plus, going through the process of eliminating answer choices
can actually help jog your memory.

Summary: Start the guessing process as soon as you decide that you can’t answer the
question based on your knowledge.

-7-
Copyright © Mometrix Media. You have been licensed one copy of this document for personal use only. Any other reproduction or redistribution is strictly prohibited. All rights reserved.

Licensed to elliot mattison (2146499335, i.astraltears@gmail.com) of 9801 royal ln, dallas, Texas 75231. If this product is distributed to others,
elliot mattison agrees to pay Mometrix the full retail price for each unlicensed recipient. Please play fair and respect the work of our authors.
How to Narrow Down the Choices
The next chapter in this book (Test-Taking Strategies) includes a wide range of strategies for how
to approach questions and how to look for answer choices to eliminate. You will definitely want to
read those carefully, practice them, and figure out which ones work best for you. Here though, we’re
going to address a mindset rather than a particular strategy.

Your chances of guessing an answer correctly depend on how many options you are choosing from.

How many choices you have How likely you are to guess correctly
5 20%
4 25%
3 33%
2 50%
1 100%

You can see from this chart just how valuable it is to be able to eliminate incorrect answers and
make an educated guess, but there are two things that many test takers do that cause them to miss
out on the benefits of guessing:

 Accidentally eliminating the correct answer


 Selecting an answer based on an impression

We’ll look at the first one here, and the second one in the next section.

To avoid accidentally eliminating the correct answer, we recommend a thought exercise called the
$5 challenge. In this challenge, you only eliminate an answer choice from contention if you are
willing to bet $5 on it being wrong. Why $5? Five dollars is a small but not insignificant amount of
money. It’s an amount you could afford to lose but wouldn’t want to throw away. And while losing
$5 once might not hurt too much, doing it twenty times will set you back $100. In the same way,
each small decision you make—eliminating a choice here, guessing on a question there—won’t by
itself impact your score very much, but when you put them all together, they can make a big
difference. By holding each answer choice elimination decision to a higher standard, you can reduce
the risk of accidentally eliminating the correct answer.

The $5 challenge can also be applied in a positive sense: If you are willing to bet $5 that an answer
choice is correct, go ahead and mark it as correct.

Summary: Only eliminate an answer choice if you are willing to bet $5 that it is wrong.

-8-
Copyright © Mometrix Media. You have been licensed one copy of this document for personal use only. Any other reproduction or redistribution is strictly prohibited. All rights reserved.

Licensed to elliot mattison (2146499335, i.astraltears@gmail.com) of 9801 royal ln, dallas, Texas 75231. If this product is distributed to others,
elliot mattison agrees to pay Mometrix the full retail price for each unlicensed recipient. Please play fair and respect the work of our authors.
Which Answer to Choose
You’re taking the test. You’ve run into a hard question and decided you’ll have to guess. You’ve
eliminated all the answer choices you’re willing to bet $5 on. Now you have to pick an answer. Why
do we even need to talk about this? Why can’t you just pick whichever one you feel like when the
time comes?

The answer to these questions is that if you don’t come into the test with a plan, you’ll rely on your
impression to select an answer choice, and if you do that, you risk falling into a trap. The test
writers know that everyone who takes their test will be guessing on some of the questions, so they
intentionally write wrong answer choices to seem plausible. You still have to pick an answer
though, and if the wrong answer choices are designed to look right, how can you ever be sure that
you’re not falling for their trap? The best solution we’ve found to this dilemma is to take the
decision out of your hands entirely. Here is the process we recommend:

Once you’ve eliminated any choices that you are confident (willing to bet $5) are wrong,
select the first remaining choice as your answer.

Whether you choose to select the first remaining choice, the second, or the last, the important thing
is that you use some preselected standard. Using this approach guarantees that you will not be
enticed into selecting an answer choice that looks right, because you are not basing your decision
on how the answer choices look.

This is not meant to make you question your knowledge. Instead, it is to help you recognize the
difference between your knowledge and your impressions. There’s a huge difference between
thinking an answer is right because of what you know, and thinking an answer is right because it
looks or sounds like it should be right.

Summary: To ensure that your selection is appropriately random, make a predetermined


selection from among all answer choices you have not eliminated.

-9-
Copyright © Mometrix Media. You have been licensed one copy of this document for personal use only. Any other reproduction or redistribution is strictly prohibited. All rights reserved.

Licensed to elliot mattison (2146499335, i.astraltears@gmail.com) of 9801 royal ln, dallas, Texas 75231. If this product is distributed to others,
elliot mattison agrees to pay Mometrix the full retail price for each unlicensed recipient. Please play fair and respect the work of our authors.
Test-Taking Strategies
This section contains a list of test-taking strategies that you may find helpful as you work through
the test. By taking what you know and applying logical thought, you can maximize your chances of
answering any question correctly!

It is very important to realize that every question is different and every person is different: no
single strategy will work on every question, and no single strategy will work for every person.
That’s why we’ve included all of them here, so you can try them out and determine which ones
work best for different types of questions and which ones work best for you.

Question Strategies
Read Carefully

Read the question and answer choices carefully. Don’t miss the question because you misread the
terms. You have plenty of time to read each question thoroughly and make sure you understand
what is being asked. Yet a happy medium must be attained, so don’t waste too much time. You must
read carefully, but efficiently.

Contextual Clues

Look for contextual clues. If the question includes a word you are not familiar with, look at the
immediate context for some indication of what the word might mean. Contextual clues can often
give you all the information you need to decipher the meaning of an unfamiliar word. Even if you
can’t determine the meaning, you may be able to narrow down the possibilities enough to make a
solid guess at the answer to the question.

Prefixes

If you're having trouble with a word in the question or answer choices, try dissecting it. Take
advantage of every clue that the word might include. Prefixes and suffixes can be a huge help.
Usually they allow you to determine a basic meaning. Pre- means before, post- means after, pro - is
positive, de- is negative. From prefixes and suffixes, you can get an idea of the general meaning of
the word and try to put it into context.

Hedge Words

Watch out for critical hedge words, such as likely, may, can, sometimes, often, almost, mostly, usually,
generally, rarely, and sometimes. Question writers insert these hedge phrases to cover every
possibility. Often an answer choice will be wrong simply because it leaves no room for exception. Be
on guard for answer choices that have definitive words such as exactly and always.

Switchback Words

Stay alert for switchbacks. These are the words and phrases frequently used to alert you to shifts in
thought. The most common switchback words are but, although, and however. Others include
nevertheless, on the other hand, even though, while, in spite of, despite, regardless of. Switchback
words are important to catch because they can change the direction of the question or an answer
choice.

- 10 -
Copyright © Mometrix Media. You have been licensed one copy of this document for personal use only. Any other reproduction or redistribution is strictly prohibited. All rights reserved.

Licensed to elliot mattison (2146499335, i.astraltears@gmail.com) of 9801 royal ln, dallas, Texas 75231. If this product is distributed to others,
elliot mattison agrees to pay Mometrix the full retail price for each unlicensed recipient. Please play fair and respect the work of our authors.
Face Value

When in doubt, use common sense. Accept the situation in the problem at face value. Don’t read too
much into it. These problems will not require you to make wild assumptions. If you have to go
beyond creativity and warp time or space in order to have an answer choice fit the question, then
you should move on and consider the other answer choices. These are normal problems rooted in
reality. The applicable relationship or explanation may not be readily apparent, but it is there for
you to figure out. Use your common sense to interpret anything that isn’t clear.

Answer Choice Strategies


Answer Selection

The most thorough way to pick an answer choice is to identify and eliminate wrong answers until
only one is left, then confirm it is the correct answer. Sometimes an answer choice may immediately
seem right, but be careful. The test writers will usually put more than one reasonable answer choice
on each question, so take a second to read all of them and make sure that the other choices are not
equally obvious. As long as you have time left, it is better to read every answer choice than to pick
the first one that looks right without checking the others.

Answer Choice Families

An answer choice family consists of two (in rare cases, three) answer choices that are very similar
in construction and cannot all be true at the same time. If you see two answer choices that are
direct opposites or parallels, one of them is usually the correct answer. For instance, if one answer
choice says that quantity x increases and another either says that quantity x decreases (opposite) or
says that quantity y increases (parallel), then those answer choices would fall into the same family.
An answer choice that doesn’t match the construction of the answer choice family is more likely to
be incorrect. Most questions will not have answer choice families, but when they do appear, you
should be prepared to recognize them.

Eliminate Answers

Eliminate answer choices as soon as you realize they are wrong, but make sure you consider all
possibilities. If you are eliminating answer choices and realize that the last one you are left with is
also wrong, don’t panic. Start over and consider each choice again. There may be something you
missed the first time that you will realize on the second pass.

Avoid Fact Traps

Don’t be distracted by an answer choice that is factually true but doesn’t answer the question. You
are looking for the choice that answers the question. Stay focused on what the question is asking for
so you don’t accidentally pick an answer that is true but incorrect. Always go back to the question
and make sure the answer choice you’ve selected actually answers the question and is not merely a
true statement.

Extreme Statements

In general, you should avoid answers that put forth extreme actions as standard practice or
proclaim controversial ideas as established fact. An answer choice that states the “process should
be used in certain situations, if…” is much more likely to be correct than one that states the “process
should be discontinued completely.” The first is a calm rational statement and doesn’t even make a

- 11 -
Copyright © Mometrix Media. You have been licensed one copy of this document for personal use only. Any other reproduction or redistribution is strictly prohibited. All rights reserved.

Licensed to elliot mattison (2146499335, i.astraltears@gmail.com) of 9801 royal ln, dallas, Texas 75231. If this product is distributed to others,
elliot mattison agrees to pay Mometrix the full retail price for each unlicensed recipient. Please play fair and respect the work of our authors.
definitive, uncompromising stance, using a hedge word if to provide wiggle room, whereas the
second choice is a radical idea and far more extreme.

Benchmark

As you read through the answer choices and you come across one that seems to answer the
question well, mentally select that answer choice. This is not your final answer, but it’s the one that
will help you evaluate the other answer choices. The one that you selected is your benchmark or
standard for judging each of the other answer choices. Every other answer choice must be
compared to your benchmark. That choice is correct until proven otherwise by another answer
choice beating it. If you find a better answer, then that one becomes your new benchmark. Once
you’ve decided that no other choice answers the question as well as your benchmark, you have your
final answer.

Predict the Answer

Before you even start looking at the answer choices, it is often best to try to predict the answer.
When you come up with the answer on your own, it is easier to avoid distractions and traps
because you will know exactly what to look for. The right answer choice is unlikely to be word-for-
word what you came up with, but it should be a close match. Even if you are confident that you have
the right answer, you should still take the time to read each option before moving on.

General Strategies
Tough Questions

If you are stumped on a problem or it appears too hard or too difficult, don’t waste time. Move on!
Remember though, if you can quickly check for obviously incorrect answer choices, your chances of
guessing correctly are greatly improved. Before you completely give up, at least try to knock out a
couple of possible answers. Eliminate what you can and then guess at the remaining answer choices
before moving on.

Check Your Work

Since you will probably not know every term listed and the answer to every question, it is
important that you get credit for the ones that you do know. Don’t miss any questions through
careless mistakes. If at all possible, try to take a second to look back over your answer selection and
make sure you’ve selected the correct answer choice and haven’t made a costly careless mistake
(such as marking an answer choice that you didn’t mean to mark). This quick double check should
more than pay for itself in caught mistakes for the time it costs.

Pace Yourself

It’s easy to be overwhelmed when you’re looking at a page full of questions; your mind is confused
and full of random thoughts, and the clock is ticking down faster than you would like. Calm down
and maintain the pace that you have set for yourself. Especially as you get down to the last few
minutes of the test, don’t let the small numbers on the clock make you panic. As long as you are on
track by monitoring your pace, you are guaranteed to have time for each question.

- 12 -
Copyright © Mometrix Media. You have been licensed one copy of this document for personal use only. Any other reproduction or redistribution is strictly prohibited. All rights reserved.

Licensed to elliot mattison (2146499335, i.astraltears@gmail.com) of 9801 royal ln, dallas, Texas 75231. If this product is distributed to others,
elliot mattison agrees to pay Mometrix the full retail price for each unlicensed recipient. Please play fair and respect the work of our authors.
Don’t Rush

It is very easy to make errors when you are in a hurry. Maintaining a fast pace in answering
questions is pointless if it makes you miss questions that you would have gotten right otherwise.
Test writers like to include distracting information and wrong answers that seem right. Taking a
little extra time to avoid careless mistakes can make all the difference in your test score. Find a pace
that allows you to be confident in the answers that you select.

Keep Moving

Panicking will not help you pass the test, so do your best to stay calm and keep moving. Taking deep
breaths and going through the answer elimination steps you practiced can help to break through a
stress barrier and keep your pace.

Final Notes
The combination of a solid foundation of content knowledge and the confidence that comes from
practicing your plan for applying that knowledge is the key to maximizing your performance on test
day. As your foundation of content knowledge is built up and strengthened, you’ll find that the
strategies included in this chapter become more and more effective in helping you quickly sift
through the distractions and traps of the test to isolate the correct answer.

Now it’s time to move on to the test content chapters of this book, but be sure to keep your goal in
mind. As you read, think about how you will be able to apply this information on the test. If you’ve
already seen sample questions for the test and you have an idea of the question format and style,
try to come up with questions of your own that you can answer based on what you’re reading. This
will give you valuable practice applying your knowledge in the same ways you can expect to on test
day.

Good luck and good studying!

- 13 -
Copyright © Mometrix Media. You have been licensed one copy of this document for personal use only. Any other reproduction or redistribution is strictly prohibited. All rights reserved.

Licensed to elliot mattison (2146499335, i.astraltears@gmail.com) of 9801 royal ln, dallas, Texas 75231. If this product is distributed to others,
elliot mattison agrees to pay Mometrix the full retail price for each unlicensed recipient. Please play fair and respect the work of our authors.
.

- 14 -
Copyright © Mometrix Media. You have been licensed one copy of this document for personal use only. Any other reproduction or redistribution is strictly prohibited. All rights reserved.

Licensed to elliot mattison (2146499335, i.astraltears@gmail.com) of 9801 royal ln, dallas, Texas 75231. If this product is distributed to others,
elliot mattison agrees to pay Mometrix the full retail price for each unlicensed recipient. Please play fair and respect the work of our authors.
The Asthma Condition
Anatomy and Physiology of Lungs

The lungs are organs in the chest cavity, enclosed within the ribcage. The lungs are attached to the
trachea or “windpipe” and comprise a series of tubes known as bronchi. The left and right main
bronchi each branch out into smaller and smaller tubes called bronchioles. Each lung has its own
separate system of tubes for breathing. This system of tubes looks similar to the underground roots
of a tree. Each bronchiole ends with air sacs called alveoli, where oxygen is transferred into the
blood during inspiration. When you breathe, you inhale oxygen, bringing it to all the organs and
systems in your body, and exhale the carbon dioxide waste. This is the main function of the lungs.
Asthma episodes can restrict your breathing and reduce the amount of oxygen that is carried to
your bloodstream.

Asthma Episode

An asthma episode is comprised of three effects on the pulmonary system: bronchoconstriction,


inflammation, and mucus production. During an asthma episode, the smooth muscle surrounding
the bronchioles constrict, often in response to the exposure of an asthma trigger. This effect causes
the bronchial tubes to become smaller and therefore harder to get air through. The muscle of the
bronchioles also becomes inflamed and swollen, resulting in chest pain and a feeling of chest
tightness when breathing. The third component of an asthma episode is mucus production. During
an asthma exacerbation, the bronchioles excrete mucus, which can plug the tubes that carry air and
oxygen throughout your lungs. These bronchioles also spasm, causing the tubes to close off and
further restrict your breathing. Many people say that these effects feel like breathing through a
straw or having an elephant sit on their chest.

Asthma and Diagnosis

Asthma is a chronic, obstructive disease of the lungs characterized by episodic airway


inflammation, bronchoconstriction, and mucus production. Asthma is chronic, meaning that it never
fully resolves; however, careful management and treatment of the disease can reduce symptoms
and exacerbations, resulting in a near normal quality of life for asthma patients. Asthma can affect
patients of any age, including adults and children. However, it is uncommon to diagnose a patient
with asthma prior to 5 years of age. The consensus for the diagnosis of asthma includes the
presence of three or more exacerbations in 1 year and/or cyclical patterns of respiratory distress.
Patients younger than 5 years of age often exhibit frequent respiratory infections such as
respiratory syncytial virus (RSV) and other infectious episodes that can cause wheezing and mucus
production in the patient’s immature airways. It is common to wait until age 5 and determine
whether the patient “grows out” of these symptoms prior to giving a definite diagnosis of asthma in
very young patients.

Review Video: Asthma and Allergens


Visit mometrix.com/academy and enter code: 799141

Long-Term Effects of Uncontrolled Asthma

Successfully controlling a patient’s asthma is the goal of every asthma education program. With
poorly controlled asthma, several long-term effects can take place, resulting in a reduced quality of
life for the asthmatic patient. When underlying airway inflammation goes untreated, it can cause

- 15 -
Copyright © Mometrix Media. You have been licensed one copy of this document for personal use only. Any other reproduction or redistribution is strictly prohibited. All rights reserved.

Licensed to elliot mattison (2146499335, i.astraltears@gmail.com) of 9801 royal ln, dallas, Texas 75231. If this product is distributed to others,
elliot mattison agrees to pay Mometrix the full retail price for each unlicensed recipient. Please play fair and respect the work of our authors.
permanent, irreversible damage to the airway, including structural changes such as airway
remodeling and scarring as well as increased episodes of bronchospasm and reduced tolerance of
environmental trigger exposure. These long-term effects can eventually become irreversible,
resulting in a chronic obstructive pulmonary disease (COPD)-like disease process. COPD is a
permanent obstructive disorder characterized by reduced expiratory capacity. COPD can be
managed but not reversed, and therefore, careful and comprehensive observation and treatment of
the asthmatic patient is imperative to ensure that asthmatic patients reverse their pulmonary
disease and do not progress into a permanent obstructive pattern.

Asthma Severity, Control, Impairment, and Risk

Asthma is characterized and measured by four main components: severity, control, impairment,
and risk. Severity refers to the intensity of the asthma symptoms and its effects on the patient’s
quality of life. Control refers to the efficacy of the patient’s current therapeutic plan, including the
effects of medications, environmental controls, and lifestyle changes on the patient’s quality of life.
Impairment measures the severity and frequency of a patient’s asthma symptoms. Last, risk is
assessed by taking into account the severity of the patient’s previous exacerbations and pulmonary
function, thereby predicting the severity and frequency of future asthma episodes. When
determining a patient’s likelihood for a life-threatening exacerbation, clinicians measure the
patient’s previous severity of exacerbations and symptoms, current level of control, level of
impairment present, and future risk. Identifying a patient’s risk is an important step in the asthma
education process as it tells the clinician the likelihood of a future event, and the clinician can
increase or decrease the patient’s therapeutic interventions accordingly.

Pulmonary Function Test

The term pulmonary function test refers to a series of spirometry tests that measure a patient’s lung
volumes and inspiratory and/or expiratory flows. Performing spirometry before and after
bronchodilation will determine the reversibility of the patient’s asthma as well as the efficacy of the
chosen pharmacological therapy. A forced vital capacity test requires the patient to inhale
completely and exhale hard and fast until he or she is unable to do so any longer. Then the patient
takes another maximum inhalation. This maneuver measures lung capacity, inspiratory flow, and
expiratory flow. An asthmatic patient will present with an obstructive flow volume loop pattern,
with reduced flows and normal volumes. The forced expiratory volume in a 1-second portion of the
test (FEV1) is a critical value for the determination of asthma severity. An FEV1 of less than 60
percent of the patient’s predicted value indicates a very poorly controlled and very severe asthma
episode.

Chronic Nature of Asthma

Asthma exacerbations are often cyclical in nature, with periods of exacerbation and resolution
dependent upon the patient’s triggers, pharmacological interventions, and lifestyle factors. Asthma
episodes are not created equal, with episodes ranging from mild to severe. Asthma is a chronic
disease that never fully resolves. Although symptoms may be well controlled at times, with near
normal lung function, exacerbations can occur at any time with varying degrees of impairment.
Asthma exacerbations may be absent for days, months, weeks, or even years and then return at a
later date due to exposure to new triggers or unsuccessful prior trigger avoidance. Patients must be
compliant in their medication regimen, even when they are asymptomatic, to successfully avoid
severe exacerbations. Careful self-monitoring and trigger avoidance by the patient, as well as
medical intervention and monitoring by the asthma educator, is critical in successfully managing a
patient’s asthma.

- 16 -
Copyright © Mometrix Media. You have been licensed one copy of this document for personal use only. Any other reproduction or redistribution is strictly prohibited. All rights reserved.

Licensed to elliot mattison (2146499335, i.astraltears@gmail.com) of 9801 royal ln, dallas, Texas 75231. If this product is distributed to others,
elliot mattison agrees to pay Mometrix the full retail price for each unlicensed recipient. Please play fair and respect the work of our authors.
Signs and Symptoms of Asthma Episodes

Asthma symptoms vary among individuals, with some patients experiencing many symptoms, while
others experience only one or two. The main signs and symptoms of asthma include but are not
limited to chest pain, increased respiratory rate, pursed-lip breathing, dyspnea, orthopnea
(difficulty sleeping or lying flat), chest tightness or heaviness, wheezing, clearing of the throat, use
of accessory muscles to breathe, nasal flaring and retractions in infants and toddlers, tracheal
tugging, and day- or nighttime cough. Asthma symptoms are often worse at night, with mucus
pooling in the airways as well as hormonal changes that naturally occur with the patient’s circadian
rhythm. A persistent nighttime cough is a common asthma symptom. However, it is important to
remember that any persistent, unexplained cough is a sign of asthma. In some cases, an unrelenting
cough can be a patient’s only complaint. It is extremely important to remember many asthmatics do
not wheeze at all.

Differing Signs and Symptoms Across All Patient Populations and Age Ranges
Signs and symptoms of asthma can vary from patient to patient as well as between patient
populations. Some signs, such as tracheal tugging and retractions, are common in infants and small
children. Tracheal tugging is identified by examining the skin at the base of the neck near the
trachea. If these tissues are “sucking in,” tracheal tugging is present. The clinician may also notice
similar movements between the patient’s ribs or at the center of the chest. These muscle
movements are known as retractions and are a sign of respiratory distress. Children may also flare
their nostrils during an asthma episode, which is also a sign of respiratory difficulty. Conversely,
adults are more likely to exhibit accessory muscle use and pursed-lip breathing. This can include
“tripod” positioning, where patients brace themselves with their arms at the end of a bed. Often the
muscles of the neck and upper chest are noticeably pulling to help the patient take in a breath.
Adults will often verbally describe their shortness of breath, whereas children do not. Pediatric
patients often can’t effectively communicate their symptoms and will often complain of stomach or
throat pain. Asking a child to point to their pain will help a clinician determine if the pain is actually
asthma induced. Small children often exhibit signs such as coughing and wheezing, an increased
respiratory rate, frequent nighttime cough, and viral infections that precede asthma exacerbations.
Adults can exhibit some of the same symptoms with the exception of retractions and nasal flaring.

Irritants vs. Allergens

Allergen and irritants can both cause asthma episodes. Patients can be allergic to virtually anything,
but the most common allergens include tree pollen, flowers, ragweed, animal dander, dust mites,
and mold. These substances cause an allergic reaction in sensitive individuals. Reactions include
symptoms such as runny and itchy nose, bronchospasm, postnasal drip, and nasal congestion. This
allergic response can provoke a patient’s asthma and result in an asthma episode. Conversely,
irritants are substances that do not trigger a true immunoglobulin E (IgE)-mediated allergic
reaction but can cause many of the same symptoms as exposure to allergens. An IgE-mediated
allergic reaction occurs when a patient is exposed to an allergen and his or her body releases IgE,
which bonds to the allergen and in turn causes the release of histamine from mast cells. Histamine
then causes the traditional symptoms of an allergic reaction and can also trigger allergic asthmatic
episodes. This reaction can be minimized by the utilization of several different therapeutic
interventions, including antihistamines. Conversely, paint fumes, perfumes, air fresheners, smoke,
chemical exposure, and cockroach droppings are all considered irritants. Although these substances
don’t cause a true allergic reaction, they cause the patient’s airways to constrict, thereby triggering
an asthma episode. Medications and immunotherapy can be utilized to minimize allergic reactions,

- 17 -
Copyright © Mometrix Media. You have been licensed one copy of this document for personal use only. Any other reproduction or redistribution is strictly prohibited. All rights reserved.

Licensed to elliot mattison (2146499335, i.astraltears@gmail.com) of 9801 royal ln, dallas, Texas 75231. If this product is distributed to others,
elliot mattison agrees to pay Mometrix the full retail price for each unlicensed recipient. Please play fair and respect the work of our authors.
whereas irritants must be carefully avoided even though they do respond to therapeutic
interventions.

Asthma Triggers

Asthmatic episodes are always triggered by either exposure to a substance, changes in the patient’s
environment, or physiological factors. Asthma varies widely from patient to patient. Some of the
common asthma triggers include allergens, such as mold, animal dander, trees and grasses, flowers,
pollen, and dust mites. Inhaled irritants such as cockroach droppings, paint fumes, strong scents,
and smoke are also common asthma triggers. Asthma episodes can also be related to changes in
breathing patterns such as laughing and crying, emotional changes such as stress, and changes in
weather conditions such as a reduction or increase in temperature, humidity, or rain storms.
Exposure to colds and viruses are also asthma triggers, with many patients experiencing asthma
episodes only during a respiratory illness. Last, some medications themselves can affect asthma
severity. These include non-steroidal anti-inflammatory medications (NSAIDS) such as aspirin,
naproxen sodium, and ibuprofen.

Triggers vary widely among patient populations and individuals with asthma. Asthma triggers are
unique to each patient and can even vary among asthmatics within the same family or environment.
Some patients, especially with a history of allergic asthma, will have increased exacerbations only
during their corresponding allergy season. For example, a patient with allergies to trees and grass
will notice asthma exacerbations in the springtime. A patient with ragweed allergies will notice an
increase in symptoms in the autumn. Others, with allergies or sensitivities to mold, will experience
asthma symptoms during rain storms or windy days with wet fallen leaves and will also have
bronchospasm if their homes and basements are damp and moldy. Some patients find strong scents
and emotions problematic for their asthma, whereas others have exacerbations solely during a
respiratory illness. It is important to remember that all triggers vary from patient to patient, and
not all triggers are problematic for every individual. Education on trigger avoidance must be
tailored to the needs of the individual patient rather than a one-size-fits-all approach.

Role of Smoke Exposure in Development of Asthma

Tobacco smoke, containing toxic chemicals and carcinogens, is a major asthma trigger. Exposure to
smoke can be firsthand, secondhand, or thirdhand. Firsthand smoke exposure is when an asthmatic
patient is the smoker. Secondhand smoke exposure is when a patient is in the presence of an active
smoker and is passively exposed to the tobacco smoke. Last, thirdhand smoke is smoke residue
found within a patient’s home, car, or on a surface. Exposure to this type of smoke residue can be
particularly dangerous as the substance is comprised of miniscule particles that are easily absorbed
into the lungs. Although current studies do not show a definite link between smoke exposure and
the development of asthma, the Asthma and Allergy Foundation of America reports that 40 percent
of children who present to the emergency room with uncontrolled asthma have been directly
exposed to secondhand smoke. These children also experience severe asthma symptoms on a
regular basis.

Effects of Smoke Exposure


All forms of smoke exposure, including firsthand, secondhand, and thirdhand exposures, causes an
asthmatic’s lungs to become inflamed and constricted, with subsequent wheezing and coughing.
Patients exposed to smoke have poor asthma control, with reduced quality of life and inability to
perform daily activities and hobbies. These patients have more frequent emergency department
visits and hospitalizations and often experience frequent, severe respiratory infections. Complete
avoidance of smoke exposure is necessary to control a patient’s asthma. This includes quitting

- 18 -
Copyright © Mometrix Media. You have been licensed one copy of this document for personal use only. Any other reproduction or redistribution is strictly prohibited. All rights reserved.

Licensed to elliot mattison (2146499335, i.astraltears@gmail.com) of 9801 royal ln, dallas, Texas 75231. If this product is distributed to others,
elliot mattison agrees to pay Mometrix the full retail price for each unlicensed recipient. Please play fair and respect the work of our authors.
smoking, not allowing smoking in your car or home, and avoiding other places where people smoke.
Tobacco smoke, however, is not the only form of smoke that is problematic for an asthma patient.
All forms of smoke, including wood smoke and leaf smoke, must be carefully avoided as these are
inhaled irritants that cause dramatic increases in asthma exacerbation severity.

Effects of Diet

Although diet has not been directly linked to the development of asthma, it is an important
component in the control of asthma symptoms. Some foods can be asthma triggers, including foods
containing sulfites or foods to which the patient is allergic. Sulfites are found in dried fruits, cured
meats, some alcoholic beverages, shellfish, and pickled products. If a patient has food allergies,
those foods must be completely avoided as exposure may cause a life-threatening allergic reaction
in addition to general asthma symptoms. Diet can also indirectly affect a patient’s asthma by
causing a patient to be overweight. Overweight or obese patients often complain of more severe
and frequent asthma symptoms. Patients with asthma may also have low levels of vitamin D, which
can be determined via a simple blood test ordered by a physician. Patients lacking vitamin D may
need to take supplements as low vitamin D levels are linked to more severe asthma symptoms.

Role of Previous Respiratory Infections in Development and Control of Asthma

Respiratory infections are a common asthma trigger. During a viral or bacterial pulmonary
infection, asthmatic patients can experience bronchoconstriction, excess mucus production, and
inflammation. Although these symptoms are common components of an asthma episode, they
become more severe when combined with an upper or lower respiratory infection. The
inflammatory response associated with infection becomes a trigger for a more severe asthma
episode. Often, patients with asthma are more susceptible to respiratory illnesses, which in turn
cause more frequent and severe asthma exacerbations. This effect is cyclical in nature, with
previous infections and subsequent asthma symptoms predicting a future pattern. Prevention of
respiratory illness is essential to the successful control of asthma symptoms. Hand washing, flu
and/or pneumonia vaccinations, and avoiding touching the face can all help prevent viral and
bacterial respiratory infections. Once an illness is contracted, the patient must carefully follow his
or her written asthma action plan and prepare to take emergency action if necessary.

Occupational Asthma

Occupational asthma is defined as asthma related to the workplace. Some patients with
occupational asthma have never had asthma symptoms before but began noticing some once they
began working at a particular location or in a certain industry. Some common occupational asthma
triggers include inhaled irritants such as ozone, fumes, dust, or gasses. Some patients state that they
have asthma symptoms at work but rarely notice them on the weekend or at home. Occupational
asthma can also refer to an underlying diagnosis of asthma that is then worsened in severity due to
occupational exposures. Allergens are another occupational hazard for asthmatic patients, with
allergies to animals, dust, mold, and latex being some of the common workplace triggers for allergic
patients. Asthma is protected under the American Disabilities Act, and reasonable accommodations
must be made for workers with occupational asthma. This may include wearing a mask, moving the
patient’s desk or office to another part of the building, or perhaps changing careers if the trigger
can’t be avoided.

Effects of Beta Blockers

Beta blockers were once considered a contraindication for asthmatic patients due to the drugs’
provocative effects on airway resistance and bronchial hyperresponsiveness. This classification of

- 19 -
Copyright © Mometrix Media. You have been licensed one copy of this document for personal use only. Any other reproduction or redistribution is strictly prohibited. All rights reserved.

Licensed to elliot mattison (2146499335, i.astraltears@gmail.com) of 9801 royal ln, dallas, Texas 75231. If this product is distributed to others,
elliot mattison agrees to pay Mometrix the full retail price for each unlicensed recipient. Please play fair and respect the work of our authors.
drug, used to treat patients with previous myocardial infarction and or heart failure, can cause
airway constriction, inflammation, and bronchospasm in an asthmatic patient. The severity of these
effects is in direct correlation with the dosage of the beta blockers, with higher doses causing more
severe symptoms. Recent studies indicate, however, that cardioselective beta blockers, which only
affect the beta-1 receptors (as opposed to both beta-1 and beta-2 receptors) cause minimal asthma
symptoms. This is mainly due to the location of the receptors, with beta-1 receptors located in the
heart muscle, and beta-2 receptors are found in the airway and muscle systems of the body.
Therefore, cardioselective beta blockers target only receptors in the heart and have minimal effects
on a patient’s asthma. Current studies show that cardioselective beta blockers are a viable option
for asthmatic patients.

Effects of NSAIDs

Non-steroidal anti-inflammatory medications (NSAIDs) can cause asthma symptoms in sensitive


patients. Although not all patients are intolerant to NSAIDs, all patients should be made aware of
the risks associated with this classification of medication. Patients with NSAID sensitivity can
experience more frequent and severe asthma exacerbations, with the use of the NSAID triggering
the symptoms and/or causing more severe symptoms. These patients should avoid the use of
NSAIDs and consider alternative pain relievers and anti-inflammatory drugs under the advisement
of their physician. The asthma educator must educate the patient on the risks of using these
medications in both sensitive populations as well as patients who might never have needed to start
an NSAID regimen in the past. Although some asthmatic patients may not be sensitive to NSAIDs,
many are. Without exposure to this drug class, it is uncertain whether a patient will have an adverse
reaction; therefore, proper education on NSAID usage is necessary and appropriate for all asthmatic
patients.

Effects of Anesthetics

Although the use of anesthesia is a necessary therapy in surgical procedures, its use can sometimes
cause adverse reactions in patients with asthma. Asthmatic patients can experience bronchospasm
due to the use of inhaled anesthetic gasses. Gasses are a common asthma trigger, and anesthetic
gasses are not an exception. The use of anesthesia is also related to an increased risk of aspiration
and subsequent pulmonary infection, which in turn can cause an asthma exacerbation. The act of
intubation during surgery is also related to asthma exacerbations due to the laryngoscope
potentially provoking bronchospasm during the probing of the airway and can cause bronchial
scarring and trauma. Conscious sedation can also cause an increased risk of asthma exacerbation as
the patient may experience mild pain or be partially aware of the procedure taking place. This
anxiety and pain can then cause bronchospasm due to stress. Careful preoperative testing,
including but not limited to pulmonary function testing, peak flow measurement, chest
auscultation, and review of the patient’s recent history of symptoms will help the clinician
determine what action should be taken prior to surgery. For example, the patient may require a
dose of bronchodilators immediately prior to surgery, or the clinician may need to choose a
different anesthetic plan for this challenging patient population.

Effects of Mold

Mold exposure is dangerous for all asthmatic patients, including those who are allergic to the
substance. Mold spores are naturally occurring and exist in every home in small numbers. However,
when a home has a mold problem, the indoor mold count is excessive, causing many troublesome
and potentially severe respiratory symptoms. Allergic reactions to mold are characterized by a
runny and itchy nose, bronchospasm, postnasal drip, and nasal congestion. This reaction can

- 20 -
Copyright © Mometrix Media. You have been licensed one copy of this document for personal use only. Any other reproduction or redistribution is strictly prohibited. All rights reserved.

Licensed to elliot mattison (2146499335, i.astraltears@gmail.com) of 9801 royal ln, dallas, Texas 75231. If this product is distributed to others,
elliot mattison agrees to pay Mometrix the full retail price for each unlicensed recipient. Please play fair and respect the work of our authors.
subsequently cause an asthma exacerbation. In non-allergic patients, mold exposure may cause
similar symptoms, with cough, congestion, shortness of breath, and headache. These nonallergic
symptoms also cause asthma exacerbations as mold is highly irritating to the lining of the airway. In
addition, mold exposure can cause serious upper and lower respiratory infections that also
exacerbate a patient’s asthma symptoms. Signs of mold growth in a home can include visible mold
growth, chalky residue on drywall and floorboards, and water stains on ceilings and walls.

Asthma Triggers in the Home Environment

Dust
Dust is a major household-based asthma trigger, with most asthma patients sensitive to the
substance. Upholstered furniture, curtains, carpeting, and stuffed animals can all hold dust mites.
The dust mite droppings become aerosolized, and the patient breathes them in, causing
bronchospasm and allergy symptoms such as nasal congestion, sneezing, and coughing. Dust mites
can also be found in bedding, with the mattress and pillow providing a perfect environment for dust
mites to live. Visible dust on surfaces, especially those areas with decorative items and other “dust
catchers” becomes especially problematic as the dust is easily blown through the environment by
attempting to clean the surface by dusting and/or the natural movement of air in the room. Air
ducts in the home can transport dust and dust mites throughout the house, causing asthma
symptoms in a patient with a seemingly visibly “clean” house. Conversely, alternative heat sources
such as radiant heat causes less dust buildup as there are no ducts used in this type of heating
system.

Pet-Related Triggers
Several animal-related asthma triggers are commonly present in the home environment. Many
people have pets in their homes, which can be problematic for patients sensitive to animal dander.
The saliva and dander of cats and dogs can flake off of the animal’s coat and skin, become airborne,
and are inhaled by the patient. Ductwork in the patient’s home can transport the dander
throughout the house, even if the patient is confined to one room or area. Although some pets shed
more than others, and some do not shed at all, there is no such thing as a nonallergenic pet. A
“hypoallergenic” pet refers to a pet that is considered “low” allergy, meaning that the pet either
doesn’t shed or sheds very little. However, this type of pet can still cause asthma exacerbations in
sensitive patients. No furry pet is considered safe for an asthmatic that is sensitive to the animal’s
dander.

Inhaled Irritants
Inhaled irritants are found in nearly every home. Cleansers, air fresheners, and scented candles are
all common asthma triggers found in patients’ homes. Cleansers, such as bleach and other heavily
scented cleaning solutions, are comprised of harsh chemicals and fragrances that can cause
immediate, severe bronchospasm in patients with asthma. The use of strong-smelling laundry
detergents and fabric softeners are particularly troublesome for patients with asthma as these
scents are carried with them throughout the day on their clothing, with no possible avoidance of the
scent. The effects of these chemicals vary from patient to patient, with some patients tolerating
particular scents and suffering from bronchospasm when exposed to other scents. Others have
asthma exacerbations from exposure to any fragrance at all. Other scents, such as food-related
odors, can also cause bronchospasm in certain patients. Cooking hot peppers can cause asthma
exacerbations, and patients with food allergies may develop respiratory distress simply from
smelling a food cooking.

- 21 -
Copyright © Mometrix Media. You have been licensed one copy of this document for personal use only. Any other reproduction or redistribution is strictly prohibited. All rights reserved.

Licensed to elliot mattison (2146499335, i.astraltears@gmail.com) of 9801 royal ln, dallas, Texas 75231. If this product is distributed to others,
elliot mattison agrees to pay Mometrix the full retail price for each unlicensed recipient. Please play fair and respect the work of our authors.
Effects of Cockroaches and Smoke
Cockroaches and smoke are both common asthma triggers found in some home environments.
Cockroach droppings become aerosolized and subsequently inhaled by the patient. These
droppings act as an inhaled irritant, causing bronchospasm, bronchoconstriction, and more severe
asthma episodes. Often this asthma trigger is more common in inner cities; however, pest
infestations can occur anywhere. Smoke, another inhaled irritant, can also contribute to asthma
exacerbations. All forms of smoke exposure, including smoking firsthand, secondhand smoke,
contact with smoke residue, and other sources of smoke exposure all cause more severe asthma
exacerbations. The use of fireplaces must be carefully controlled to ensure that the smoke does not
drift into the home. Careful monitoring of any fireplaces and wood-burning stoves are critical for
patients with asthma. These patients also must not burn leaves in their yards, as leaf and wood
smoke is just as irritating to the airways as tobacco smoke.

Classroom Pets and Inhaled Chemicals in Schools

Several common asthma triggers can be found in the school environment, with many triggers
varying among schools and classrooms. Environmental classroom triggers can include ozone
released by computer printers, chalk dust from chalkboards, and fumes from wipe-off markers used
by instructors instead of chalkboards. Students may also notice increased asthma symptoms when
the gym floors are newly varnished, or the lockers are freshly cleaned as these cleansers and
sealants contain harsh chemicals and are generally strong smelling. Class pets, although more
uncommon today, can also be an environmental asthma trigger in the classroom. These pets are
often small, furry rodents or birds that can release dander into the air. Small rodent pets also
require fluffy bedding, which can hold dander, mold, dust, and other irritants. Cleaning the pet’s
enclosure is especially dangerous for the asthmatic patient as the irritants will become airborne as
the bedding is moved and replaced.

Role of Sports and Emotional Stress in Student’s Risk of Asthma Exacerbations

Sports and physical education can also be common asthma triggers. Many patients with asthma cite
physical activity as one of their main triggers. This can include playing sports, running, or light
physical activity with peers. Although the physical activity itself can be a common school-based
asthma trigger, sometimes the environment in which the activity is played can contribute to the
asthma exacerbation. If physical education or recess is conducted outside in the spring and fall,
students with seasonal allergies may notice an increase in the frequency and severity of asthma
symptoms at school. Although physical stress and exertion are common asthma triggers, emotional
stress can also cause increased asthma exacerbations in students. Some students with asthma may
notice an increase in asthma symptoms during testing periods or when projects are due. This type
of stress is a common asthma trigger for schoolchildren.

Asthma Triggers in Office Environment

Several common asthma triggers can be found in indoor workplaces: ozone released by computer
printers, dust from files and surfaces, and fumes from wipe-off markers used by instructors instead
of chalkboards. Workers may notice an increase in asthma symptoms in the early morning hours
after the office has been cleaned after hours in the evening. The use of cleansers and air fresheners
in the office can cause increased asthma severity. Plants in workers’ offices can harbor mold growth
in their soil, which can cause bronchospasm and allergy symptoms in allergic individuals. Wipe-off
markers, often used on whiteboards during meetings, can release strong scents and fumes that can
exacerbate a worker’s asthma. Stress and strong emotions can also be common workplace asthma

- 22 -
Copyright © Mometrix Media. You have been licensed one copy of this document for personal use only. Any other reproduction or redistribution is strictly prohibited. All rights reserved.

Licensed to elliot mattison (2146499335, i.astraltears@gmail.com) of 9801 royal ln, dallas, Texas 75231. If this product is distributed to others,
elliot mattison agrees to pay Mometrix the full retail price for each unlicensed recipient. Please play fair and respect the work of our authors.
triggers. The stress of giving a presentation or an argument with an employer or colleague can
cause physical effects in the asthmatic patient, including bronchospasm and airway hyperreactivity.

Asthma Triggers in Workplaces with Animals

For asthmatic patients who work with animals, controlling their asthma becomes a more difficult
task. Animal dander is often one of the most difficult triggers to control for these patients as
exposure to this substance is inevitable. During the bathing, brushing, grooming, and examining of
animals, dander becomes airborne. The patient then inhales the dander and can experience allergy
and asthma symptoms. However, dander is not the only airborne animal-related trigger in this
workplace. Although dander is a problem for many asthmatics, sometimes the trigger is not the
dander itself. Pets often roam outdoors and can carry environmental allergens on their coats. Tree,
flower, or grass pollens and other outdoor allergens can be subsequently transported into the
workplace on the animal. The patient then is exposed to those triggers, which can exacerbate their
asthma. In addition, the chemicals and cleansers used in these workplaces (including surface
cleaners, air fresheners, and animal shampoos) can act as inhaled irritants, causing an increase in
the frequency and severity of the patient’s asthma symptoms.

Asthma Triggers Found in Outdoor Workplaces

For patients who work outdoors, the workplace can hold unexpected asthma triggers. Patients with
environmental allergies can have an increased risk of severe asthma exacerbation while working
outdoors due to the exposure to environmental allergens, weather changes, and other
environmental factors. Exposure to allergens such as pollen and ragweed can increase the
frequency and severity of asthma symptoms in these patients. Humidity in the summer months
often causes bronchospasm due to the thick, heavy nature of the air. Conversely, the winter months
can be especially difficult for the asthmatic worker working outdoors because cold, dry air,
especially at below-zero temperatures, is often not warmed well enough by the nasal passages and
subsequently causes severe bronchospasm. Weather changes in any season can cause increased
asthma exacerbations as the lungs in an asthmatic patient are hyperresponsive, reacting to changes
both in barometric pressure as well as temperature and humidity.

Outdoor Asthma Triggers

Outdoor asthma triggers can vary based on the season and region. Patients who are allergic to
trees, flowers, and ragweed will notice increased asthma symptoms when exposed to these triggers.
The effects of these triggers are worse on windy, dry days when the wind carries allergens and
irritants from miles away. Changes in humidity and air temperature can cause asthma
exacerbations in sensitive patients. Many patients find that abrupt changes in weather, particularly
after a thunderstorm, cause asthma exacerbations. Rainy days also correspond to a higher mold
count, and wet, fallen leaves hold the mold until they are blown by the wind, causing the mold to
become airborne and further exacerbate the patient’s asthma. Other inhaled particles, such as leaf
or wood smoke, are other outdoor asthma triggers. These inhaled irritants cause increased severity
of asthma exacerbations in patients due to the toxic by-products of the smoke provoking a patient’s
already hyperresponsive airways.

Obesity and Asthma

Obesity is directly related to severe, poorly controlled asthma as well as the development of asthma
in otherwise healthy patients. A body mass index (BMI) consistent with an obesity diagnosis is a
comorbidity for asthma in all patient populations, including adults and children. Studies indicate
that obese females are more likely to have obesity-related asthma than males. In obese patients

- 23 -
Copyright © Mometrix Media. You have been licensed one copy of this document for personal use only. Any other reproduction or redistribution is strictly prohibited. All rights reserved.

Licensed to elliot mattison (2146499335, i.astraltears@gmail.com) of 9801 royal ln, dallas, Texas 75231. If this product is distributed to others,
elliot mattison agrees to pay Mometrix the full retail price for each unlicensed recipient. Please play fair and respect the work of our authors.
with asthma, subcutaneous fatty tissue in the body releases inflammatory cells that can cause a
hyperreactive response in the airways. However, the main cause of obesity-related asthma
symptoms is the restrictive effect of an increased body weight. Obese patients have more weight on
their chest walls and tracheas, causing inadequate respiration and ventilation. This effect is also
known as reduced lung capacity. Some studies indicate that utilizing low tidal volume in resting
breaths can cause changes in the airway, which subsequently cause an increase in airway
hyperreactivity.

Effects of Obstructive Sleep Apnea

Obstructive sleep apnea is a disorder in which a patient stops breathing during sleep due to an
obstruction in the airway. This disease may be caused by the tongue falling back in the oral cavity
and covering the oral airway or from relaxation and compression of the airway due to increased
body weight and/or obesity. Obstructive sleep apnea is especially dangerous in patients with
asthma. During the airway obstruction, the patient’s vagal nerve and other neurological receptors
can be stimulated, resulting in bronchoconstriction and corresponding reduction in lung capacity. A
dramatic drop in intrathoracic pressure during these episodes can cause the capillaries within the
lung to fill, both effects contributing to subsequent bronchoconstriction and low lung volume.
Hypoxia and inflammation of both the upper and lower airways are common in asthmatic patients
with obstructive sleep apnea.

Vocal Cord Dysfunction

Vocal cord dysfunction is one disease that perfectly mimics asthma. The respiratory symptoms of
asthma and vocal cord dysfunction are identical, with wheeze, shortness of breath, and cough
present in both patient populations. Some patients with vocal cord dysfunction may also experience
a raspy voice and throat irritation. Triggers for vocal cord dysfunction are also identical to some of
the triggers associated with asthma. Inhaled irritants, exertion, acid reflux, and upper respiratory
infections are all triggers for both asthma and vocal cord dysfunction. It is difficult for a clinician to
diagnose vocal cord dysfunction as it is not a common disorder and is similar in appearance to
asthma. To diagnose vocal cord dysfunction, the patient must undergo spirometry testing as well as
a visual inspection of the structures of the airway via laryngoscope. If the vocal cords are not open
upon inhalation, vocal cord dysfunction is suspected. This patient may also have a flattened flow
volume loop pattern on their spirometry testing results due to the closure of the vocal cords during
inhalation, which is the hallmark of the disease.

Effects of Stress

Stress directly affects the severity of asthma and is a common root cause of asthma symptoms. The
physiological effects of stress on the airway cause bronchial hyperresponsiveness and subsequent
bronchoconstriction. During a stressful event, a patient may be anxious, afraid, or upset. These
feelings then trigger the nervous system to respond, often with a flight-or-fight response. The
systems of the body prepare for action by increasing blood pressure, increasing respiratory rate,
reducing tidal volumes during respiration, and constriction of the airways. The stressed nervous
system triggers bronchospasm and tissue inflammation, resulting in an asthma exacerbation.
Patients with stress-induced asthma may complain of their heart racing or pounding, cough,
wheeze, chest tightness, headache, and dyspnea. Stress-relieving exercises can be particularly
helpful for these asthmatic patients to prevent an asthma episode before it begins and to help
reduce asthma symptoms when an episode is occurring.

- 24 -
Copyright © Mometrix Media. You have been licensed one copy of this document for personal use only. Any other reproduction or redistribution is strictly prohibited. All rights reserved.

Licensed to elliot mattison (2146499335, i.astraltears@gmail.com) of 9801 royal ln, dallas, Texas 75231. If this product is distributed to others,
elliot mattison agrees to pay Mometrix the full retail price for each unlicensed recipient. Please play fair and respect the work of our authors.
Depression and Asthma

Many patients with asthma have a history of depression. Uncontrolled asthma and frequent, severe
asthma symptoms have an adverse effect on a patient’s quality of life and can cause symptoms of
depression. Although depression itself does not cause asthma, asthma can cause depression in
some patients. When a patient has uncontrolled or poorly controlled asthma, they undergo many
changes in lifestyle. Their jobs may become adversely affected by their chronic disease, perhaps due
to frequent absences or accommodations that must be made on the job. Complicated medication
regimens with unpleasant side effects can also cause feelings of hopelessness and annoyance in
patients with asthma. Patients with poorly controlled asthma may have a reduction in ability to
perform daily activities and participate in their favorite hobbies, which has a negative impact on
their quality of life and mental health, also possibly contributes to depression. Lack of empathy and
understanding from family, friends, and employers also add to an asthmatic patient’s risk of
depression.

Effects of Asthma on Pregnant Patients

Pregnancy has several possible effects on the asthmatic patient. Changes in hormones, weight, and
blood volume can all cause an increase in asthma symptoms. However, many pregnant women
don’t notice any increase in asthma symptoms at all. Other women notice their asthma either
improves or becomes worse during pregnancy. Unfortunately, there is no definitive evidence
explaining this variation in asthma severity. The expected severity or frequency of asthma
symptoms in a pregnant patient cannot be accurately predicted; however, some components of the
birth process can cause respiratory distress in a pregnant patient. This may include the position in
which the woman gives birth. Often, a patient gives birth in a supine position, with her knees
brought to her chest. In this position, the lungs and chest are constricted, causing a decrease in lung
capacity and feeling of breathlessness in some patients. A simple change in position may alleviate
these symptoms without pharmacological treatment.

Role of GERD in Asthma Exacerbations

Gastroesophageal acid reflux disease (GERD) is a common asthma trigger affecting millions of
Americans, especially those who also have obstructive sleep apnea and obesity. During a GERD
episode, stomach acids rise up through the esophagus due to a weakened lower esophageal
sphincter. The stomach acids then irritate the patient’s airways, causing cough, bronchospasm,
bronchoconstriction, wheeze, and chest pain. In addition, vagal nerve stimulation from the presence
of stomach acids in the esophagus causes an increase in airway resistance secondary to airway
inflammation and constriction. Asthma symptoms can also be exacerbated by the aspiration of
stomach acids in varying amounts during sleep. Patients with severe asthma that responds poorly
to conventional treatment methods may unknowingly have underlying GERD. Treatment of GERD,
including medications, sleep positional changes, and other therapeutic interventions, can help
improve a patient’s quality of life, reduce asthma symptoms, and minimize severe asthma
exacerbations in this patient population.

Role of Allergic Rhinitis in Triggering Asthma Episodes

Allergic rhinitis, commonly known as “hay fever” or “seasonal allergies,” is a common asthma
trigger in the spring and fall months; however, rhinitis can also occur in any season due to the
presence of indoor allergens. Exposure to allergens cause an allergic reaction characterized by an
itchy nose, nasal congestion, runny nose, headache, itchy eyes, and postnasal drip. All of these
symptoms may be present, or the patient may only present with one or two symptoms. Allergic

- 25 -
Copyright © Mometrix Media. You have been licensed one copy of this document for personal use only. Any other reproduction or redistribution is strictly prohibited. All rights reserved.

Licensed to elliot mattison (2146499335, i.astraltears@gmail.com) of 9801 royal ln, dallas, Texas 75231. If this product is distributed to others,
elliot mattison agrees to pay Mometrix the full retail price for each unlicensed recipient. Please play fair and respect the work of our authors.
rhinitis can disrupt sleep, making asthma worse during the night and in the early morning hours.
Patients waking up with a stuffy nose and asthma symptoms should suspect allergic rhinitis as a
trigger. When paired with rhinitis, asthma symptoms become more severe as the increased mucus
production, inflammation, and cough can all trigger bronchospasm and bronchoconstriction in an
asthmatic patient. Careful management of both asthma and allergic rhinitis is crucial in the allergic
asthma patient.

Sinusitis and Asthma Episodes

Sinusitis can cause frequent and severe asthma episodes. The root cause of sinusitis can be allergic,
infectious, or environmental in nature. In sinusitis, patients develop swollen, inflamed sinus tissue
in response to inhaled allergens, irritants, or the presence infections. The then partially or fully
obstructed sinus cavities collect mucus, harboring bacteria and resulting in an infectious sinusitis in
some cases. In other patients, the excess mucus production results in postnasal drip, which
provokes the airways into spasm and constriction, causing an asthma exacerbation. The sinusitis’s
inflammatory response also affects a patient’s airway, with airway inflammation common during a
sinusitis exacerbation. Chronic and frequent episodes of sinusitis cause asthma symptoms to be
more frequent and severe, sometimes resulting in poorly controlled asthma. The restricted airflow
through the sinuses also results in changing breathing patterns and/or snoring, which can
contribute to airway inflammation and bronchoconstriction.

Role of Eczema in Development of Asthma

Eczema, although not a direct asthma trigger, does affect the development of asthma in children.
Atopic dermatitis, or eczema, is considered the first step in the “atopic march” or “allergic march.”
This phenomenon is characterized by a progression of symptoms and diseases over a child’s
lifetime. Often the “march” begins with the development of eczema, which then progresses into a
subsequent diagnosis of asthma and allergies (including potentially life-threatening food allergies.)
Not all patients with eczema will be later diagnosed with asthma, but historical data and studies
indicate a strong connection between the two diseases. Eczema is characterized by scaly, itchy
patches of skin that tend to flare up in a cyclical pattern. Asthma is also a cyclical disease, with
patterns of symptoms corresponding to exposures and viral or bacterial illnesses. It is very
common for patients with eczema to later develop asthma and allergies.

Allergic Bronchopulmonary Aspergillosis

Allergic bronchopulmonary aspergillosis (ABPA) is an allergic disorder present in some individuals


with a weakened immune system. These patients have allergic reactions as a result of exposures to
the Aspergillus fumigatus, which is a fungus that is naturally occurring in soil. Patients with asthma
are particularly susceptible to ABPA due in part to both increased incidence of allergic reactions
and the presence of hypersensitive airway tissue. Symptoms of ABPA are similar to those of an
asthma episode, with cough being a common symptom. However, this disorder is infectious in
nature, with most patients presenting with a fever. This allergic and infectious reaction causes
asthma exacerbations due to the body’s inflammatory response to the Aspergillus fumigatus fungi.
Patients will notice a more severe asthma exacerbation when suffering from ABPA and will need
careful monitoring and treatment to successfully control both their asthma and their allergic
bronchopulmonary aspergillosis.

Effects of Pneumonia

Patients with asthma are often more susceptible to respiratory infections as their airways are
sensitive and hyperreactive. When asthmatic patients contract pneumonia, they are at a high risk

- 26 -
Copyright © Mometrix Media. You have been licensed one copy of this document for personal use only. Any other reproduction or redistribution is strictly prohibited. All rights reserved.

Licensed to elliot mattison (2146499335, i.astraltears@gmail.com) of 9801 royal ln, dallas, Texas 75231. If this product is distributed to others,
elliot mattison agrees to pay Mometrix the full retail price for each unlicensed recipient. Please play fair and respect the work of our authors.
for severe, life-threatening asthma exacerbations. Pneumonia can cause an inflammatory response
in the lungs, triggering an asthma exacerbation. This infection is characterized by symptoms such
as high fever, chills, shortness of breath, chest pain, cough, and areas of consolidation in the lower
lung fields. The consolidation prevents normal oxygenation and can result in hypoxia. Asthmatic
patients with pneumonia must be carefully monitored and treated aggressively to resolve the
infection and reduce asthma symptoms. An increase in the patient’s usual asthma medication
regimen may be necessary, with the use of bronchodilators and potentially increased dosage of
their maintenance medications. In addition, clinicians may consider administering a pneumococcal
vaccine in some susceptible patient populations to prevent future pneumonia infections. This
vaccination would occur when the patient is otherwise healthy.

Difference Between Asthma and COPD

Chronic obstructive pulmonary disease (COPD) is a permanent obstructive disorder characterized


by reduced expiratory flows and air trapping within the lungs. Patients commonly present with
breathlessness, wheeze, cough, and orthopnea (inability to lie flat). COPD is a permanent,
obstructive disease that is irreversible. Treatment can make the patient more comfortable but will
not resolve the underlying disease process. Conversely, asthma is a reversible obstructive
respiratory disease where pharmacological therapy and trigger avoidance can successfully resolve
the patient’s symptoms. It is chronic in the sense that it is a cyclical disease, with exacerbations
precipitated by triggers, causing patterns of symptoms. However, the symptoms can resolve in each
episode, allowing a patient a better quality of life. It is important to note that whereas asthma and
COPD are separate diseases, asthma can progress to COPD when uncontrolled. With severe, poorly
controlled asthma, untreated airway inflammation can cause permanent, irreversible damage to the
airway, including structural changes such as airway remodeling and scarring as well as increased
episodes of bronchospasm and reduced tolerance of environmental triggers exposure. These long-
term effects can eventually become irreversible, resulting in a COPD-like disease process.

- 27 -
Copyright © Mometrix Media. You have been licensed one copy of this document for personal use only. Any other reproduction or redistribution is strictly prohibited. All rights reserved.

Licensed to elliot mattison (2146499335, i.astraltears@gmail.com) of 9801 royal ln, dallas, Texas 75231. If this product is distributed to others,
elliot mattison agrees to pay Mometrix the full retail price for each unlicensed recipient. Please play fair and respect the work of our authors.
Assessment of an Individual with Asthma and Family
Educating Patients on Asthma Symptoms and Determining Personal Asthma Symptom
Pattern

Adult Patients
When educating a patient on asthma symptoms and trying to determine their own patterns, it is
important to ask open-ended questions. First, explain that symptoms vary from patient to patient
and that no patient’s symptoms are exactly alike. Explain that patients may experience cough,
wheeze, chest tightness, throat clearing, shortness of breath, chest pain, and trouble sleeping flat
(this disorder is known as orthopnea—many asthma patients need two or more pillows to elevate
themselves during sleep, which helps them breathe more easily). Ask how many pillows the patient
uses at night. Ask if the patient experiences any asthma symptoms with exercise or exertion, such
as walking up stairs, carrying groceries, or participating in recess or gym class. Use phrases like “tell
me about your breathing” or “describe some of the symptoms you notice when you have asthma
trouble.”

Pediatric Patients
Educating pediatric patients on the topic of asthma can be challenging. It is important not to
frighten the child. The words asthma attack should never be used with patients of any age. This
type of language instills fear in patients. The educator must use positive language and meet the
child at eye level during education. Use simple, clear terms. For example, bronchioles can be
described as the breathing tubes that are inside the lungs. The asthma educator might say, “These
tubes are like straws that can get plugged up with mucus when your asthma is bad. You might feel
like an elephant is sitting on your chest, or you might have a sore chest. It might feel hard to
breathe, and you might cough a lot, especially at night and in the early morning.” When attempting
to determine the patient’s individual pattern of symptoms, ask simple questions such as these:
“How do you feel when your breathing is bad?” “Can you show me where it hurts when your asthma
is bothering you?”

Determining Patient’s Quality of Life and Activity Tolerance

Adult Patients
When attempting to determine a patient’s quality of life or activity level, it is important to ask open-
ended questions such as these: “What hobbies do you enjoy?” “Can you tell me about some of your
daily activities?” “Has your activity level changed since you found out you had asthma?” “How often
does your asthma prevent you from doing the things you enjoy?” These simple questions help build
rapport with patients by not lecturing a patient but engaging in discussion about lifestyle choices.
Asthma education requires the patient to do the majority of the talking, with the educator taking
careful notes and making a determination on the severity and frequency of symptoms and how they
affect the patient’s home life. At that point, the educator can help tailor a treatment plan for the
individual to maximize his or her ability to join in activities of choice. Keeping active is essential in
the management of asthma and the preservation of a patient’s optimal quality of life.

Pediatric Patients
Although interviewing a pediatric patient may not be an easy task, it is essential to the proper
education and treatment of the asthmatic child. Education can involve the parents too, but the
patient must be part of the discussion, sharing his or her feelings and opinions to reach the goals for
therapy. With pediatric patients, asking simpler and more age-appropriate questions is essential.

- 28 -
Copyright © Mometrix Media. You have been licensed one copy of this document for personal use only. Any other reproduction or redistribution is strictly prohibited. All rights reserved.

Licensed to elliot mattison (2146499335, i.astraltears@gmail.com) of 9801 royal ln, dallas, Texas 75231. If this product is distributed to others,
elliot mattison agrees to pay Mometrix the full retail price for each unlicensed recipient. Please play fair and respect the work of our authors.
Some examples might include: “What kinds of things do you play with at home?” “Do you like to
play any running or jumping games?” These questions give the educator an idea of what the
patient’s interests are and can then build the conversation further. Follow-up questions might
include: “Can you tell me how your breathing feels when you play these games?” “Does your
breathing ever stop you from playing?” These types of questions are crucial for the determination
of a patient’s quality of life and exercise tolerance.

Determining Patient’s Level of Impairment

School Environment
The school environment is often where school-age pediatric patients spend the majority of their
time. It is imperative for the asthma educator to understand these patients’ level of control in the
school environment to successfully treat the patients’ asthma. In addition to asking about the
patient’s severity of symptoms at school, the educator should ask about the frequency of symptoms.
Questions such as “How often do you notice asthma trouble at school?” are an easy way to start the
discussion with school-age children. Smaller children might not be able to quantify their symptoms,
but there is value in starting the conversation as it promotes self-awareness in the patient’s future
school experience. In addition, it is necessary to determine how often asthma affects their academic
progress. Asking the parent or guardian how many school days the student has missed in the past
month or year is helpful in determining the patient’s level of control and impairment. Impairment
can also be measured by asking the patient a question such as these: “Do you play sports or
participate in gym class?” “Does your asthma stop you from playing?” These questions can help
determine the patient’s level of impairment.

Workplace
American workers spend many hours each day in the workplace, where successful asthma control
is imperative. Therefore, the asthma educator must investigate the patient’s level of control at
work. The educator should ask questions about the patient’s perception of asthma control in the
workplace as well as more easily measured data regarding workplace control, such as attendance.
Some questions on the topic of frequency and severity of symptoms and attendance might include
these: “How does your asthma affect your work life and career goals?” “Have you noticed your
asthma getting in the way of your work performance?” These perception-based questions can help
the patient be observant in the workplace and determine future levels of control, even if the patient
was not acutely aware of his or her impairment level prior to the asthma education session. In
addition, asking patients quantitative questions such as “How many days of work have you missed
in the past month or year due to your asthma?” is a good way to gain information that is easily
tracked and measured in the future.

Educating on Signs and Symptoms of Respiratory Distress Requiring Immediate Treatment

Pediatric patients often can’t effectively identify and communicate their symptoms to caregivers. As
a result, the parent or caregiver must carefully monitor the patient during an asthma episode and
identify signs of respiratory distress as soon as they become apparent. Some of these symptoms
may include intercostal or subcostal retractions. The asthma educator should describe the
appearance of this symptom, explaining that retractions look like the areas around the stomach and
ribs are “sucking in” during breathing. Tracheal tugging can be described as the area at the base of
the neck “sucking in” during inhalation, sometimes with the added symptom of neck muscles
bulging or flexing at the same time. Nasal flaring, another sign of respiratory distress, can be
described as flaring of the nostrils during inhalation. Parents should also watch for fast breathing,

- 29 -
Copyright © Mometrix Media. You have been licensed one copy of this document for personal use only. Any other reproduction or redistribution is strictly prohibited. All rights reserved.

Licensed to elliot mattison (2146499335, i.astraltears@gmail.com) of 9801 royal ln, dallas, Texas 75231. If this product is distributed to others,
elliot mattison agrees to pay Mometrix the full retail price for each unlicensed recipient. Please play fair and respect the work of our authors.
pale or bluish skin tone, and inability to speak (in older children). Any of the aforementioned
symptoms require immediate medical intervention.

Educating Adult Patients on Signs and Symptoms of Severe, Life-Threatening Asthma


Episode

Although all asthma symptoms require treatment, some are more dangerous than others. Some
patients may not be aware that asthma can be a life-threatening disease if left untreated. Therefore,
the asthma educator must explain in clear terms exactly what constitutes a medical emergency.
Explain to the patient that if he or she is too breathless to speak in full sentences, it is considered an
emergency. The use of accessory muscles, such as in the neck, a shoulder, and arms, is also a sign of
a severe asthma exacerbation. Describe these symptoms to the patient using simple language, such
as “If you are bracing yourself or leaning over to breathe, you need to go to the hospital.” It is
imperative that the educator instruct the patient to either call an ambulance or get a ride to the
hospital. Patients must never drive themselves to the hospital in the midst of an asthma episode.

Increase in Asthma Risk with Past Intubations and Hospitalizations

A patient’s asthma risk is determined in part by taking into consideration the patient’s past
hospitalizations and intubations. The number of hospitalizations the patient has had in the past
year directly correlates with the patient’s future asthma exacerbation severity and risk. A history of
multiple hospitalizations indicates that the patient is likely to be hospitalized again in the near
future. In addition, a history of intubation is a major risk factor for future severe, life-threatening
asthma exacerbations. Once intubated in the past, the patient is at a greater risk for future
intubations and possible ventilator dependency or death. Asking a patient about the frequency of
past hospitalizations and the number of past intubations is essential. Tell patients that once they
are hospitalized or intubated for asthma, they are more likely to have similar outcomes in the
future, unless the cycle is broken.

Role of Bronchodilator Overuse in Severity of Asthma Exacerbations

The overuse of beta agonists, commonly known as asthma “rescue” medications and or
bronchodilators, is a potentially serious problem for poorly controlled asthmatics. Some patients
with asthma choose to medicate solely with rescue medications, as opposed to incorporating
suggested maintenance medications into their regimens, such as inhaled corticosteroids or
leukotriene modifiers. The overuse of beta agonists can be associated with cardiac side effects,
rebound bronchospasm, and increased tolerance to the medication. A patient’s written action plan
outlines the suggested frequency of use for bronchodilators. Any deviation from this recommended
dose can cause adverse events including severe, life-threatening asthma exacerbations. If a patient
is not well controlled on his or her current regimen of bronchodilators, the asthma educator and
health-care team must reevaluate the plan and make adjustments as needed to improve the
frequency and severity of the patient’s asthma symptoms.

Frequent Use of Systemic Corticosteroids

The use of systemic corticosteroids is a helpful tool in the control of severe, uncontrolled asthma.
However, due to the serious side effects associated with this classification of drug, the therapy must
be utilized sparingly and only when absolutely necessary. Systemic corticosteroids are used to
reduce inflammation and relax the smooth muscle in the patient’s airway. However, serious side
effects such as mood swings, Cushing’s syndrome (a metabolic disorder associated with
corticosteroid use), and osteoporosis can occur with overuse or misuse of the medications.
Frequent use of systemic steroids might indicate that the patient is either not using quick relief

- 30 -
Copyright © Mometrix Media. You have been licensed one copy of this document for personal use only. Any other reproduction or redistribution is strictly prohibited. All rights reserved.

Licensed to elliot mattison (2146499335, i.astraltears@gmail.com) of 9801 royal ln, dallas, Texas 75231. If this product is distributed to others,
elliot mattison agrees to pay Mometrix the full retail price for each unlicensed recipient. Please play fair and respect the work of our authors.
medications or not taking maintenance medications as prescribed. The asthma educator must
carefully review the patient’s written asthma action plan, ask the patient about medication use, and
perhaps even call the patient’s local pharmacy to determine how often the rescue and maintenance
medications are being refilled. A lack of refills indicates a lack of use.

Poor Perceiver

A poor perceiver is a patient who does not have full awareness of his or her asthma exacerbation
symptoms, frequency, and or severity. In some cases, a poor perceiver does not know how to
determine what is normal breathing and what is an emergency. Other patients might not
understand the difference between mild, moderate, and severe asthma symptoms. These patients
are at an increased risk for severe, life-threatening asthma exacerbations and asthma related death
as they are not able to correctly identify severe symptoms. Some poor perceivers simply do not
notice any asthma symptoms until they become severe. In patients with poor perception, it is the
asthma educator’s role to educate these patients on common asthma symptoms, including examples
of mild, moderate, and severe forms of each symptom. For example, the educator might describe a
mild shortness of breath as chest heaviness or tightness, whereas severe shortness of breath could
be described as breathlessness that interrupts a patient’s speech. Tools such as peak flow meters,
which measure the patient’s peak expiratory flow, may be of considerable help for poor perceivers
as they can use this tool to directly measure their level of asthma control.

Educating Asthmatic Patients on Possible Reasons for Lack of Asthma Control

Loss of asthma control can be caused by many different factors. Lifestyle changes, such as an
increase in exercise, change in environment, or change in physical health can all affect a patient’s
asthma control. Abrupt changes in the severity of a patient’s asthma must be carefully investigated
by the asthma educator. The educator must interview the patient, asking open-ended questions. For
example, the educator might ask, “Can you tell me about any changes that might have taken place at
home or at work? Has anything changed in your overall health since your asthma became worse?”
Discovering the addition of a new pet, recent travel to a different climate, changes in workplace or
job, and other environmental factors can all contribute to a loss of asthma control. Addressing these
changes, and offering solutions, can help patients better control their asthma and improve their
quality of life.

Measuring Patient’s Severity, Control, and Risk

An asthma educator can assess a patient’s impairment and risk by interviewing the patient about
his or her current lifestyle. Asking questions about environmental triggers, past hospitalizations,
medication regimen, and perception of asthma symptoms can all help determine a patient’s level of
asthma control. In addition, the educator must ask the patient to quantify the asthma exacerbations
by explaining how often the exacerbations occur as well as how severe they are. Keeping an asthma
diary can be helpful in determining a level of control as this serves as a record of all asthma
symptoms, the severity of those symptoms, actions taken to resolve the exacerbation, and any
trigger exposures that may have occurred. A full medical history, including comorbidities, must be
taken. Patients with comorbidities such as acid reflux disease, obesity, and obstructive sleep apnea
all have an increased risk of poor asthma control.

Referral to Asthma Specialist

Not all patients with asthma require a specialist’s care. Patients who are well controlled under their
primary physician’s care do not usually require a specialist’s referral. However, patients with
increased risk and impairment may require more specialized care from a physician who has a

- 31 -
Copyright © Mometrix Media. You have been licensed one copy of this document for personal use only. Any other reproduction or redistribution is strictly prohibited. All rights reserved.

Licensed to elliot mattison (2146499335, i.astraltears@gmail.com) of 9801 royal ln, dallas, Texas 75231. If this product is distributed to others,
elliot mattison agrees to pay Mometrix the full retail price for each unlicensed recipient. Please play fair and respect the work of our authors.
concentration in refractory (severe, uncontrolled) asthma. Some of the criteria that leads to a
specialist referral include frequent past hospitalizations (especially within the past year), frequent
or daily asthma symptoms, severe asthma exacerbations requiring intubation, severe symptoms
impacting the patient’s ability to perform daily tasks and hobbies, and inability to control
symptoms despite increasing medication dosage or adding pharmacological therapy. These criteria
are all signs of very poorly controlled asthma. In addition, any prolonged reduction in a patient’s
quality of life, despite medical care, is a sign that the patient needs further assistance from an
asthma specialist such as a pulmonologist.

Helping Patients Identify Asthma Triggers

The first step in identifying a patient’s asthma triggers is to educate the patient on possible causes
of exacerbations. Explain common asthma triggers such as allergies, mold, dust, dander, weather
changes, exercise, strong emotions, certain foods, and occupational or school-based exposures.
Then interview the patient, asking open-ended questions about the home and work or school
environment. The educator may ask questions such as these: “Tell me about your home.” “What
kind of work do you do?” After the educator gathers information about the possible environmental
triggers of the patient’s asthma, the educator can then ask questions regarding other possible
triggers. These questions might include the following: “Do you have any pets at home?” “What kinds
of hobbies and exercise do you enjoy?” “Have you noticed anything that might make your asthma
worse?” After the educator describes common asthma triggers and asks questions about their
presence in the patient’s daily environments, the patient will be more likely to identify some of his
or her possible asthma triggers.

Effects of Exercise on Patient’s Asthma

Exercise is a common asthma trigger, with millions of asthmatics experiencing asthma symptoms
during or after physical exertion. Breathlessness, cough, chest tightness, chest pain, and wheeze are
all common exercise-induced asthma symptoms. The act of exercise causes an increased oxygen
demand in the systems of the body. To keep up with the demand, the patient increases his or her
respiratory rate and tidal volume. The change in breathing pattern can trigger bronchospasm in
patients with asthma. Other exercise-related asthma triggers, such as breathing in hot or cold air or
inhaling allergens, are associated mainly with outdoor exercise. The presence of exercise-induced
asthma is easily treated and managed, but the asthma educator must first determine if exercise and
physical exertion are asthma triggers for their patient. This requires careful interviewing and
discussion about the patient’s activity level and tolerance.

Interviewing Patients to Determine Presence of Exercise-Induced Asthma

Although exercise-induced asthma is common, not all asthmatic patients experience symptoms
during exercise and exertion. To determine the presence of exercise-induced asthma, the asthma
educator must interview the patient about his or her asthma triggers. If the patient doesn’t mention
exercise specifically, then the educator should ask open-ended questions such as these: “Tell me
about your activity level” “What kind of exercise and hobbies do you enjoy?” At that point, the
educator can ask follow-up questions such as, “Have you noticed any changes in your stamina or
breathing while enjoying these activities?” If the patient complains of asthma symptoms during
exertion, such as walking up flights of stairs, running, playing sports, or carrying heavy items,
exercise-induced asthma is suspected. This specialized form of asthma can stand alone or be
present in patients who have a general diagnosis of asthma. In other words, exercise can be one of
many asthma triggers for a patient, or it may be the only asthma trigger for the patient.

- 32 -
Copyright © Mometrix Media. You have been licensed one copy of this document for personal use only. Any other reproduction or redistribution is strictly prohibited. All rights reserved.

Licensed to elliot mattison (2146499335, i.astraltears@gmail.com) of 9801 royal ln, dallas, Texas 75231. If this product is distributed to others,
elliot mattison agrees to pay Mometrix the full retail price for each unlicensed recipient. Please play fair and respect the work of our authors.
Comorbidities

Sinusitis
Sinusitis, or inflammation of the sinus cavities, is a common comorbidity in asthma patients,
especially in those who have environmental or seasonal allergies and sensitivities. This condition
can exacerbate a patient’s asthma due to chronic postnasal drip, which irritates the airway and
provokes airway inflammation and constriction. Chronic and or frequent episodes of sinusitis can
cause asthma symptoms to be more frequent and severe, sometimes resulting in poorly controlled
asthma. The asthma educator can determine the presence of sinusitis by taking a thorough medical
history and asking the patient about past and present symptoms. For example, the educator might
ask if the patient suffers from sinusitis symptoms such as facial pain, pressure, and swelling as well
as “ear popping.” If the patient does suffer from allergies and/or sinusitis symptoms, the educator
can ask follow-up questions such as these: “How often would you say you feel these symptoms?”
“Do you notice these symptoms at certain times of year?” Questions such as “Have you noticed your
asthma is worse during your allergy season?” and “How often do you notice pressure around your
eyes and cheekbones or popping in your ears?” can help the patient identify these symptoms as
those consistent with sinusitis.

Nasal Polyps
Nasal polyps are noncancerous growths found in the nasal passages, which can be large enough to
affect normal breathing and sinus drainage. This abnormal tissue develops due to chronic, inflamed
nasal passage and sinuses, often as a result of uncontrolled allergies and sinusitis. This
inflammatory process is a common comorbidity for asthma as both sinusitis and severe allergies
are also considered comorbidities. These disorders go hand in hand, often with more than one of
these conditions present simultaneously. This cyclical inflammatory process (allergies lead to
sinusitis, which leads to polyps) is a direct trigger for asthma exacerbations. The symptoms of nasal
polyps, including nasal congestion, stuffiness, and postnasal drip or runny nose, all irritate the
lower airways. This can cause bronchoconstriction and airway inflammation in patients with
asthma. These subsequent asthma exacerbations are often severe in nature and can be poorly
controlled due to these comorbidities.

GERD
Gastroesophageal acid reflux disease (GERD) is a comorbidity that often pairs with asthma,
especially in obese patients. GERD is a condition in which stomach acids rise up through the
esophagus, causing airway irritation and exacerbating a patient’s asthma. Some patients with GERD
experience heartburn and indigestion while sleeping or while resting in a reclined position. Others
experience heartburn that is not positional in nature. The asthma educator must screen all patients
for GERD. The educator can directly ask the patient if they have a GERD diagnosis. If the answer is
no, but the patient complains of increasing nighttime asthma symptoms and/or uncontrolled,
severe, and frequent asthma exacerbations, the asthma educator must interview the patient further.
The educator can ask questions such as these: “How often, if at all, do you experience heartburn and
indigestion?” “Do you notice a change in symptoms at night?” These questions can help determine
the presence of GERD, but the final diagnosis is made by the patient’s physician.

Review Video: Peptic Ulcers and GERD


Visit mometrix.com/academy and enter code: 184332

Obesity
A high body mass index (BMI) is associated with a diagnosis of obesity. This condition is a common
comorbidity in asthma. Obesity is currently considered an epidemic in the United States. It can
- 33 -
Copyright © Mometrix Media. You have been licensed one copy of this document for personal use only. Any other reproduction or redistribution is strictly prohibited. All rights reserved.

Licensed to elliot mattison (2146499335, i.astraltears@gmail.com) of 9801 royal ln, dallas, Texas 75231. If this product is distributed to others,
elliot mattison agrees to pay Mometrix the full retail price for each unlicensed recipient. Please play fair and respect the work of our authors.
cause poorly controlled asthma due to the inflammatory process that originates in the patient’s
adipose tissue. The patient’s overall increased body weight restricts breathing, causing asthma
exacerbations. Obesity can be determined by measuring the patient’s BMI. This measurement is a
weight-to-height ratio, which can be easily measured by taking the information directly from the
patient’s chart. If this information is not available, the asthma educator can ask the patient for his or
her height and weight and then use a BMI calculator to determine the patient’s obesity risk. The
educator can also ask the patient about recent weight fluctuations to determine whether the patient
has a previous history of obesity and achieved subsequent weight loss or if the patient is currently
gaining or maintaining weight. This information is important as more in-depth asthma education on
the subject of obesity, and possible referral to a weight management physician or program, could be
required for some patients.

Obstructive Sleep Apnea


Obstructive sleep apnea is a common comorbidity in asthma patients, especially in the overweight
or obese patient population. Obstructive sleep apnea is a disorder in which a patient stops
breathing during sleep due to an obstructed airway. This may be caused by the tongue falling back
in the mouth and covering the airway or from compression of the airway due to increased body
weight. Obstructive sleep apnea exacerbates asthma due to the patient’s vagal nerve being
stimulated by the airway obstruction. This results in bronchoconstriction and hypoxia. Some
patients may not be aware that they suffer from obstructive sleep apnea. Some of the common
symptoms of this disorder include nighttime symptoms such as snoring and periods of apnea
during sleep as well as daytime symptoms such as headache, grogginess, and fatigue. The asthma
educator should interview these patients and determine their levels of control as well as the extent
to which the obstructive sleep apnea affects their asthma. The educator can ask questions such as
these: “How is your energy level during the day?” “Has anyone ever told you that you snore at
night?” These questions can help the educator decide whether to refer this patient to a physician
specializing in sleep disorders and/or discuss ways to treat this disorder on his or her own.

Determining Using Over-the-Counter Asthma Medications

The use of over-the-counter asthma medications is of vital importance to the patient’s health,
safety, and quality of life. These types of medications are strictly prohibited for the treatment of
asthma as they are ineffective and potentially dangerous. The asthma educator must interview the
patient to determine whether these medications are being used. The educator must exhibit no signs
of judgment for patients utilizing over-the-counter medications as many patients have not been
educated against their dangers and assume that they are safe because they do not require a
prescription. The educator might need to ask a more direct version of the question such as, “Do you
use any over-the-counter asthma medications?” If the patient is unsure of which medications they
are taking, the educator may need to make reference to some brand names of over-the-counter
medications such as Bronkaid or Primatene. Many patients do not know the generic names of their
medications.

Determining Which Prescription Asthma Medications a Patient is Taking

Adhering to a medication regimen is one of the post important components of asthma education.
The asthma educator must interview the patient and ask appropriate questions to determine which
medications are a part of the patient’s current and past medication regimens. Some patients start
and stop medications on their own without their physician’s knowledge, and this behavior must be
corrected. The asthma educator can ask questions such as, “What prescription medications do you
take for your asthma?” The educator may also ask a follow-up question such as, “Have you started
or stopped any medications in the past few months?” This will help the educator determine if the

- 34 -
Copyright © Mometrix Media. You have been licensed one copy of this document for personal use only. Any other reproduction or redistribution is strictly prohibited. All rights reserved.

Licensed to elliot mattison (2146499335, i.astraltears@gmail.com) of 9801 royal ln, dallas, Texas 75231. If this product is distributed to others,
elliot mattison agrees to pay Mometrix the full retail price for each unlicensed recipient. Please play fair and respect the work of our authors.
medication regimen is complete or if perhaps some medications have been stopped without being
instructed to do so.

Determining Which Herbal or Nutritional Supplements a Patient is Taking

Herbal remedies and supplements are a popular business today, but these unregulated products
can be dangerous for patients with asthma. Without a physician’s approval, there is no way to know
if these therapies will interact with other medications or with the patient’s asthma itself. The
asthma educator has a responsibility to thoroughly interview the patient about these products as
the patient may not know that they can possibly interact with his or her current. physician-
approved medication regimen. Asking a question such as “Do you take any herbal or nutritional
supplements?” can be helpful; however, some patients are not aware that some of these products
are considered supplements, which are not approved by the Food and Drug Administration. They
may refer to these products as “vitamins” or “herbs,” so the asthma educator must carefully listen to
the patient’s verbal cues and ask follow-up questions to determine exactly what substance the
patient is utilizing and what form the substance is in (i.e., pills, powders, oils, and inhalers.)

Role of Food Additives and Preservatives in Triggering Asthma

Asthma triggers vary among patients, and some triggers are rarer than others. Some patients
experience asthma symptoms due to food additives and preservatives. This trigger isn’t as common
in the general asthma population but can be particularly bothersome for some patients. Patients
affected by these substances notice an increase in asthma symptoms within minutes to hours after
eating the food. This can include food dyes, additives, preservatives, and chemicals. Many patients
choose to purchase and consume natural food products that do not contain these substances to
better control their asthma. These products do not have any inherent risk to the patient; in fact, the
patient’s asthma can be better controlled due to avoidance of this asthma trigger. If food additives
appear to be an asthma trigger for a patient, the asthma educator should then suggest total
avoidance of that substance.

Potential Role of Natural Food Products in Patient’s Asthma Action Plan

Some patients’ asthma triggers include food additives, preservatives, and dyes. For these patients,
total avoidance is essential. Therefore, the asthma educator must ask the patient about avoidance
techniques and determine if the patient’s asthma is well controlled using the patient’s current
avoidance methods. The educator also must determine whether the patient is using any “natural”
supplements, herbal remedies, or alternative medications in his or her regimen as these products
can be hidden sources of foodborne asthma triggers. These products are not approved or regulated
by the Food and Drug Administration and do not have to follow the same stringent procedures and
labeling laws that conventional food and beverage companies have to follow. Some patients
discover hidden sources of their triggers in unregulated supplement products. Careful reading of
the product label and perhaps even contacting the product company might be required to ensure
that the product is safe for patients with food-based intolerances.

Asthma Management

Acupuncture
Acupuncture is a therapy originating in Chinese medicine. In this therapy, needles are inserted into
areas of the patient’s skin to help balance the energies that flow throughout the body. The needles
can be stationary or rotated within the skin by the practitioner. Acupuncture practitioners believe
that acupuncture aligns the body’s energies and cures diseases that occur do to imbalances. This

- 35 -
Copyright © Mometrix Media. You have been licensed one copy of this document for personal use only. Any other reproduction or redistribution is strictly prohibited. All rights reserved.

Licensed to elliot mattison (2146499335, i.astraltears@gmail.com) of 9801 royal ln, dallas, Texas 75231. If this product is distributed to others,
elliot mattison agrees to pay Mometrix the full retail price for each unlicensed recipient. Please play fair and respect the work of our authors.
concept is scientifically unfounded, and current research does not show any correlation between
acupuncture and asthma control. This therapy is not conducted by board-certified physicians, and
therefore, the safety and efficacy of this practice is unknown. Current studies indicate that
acupuncture does not lead to better asthma control. The therapy does have risks, including blood-
borne illness, tissue damage, piercing of the internal organs, bruising, pain, and minor discomfort.
Therefore, the use of acupuncture is not recommended for the control of asthma symptoms.

Yoga
Yoga, the practice of rhythmic deep breathing and stretching, is a form of exercise that is popular in
the United States today. Students of yoga perform a series of deep breathing exercises as well as
slow, sustained movements and stretches during class sessions. Although yoga is not considered a
treatment for asthma, it may have a complimentary role for some asthmatic patients. Pursed-lip
breathing is a natural way to alleviate shortness of breath in diseases such as asthma and chronic
obstructive pulmonary disease (COPD). This method of breathing creates peak end expiratory
pressure (PEEP), which helps the airways to remain open during exacerbations. Similarly, the deep-
breathing exercises in yoga sessions can help a patient remain calm in an asthma emergency and
alleviate some of the air hunger in an asthma episode. Some patients may experience orthopnea
(difficulty breathing while lying flat) during yoga, but a simple position change may be helpful. Yoga
is not a proven treatment for asthma but can be utilized safely in many asthmatics as a relaxation
method for exacerbations.

Significance of Family History of Asthma or Allergies in Asthmatic Patients

Asthma and allergies are genetic disease processes that “run” in families. Parents with asthma have
a much higher incidence of having children with asthma. The same is true for allergies, with food
and environmental allergies showing a strong multigenerational genetic component in families.
Some family members may have identical allergies, whereas others have completely different ones;
however, the mere presence of allergies indicates that this disease process is passed genetically to
children from their parents and grandparents. It is important to get an accurate familial medical
history from the asthma patient as the patient’s asthma and allergy history helps determine the
patient’s asthma risk. Therefore, identifying familial patterns of asthma and allergies can help the
asthma educator tailor an asthma and allergy action plan to the individual patient, which will help
control these disease processes and improve the patient’s quality of life.

Gathering Clinical History of Current Illness

A full assessment of the patient’s current asthma exacerbation and/or illness is essential to the
successful treatment of an asthma episode. The asthma educator must ask the patient several
questions about his or her current illness, even if this information is previously documented in the
chart. Often, after a friendly conversation with the patient, the asthma educator will learn new facts
about the case that were not previously shared with the medical staff. Asking the patient to describe
the exacerbation with statement such as “Tell me how this all started” and/or “Tell me how your
asthma has changed over the past day or two” can give the educator an idea of not only how the
exacerbation began but how it has changed with treatment. The educator must document when the
exacerbation started, its symptoms and potential triggers, any action taken to treat the
exacerbation, and the patient’s current symptoms and comfort level. Vital signs must be
documented as well as breath sounds and physical examination findings.

- 36 -
Copyright © Mometrix Media. You have been licensed one copy of this document for personal use only. Any other reproduction or redistribution is strictly prohibited. All rights reserved.

Licensed to elliot mattison (2146499335, i.astraltears@gmail.com) of 9801 royal ln, dallas, Texas 75231. If this product is distributed to others,
elliot mattison agrees to pay Mometrix the full retail price for each unlicensed recipient. Please play fair and respect the work of our authors.
.

Including Patient’s Past Asthma-Related Medical History in Patient’s Chart

The patient’s past asthma-related hospitalizations, symptoms, and possible triggers are important
pieces of information to include in the patient’s record. This information helps the asthma educator
determine risk and impairment, which are two main components to successful asthma education
and management. For example, a patient with numerous past hospitalization and subsequent
intubations is treated differently than a patient who has a cough and mild shortness of breath only
on exertion. Obtaining a full and accurate description of past exacerbations gives the educator
valuable information that helps determine the course of treatment for the patient. Knowing a
patient has an increased asthma risk could result in ordering additional asthma medications or
increasing the dosage of the patient’s current home medications. Conversely, if the patient has mild,
intermittent symptoms that do not require daily medication, the asthma educator may simply
reiterate the patient’s current asthma action plan and make no changes.

Importance of Obtaining Patient’s General Past Medical History

When obtaining an asthmatic patient’s medical history, the asthma educator is looking for
comorbidities that might make the patient’s asthma more difficult to treat and control. The asthma
educator must ask the patient about any prior medical history that could contribute to the patient’s
current exacerbation, such as common asthma comorbidities. Diseases such as obesity, nasal
polyps, allergies of any kind, obstructive sleep apnea, and gastroesophageal acid reflux disease all
can cause bronchospasm and airway inflammation in patients with asthma. These underlying
symptoms can cause the patient’s exacerbations to be more difficult to control and can lead to
severe and possibly even life-threatening asthma episodes. Treating the patient’s underlying
comorbidities is an essential part of the asthma educator’s treatment plan as the patient’s asthma
symptoms cannot be successfully treated until their comorbidities are under control. The asthma
educator should be prepared to refer the patient to other practitioners or specialists for
comorbidity treatment if necessary.

Physical Examination of Asthma Patient

A complete physical examination is part of an asthma educator’s assessment. Several important


clinical findings must be assessed and included in the patient’s record. The asthma educator must
assess and record the patient’s inspiratory and expiratory breath sounds. The most common breath `
sounds noted in asthmatic patients include inspiratory or expiratory wheeze and diminished breath
sounds in any lung field. Patients may also have rhonchi due to increased mucus production from
either an underlying respiratory infection or the asthma exacerbation itself. The patient’s position
and level of comfort must also be recorded in the patient’s chart. If a patient is positioned as a
“tripod” where the patient is bracing him- or herself to breathe, the patient is likely experiencing
shortness of breath. Other signs of respiratory distress such as accessory muscle usage in the chest
and neck must be noted. In pediatric patients, tracheal tugging and substernal or intercostal
retractions might be present during an asthma episode. Tracheal tugging looks like “sucking in” at
the base of the patient’s neck, whereas retractions look like “sucking in” between the patient’s ribs
and at the center of the chest.

Important Vital Signs in Asthmatic Patient’s Record

The asthma educator’s physical examination of the patient includes assessing and recording the
patient’s vital signs. First, the asthma educator must measure the patient’s heart rate and
respiratory rate. An elevated respiratory rate and heart rate is expected in a patient with an asthma
exacerbation due to the exertion required to breathe during these episodes of inflammation and

- 37 -
Copyright © Mometrix Media. You have been licensed one copy of this document for personal use only. Any other reproduction or redistribution is strictly prohibited. All rights reserved.

Licensed to elliot mattison (2146499335, i.astraltears@gmail.com) of 9801 royal ln, dallas, Texas 75231. If this product is distributed to others,
elliot mattison agrees to pay Mometrix the full retail price for each unlicensed recipient. Please play fair and respect the work of our authors.
bronchoconstriction. The patient’s blood pressure must also be assessed either manually or by
automatic blood pressure cuff as part of continuous inpatient monitoring. Last, a patient’s oxygen
saturation must be measured via oxygen saturation (SPO2) monitor and recorded in the patient’s
chart. Hypoxia is a serious complication associated with asthma exacerbations, and the asthma
educator must be ready to report any drop in oxygen saturation and take action to treat it.
Supplemental oxygen may be required for oxygen desaturation in a patient presenting with an
asthma exacerbation.

Arterial Blood Gas Testing

For a patient with asthma exacerbations, laboratory testing is often not necessary, as the patient is
usually treated based on clinical signs as opposed to blood tests; however, some laboratory testing
is helpful for patients with severe asthma. Some potentially life-threatening exacerbations with
profound air hunger, dyspnea, and increased work of breathing may indicate that the patient is in
impending respiratory failure. In this case, an arterial blood gas (ABG) might be ordered, which will
show the acid-base balance in the body as well as the patient’s oxygen saturation (SPO2), partial
pressure of oxygen in arterial blood (PaO2), partial pressure of carbon dioxide (PCO2), and blood
PH. These measurements can determine whether the patient is in impending respiratory failure
and subsequently requires intubation with mechanical ventilation or noninvasive mechanical
ventilation. Although noninvasive SPO2 monitoring is the gold standard for measuring a patient’s
oxygen saturation, the invasive ABG test is helpful for more severe patients who might require
emergency interventions.

RAST Allergy Testing

Some patients with uncontrolled asthma have underlying allergies either to food or the
environment. If the patient’s clinical and past medical histories indicate a possible allergy, the
patient can undergo allergy testing. This would help pinpoint the substance to which the patient is
allergic and, in turn, help the asthma educator instruct the patient on allergic trigger avoidance. One
common laboratory test for allergies is called the radioallergosorbent (RAST) allergy test. This
simple blood collection tests for the presence of allergic sensitivity to a variety of allergens. This can
be particularly helpful for patients with unknown allergies; however, the test is not accurate and
often requires further testing via skin prick for comparison and confirmation. Keeping an asthma
diary, and recording any allergy symptoms, activities performed, and environmental conditions, can
be helpful in pinpointing which allergens might be the culprit. Allergic reactions can cause more
frequent and severe asthma exacerbations, and the cause of these reactions must be thoroughly
investigated.

PFT

Pulmonary function testing (PFT) refers to a variety of spirometry tests that measure many
different components of respiration. Some of the common measurements include lung volumes,
inspiratory and expiratory flows, nitric oxide measurement (FENO), diffusion capacity (DLCO), vital
capacity, and other more specialized lung function measurements and maneuvers. Simple
spirometry, the most common PFT ordered for patients with asthma measures the patient’s flows
and volumes. This can be used to obtain a baseline value for the patient’s respiratory status when
the patient is symptom free. This test can also be performed before and after bronchodilation to
determine the reversibility of the patient’s disease and his or her response to pharmacological
treatment. More specialized pulmonary function testing can confirm a patient’s asthma diagnosis by
provoking the airways to constrict if asthma is present. PFT should be performed at least once per
year for patients with asthma. Patients who have more frequent and severe exacerbations may

- 38 -
Copyright © Mometrix Media. You have been licensed one copy of this document for personal use only. Any other reproduction or redistribution is strictly prohibited. All rights reserved.

Licensed to elliot mattison (2146499335, i.astraltears@gmail.com) of 9801 royal ln, dallas, Texas 75231. If this product is distributed to others,
elliot mattison agrees to pay Mometrix the full retail price for each unlicensed recipient. Please play fair and respect the work of our authors.
require more frequent monitoring. It is also important to note that a PFT serves as a “snapshot in
time” for an asthmatic patient. The test shows the patient’s condition only in that moment in time
and cannot predict future exacerbations or give clues as to the severity of past exacerbations.

Radiological Testing

Radiological testing, such as X-rays, can be helpful in the treatment of asthma. X-rays can be
portable or stationary in nature. A patient can have a chest X-ray with a portable machine without
leaving the hospital bed or may go to the radiology department for a standard chest X-ray.
Regardless of the type of chest X-ray, the test itself is helpful in determining both the cause and
severity of the asthma exacerbation. A severe asthma exacerbation presents on a chest X-ray as air
trapping in the lungs, with hyperinflation noted. This often looks like large, black spaces in the
lungs with the sixth rib visible in the X-ray. The chest X-ray may also show an underlying infection
such as pneumonia. This often looks like areas of consolidation in the lung bases, giving the X-ray
image a fuzzy appearance toward the bottom of the lungs.

Importance of Asthma Patient’s Previous Asthma Therapies in Asthma Management

Obtaining a complete history of past medical interventions is important in managing a patient’s


asthma. The clinician should ask questions such as these: “What medications do you currently take
for your asthma?” “Have you started or stopped any medications in the past year?” “Tell me about
how these medications are working for your asthma.” These questions will help the educator
determine the patient’s compliance in his or her medication regimen as well as informing the
educator as to past and current medication regimens. For example, if the asthma educator discovers
that the patient stopped using his or her inhaled corticosteroid because the inhaler made him or
her hoarse, the educator can change the route of administration for the medication and suggest
ways to minimize side effects. Understanding a patient’s previous medication regimen, the reasons
for any changes in the regimen, and any adverse side effects experienced can help the educator
offer alternative therapies that might be more tolerable to the patient.

Objective Testing Measures

Although invasive and noninvasive testing measures exist for the diagnosis and treatment of
asthma, there is no one standard test that can diagnose asthma. Often, the diagnosis is made with a
combination of objective testing such as oxygen saturation measurement (SPO2), arterial blood gas
measurement (ABG), chest X-ray, pulmonary function testing (PFT), and radioallergosorbent
(RAST) allergy testing in addition to other subjective anecdotal information. Although each of these
tests can be helpful in the diagnosis of asthma, none can accurately diagnose asthma on its own.
These tests, used in tandem with a thorough physical exam, medical history, medication regimen
review, and environmental exposure history, can help determine an asthma diagnosis. Only the
specialized pulmonary function test called a methacholine challenge can singularly diagnose
asthma. This test provokes an asthma exacerbation in asthmatic patients upon inhaling the
chemical methacholine; however, this test is not always completely accurate and should be used in
conjunction with the aforementioned assessment tools.

Physical Signs of Asthma Exacerbation

During an asthma exacerbation, a patient will present to the clinic complaining of shortness of
breath at rest or with exertion. The most common symptoms of an asthma exacerbation are
wheeze, cough, and dyspnea. Signs of a more severe exacerbation can include cyanosis, which is
characterized by a bluish tint to the patient’s lips or skin and/or pale or dusky-looking skin. Severe
asthma episodes can also include trouble speaking in full sentences. Very labored breathing, often

- 39 -
Copyright © Mometrix Media. You have been licensed one copy of this document for personal use only. Any other reproduction or redistribution is strictly prohibited. All rights reserved.

Licensed to elliot mattison (2146499335, i.astraltears@gmail.com) of 9801 royal ln, dallas, Texas 75231. If this product is distributed to others,
elliot mattison agrees to pay Mometrix the full retail price for each unlicensed recipient. Please play fair and respect the work of our authors.
presenting as an increased respiratory rate and low tidal volume or gasping breaths, is also
common in severe asthma exacerbations. More mild exacerbations may include more subtle
symptoms such as cough, chest tightness, throat clearing, and throat irritation. It is imperative to
note that a chronic cough may be the patient’s only symptom of asthma. Although wheeze is the
most common asthma symptom, it may be completely absent in an asthma exacerbation.

Breath Sounds

Rales
Crackles or rales are breath sounds commonly auscultated in the lowest parts of the patient’s lungs.
The base of the lungs is where consolidation commonly occurs in patients with pneumonia. This
consolidation is caused by the alveoli filling with fluid and impairing gas exchange through the
alveolar capillary membranes. When the clinician auscultates these patients’ backs, they are
listening to sounds in the posterior lung fields. This is where crackles will be heard. The crackling
sound is heard only on inspiration and is caused by the consolidated areas of the lungs and the
fluid-filled alveoli popping open as air fills the lungs. Crackles sound like Rice Krispie-type breakfast
cereal when it comes into contact with milk: a faint popping or crackling sound during auscultation.
A clinician who hears crackles should investigate the cause further as a pneumonia infection can
complicate the patient’s asthma treatment plan and result in more severe, harder-to-manage
asthma symptoms.

Wheezing
Wheezing is the most common breath sound associated with an asthma diagnosis. Wheezing
sounds like whistling in the airways. This can occur on inspiration, expiration, or during both
inhalation and exhalation. An expiratory wheeze is more common with asthma exacerbations;
however, and inspiratory wheeze can also be noted in severe asthma episodes. Wheezing can be
present in the anterior lung fields during auscultation of the patient’s chest and/or posterior lung
fields upon auscultation of a patient’s back. Wheezing can be diffuse, meaning that it is present in
multiple lung fields anteriorly and or posteriorly, or they can be localized, meaning that they are
present in only one area of the patient’s lungs. The squeaky wheeze sound is due in part to the
constriction of the bronchioles. A similar sound can be demonstrated when a person attempts to
whistle through pursed lips.

Diminished Breath Sounds


Diminished breath sounds can be some of the most dangerous breath sounds for asthmatics.
Diminished or silent breath sounds are heard when the clinician auscultates the lungs and hears
minimal air movement or possibly even none at all. The clinician will hear faint sounds of normal
air movement (whooshing sounds on inspiration or expiration), wheezing, and/or rales. (This can
be heard anteriorly or posteriorly.) If none of these sounds are heard, the patient is said to have
silent chest. This is the most dangerous type of breath sound because silent chest and diminished
breath sounds are associated with minimal to zero air movement in the lung fields. This is an
emergency situation as the patient is no longer effectively ventilating and may require
supplemental oxygen, bronchodilation, and emergency ventilator support such as intubation and
mechanical ventilation or noninvasive mechanical ventilation. It is also important to note that some
patients never wheeze but might have only slightly diminished, clear breath sounds. Although this
is not necessarily a medical emergency, it is still a sign of an asthma exacerbation and should be
treated appropriately.

- 40 -
Copyright © Mometrix Media. You have been licensed one copy of this document for personal use only. Any other reproduction or redistribution is strictly prohibited. All rights reserved.

Licensed to elliot mattison (2146499335, i.astraltears@gmail.com) of 9801 royal ln, dallas, Texas 75231. If this product is distributed to others,
elliot mattison agrees to pay Mometrix the full retail price for each unlicensed recipient. Please play fair and respect the work of our authors.
Determining Necessity of Referral to Emergency Services

Identifying the clinical signs of a severe asthma exacerbation is critical for all members of the care
team but especially for the asthma educator. The asthma educator often spends the most time
talking to the patient and educating him or her; therefore, the educator’s assessment skills must be
exceptional. Some asthma patients must be referred to emergency services by their asthma
educator. In these instances, the educator observes air hunger, increased work of breathing, trouble
speaking, and fatigue. If the patient must stop to take a breath during regular speech, and cannot
speak in full sentences, it is considered a potentially life-threatening emergency. The use of
accessory muscles, such as in the neck and shoulders for adults, and retractions or tracheal tugging
for pediatric patients, are all signs of a severe, emergent exacerbation. Last, if patients are using a
tripod stance, in which they are bracing themselves with their arm muscles during respiration, they
need to be referred to emergency services. It is imperative that the educator calls an ambulance
immediately. The patient must not be permitted to drive him- or herself to the hospital.

Asthma Diary

Asthma diaries can be helpful for both the patient with asthma as well as the asthma educator
treating that patient. An asthma diary can be a calendar or day planner booklet. Each day, the
patient logs his or her symptoms, environment, activities, medication use, peak flow readings (if
applicable), and any other information that might be pertinent to asthma management. The patient
can then look back through the days of the past week, month, and even year to identify patterns of
asthma and allergy symptoms. If, for example, the patient discovers that every time he or she visits
his or her grandmother’s house, they have an asthma exacerbation within a day or two, he or she
can look for environmental triggers at that home and work to minimize them. They may also be
advised to pre-medicate with a bronchodilator prior to entering that environment. Identifying
possible triggers and resulting patterns of symptoms is the first step in successfully treating a
patient’s asthma.

Asthma Control Test®

The Asthma Control Test® is a standardized test that patients can use to quickly determine their
level of asthma control. This is a helpful tool for poor perceivers who may not be able to identify a
severe asthma exacerbation. This test asks patients to look back over the past month and use a five-
point scale to rate their asthma. First, patients must record how often their asthma affected their
normal activities and hobbies. Next, patients must record how often they experienced shortness of
breath. Patients then record how many times nighttime symptoms have interrupted their sleep.
Then, patients record how many times they’ve used their rescue medications. Last, patients give
themselves a general asthma control rating for the past month. Each criterion is rated on a five-
point scale. Patients then add up their scores. If the score is less than 20, the patient is likely to be
suffering from uncontrolled asthma.

Childhood Asthma Control Test®

The Childhood Control Test® is a standardized test designed for use by the pediatric asthma
population. Patients can use this test to quickly determine their level of asthma control. This is an
age-appropriate tool that can help children ages 4 to 11 identify a severe asthma exacerbation. This
test asks patients to use a three-point scale to rate their asthma control. First, patients must rate
how their asthma feels at that moment. Next, they record how often their asthma affected their
normal activities and hobbies. Next, patients must record how often they experienced coughing.
Patients then record how many times nighttime symptoms have interrupted their sleep. Then, the

- 41 -
Copyright © Mometrix Media. You have been licensed one copy of this document for personal use only. Any other reproduction or redistribution is strictly prohibited. All rights reserved.

Licensed to elliot mattison (2146499335, i.astraltears@gmail.com) of 9801 royal ln, dallas, Texas 75231. If this product is distributed to others,
elliot mattison agrees to pay Mometrix the full retail price for each unlicensed recipient. Please play fair and respect the work of our authors.
parents answer the last three questions, which include how often the patient has daytime asthma
symptoms, how many times the patient had a daytime wheeze in the past month, and how often the
parent noticed nighttime awakenings due to asthma. The score is added up, and if it is less than 20,
the patient is likely to be suffering from uncontrolled asthma.

Importance of Allergy Testing

Asthmatic patients with suspected allergies should be evaluated by an allergist to determine the
allergens to which the patient is sensitive. Pinpointing allergens helps the asthma educator offer
appropriate suggestions for allergic trigger avoidance, which is essential to asthma management.
The educator should explain the importance of these tests using layperson’s terms. For example,
the educator might say, “Based on the symptoms you are describing, it sounds like you might have
some allergies. It is important to consider allergy testing so we can pinpoint exactly what is
bothering your asthma. That way, I can help you avoid those things, and that will help improve your
symptoms. Allergies and asthma are related and they affect each other, so treating your allergies
helps treat your asthma too.” The asthma educator can also give a basic description of the testing
process. The educator might say, “Allergies are tested in a few different ways. Sometimes it’s a
simple blood test, whereas other times it is a series of tiny skin pricks. The allergist can tell you
more about what testing is best for you.”

Importance of Evaluating Presence of Nasal Polyps and Sinus Congestion

Sinus congestion and/or nasal polyps can be problematic for asthma patients. Uncontrolled
sinusitis and the presence of nasal polyps can affect the patient’s breathing as well as cause
postnasal drip, which irritates the patient’s lungs. This acts as an asthma trigger, causing poorly
controlled asthma. The asthma educator must explain how these conditions are evaluated using
simple terminology. The educator might say, “Treating sinus problems is an important part of the
asthma puzzle. Sinus trouble can cause your asthma to become worse and can make it much harder
to control. The symptoms you’ve described sound like you might have a problem with your sinuses.
A doctor can help figure that out by doing some simple tests. They might numb your nose with
some spray and take a look with a long scope or camera. Or they might send you for a simple X-ray
of your sinuses. Both of these tests help the doctor see exactly what’s going on in your nose and
sinuses, so they know the best way to treat them.”

Importance of Testing for Obstructive Sleep Apnea

Obstructive sleep apnea is a serious comorbidity for some asthma patients. Patients who are
overweight, snore, and/or stop breathing at night might have obstructive sleep apnea. The asthma
educator must explain the testing process and its importance using simple language that all
patients can understand. The educator might say, “Based on the symptoms you’ve talked about, it
sounds like you might need to be checked for obstructive sleep apnea. That’s a condition where you
have trouble breathing during sleep. It can be dangerous if it is not treated, and it can make your
asthma much worse. A doctor can test for this condition by sending you for a sleep study. In this
test, you either sleep overnight in the hospital or possibly sleep at home with special monitors. The
doctor places a few stickers and sensors on you to measure your breathing while you sleep. Then
after you wake up, the doctor will check over all the information from overnight and let you know
what the next step is for your care.”

Importance of Testing for GERD

Gastroesophageal acid reflux disease (GERD) is a comorbidity for many asthma patients. In this
condition, stomach acids back up into the esophagus, causing inflammation in the airway. This

- 42 -
Copyright © Mometrix Media. You have been licensed one copy of this document for personal use only. Any other reproduction or redistribution is strictly prohibited. All rights reserved.

Licensed to elliot mattison (2146499335, i.astraltears@gmail.com) of 9801 royal ln, dallas, Texas 75231. If this product is distributed to others,
elliot mattison agrees to pay Mometrix the full retail price for each unlicensed recipient. Please play fair and respect the work of our authors.
effect then causes asthma exacerbations due to bronchospasm and airway inflammation. The
asthma educator must explain the importance of testing for this dangerous comorbidity using
simple language that the patient can understand. The educator might say, “Because your asthma is
not well controlled, and you have mentioned to me that you notice heartburn at night, it might be a
good idea to check you for something called GERD. That’s another name for acid reflux. A doctor can
test you for that by doing a PH test, where they test your stomach acids using a tiny sensor. If the
doctor thinks this test is necessary, he or she can explain it in more detail. He or she might also be
able to determine if you have GERD simply from talking with you about your symptoms.”

Spirometry Testing

Spirometry is a common test used to help manage a patient’s asthma. When spirometry is ordered,
the asthma educator must describe the use of spirometry testing as part of a successful asthma
management plan. The asthma educator might say, “Spirometry testing is a type of breathing test
that measures different parts of your breathing. It can measure how much air your lungs hold, how
fast you can exhale, and how powerful your lungs are. Depending on which test your doctor orders,
you would breathe into a machine and breathe in certain patterns for each test. For example, one
spirometry test has you take a giant breath in, blast it out as hard and fast as you can, and then take
a huge breath in. In other tests, you might pant like a puppy or breathe normally. The tests are
painless and helpful in both diagnosing asthma and seeing how bad your asthma is at any given
moment. These tests are important for patients with asthma, and many doctors have their patients
do these tests at least once a year to compare your breathing from year to year.”

Obstructive Flow Volume Loop and Reduced FEV1

When discussing test results in a clinical setting, the physician ordering the test interprets the test
results and records the findings. The asthma educator can then further discuss the findings with the
patient. During spirometry testing, an asthmatic patient’s flow volume loop will show an
obstructive pattern, with reduced flows and normal volumes. The forced expiratory volume in 1-
second (FEV1) portion of the test indicates the severity of bronchospasm and obstructed airways.
An FEV1 of less than 60 percent of the patient’s predicted value indicates a poorly controlled and
severe asthma episode. The asthma educator must explain these results using simple language that
all patients can understand. The educator might say, “Take a look at this graph with me. You might
notice that this hill-looking shape is scooped out. That means that you are able to get the air in, but
you’re having trouble getting it out. That is what makes asthma different from other lung diseases.
Also, this FEV1 number tells me how much you can breathe out in 1 second. The lower the number,
the worse your asthma is at that moment.”

Proper Peak Flow Meter Technique

A peak flow meter is a tool that measures a patient’s forced expiratory flow. This reading is helpful
for asthmatics as the patient can track results daily and identify patterns and changes over time. A
peak flow reading is taken by instructing the patient to exhale completely. Then the patient puts the
peak flow meter mouthpiece into the mouth, closing the lips tightly around it and keeping the teeth
away from the opening. Then the patient takes a maximum inhalation and exhales as hard and fast
as he or she can. They follow these steps twice more, and they record their highest number out of
the three attempts. This is the number that is tracked over time. The patient must record this
number daily, especially when first diagnosed or if the patient is a poor perceiver. The patient can
then take action when the peak flow drops. A drop in peak flow reading indicates an asthma
exacerbation and can predict an asthma episode even before the patient feels symptoms.

- 43 -
Copyright © Mometrix Media. You have been licensed one copy of this document for personal use only. Any other reproduction or redistribution is strictly prohibited. All rights reserved.

Licensed to elliot mattison (2146499335, i.astraltears@gmail.com) of 9801 royal ln, dallas, Texas 75231. If this product is distributed to others,
elliot mattison agrees to pay Mometrix the full retail price for each unlicensed recipient. Please play fair and respect the work of our authors.
Personal Best and Predicted Peak Flow Values

Peak flow monitoring is helpful in both identifying an asthma exacerbation and observing patterns
of symptoms over time. The patient’s daily peak flow readings are compared to either the personal
best reading or the predicted value, and then an action plan is designed utilizing those reference
values. If the patient has a personal best peak flow reading (the highest peak flow reading ever
achieved), he or she would compare the current peak flow reading to that number. If the patient
does not have a personal best, then he or she would compare the current reading with a predicted
value, which is calculated using height and sex. Each peak flow meter has a table with predicted
peak flow values that the clinician or patient can use to calculate his or her predicted peak flow
value. Utilizing a personal best peak flow gives much more accurate results than a predicted value
because some patients may never reach their predicted value even when completely healthy. This is
because the predicted peak flow value is based on a healthy individual with no underlying lung
disease.

Peak Flow Zones

Peak flows are used as a tool to rate an asthma exacerbation and take appropriate action to treat
the episode. The system is designed like a traffic light with green, yellow, and red zones. In the
green zone, the patient is between 80 and 100 percent of his or her personal best or predicted value
(the clinician must always use a personal best number if one is available). In this zone, no action
needs to be taken. The patient is well controlled. In the yellow zone, the patient’s peak flows are
between 79 and 50 percent of his or her personal best or predicted value. This is the most
important zone in asthma management as this is where an exacerbation is beginning. Here, the
patient must follow the written asthma action plan designed by the physician and asthma educator.
Some common actions taken in this zone include starting to use a rescue inhaler, increasing the
inhaled corticosteroid dose, and notifying a physician. In the red zone, the patient is less than 50
percent of his or her personal best or predicted value and must head to the nearest emergency
room.

Proper Peak Flow Meter Technique, and Personal Best vs. Predicted Value

When educating a patient on peak flow meters, the asthma educator might say, “A peak flow meter
is a tool that tells you how bad your breathing is at any given moment. Exhale all the way, and then
put the mouthpiece in your mouth. Inhale as far as you can and hold it. Now blast it out as hard and
fast as you can. The number you see on the meter is what you want to watch. Now let’s do that
twice more. The highest number out of the three tries is your best number. Now let’s compare that
to your peak flow zones. These zones are like a traffic light, with red, yellow, and green. If your
number falls in the green zone, you’re doing well. In the yellow zone, your asthma is starting to
become worse. Here you need to take action so that your asthma improves. Follow the steps your
doctor outlined in your action plan when you reach the yellow zone. If your numbers fall in the red
zone, it is an emergency. Get help immediately.”

Peak Flow Predicted Values vs. Personal Best Values

When explaining the difference between a personal best and predicted peak flow value, the asthma
educator might say, “Finding your personal best peak flow number is important. The peak flow
meter package instructions can give us a number to strive for based on your height, but those
numbers are based on healthy people who do not have asthma. You may never reach that number
even when you are feeling great, so the best way to monitor your peak flows is to find out what
your personal best is. To do that, you use your peak flow meter in the morning and at night for 2

- 44 -
Copyright © Mometrix Media. You have been licensed one copy of this document for personal use only. Any other reproduction or redistribution is strictly prohibited. All rights reserved.

Licensed to elliot mattison (2146499335, i.astraltears@gmail.com) of 9801 royal ln, dallas, Texas 75231. If this product is distributed to others,
elliot mattison agrees to pay Mometrix the full retail price for each unlicensed recipient. Please play fair and respect the work of our authors.
weeks when you’re feeling well. Take the best number out of three tries each time and record it.
After the 2 weeks are up, check your numbers, and find the highest reading. That is your new
personal best. This is the number we will base your peak flow zones on.”

Improving Adherence to Peak Flow Meter Regimen

Patients often find difficulty in adhering to a peak flow regimen when it is not designed around
their usual schedule and daily activities. One way to improve adherence is to recommend ways to
incorporate peak flow measurement into a patient’s daily routine. For example, ask the patient
what his or her usual weekday entails. Weekdays are often more busy and regimented than
weekends, so this is the time frame that needs to be investigated first. If the patient says that he or
she works long hours and can’t find the time to perform the peak flows, the educator can suggest
that the patient keep the peak flow meter on the bedside table and use it as soon as he or she wakes
up in the morning and right before he or she goes to bed at night. Patients may also keep it in their
medicine cabinet next to their toothbrush and use it after brushing their teeth in the morning and at
night. Once their asthma is stabilized, daily peak flow monitoring may no longer be required.

Pre- and Post-Bronchodilation Pulmonary Function Testing

Measuring a patient’s response to bronchodilation is helpful in determining the appropriate course


of treatment for that individual. Pulmonary function testing measures a patient’s lung volumes and
flows and can be used to determine the reversibility of a patient’s asthma when performed before
and after the administration of a bronchodilator. In this type of testing, the patient would perform
simple spirometry without any use of a bronchodilator or other asthma medications. Then, the
patient would be administered a bronchodilator either by nebulizer treatment or via inhaler. The
patient would then repeat the pulmonary function testing. If the patient’s post-bronchodilation
forced vital capacity and/or forced expiratory volume in 1 second (FEV1) increases by 12 percent
or more, the results show an adequate response to therapy with reversibility of bronchospasm.
Another indicator of reversibility is a forced expiratory flow of 25 to 75 percent of vital capacity
(FEF25-75) value increasing by at least 25 percent after bronchodilation.

Importance of Pre- and Post-Bronchodilation Pulmonary Function Testing


When educating a patient on the use of pre- and post-bronchodilation pulmonary function testing,
the educator might say, “The doctor has ordered an important breathing test for your asthma,
called a pulmonary function test. This is a test where you breathe in and out through a machine. The
machine measures how much air goes in and out of your lungs as you breathe, and it also measures
how hard and fast you can exhale and inhale. It is a painless test that tells us about your asthma and
how it responds to asthma medicines. You will do the test twice: once without medication and once
after taking a breathing treatment. If your breathing numbers are better after the breathing
treatment, then we know that the asthma medication is doing its job.”

Pulse Oximetry Monitoring

Pulse oximetry testing is a noninvasive method of measuring a patient’s oxygen saturation. During
pulse oximetry, the patient wears a lighted sensor either on his or her finger, toe, or forehead. The
red light in the sensor penetrates the skin and is absorbed by the patient’s hemoglobin, measuring
the hemoglobin’s oxygen saturation. This can be performed either via a spot check, meaning that
the clinician performs a one-time measurement of the patient’s saturation, or pulse oximetry can be
performed via continuous monitoring, in which the patient remains on the pulse oximeter for an
extended period of time. Pulse oximetry is an important measurement for patients with asthma as
hypoxia and periods of desaturation are common in asthma exacerbations, especially those with

- 45 -
Copyright © Mometrix Media. You have been licensed one copy of this document for personal use only. Any other reproduction or redistribution is strictly prohibited. All rights reserved.

Licensed to elliot mattison (2146499335, i.astraltears@gmail.com) of 9801 royal ln, dallas, Texas 75231. If this product is distributed to others,
elliot mattison agrees to pay Mometrix the full retail price for each unlicensed recipient. Please play fair and respect the work of our authors.
severe and life-threatening symptoms. If a patient’s saturation is dropping, supplemental oxygen
may be required. A normal saturation is 97 percent or higher.

Properly Setting Up Pulse Oximeter and Troubleshooting the Machine


An accurate pulse oximetry reading is essential to the proper care and monitoring of an asthmatic
patient. Several factors can cause a pulse oximeter to malfunction or not read properly. When
placing the pulse oximetry sensor on a patient, the practitioner must choose the correct size for the
patient. The sensors come in neonatal, infant, pediatric, and adult sizes. Not all facilities will have all
of the sizes, so the clinician will need to choose the best-fitting option for the patient. Once these
issues are corrected, the clinician must ensure that the monitor’s saturation reading is accurate.
One way to do this is to take the patient’s pulse and compare it to the reading on the pulse oximeter.
If there is a discrepancy in the heart rate reading, the saturation reading is not accurate.

Troubleshooting a Pulse Oximeter That is Not Displaying Saturation Results


When monitoring a patient with an asthma exacerbation, accurate measurement of the patient’s
oxygen saturation is essential to the evaluation of the patient’s ventilatory status. Several factors
can adversely affect the accuracy of a pulse oximetry measurement, resulting in low readings and or
no readings at all. First, the clinician must ensure that the pulse oximetry sensor is wrapped around
the patient’s finger or toe, with the light side of the probe directly opposite the sensor. If the
sensor’s adhesive is no longer sticky, replace the sensor. Ensure that the patient’s skin is not too
cold or poorly perfused. If it is, the sensor will either not measure the saturation at all or will show
a very low reading. The clinician must also remove any dark or red-colored nail polish on the
patient’s fingers or toes as this will inhibit the sensor’s light from permeating the patient’s nail bed.
If a patient has artificial fingernails, the sensor can be attached to a patient’s toe.

Educating on Use of Pulse Oximetry


When educating an asthmatic patient on the use of pulse oximetry, the educator might say, “The
doctor has ordered a special monitor so we can keep a close watch on your oxygen levels. This
machine is called a pulse oximeter. It measures the oxygen level in your blood without even
pricking your finger. The light in the sticker on your finger bounces off of your blood and measures
the amount of oxygen in it. Your oxygen levels will display on this bedside monitor, and we will
know if you need a little extra oxygen or not. If your oxygen numbers are too low, we can give you a
mask or nose piece that will give you a little extra air and help you to breathe easier. Unfortunately,
I might have to remove any nail polish that you might be wearing because it interferes with the
sticker’s light. The light can’t pass through nail polish and won’t give us an accurate oxygen
number.”

Exhaled Nitric Oxide Measurement

Exhaled nitric oxide measurement via pulmonary function test is helpful in determining the extent
of a patient’s airway inflammation. It can also be used as a monitoring tool, allowing clinicians to
compare past results with future values to identify patterns of exacerbation as well as changes in
asthma episode severity. This test can be performed as a baseline record of the patient’s underlying
inflammation and then repeated after therapeutic intervention with inhaled or systemic
corticosteroids. A decreased level of exhaled nitric oxide after intervention would then indicate if
the steroids are resolving the patient’s airway inflammation. The patient places the pulmonary
function test machine’s mouthpiece into his or her mouth, inhales for several seconds, and then
performs a prolonged exhalation into the mouthpiece. The pulmonary function machine will then
measure the levels of nitric oxide in the patient’s exhaled breath. Elevated levels of exhaled nitric

- 46 -
Copyright © Mometrix Media. You have been licensed one copy of this document for personal use only. Any other reproduction or redistribution is strictly prohibited. All rights reserved.

Licensed to elliot mattison (2146499335, i.astraltears@gmail.com) of 9801 royal ln, dallas, Texas 75231. If this product is distributed to others,
elliot mattison agrees to pay Mometrix the full retail price for each unlicensed recipient. Please play fair and respect the work of our authors.
oxide indicate an inflammatory response in the patient’s airways and can help confirm an asthma
diagnosis (if one has not been made previously).

When educating a patient on the use of exhaled nitric oxide measurement, the educator must
explain the purpose of the test as well as what the test entails. The educator might say, “The doctor
has ordered a special breathing test to help find out how inflamed your lungs are due to your
asthma. The test is called an exhaled nitric oxide test. The test is painless and simple to perform.
You will use a mouthpiece that is hooked up to a special machine that measures your breaths. You
will inhale for a few seconds and then exhale into the machine for several seconds. The machine
then measures how much nitric oxide gas is in your breath. The inflamed cells in your lungs release
nitric oxide gas, so measuring it can give us a better idea of how inflamed your lungs are.”

In Vitro-Specific IgE Testing for Allergies

In vitro-specific immunoglobulin E (IgE) testing for allergies is performed via blood test. Blood is
drawn from a patient’s veins and is tested in the lab for IgE responses to several different allergens.
Hundreds of allergens can be tested, including foods, animals, trees, flowers, dust mites, and other
substances. Based on a patient’s pattern of symptoms and suspected triggers, the physician will
order testing for a variety of allergens to which the patient may be allergic. The presence of an IgE
response indicates sensitization to that specific allergen. The test is scored on a numeric scale;
however, a higher number does not necessarily indicate how sensitive the patient is to that
allergen, nor does it indicate the possible severity of a reaction. This test measures only
sensitization, which may not result in an allergic reaction to the substance. Many patients are
sensitized to a substance and never experience allergy symptoms. This test is not always accurate,
with a high false negative rate. This test is often performed in addition to skin prick testing,
comparing the two tests’ results to determine the likelihood of an allergic reaction.

Procedure and Interpretation


When educating a patient on the procedure for in vitro immunoglobulin E (IgE) allergy testing, the
asthma educator might say, “Your doctor has ordered a blood test for allergies to see if you might be
sensitive to something that is making your asthma worse. For this test, blood will be drawn from a
vein in your arm, and then it will be sent off to a lab that tests for an allergic response to different
allergens. It takes up to a week for your results to come, and then the doctor will go over the results
with you. The test will tell you if you are sensitive to different allergens, but it is important to know
that sometimes there are false negatives with this test. Many doctors do both blood tests and skin
tests for allergies; this helps the doctor confirm whether or not you’re allergic to something.”

Skin Prick Allergy Testing

Skin prick allergy testing often takes place in a physician’s office. Depending on the patient’s
symptoms and suspected triggers, the physician will order testing for a variety of allergens to which
the patient may be allergic. This test can be performed in addition to allergy testing via blood (in
vitro-specific immunoglobulin E [IgE] testing) because neither test is completely accurate on its
own. In skin prick testing, the clinician pricks or scratches the patient’s skin with a lancet that
contains an allergen. This action is performed as many times as needed to test for all possible
allergens. If the patient is being tested for only a few allergens, the test can be performed on the
forearm. Otherwise, the testing can be performed on the patient’s back. In addition to testing for
allergens, the patient has two control pricks: one for histamine and one for saline. It is expected that
the patient will react to the histamine but not the saline. If this doesn’t happen, then the test is null
and void.

- 47 -
Copyright © Mometrix Media. You have been licensed one copy of this document for personal use only. Any other reproduction or redistribution is strictly prohibited. All rights reserved.

Licensed to elliot mattison (2146499335, i.astraltears@gmail.com) of 9801 royal ln, dallas, Texas 75231. If this product is distributed to others,
elliot mattison agrees to pay Mometrix the full retail price for each unlicensed recipient. Please play fair and respect the work of our authors.
Interpretation
In skin prick allergy testing, the patient’s forearm or back is pricked in several areas with a lancet
containing different allergens. There is one prick for each allergen, and there are two additional
pricks for histamine and saline. The patient should have a positive reaction to the histamine and a
negative reaction to the saline; otherwise, the test is not accurate. A positive reaction is determined
when a raised wheal, which looks similar to a mosquito bite, and reddened surrounding skin (a
flare) occur at an allergen prick site. The physician will then measure both the wheal and the flare
to give the reaction a numeric score. This score does not indicate the future severity of a reaction;
instead it is used as a comparison tool for future allergy testing. A positive test indicates
sensitization to the allergen. A negative result is often accurate; however, false positives can be a
common problem in this type of testing.

Educating Patients
When educating a patient on skin prick allergy testing, the educator might say, “Your doctor has
ordered skin testing for allergies. This will help the doctor determine if you might be sensitive to
something that is making your asthma worse. Skin testing is done in a doctor’s office, where the
nurse will prick your skin in different places with a special tool that has a little bit of an allergen on
it. They might do this on your forearm or on your back, depending on how many pricks they have to
do. It feels like little scratches with a fingernail. If you’re allergic to the substance, your skin will puff
up like a mosquito bite at the spot where they pricked you, and it will become red and possibly a
little itchy. The doctor will measure the spot and then give you some allergy cream or maybe some
antihistamines to take the itch away. You will have your results in less than an hour, so you will
know right away if you’re allergic to something or not. But sometimes there are false positives with
this test, so many doctors do both skin tests and blood tests.”

Methacholine Challenge

Methacholine is a drug that provokes the airways into bronchospasm in a patient with asthma. A
methacholine challenge is a specialized pulmonary function test in which a patient with suspected
asthma performs spirometry testing before and after the administration of methacholine. This test
is often performed to confirm a diagnosis of asthma in a patient with unusual or sporadic asthma
symptoms. This test does have potentially serious risks associated with it as the clinician is
essentially forcing the patient into an asthma exacerbation. Therefore, emergency medical
equipment must be available at all times. This includes but is not limited to a crash cart, intubation
equipment, supplemental oxygen, and resuscitation bag with mask. The clinician must also be
prepared to administer a bronchodilator immediately upon completion of the post-methacholine
administration spirometry maneuver to reverse the patient’s bronchospasm.

Procedure
In a methacholine challenge, spirometry is performed before and after the administration of the
drug methacholine. This bronchoprovocation drug causes bronchospasm in patients with asthma.
The patient must be informed of all aspects of the procedure, including its risks, prior to the test.
The patient will first perform simple spirometry testing to obtain baseline measurements of lung
volumes and flows. The clinician will then administer methacholine treatments, carefully
monitoring the patient for signs of respiratory distress. The patient then repeats the spirometry
maneuvers. In a patient with asthma, the post-methacholine maneuvers will show an obstructive
pattern with an expiratory flow rate of 20 percent less than baseline and normal lung volumes. The
clinician will then administer a bronchodilator treatment to resolve the bronchospasm while
constantly monitoring the patient. If respiratory distress occurs at any time during testing, the test
is terminated, and emergency treatment is initiated.

- 48 -
Copyright © Mometrix Media. You have been licensed one copy of this document for personal use only. Any other reproduction or redistribution is strictly prohibited. All rights reserved.

Licensed to elliot mattison (2146499335, i.astraltears@gmail.com) of 9801 royal ln, dallas, Texas 75231. If this product is distributed to others,
elliot mattison agrees to pay Mometrix the full retail price for each unlicensed recipient. Please play fair and respect the work of our authors.
Educating Patients
When educating a patient about methacholine challenge testing, the clinician must explain the
procedure as well as any risks without causing the patient any unnecessary fear or anxiety. The
educator might say, “The doctor has ordered a breathing test for you called methacholine challenge
testing. This test can mimic an asthma episode in patients who are sensitive to the medication. This
can help the doctor determine if you have asthma. First, you will breathe in and out of a special
machine that measures your breaths. Then the staff will give you a methacholine breathing
treatment, and you will do the test a second time. After that, the staff will give you another
breathing treatment to reverse the effects of the methacholine. You might notice some chest
tightness after the methacholine, but don’t worry; the staff will be watching you very closely and
will treat you right away if you’re having any troublesome symptoms.”

Determining Peak Flow Technique Result Accuracy

Peak flow meter result accuracy is completely dependent on patient effort and technique. If a
patient blocks the mouthpiece with his or her tongue, or spits into the mouthpiece, the indicator
will rise rapidly. This results in falsely elevated results. The best way to combat this issue is to
carefully monitor the patient, while he or she uses the peak flow meter, and coach them on the best
practices. Instruct them to place the mouthpiece between the teeth, keeping the tongue beneath the
mouthpiece and away from the mouthpiece opening. Conversely, inaccurate low results can occur
when patients do not exhale hard and fast enough. The patient should be instructed to take the
biggest breath in that they can and blast it out as hard and fast as they can. The asthma educator
may need to bring along a demo peak flow meter that they can use to demonstrate the proper
technique to the patient.

Phenomenon of Severe Asthma Symptoms with No Drop in Peak Flow Values

Peak flow meters, although a helpful tool in asthma management, are not helpful for all patients.
Most patients find a direct correlation between reduced peak flow meter results and the presence
of asthma symptoms. However, a smaller patient population can experience asthma symptoms with
no reduction in peak flow meter results. This is often seen in patients who do not wheeze during
asthma episodes. These patients often present with a cough as well as clear to slightly diminished
breath sounds during asthma exacerbations. These atypical patients often have inaccurate peak
flows. In these cases, peak flow measurements may not be helpful and can be discontinued.
Otherwise, the patient may be experiencing a severe asthma episode but may still wait until they
are in the red or yellow peak flow zone to take action. This late phase reduction in peak flows (and
subsequently delayed treatment) might not happen until the exacerbation has progressed to a
potentially life-threatening severity level. Peak flow measurement is not a sole indicator of an
exacerbation’s presence or severity.

Variances Between Peak Flow Meters

Not all peak flow meters are “created equal.” Peak flow meter results can vary up to 20 percent
among meters, depending on the meter that is used. Therefore, it is essential that the asthmatic
patient use the same peak flow meter every time. Each meter can vary, even among the same brand
and model. A 20 percent variance can completely negate the patients’ peak flow zones, causing an
inaccurate zone system and inappropriate asthma action plan. For example, a large difference
among peak flow meters (such as 20 percent) can cause a patient’s peak flow zones to be set
significantly too high or too low. This can cause potentially adverse events such as severe reduction
in pulmonary function before therapeutic action is taken, based on the patient’s written asthma
action plan and calculated peak flow zones. Therefore, the clinician must ensure the patient uses

- 49 -
Copyright © Mometrix Media. You have been licensed one copy of this document for personal use only. Any other reproduction or redistribution is strictly prohibited. All rights reserved.

Licensed to elliot mattison (2146499335, i.astraltears@gmail.com) of 9801 royal ln, dallas, Texas 75231. If this product is distributed to others,
elliot mattison agrees to pay Mometrix the full retail price for each unlicensed recipient. Please play fair and respect the work of our authors.
the same peak flow meter at home as they do in the asthma clinic, bringing the meter along to each
appointment if needed.

Determining Accuracy and Reproducibility of Pulmonary Function Test Results

When conducting pulmonary function testing for the diagnosis or monitoring of asthma, the
clinician must determine the accuracy and reproducibility of the test prior to reporting the results
as accurate. The accuracy of a pulmonary function test is completely dependent on the effort and
performance of the patient performing the maneuvers. Several criteria must be met for pulmonary
function test results to be considered accurate. In simple spirometry with flow volume loops, each
attempt must vary less than 0.2 liters. The loops must not show any unusual patterns such as
coughing or other artifact. Last, the expiration in the volume-time curve must plateau, and the
duration of the expiration portion of the maneuver must reach 6 seconds or more; less than that is
unacceptable. Flattened loops are not expected in asthmatic patients, and these results must be
carefully investigated as they may indicate poor performance in the maneuver or possibly upper
airway obstruction disorders.

Determining Patient’s Knowledge of Asthma Treatment Process

Asthma educators must carefully assess a patient’s knowledge and comfort level with the basics of
asthma management as well as the disease process itself. Simple questions such as the following are
important: “What kind of information did you receive when you were first diagnosed with asthma?”
“Has anyone talked with you about what causes asthma and how it is treated?” “Can you describe to
me the kinds of things that happen inside your body when you have an asthma episode?” Asking
these types of questions can prompt the patient to share any preconceived notions about asthma as
well as any formal education they might have received either at the doctor’s office or during a
hospital admission. The educator might also ask if the patient has ever participated in asthma
education before and, if so, why they stopped visiting that particular educator.

Determining Patient’s Knowledge and Comfort Level with Asthma Management Techniques

One of the first steps in asthma education is to assess the patient’s level of experience in asthma
management tools such as the asthma action plan, spacer use, and peak flow meter use. Then, the
educator knows exactly what material to cover in detail. The educator can ask questions such as
these: “Do you have a written asthma action plan from your doctor?” “Do you use a spacer with
your metered dose inhalers?” “Do you use peak flow meters at home?” If the answer to any of the
aforementioned questions is no, the educator should then ask why the patient does not use these
tools. If the answer is that he or she was never given these tools, then the educator must provide
them to the patient and educate on their use. If, however, the patient says he or she has the tools
but dislike using them, the educator should then work on ways to incorporate these tools into the
patient’s home regimen.

Determining Adherence Issues in Asthma Management

Financial Problems
One of the possible causes of adherence issues in asthmatic patients is financial problems. This
topic is sensitive and must be investigated in a careful, respectful manner. The asthma educator
must ask carefully worded questions to avoid causing feelings of shame or anger for the patient.
The educator must not assume financial problems are an issue as this may not be the case;
however, finances should be respectfully investigated as part of a comprehensive asthma education
plan. The educator might ask questions such as these: “Have you been having any trouble filling

- 50 -
Copyright © Mometrix Media. You have been licensed one copy of this document for personal use only. Any other reproduction or redistribution is strictly prohibited. All rights reserved.

Licensed to elliot mattison (2146499335, i.astraltears@gmail.com) of 9801 royal ln, dallas, Texas 75231. If this product is distributed to others,
elliot mattison agrees to pay Mometrix the full retail price for each unlicensed recipient. Please play fair and respect the work of our authors.
your prescriptions?” “What kind of work do you and your spouse do?” If it is determined that the
patient cannot afford medications, it is imperative for the educator to investigate possible cost-
saving measures and or refer the patient to a social worker who can help further.

Cultural Beliefs
Cultural beliefs directly affect a patient’s adherence to asthma treatment plans as these beliefs are
often ingrained in the patient’s family. Each culture has its own opinions and beliefs for asthma
treatment. For example, in the Latino culture, many people believe that asthma is caused by a
temperature imbalance in the body, which causes wheezing. Other Latino patients believe asthma is
caused by an emotional imbalance, weak lungs, bathing, or being wet and cold. Sometimes these
patients use home remedies or folk remedies from healers in their communities. Cultural beliefs
and home remedies must be respected, especially if they are not inherently harmful. Incorporating
these beliefs and remedies into the treatment plan can help improve adherence. A patient who feels
respected is more likely to actively participate in asthma treatment.

Effects of Different Attitudes Regarding Asthma Treatment on Adherence and Long-Term


Symptom Management

A patient’s or caregiver’s attitude about the asthma treatment regimen directly affects the
successfulness of the asthma treatment plan. Many patients have fears associated with certain
medication such as corticosteroids. They hear the word “steroid” and think that the educator is
referring to anabolic “body-building” steroids that can cause serious side effects. It is the educator’s
job to allay these fears, explaining all potential side effects of medications as well as explaining the
likelihood of these side effects occurring. Some side effects are rare but serious. This can frighten
patients and caregivers and cause them to stop their asthma medications and/or take less
medication than directed. Other patients might be concerned that they may become addicted or
dependent on medications. It is essential for the asthma educator to explain that although
maintenance drugs are taken daily, they are not habit forming. In fact, they heal and condition the
airways to not respond as severely to asthma triggers.

Determining Patient’s Knowledge of Triggers

Home Environment
An asthma educator must carefully assess an asthmatic patient’s knowledge of triggers in the home
environment and especially in the bedroom. Bedrooms should be as trigger free as possible as
many people sleep 6 to 8 hours per night or more. This constant exposure to triggers, in addition to
the fact that asthma is often worse at night, is a potentially harmful situation. The asthma educator
can ask questions such as these: “Have you noticed anything in your home that might be making
your asthma worse?” “What kinds of things seem to worsen your breathing?” If the patient is not
able to identify any triggers, the educator should make suggestions based on the home
environment. For example, if the patient’s bedroom is in the basement, the educator should ask
about mold, mildew, and dust. These triggers are common in basement environments. The educator
can also educate the patient on other common indoor asthma triggers such as pets.

School Environment
Asthma educators must interview asthmatic patients and determine which school-based triggers
might be affecting a patient’s asthma. They can ask questions such as these: “Does your classroom
have a pet or a chalkboard?” “Have you noticed anything in particular that bothers your asthma at
school?” They can also ask questions about exercise-induced asthma, which is a common form of
asthma that is often noticed for the first time in physical education classes. Asking questions such as

- 51 -
Copyright © Mometrix Media. You have been licensed one copy of this document for personal use only. Any other reproduction or redistribution is strictly prohibited. All rights reserved.

Licensed to elliot mattison (2146499335, i.astraltears@gmail.com) of 9801 royal ln, dallas, Texas 75231. If this product is distributed to others,
elliot mattison agrees to pay Mometrix the full retail price for each unlicensed recipient. Please play fair and respect the work of our authors.
“Are you able to participate in gym class without getting winded?” or “Do you ever have trouble
when you play at recess?” can help ascertain possible triggers in a school-age patient. If the patient
can’t identify any triggers, the educator can suggest some possibilities. In addition to pets and
exercise-induced asthma, indoor triggers such as varnishes and cleansers, dust, mold, and strong
scents are all possible school-based triggers.

Work Environment
Asthma educators must interview the patient about possible triggers in the work environment. The
asthma educator must first find out what kind of work the patient does as different work
environments contain different potential asthma triggers. The educator should ask questions such
as these: “Have you noticed anything in your work environment that affects your asthma?” “Do you
notice more asthma symptoms at work than at home?” These questions can help the educator
determine whether the patient has identified any asthma triggers in the work environment or if he
or she needs further education and help identifying potential triggers. Occupational asthma triggers
can include animals, dust, mold, and latex. In the office setting, ozone from printers, paper dust, and
dust from carpets can all affect a patient’s asthma. In an outdoor setting, outdoor allergens, poor air
quality days (ozone action days), and physical exertion can all trigger asthma episodes.

Determining Patient’s Ability to Participate in Asthma Education

The asthma educator is required to educate a patient at a level to which he or she is comfortable
and able to learn. This level varies from patient to patient. Some patients are highly educated,
whereas others might not read at grade level. Others may have a primary language that is not
English, causing a language barrier. All of these issues must be taken into consideration when the
educator formulates an education plan for the patient. Written education booklets, pamphlets, and
action plans should be written at a sixth-grade reading level or lower. Clear, simple language should
be used while limiting medical jargon and unnecessarily “scary” language. For example, an asthma
exacerbation should never be described as an “asthma attack.” Written action plans should use the
“traffic light” format, in which clear zones are set for peak flow meter results and asthma
symptoms. These steps can help the educator communicate with patients at all reading levels.

Determining the Best Way to Educate Patients

An asthma educator must determine the optimal way to educate each asthmatic patient. Every
patient is different, with varying educational needs. Some patients prefer visual aids to help them
grasp concepts, whereas others prefer hearing the information presented. Some patients prefer
discussing the concepts at length in a group setting, whereas others are successful in a more self-
study style of learning. Educators can ask questions such as “When you were in school, which
classes or teachers were your favorite?” and then follow up with “What made them so special?”
These open-ended questions can prompt the patient to evaluate previous educational experiences
and determine which ones were most beneficial. A more direct question such as “Which learning
style is most helpful for you: written information, talking about the issue, or talking in a group
setting?” can also be helpful.

Assessing Patient’s Coping Mechanisms

Patients cope with an asthma diagnosis in different ways. Some worry unnecessarily about the
possibility of severe and life-threatening exacerbations, restricting their physical activity and
limiting social interactions in an effort to minimize their asthma exacerbations. Others ignore their
asthma completely, denying their asthma even exists. All of these coping methods are unhealthy
and dangerous. The asthma educator must evaluate each patient for any unhealthy coping

- 52 -
Copyright © Mometrix Media. You have been licensed one copy of this document for personal use only. Any other reproduction or redistribution is strictly prohibited. All rights reserved.

Licensed to elliot mattison (2146499335, i.astraltears@gmail.com) of 9801 royal ln, dallas, Texas 75231. If this product is distributed to others,
elliot mattison agrees to pay Mometrix the full retail price for each unlicensed recipient. Please play fair and respect the work of our authors.
mechanisms and replace those actions with appropriate, healthy interventions that can improve a
patient’s quality of life. Asking questions such as “How do you feel about your asthma diagnosis?”
and “What kinds of things do you do to control your asthma at home?” can help the educator
determine if any unhealthy coping actions exist. If poor coping mechanisms are found, the educator
can suggest healthier interventions such as pre-medicating before physical activity.

Determining Patient’s Source of Health Care

Comprehensive health care is essential for patients with asthma. The asthma educator must
determine whether or not the patient has continuing health care via either a primary care physician
or pulmonologist. The asthma educator can ask questions such as these: “Do you have a family or
primary doctor?” “Who do you see for your asthma?” “Where do you go when you have an asthma
episode?” If a patient does not have a primary care physician or pulmonologist, he or she needs to
be referred to one that is in his or her health insurance network. (If he or she does not have
insurance, social workers can be of help.) Continuity of care is important for asthma patients.
Primary care physicians and/or pulmonologists assess patients over long periods of time. They
keep careful records of the patient’s visits as well as any diagnostic testing ordered by the office or
other physicians and hospitals that the patient might have visited for treatment.

Challenges Faced by Using Clinics and Urgent Care Facilities for Asthma Management

Some patients do not have primary care physicians, family physicians, or pulmonologists. These
patients are at an increased risk for poor asthma control as they have no continuity of care. These
patients often do not have baseline spirometry or examination results, which makes a
comprehensive assessment of the patient’s asthma difficult. Many patients utilize urgent care
facilities, walk-in clinics, and emergency rooms for asthma management as well as treatment of
acute symptoms. This is not an ideal plan as there is no continuity of care and little to no follow-up
assessment. The asthma educator must determine if the patient utilizes these facilities. The
educator can ask questions such as these: “How often do you see your doctor for asthma checkups?”
“How many times have you seen your doctor for asthma in the past year?” Determining the
frequency of doctor visits can help the educator determine the patient’s asthma severity and level
of control.

Determining Patient’s Perception of Asthma Symptom Occurrence

A patient’s recognition of symptoms is one of the most important components of asthma


management. Poor perceivers have an increased risk of severe and potentially life-threatening
asthma exacerbations. A poor perceiver is a patient who does not have full awareness of his or her
asthma exacerbation symptoms, frequency, and/or severity. The first step in assessing a patient’s
perception is to determine if the patient is able to identify the symptoms. The asthma educator can
ask questions such as this: “What are your usual asthma symptoms?” If the patient is unable to
identify symptoms, the educator can give examples of some common asthma symptoms. Then, the
educator must assess the patient’s perception of the frequency of symptoms. The educator can ask
questions like these: “How often do you experience asthma symptoms?” “How many days per week
do you notice shortness of breath?”

Determining Patient’s Perception of Changes in Asthma Severity

An asthma educator must assess for poor perception of asthma symptom severity in addition to
frequency of symptoms and symptom identification. To assess for symptom severity perception, the
educator must ask questions regarding changes in the patient’s asthma symptoms. The asthma
educator can ask questions such as these: “When do you notice that your asthma is getting worse?”

- 53 -
Copyright © Mometrix Media. You have been licensed one copy of this document for personal use only. Any other reproduction or redistribution is strictly prohibited. All rights reserved.

Licensed to elliot mattison (2146499335, i.astraltears@gmail.com) of 9801 royal ln, dallas, Texas 75231. If this product is distributed to others,
elliot mattison agrees to pay Mometrix the full retail price for each unlicensed recipient. Please play fair and respect the work of our authors.
“Have you noticed anything that makes your asthma symptoms better?” If the patient uses a peak
flow meter, the asthma educator could ask action plan-based questions such as these: “How do you
know you’re in the yellow peak flow zone?” “What does it feel like to be in the red peak flow zone?”
Simple modifications to these questions can help tailor the interview to the patient rather than
asking form questions that would be asked of all asthma patients. A personalized approach to
asthma education is always more successful.

Determining if Patient is Acting on and Appropriately Treating Symptoms

The asthma educator must carefully evaluate a patient’s actions in treating asthma. Some patients
have a written action plan but do not follow it. Others do not have an action plan, have never
completed an asthma education session, and therefore do not have any tools by which to measure
their asthma and effectively treat it. The asthma educator can ask open-ended questions about the
patient’s self-treatment such as these: “How you do treat your asthma at home?” “What do you do
to help yourself when your asthma is acting up?” If the patient has an action plan, the asthma
educator can ask questions like these: “How is your action plan working out for you?” “Do you
normally follow an action plan, or do you treat your asthma as you see fit?” These are important
questions because they give the educator information on adherence as well as the patient’s
potential need for a modified asthma action plan.

Making Goals of Asthma Management for Patients

Although asthma educators are highly educated in guidelines for asthma management, the goals of
therapy always lie with the patient. The patient is the person who makes goals, adjusts action plans,
and modifies trigger avoidance methods with the help of the asthma educator and health-care team.
The patient is the best assessor of quality of life issues and must determine what goals are best for
his or her own personal hobbies, lifestyle, career, and family life. Asthma educators can guide
patients in the goal-making process by giving suggestions based on an interview of the patient’s
interests, activity level, and other factors. For example, if the patient enjoys sports, the asthma
educator can suggest a goal such as being well enough to join a soccer team or take a dance class.
Goals can also be less specific and more general. One such goal might be reducing asthma
symptoms and severity.

Asthma Education Sessions

Successful Interview Style


Asthma education sessions have two components: communication and sharing of knowledge. With
the proper interview technique, the educator and patient learn from each other rather than the
patient learning only from the educator. Interviewing skills are important for successful asthma
education sessions. Asthma educators should talk less and listen more. Many have found that the
most important information about a patient is offered during conversation rather than lecture.
Without a conversational approach to asthma education, the educator relies solely on lecturing
skills to educate a patient. Instead, the educator should take an active listening role during
education and carefully record the information offered by the patient. Asking yes-or-no questions is
not helpful as it does not open up discussion. Open-ended questions and a nonjudgmental attitude
are essential to successful asthma education sessions. A free-flowing conversation, rather than a
lecture-based education session, is the most successful type of asthma education session.

Questions to Ask
One-on-one asthma education is not a lecture-based exchange. In fact, asthma education is a
partnership between the patient and educator with the sharing of ideas between the two

- 54 -
Copyright © Mometrix Media. You have been licensed one copy of this document for personal use only. Any other reproduction or redistribution is strictly prohibited. All rights reserved.

Licensed to elliot mattison (2146499335, i.astraltears@gmail.com) of 9801 royal ln, dallas, Texas 75231. If this product is distributed to others,
elliot mattison agrees to pay Mometrix the full retail price for each unlicensed recipient. Please play fair and respect the work of our authors.
individuals. The best way to conduct asthma education is to utilize open-ended questions that
require the patient to reflect on the topic and share opinions rather than just stating facts. For
example, asking a patient “Do you use your rescue inhaler?” is not as helpful as asking “Tell me
about how your rescue inhaler is working out for you.” The first question asks if the patient is using
the inhaler but does not allow for any discussion as to how the inhaler is actually being used. The
educator asking the second question may discover that the patient uses a rescue inhaler but only
during severe episodes because he or she doesn’t like the side effects. This is a serious problem and
needs to be addressed, but it never would have been discovered if the “right” question wasn’t asked.

Appropriate Body Language for Asthma Educator


During an asthma education session, the patient must be comfortable and relaxed. An anxious or
worried patient can quickly become defensive and closed to new information. To keep a patient
engaged in the lesson, the educator must utilize techniques to make the patient comfortable. This
can include taking a seated position at the same level as the patient. If a health-care practitioner is
standing while giving instructions and information, the seated patient can perceive that action as
aggressive or domineering. It is also perceived as looking down upon the patient from a person in a
position of authority. The asthma educator must maintain eye contact at all times, keeping a neutral
body position with arms uncrossed. An asthma educator who does not maintain eye contact can be
perceived as distracted and uninterested. If the educator crosses his or her arms, the body language
appears closed off and unapproachable. Maintaining neutral body language is important for
education sessions.

Importance of Nonjudgmental Approach


In addition to appropriate body language, the asthma educator must also take care in how he or she
speaks to a patient. The educator may use wording that is seemingly innocent but can cause a
patient to feel shamed and/or degraded, which in turn causes the patient to “check out” of an
education session. An example of inappropriate language might include saying something like this:
“I see you live in the inner city. Does your apartment have cockroaches?” This question, although
relevant and important, can be worded more delicately so as to not be accusatory; for example,
“Pests control measures are helpful for some patients who have asthma. If you have any concerns
about that, let me know, and I’d be happy to help.” The question worded this way does not imply
that the patient has any pest problems in his or her home and, instead, keeps the lines of
communication open by using a more generalized statement.

Signs of Disinterest, Shame, and Frustration


Asthma educators must pay careful attention to their body language as well as the verbal language
they choose to use during education sessions. Being sure to avoid causing a patient to come
disinterested, shamed, or frustrated is one way to ensure a successful asthma education session.
Asthma educators find that a conversational style of education is most successful; however, the
patient has to be a willing participant in the conversation. If a patient is feeling shamed, frustrated,
or disinterested, he or she is no longer an active participant in education. Identifying these signs is
critical so the educator can take steps to correct the issue. Some of the common signs of an upset or
disinterested patient include not maintaining eye contact, bowing his or her head, shifting in the
seat, and appearing otherwise distracted.

Combating Issues of Fear, Disinterest, or Frustration


When confronting the reality of a chronic illness diagnosis, many patients experience a grieving
process. They may become disinterested, afraid, or frustrated with the asthma education session.
Some want to ignore their diagnosis completely, whereas others want to know everything they can
possibly learn about it. For those who are disinterested in the educations session, the asthma

- 55 -
Copyright © Mometrix Media. You have been licensed one copy of this document for personal use only. Any other reproduction or redistribution is strictly prohibited. All rights reserved.

Licensed to elliot mattison (2146499335, i.astraltears@gmail.com) of 9801 royal ln, dallas, Texas 75231. If this product is distributed to others,
elliot mattison agrees to pay Mometrix the full retail price for each unlicensed recipient. Please play fair and respect the work of our authors.
educator needs to carefully address the problem and open up communication. The educator might
ask the patient: “Tell me some of your concerns about asthma.” “Is there anything specific you want
to change about your asthma?” This makes the patient the center of the education session and gives
him or her control over the educational topics discussed. If the patient seems to be upset or
frustrated, the educator might need to address the issue directly and say something like this: “You
seem frustrated with this. What can I do to help?”

Asthma Control Among Patients with High Incomes and Medium Incomes

A patient’s socioeconomic status is directly related to the quality of health care and level of asthma
control. Patients in the highest income group often have the highest quality of health care, with
primary care physicians or even pulmonary specialists monitoring their asthma. These patients can
afford their doctor visits and medications. Many of these patients get annual flu shots, which helps
protect these individuals from life-threatening asthma exacerbations due to influenza infection.
Similar to the high-income patients, middle-income patients often have the financial means to
successfully treat their asthma. They may have some trouble affording some of the more expensive
asthma medications and therapies but often have adequate preventative care. They often have a
higher incidence of chronic diseases when compared to the high-income patients but have a lower
incidence of chronic diseases when compared to the low-income patients.

Challenges Facing Low-Income Patients

Asthmatic patients with a low income have the most challenges in managing their asthma. They
have a general increase in the presence of chronic diseases as compared to patients with medium or
high incomes, and they have an increased risk of severe asthma. Their financial challenges can
cause them to experience the more severe asthma exacerbations as they often cannot afford to fill
their preventative and rescue asthma medications. These patients may not have any health
insurance, which leads to avoidance of preventative care doctor visits. They may not have a family
physician and therefore use clinics or emergency departments for asthma treatment. This is not
optimal as these locations often do not offer comprehensive preventative asthma care, and there is
no continuity of care over time. Low-income patients often have suboptimal living conditions in the
home, which can exacerbate their asthma. They also suffer from more daily stress than some of the
other socioeconomic groups, which can also trigger asthma episodes.

Racial Disparities Among Patients

The incidence of asthma varies widely among ethnic groups. Caucasians have the lowest incidence
of asthma as compared to Latino and Hispanic groups, Native Americans, and African Americans. In
the pediatric population, African American patients have a much higher hospitalization rate, rate of
emergency room visits, and mortality rate than Caucasian patients, and Native American children
are six times more likely to be diagnosed with asthma as compared to Caucasian children. These
racial groups often have a higher incidence of asthma as well as a higher risk of severe or
uncontrolled asthma resulting in emergency room visits and or hospitalization. These inequities
between racial and ethnic groups are based on several factors. Less access to affordable and
comprehensive health care, inability to afford medications, poor living conditions, and genetics are
all factors in the higher incidence of asthma among ethnic groups other than Caucasians.

Disparity of Incidence Between Male and Female Patients

The incidence of asthma in the United States varies between males and females. Females have a
higher incidence of asthma, with more severe exacerbations. Conversely, males have a lower
incidence of asthma with less severe exacerbations. Some of these differences can be attributed to

- 56 -
Copyright © Mometrix Media. You have been licensed one copy of this document for personal use only. Any other reproduction or redistribution is strictly prohibited. All rights reserved.

Licensed to elliot mattison (2146499335, i.astraltears@gmail.com) of 9801 royal ln, dallas, Texas 75231. If this product is distributed to others,
elliot mattison agrees to pay Mometrix the full retail price for each unlicensed recipient. Please play fair and respect the work of our authors.
female hormones, which can trigger asthma episodes at different points in the menstrual cycle.
Fluctuation in estrogen levels can cause inflammation in the body, including the lungs. Female
patients are more likely to seek medical assistance when experiencing asthma symptoms and often
are better perceivers of asthma symptoms. These differences contribute to the higher incidence of
female asthma diagnosis as compared to males. Females are also more likely to have severe asthma
exacerbations resulting in hospitalization. Males are less likely to visit a health-care practitioner
and often do not perceive asthma symptoms as effectively as females.

Effects of Socioeconomic Status on Living Conditions and Subsequent Asthma Severity

A patient’s socioeconomic status encompasses more than just his or her financial state.
Socioeconomic status affects a patient’s living conditions, which in turn affect asthma exacerbation
severity. Patients who live in the inner city are more likely to have severe asthma as compared to
patients who live in the suburbs or rural areas. The air quality of the city, including fumes, car and
bus exhaust, increased ozone levels, smog, and other irritants, can provoke a patient’s asthma,
resulting in more severe exacerbations. Some patients of lower socioeconomic status may live in
substandard housing with water leaks and subsequent mold growth, pests such as cockroaches or
rodents, and lead paint. The patient can take steps to correct the indoor asthma triggers; however,
the outdoor triggers cannot be avoided. Therefore, the location in which the patient lives has a
direct correlation with the successfulness of asthma management.

Effects of Workplaces of Low-Income Patients on Level of Asthma Control

A patient’s work environment can adversely affect his or her asthma, causing more frequent and
severe asthma exacerbations. In the low-income population, this problem is more apparent, with
many of these patients experiencing difficult working conditions. Factory workers, such as those in
textile factories, can inhale both fumes from the machines as well as dust and fibers from the cloth
and products they are manufacturing. These factors become inhaled irritants, causing potentially
severe asthma symptoms. Factory workers may also experience extremes in temperature or
changes in air quality that can also adversely affect their asthma. Other low-income patients might
work in housekeeping positions or outdoor environments where exposure to dust, mold, cleansers,
bleach, and other irritants can trigger severe and potentially life-threatening asthma episodes. Any
patient may have a work environment that contains asthma triggers; however, lower-income
patients have a higher incidence of workplace-related asthma symptoms.

Effects of Low-Income Patient’s School Environment on Asthma

A low-income patient’s school environment can directly affect the level of asthma control. Often
patients in lower-income areas go to school in facilities that are subpar or in need of repair. These
schools might lack adequate ventilation and air-conditioning, which in turn causes temperature
changes that trigger asthma episodes. These schools might have pest control problems or peeling
lead paint, which are irritants that can cause more severe asthma exacerbations. These facilities
might also lack the funding for full-time school nurses, who would help monitor and treat the
students with asthma. A poorly funded school might also have mold, dust, water leaks, and other
poor conditions that can affect a patient’s asthma. These schools might be located in the inner city,
with ozone, fumes, and smog affecting a patient’s level of asthma control. All of these factors can
cause more severe and frequent asthma exacerbations in school-age, low-income patients.

Emotional Factors That Can Adversely Affect Patient’s Asthma Control

Patients with chronic diseases such as asthma often find themselves in a state of mourning over the
life they expected to have. In other words, the chronic disease diagnosis can cause a patient to

- 57 -
Copyright © Mometrix Media. You have been licensed one copy of this document for personal use only. Any other reproduction or redistribution is strictly prohibited. All rights reserved.

Licensed to elliot mattison (2146499335, i.astraltears@gmail.com) of 9801 royal ln, dallas, Texas 75231. If this product is distributed to others,
elliot mattison agrees to pay Mometrix the full retail price for each unlicensed recipient. Please play fair and respect the work of our authors.
experience a state of anxiety and or depression, which in turn can exacerbate a patient’s asthma.
Strong emotions, including anxiety, change the patient’s breathing patterns and can cause asthma
exacerbations. High stress levels can cause a fight-or-flight response, increasing the respiratory rate
and triggering an inflammatory response in the body. These feelings can coincide with feelings of
hostility and denial, with some patients wanting to “forget” they have asthma and therefore
ignoring the disease entirely. This is an extremely dangerous practice as the patient is then at
increased risk for severe and potentially life-threatening exacerbations, especially if the patient
refuses to take prescribed medications and utilize suggested monitoring tools. If an asthma
educator suspects that a patient has anxiety and depression that adversely affects his or her asthma
and general well-being, the educator may refer the patient to cognitive therapists.

Health Literacy

The term health literacy refers to a patient’s ability to comprehend health care-related information.
Patients with good health literacy are able to communicate effectively with the medical team, share
ideas and goals of therapy, design an asthma treatment plan that meets their needs and personal
goals of therapy, and understand and participate in an asthma education session. Patients with
adequate or high-level health literacy often have more successful treatment outcomes than patients
with poor health literacy because the patients can act as their own health advocates, participating
in discussions with the health-care team and setting treatment goals. These patients are able to
read and understand medical paperwork, educational materials, and their asthma action plans.
These patients also have at least a basic understanding of the body and its systems and can absorb
and comprehend information about how asthma is treated.

Contributing Factors to Poor Health Literacy


Poor health literacy is comprised of several factors, including language, age, cultural beliefs, and
socioeconomic status. A patient’s native language is an important factor in determining the
patient’s health literacy level. Language barriers have a direct correlation with poor health literacy
as patients who do not speak English may not be able to effectively communicate with the asthma
educator. Age is another factor in poor health literacy. Elderly patients may have reduced or slowed
cognition and failing eyesight. These factors are common, age-related complications that can result
in poor health literacy. Low socioeconomic status can also cause poor health literacy due to the
patient having poor access to education, a low reading level, and poor access to health care in
general. Cultural issues and beliefs can also contribute to poor health literacy as misinformation
regarding the asthmatic disease process can inhibit the patient from understanding the diagnosis
and treating the condition effectively.

Signs of Illiteracy or Limited Literacy Levels in Patients

Assessing literacy is an important part of the asthma education session as the patient will be given a
written asthma action plan as well as written reference materials during the education session.
Assessing health literacy is not a simple task for an asthma educator. The educator cannot simply
rely on asking the patient about his or her educational background. Many people have poor literacy
skills even after finishing high school. Therefore, the asthma educator needs to carefully assess the
patient’s literacy level by observing clues in his or her behavior. For example, patients with poor
literacy may make excuses about why they can’t participate in education sessions. They might say
they’ve forgotten their reading glasses or ask if they can fill out forms later and bring them back
another time. Patients with low literacy skills might avoid participating in asthma education or ask
family members to be present during education to “take notes so I can pay attention.”

- 58 -
Copyright © Mometrix Media. You have been licensed one copy of this document for personal use only. Any other reproduction or redistribution is strictly prohibited. All rights reserved.

Licensed to elliot mattison (2146499335, i.astraltears@gmail.com) of 9801 royal ln, dallas, Texas 75231. If this product is distributed to others,
elliot mattison agrees to pay Mometrix the full retail price for each unlicensed recipient. Please play fair and respect the work of our authors.
Low Health Literacy in Patients

Signs
Similar to assessing a patient for literacy and reading ability, assessing for health literacy is a
challenging task for an asthma educator. Determining a patient’s educational background is not
helpful in assessing health literacy as a patient’s reading level is not the only factor that contributes
to poor health literacy. Health literacy depends on a patient’s knowledge and experience with
health care. Some signs of poor health literacy might be a lack of refilling medications due the
patient not understanding what the medication is for. Or perhaps the patient takes too much or too
little medication because he or she can’t read the bottle or doesn’t understand the clinician’s
instructions. He or she might not avoid asthma triggers or refuse to incorporate environment
controls in the home because of misinformation regarding what triggers asthma exacerbations. All
of these factors directly contribute to a patient’s health literacy level.

Challenges
Patients with low health literacy have unique challenges to overcome to actively participate in
asthma education sessions and successfully treat their asthma. Patients with poor health literacy
often experience more frequent and severe symptoms, poor quality of life, and poor adherence to
their medication regimen. These patients might be unable to make or attend medical appointments,
fill prescriptions, and take medications on an appropriate schedule. They might not be able to
effectively communicate with medical staff and become their own health advocate. These patients
often don’t understand or aren’t aware of the choices in asthma management and aren’t able to help
the asthma educator tailor an asthma treatment plan to their specific goals. All of these factors
cause a breakdown of communication between the patient and the health-care team and can
adversely affect treatment outcomes for the patient with asthma.

Addressing Low Health Literacy


Addressing a patient’s low health literacy is a difficult task for an asthma educator; however, it is
the educator’s responsibility to effectively communicate with the patient and help him or her
understand the educational topics being discussed. This can include using simple, clear language
that is free from medical jargon and other advanced terminology. During the education session, the
asthma educator should speak slowly and clearly and pause to allow the patient to ask questions.
The education session should be a conversation, not a lecture. The asthma educator must provide
written reference materials at a sixth-grade reading level or lower. The materials should contain
pictures and graphics that can help the patient understand the concepts presented. The educator
should provide the patient with a written asthma action plan that contains simple language with
clear treatment instructions in each zone. Written action plans often use a traffic light design, which
patients can easily identify and understand based on the stop, go, and slow down instructions
associated with a real-life traffic light.

Common Cultural Beliefs Related to Asthma Diagnosis

Asian Indian
Patients of the Asian Indian culture have specific cultural practices for asthma treatment. Some rely
on dietary restrictions or supplements, including teas and herbal products, for the treatment of
asthma. Others use prayer or faith healers to assess the patient and make recommendations.
Sometimes these healers do not physically examine the patient, but use information relayed to
them by the patient’s family members, and then make their determinations. Often these treatment
plans include dietary additions or restrictions as well as prayer and meditation. Some Asian Indian
patients have a unique environmental trigger that is common in their culture. This trigger is

- 59 -
Copyright © Mometrix Media. You have been licensed one copy of this document for personal use only. Any other reproduction or redistribution is strictly prohibited. All rights reserved.

Licensed to elliot mattison (2146499335, i.astraltears@gmail.com) of 9801 royal ln, dallas, Texas 75231. If this product is distributed to others,
elliot mattison agrees to pay Mometrix the full retail price for each unlicensed recipient. Please play fair and respect the work of our authors.
.

incense, which is commonly burned in some Indian patients’ homes. The heavily scented smoke is
an asthma trigger due not only to the smoke but to the added fragrance as well. This acts as an
inhaled irritant that can cause asthma exacerbations.

Hispanic and Latino


In many Latino and Hispanic cultures, asthma is considered a disease of imbalance. Some of the
patients of these cultures believe that asthma is caused by an imbalance of hot and cold
temperatures in the body. Some believe that asthma is caused by being cold or wet. Others believe
that strong emotions cause asthma and that it is a “nervous” disorder. Some of these patients use
home remedies or herbal remedies to treat their asthma, such as teas, eucalyptus ointments, and
other dietary supplements. Other patients from the Hispanic and Latino cultures rely on prayer and
massage to treat their asthma. None of these therapies are inherently dangerous, but the asthma
educator must carefully evaluate the herbal remedies as many of these plant-derived supplements
can trigger a patient’s seasonal or environmental allergies, resulting in asthma exacerbations.

Asian
Similar to Hispanic and Latino patients, patients of Asian descent believe that asthma is caused by
an imbalance in hot and cold (yin and yang) temperatures in the body. These patients believe that
once balance is restored, the asthma symptoms will resolve. Many of these patients utilize
traditional Chinese medicine to treat their asthma. These interventions can include teas, herbal
supplements, and other dietary changes. Some of these treatments will not interfere with
conventional medical treatment; however, herbal supplements can be asthma triggers as they are
derived from plants and flowers. Some traditional Chinese treatment, such as energy measurement
and massage, are often harmless and can be used in conjunction with conventional medical
treatment. However, Chinese medical interventions, including cupping, coining, and acupuncture,
can be harmful to the patient. Cupping and coining can leave bruising and cause pain, whereas
acupuncture needles can cause blood-borne infections and tissue damage.

Addressing Language Barriers Between Patient and Asthma Educator

Addressing a language barrier is an important part of successful asthma education. The asthma
educator should provide all printed pamphlets, booklets, and instruction written in the patient’s
native language. In addition, the asthma educator must use a professional translator to translate `
both the asthma educator’s instruction as well as the patient’s responses and any questions the
patient may ask. When educating on activity-related topics, such as peak flow meter use and inhaler
with spacer technique, the asthma educator must ensure that the patient understands the
instructions and can perform the activity correctly. The asthma educator should provide written
instruction as well as demonstrating the proper technique. To assess the patient’s level of
understanding, the educator should watch the patient perform the activity. This ensures that
despite a language barrier, education has been delivered and understood.

Using Professional Translators in Asthma Education Sessions

Challenges
When educating a non-English speaking patient on asthma-related topics, the asthma educator
must utilize a professional, medically trained translator. The utilization of a translator helps when
language barriers exist between the educator and the patient. However, using a translator has its
challenges, too. The process of translation takes a great deal of time to complete. In a translated
conversation, the educator speaks to the translator, the translator translates to the patient, the
patient responds to the translator, and the translator responds to the educator. This exchange takes

- 60 -
Copyright © Mometrix Media. You have been licensed one copy of this document for personal use only. Any other reproduction or redistribution is strictly prohibited. All rights reserved.

Licensed to elliot mattison (2146499335, i.astraltears@gmail.com) of 9801 royal ln, dallas, Texas 75231. If this product is distributed to others,
elliot mattison agrees to pay Mometrix the full retail price for each unlicensed recipient. Please play fair and respect the work of our authors.
more than twice as long as a conventional education session, and the educator must utilize careful
time management skills to ensure a timely education session that stays on track. The educator must
be prepared to take detailed notes while the translator speaking so that no details are missed
during the exchange. It is important to note that a professional translator will translate the
educator’s words exactly, so the educator must carefully plan sentences so that extraneous words
and phrases are not translated.

Importance
Asthma educators must utilize professional, medically trained interpreters during education
sessions rather than using laypeople or family members as translators. Professional translators are
trained in medical terminology and can relay information to patients without changing the
information inadvertently. Patient family members and laypeople will sometimes make small
changes to the educator’s words, causing a different meaning in the final version. Like a childhood
game of telephone, the end result is often different than the original phrase. Also, a professional
translator will not add any interpretation to the conversation and will translate the educator’s
words exactly. Family members may not make exact translations and may include opinion or
misinformation in the translation. Using a patient’s family member or friend can also cause an
inadvertent Heath Insurance Portability and Accountability Act (HIPAA) violation, with the family
member now being aware of the patient’s medical history and treatment plan.

- 61 -
Copyright © Mometrix Media. You have been licensed one copy of this document for personal use only. Any other reproduction or redistribution is strictly prohibited. All rights reserved.

Licensed to elliot mattison (2146499335, i.astraltears@gmail.com) of 9801 royal ln, dallas, Texas 75231. If this product is distributed to others,
elliot mattison agrees to pay Mometrix the full retail price for each unlicensed recipient. Please play fair and respect the work of our authors.
Asthma Management
Reducing Pharmacological Therapy Dosage as Needed

For long-term asthma management, a patient should be on the lowest possible dose of medication
while still maintaining a good quality of life, reducing the frequency and severity of symptoms, and
reducing risk and impairment. If a patient’s pulmonary function spirometry testing is normal or
close to normal, the patient is using a rescue medication less than twice a week, and the patient can
maintain a normal activity schedule, he or she is considered under control. At this point, the
medication regimen can be maintained at the current therapeutic dosage, and the clinician can
consider reevaluating the patient’s level of control after 3 months. At that time, the clinician can
attempt to reduce the patient’s interventions. If the patient’s maintenance medication dosage is
reduced, and the patient is still experiencing infrequent asthma symptoms and is able to participate
in all scheduled activities, the patient can be maintained at that newer, lower therapeutic dose. The
patient can be reevaluated again in 3 months. The patient should take the smallest amount of
medication possible to maintain quality of life. This action helps mitigate the patient’s possible side
effects from medications.

Stepwise Approach to Asthma Management

The term stepwise approach to asthma management refers to the EPR-3 Guidelines for Asthma
Management’s asthma management plan. This plan is the gold standard for asthma treatment and
offers a detailed chart by which clinicians can titrate a patient’s therapeutic interventions to better
control asthma symptoms. The plans are considered an important tool in the management of
asthma, but they do not negate the importance of good clinical judgment and careful patient
assessment. There are several stepwise asthma management plans for various age groups. Each
treatment plan includes medication guidelines for specific age groups. There are separate plans for
patients age 4 years and younger, 5 to 11 years old, and patients 12 years old or older. Each plan
has six therapeutic steps based on asthma symptoms, risk, and impairment. It is expected that a
patient will move up and down the steps as symptoms improve or worsen over his or her lifetime.

Patient Education
Patients often believe that once they are placed on a medication regimen, the treatment plan will
not change over his or her lifetime. Patients expect that their asthma will not change over time, and
no titration of dosage or change in medication is needed. This is a major misconception in asthma
education and must be addressed by the asthma educator in an education session. The educator
might say, “It is common that a person’s asthma will change over his or her lifetime. Different
factors like a person’s lifestyle, activity level, environment, and adding pets to the home can cause
asthma to change. It is expected that over the years, your breathing will change, and your
medications will change too to properly control your symptoms. This is normal and should be
expected. We will increase and decrease your medications as needed to keep you healthy, and we
will talk about those changes as they happen.”

Adults
Step 1: Step 1 of the stepwise approach to asthma management for adults incorporates only rescue
medications into a patient’s asthma action plan. These patients have intermittent asthma symptoms
that do not interfere with daily life. These mild, intermittent symptoms require only a rescue
medication for treatment. This can include short-acting beta agonists (also known as short-acting
bronchodilators), which treat mild to moderate asthma symptoms within minutes. These

- 62 -
Copyright © Mometrix Media. You have been licensed one copy of this document for personal use only. Any other reproduction or redistribution is strictly prohibited. All rights reserved.

Licensed to elliot mattison (2146499335, i.astraltears@gmail.com) of 9801 royal ln, dallas, Texas 75231. If this product is distributed to others,
elliot mattison agrees to pay Mometrix the full retail price for each unlicensed recipient. Please play fair and respect the work of our authors.
medications are used on an as-needed (PRN) basis and can also be used for pre-medicating prior to
exercise, exertion, general activity, or other expected and unavoidable exposure to triggers. These
short-acting bronchodilators can be used up to three times, 20 minutes apart, for acute asthma
exacerbations. If the patient needs more medication than this, he or she needs to be seen by a
clinician. The patient may require a change in medication or a step up in therapeutic interventions.

Step 2: Step 2 of the stepwise approach to asthma management for adults incorporates new
medications as well as medications from Step 1 on the treatment chart. In this category, a patient
requires both a short-acting bronchodilator for as-needed use plus a low dose of inhaled
corticosteroids. The inhaled corticosteroids act as a maintenance medication, healing and
conditioning the lungs to reduce reactivity to triggers, improve airflow, reduce inflammation, and
reduce bronchoconstriction. The patient will still require short-acting bronchodilators for acute
exacerbations. Alternative therapies in this category can include leukotriene receptor antagonists,
Cromolyn sodium, Nedocromil, or Theophylline; however, these are not front-line therapies for
asthma and are not commonly used in the treatment of asthma. These alternative therapies can be
utilized in specific patient populations as deemed appropriate by a physician. Some of these
medications are not commonly stocked in pharmacies and may be difficult to obtain.

Step 3: Step 3 of the stepwise approach to asthma management for adults incorporates new
medications as well as medications from earlier steps on the treatment chart. In this category, a
patient requires both a short-acting bronchodilator for as-needed use plus a low-dose inhaled
corticosteroid and long-acting bronchodilator. Or, the physician can choose to order the short-
acting bronchodilator plus a medium-dose inhaled corticosteroid. The choice depends on the
patient’s preferences, side effects experienced, possible additional side effects from the new
medication, and patient’s current level of adherence. Alternatives to these two common treatment
plans include a low-dose inhaled corticosteroid plus a choice of Theophylline, Zileuton, or
leukotriene receptor antagonist. These are all additional maintenance medications that are taken
on a daily basis to control asthma symptoms. The short-acting bronchodilator would be used for
acute exacerbations. In addition, a consultation with an asthma or pulmonary specialist may be
considered at this point in the patient’s asthma management plan.

Step 4: Step 4 of the stepwise approach to asthma management for adults incorporates new
medications as well as medications from earlier steps on the treatment chart. In this category, a
patient requires both a short-acting bronchodilator for as-needed use plus a medium-dose inhaled
corticosteroid with a long-acting bronchodilator. As an alternative to this therapy, the physician
could order a medium-dose inhaled corticosteroid with Theophylline, Zileuton, or a leukotriene
receptor antagonist. At this point, if allergy testing has not been performed, and the patient either
has allergic asthma or is presumed to have an allergic component to their asthma, allergy testing
should be considered. After testing is complete, immunotherapy including allergy shots or other
treatment methods should be considered. Uncontrolled allergies can contribute to severe asthma
that does not respond to conventional therapy. A consultation with an asthma or pulmonary
specialist should be considered at this point in the patient’s asthma management plan.

Step 5: Step 5 of the stepwise approach to asthma management for adults incorporates new
medications as well as medications from earlier steps on the treatment chart. In this category, a
patient requires both a short-acting bronchodilator for as-needed use plus a high dose of inhaled
corticosteroids with a long-acting beta agonist. Patients in this category who also have a history of
allergies might also benefit from Omalizumab therapy. This intervention is a medication given via
injection that reduces a patient’s sensitivity to allergens and thereby treats allergic asthma.
However, this therapeutic intervention is not without risk. All possible side effects of medications
should be discussed with the patient prior to incorporating new medications and interventions into

- 63 -
Copyright © Mometrix Media. You have been licensed one copy of this document for personal use only. Any other reproduction or redistribution is strictly prohibited. All rights reserved.

Licensed to elliot mattison (2146499335, i.astraltears@gmail.com) of 9801 royal ln, dallas, Texas 75231. If this product is distributed to others,
elliot mattison agrees to pay Mometrix the full retail price for each unlicensed recipient. Please play fair and respect the work of our authors.
the patient’s asthma action plan. These patients also require short-acting bronchodilators for quick
relief of acute asthma exacerbations. A consultation with an asthma or pulmonary specialist should
be considered at this point in the patient’s asthma management plan if the patient hasn’t been
assessed already by this specialty.

Step 6: Step 6 of the stepwise approach to asthma management for adults incorporates new
medications as well as medications from earlier steps on the treatment chart. In this category, a
patient requires both a short-acting bronchodilator for as-needed use plus a high-dose inhaled
corticosteroid, long-acting bronchodilator, and oral corticosteroid. Patients in this category who
have allergies might also benefit from Omalizumab therapy. Omalizumab is a medication given via
injection that reduces a patient’s sensitivity to allergens and thereby treats allergic asthma.
However, this therapeutic intervention carries a risk of anaphylaxis and other serious side effects.
Clinicians should be prepared to educate the patient about any side effects and be prepared to treat
side effects that might occur. These patients also require short-acting bronchodilators for quick
relief of acute asthma exacerbations. A consultation with an asthma or pulmonary specialist should
also be considered at this point in the patient’s asthma management plan if the patient hasn’t been
assessed already by this specialty.

Children 4 Years Old and Younger


Step 1: Step 1 of the stepwise approach to asthma management for children less than 4 years of age
incorporates only rescue medications into a patient’s asthma action plan. These patients have
intermittent asthma symptoms that do not interfere with daily life. These mild intermittent
symptoms require only a rescue medication for treatment. This can include short-acting beta
agonists (also known as short-acting bronchodilators), which treat mild to moderate asthma
symptoms within minutes. These medications are used on an as-needed (PRN) basis and can also
be used for pre-medicating prior to exertion or expected and unavoidable exposure to triggers.
These short-acting bronchodilators can be used every 4 to 6 hours for up to 24 hours. If the patient
needs more medication than this, he or she needs to be seen by a clinician. The patient may require
a change in medication or a step up in therapeutic interventions.

Step 2: Step 2 of the stepwise approach to asthma management for children less than 4 years of age
incorporates new medications as well as medications from earlier steps on the treatment chart. In
this category of asthma management, patients use a rescue short-acting bronchodilator for acute
asthma symptoms and exacerbations in addition to a low dose of inhaled corticosteroids. This step
up in treatment incorporates both rescue and maintenance medications for the successful
management of increased asthma symptoms and severity. And although every medication is
associated with potential side effects, the expected side effects associated with inhaled
corticosteroid use in the pediatric population are minimal. The risk associated with these
medications is small compared to the many benefits of improving a patient’s breathing and quality
of life. However, a physician may choose to incorporate an alternative therapy to the inhaled
corticosteroid. These interventions include Montelukast or Cromolyn sodium.

Step 3: Step 3 of the stepwise approach to asthma management for children less than 4 years of age
incorporates new medications as well as medications from earlier steps on the treatment chart. In
this category of asthma management, medium doses of inhaled corticosteroids are used to help heal
and condition the lining of the bronchioles as part of a maintenance regimen. Patients in this
category will also use a short-acting bronchodilator for quick relief of asthma exacerbations. It is
important to note that although some parents are uncomfortable with an increase in inhaled
corticosteroid medications due to possible side effects, the parents should be educated on the fact
that a step up in therapy may not be long-lasting. Some patients step up and step down therapy

- 64 -
Copyright © Mometrix Media. You have been licensed one copy of this document for personal use only. Any other reproduction or redistribution is strictly prohibited. All rights reserved.

Licensed to elliot mattison (2146499335, i.astraltears@gmail.com) of 9801 royal ln, dallas, Texas 75231. If this product is distributed to others,
elliot mattison agrees to pay Mometrix the full retail price for each unlicensed recipient. Please play fair and respect the work of our authors.
frequently, sometimes on a seasonal basis depending on their asthma triggers. In addition, a
consultation with an asthma or pulmonary specialist might be considered at this point in the
patient’s asthma management plan due to increasing frequency and severity of symptoms.

Step 4: Step 4 of the stepwise approach to asthma management for children less than 4 years of age
incorporates new medications as well as medications from earlier steps on the treatment chart. In
this category of asthma management, patients utilize a short-acting bronchodilator for quick relief
of asthma symptoms and exacerbations as well as a medium dose of inhaled corticosteroids with
either Montelukast or a long-acting bronchodilator. Montelukast is a leukotriene modifier
medication that is taken by mouth in tablet form and that treats allergic asthma as well as general
indoor and outdoor allergies. This medication is used only in patients with allergic asthma and or
allergies; therefore, if the patient has not been tested for allergies, allergy testing may be
appropriate for a patient who falls within this category of asthma management. In addition, a
consultation with an asthma or pulmonary specialist should be considered at this point in the
patient’s asthma management plan if the patient hasn’t been assessed already by this specialty.

Step 5: Step 5 of the stepwise approach to asthma management for children less than 4 years of age
incorporates new medications as well as medications from earlier steps on the treatment chart. In
this category of asthma management, patients utilize a short-acting bronchodilator for quick relief
of asthma symptoms and exacerbations as well as a high dose of inhaled corticosteroids and a long-
acting bronchodilator. If a patient has allergic asthma or a history of indoor or outdoor allergies, the
physician may choose to incorporate Montelukast, a leukotriene modifier, instead of the long-acting
bronchodilator. Either way, at this point in the stepwise approach to asthma management, patients
are experiencing frequent asthma symptoms and is not properly controlled on lower doses of
medications. Therefore, they must increase their dose and/or frequency of medications in their
treatment regimen. A consultation with an asthma or pulmonary specialist should be considered at
this point in the patient’s asthma management plan if the patient hasn’t been assessed already by
this specialty.

Step 6: Step 6 of the stepwise approach to asthma management for children less than 4 years of age
incorporates new medications as well as medications from earlier steps on the treatment chart. In
this category of asthma management, patients utilize a short-acting bronchodilator for quick relief
of asthma symptoms and exacerbations as well as a high dose of inhaled corticosteroids and a long-
acting bronchodilator. As an alternative, if a patient has allergic asthma or a history of indoor or
outdoor allergies, the physician may choose to incorporate Montelukast, a leukotriene modifier,
instead of the long-acting bronchodilator. In addition, due to the severe and persistent nature of
these patients’ asthma exacerbations and daily asthma symptoms, these patients require oral
corticosteroid therapy. In addition, a consultation with an asthma or pulmonary specialist should
be considered at this point in the patient’s asthma management plan if the patient hasn’t been
assessed already by this specialty.

Children Age 5 to 11 Years Old


Step 1: Step 1 of the stepwise approach to asthma management for children 5 to 11 years of age
incorporates only rescue medications into a patient’s asthma action plan. These patients have
intermittent asthma symptoms that do not interfere with daily life. These mild intermittent
symptoms require only a rescue medication for treatment. This can include short-acting beta
agonists (also known as short-acting bronchodilators), which treat mild to moderate asthma
symptoms within minutes. These medications are used on an as-needed (PRN) basis and can also
be used for pre-medicating prior to physical activity, exercise, exertion or expected and
unavoidable exposure to triggers. These short-acting bronchodilators can be used every 20 minutes

- 65 -
Copyright © Mometrix Media. You have been licensed one copy of this document for personal use only. Any other reproduction or redistribution is strictly prohibited. All rights reserved.

Licensed to elliot mattison (2146499335, i.astraltears@gmail.com) of 9801 royal ln, dallas, Texas 75231. If this product is distributed to others,
elliot mattison agrees to pay Mometrix the full retail price for each unlicensed recipient. Please play fair and respect the work of our authors.
for up to three treatments. If the patient needs more medication than this, he or she needs to be
seen by a clinician. The patient may require a change in medication or a step up in therapeutic
interventions. Often, if this schedule of medication isn’t enough, the patient will require a short-
term oral corticosteroid course.

Step 2: Step 2 of the stepwise approach to asthma management for children 5 to 11 years of age
incorporates new medications as well as medications from earlier steps on the treatment chart. In
this category, patients utilize rescue medications on an as needed basis for asthma exacerbations
and can also pre-medicate with these medications prior to physical activity, exercise, exertion, or
expected and unavoidable exposure to triggers. In addition, Step 2 incorporates a low dose of
inhaled corticosteroids for daily maintenance of asthma symptoms. As an alternative, a physician
may choose to order Theophylline, Cromolyn sodium, or leukotriene modifiers. Some risks and
challenges are associated with these medications, however. Cromolyn sodium is well tolerated, but
sometimes hard to find at pharmacies. Theophylline has many troublesome and potentially
dangerous side effects. And leukotriene modifiers require an allergy or allergic asthma diagnosis
for use.

Step 3: Step 3 of the stepwise approach to asthma management for children 5 to 11 years of age
incorporates new medications as well as medications from earlier steps on the treatment chart. In
this category of asthma management, a patient will be prescribed a low-dose inhaled corticosteroid
used in combination with a long-acting bronchodilator or Theophylline. Leukotriene modifiers can
also be used as an alternative to Theophylline and long-acting bronchodilators in patients with
allergies or allergy-triggered asthma. Patients in this category will also use a short-acting
bronchodilator for quick relief of asthma exacerbations. In this step of asthma management, a
consultation with an asthma or pulmonary specialist might be considered due to the increased
severity and frequency of asthma symptoms and subsequent effects on the patient’s quality of life.

Step 4: Step 4 of the stepwise approach to asthma management for children 5 to 11 years of age
incorporates new medications as well as medications from earlier steps on the treatment chart. In
this category of asthma management, patients utilize a short-acting bronchodilator for quick relief
of asthma symptoms and exacerbations as well as a maintenance regimen that includes a medium
dose of inhaled corticosteroid with a long-acting bronchodilator. As an alternative treatment, the
physician may choose to order a medium-dose inhaled corticosteroid with a Theophylline or
leukotriene modifier. The leukotriene modifier would be appropriate for patients with allergic
asthma and/or allergies; therefore, if the patient has not been tested for allergies, allergy testing
may be appropriate for a patient who falls within this category of asthma management. In addition,
a consultation with an asthma or pulmonary specialist should be considered at this point in the
patient’s asthma management plan if the patient hasn’t been assessed already by this specialty.

Step 5: Step 5 of the stepwise approach to asthma management for children 5 to 11 years of age
incorporates new medications as well as medications from earlier steps on the treatment chart. In
this category of asthma management, patients utilize a short-acting bronchodilator for quick relief
of asthma symptoms and exacerbations as well as a high dose of inhaled corticosteroids and a long-
acting bronchodilator. If a patient has allergic asthma or a history of indoor or outdoor allergies, the
physician may choose to incorporate a leukotriene modifier instead of the long-acting
bronchodilator. A second alternative to the long-acting bronchodilator would be the addition of
Theophylline. However, Theophylline is not well tolerated and is rarely incorporated into an
asthma action plan. In addition, a consultation with an asthma or pulmonary specialist should be
considered at this point in the patient’s asthma management plan if the patient hasn’t been
assessed already by this specialty.

- 66 -
Copyright © Mometrix Media. You have been licensed one copy of this document for personal use only. Any other reproduction or redistribution is strictly prohibited. All rights reserved.

Licensed to elliot mattison (2146499335, i.astraltears@gmail.com) of 9801 royal ln, dallas, Texas 75231. If this product is distributed to others,
elliot mattison agrees to pay Mometrix the full retail price for each unlicensed recipient. Please play fair and respect the work of our authors.
Step 6: Step 6 of the stepwise approach to asthma management for children 5 to 11 years of age
incorporates new medications as well as medications from earlier steps on the treatment chart. In
this category of asthma management, patients utilize a short-acting bronchodilator for quick relief
of asthma symptoms and exacerbations as well as a high dose of inhaled corticosteroids, a long-
acting bronchodilator, and a course of oral corticosteroids. As an alternative, if a patient has allergic
asthma or a history of indoor or outdoor allergies, the physician may choose to incorporate a
leukotriene modifier instead of the long-acting bronchodilator. The physician may also consider
Theophylline as an alternative to both the leukotriene modifier and the long-acting bronchodilator.
In addition, a consultation with an asthma or pulmonary specialist should be considered at this
point in the patient’s asthma management plan if the patient hasn’t been assessed already by this
specialty.

Step Up in Treatment

Stepping up treatment is necessary when a patient’s frequency and severity of symptoms do not
improve under the patient’s current treatment plan. Prior to increasing a patient’s dosage of
medication or adding a new medication to the medication regimen, the educator must first ensure
that the patient is taking his or her asthma medications as directed. Watch the patient perform his
or her inhalers and nebulizers, ensuring that the technique is adequate and appropriate. Next,
discuss environmental controls with the patient. Find out if the patient has successfully
incorporated environmental controls into the home. Check if the patient’s comorbidities are being
appropriately and successfully treated. If all of these factors have been addressed, then it is time to
either increase the patient’s inhaled corticosteroid dosage, or add medications to the regimen,
according to the stepwise approach for asthma management guidelines. The clinician should wait at
least 1 month on the current treatment plan before stepping up therapy.

Step Down in Treatment

Stepping down treatment is the goal of every clinician treating patients with asthma. Patients
should be utilizing the lowest possible dosage of medication and least possible number of
medications while still maintaining symptoms and quality of life. If a patient is stable for at least 3
months on the current treatment plan and the severity and frequency of symptoms have improved,
then the clinician can consider going one step down on the stepwise approach to asthma
management chart. If the patient is well controlled on that step for 3 months or longer, the clinician
can consider stepping the treatment down once more. In addition, the educator must reeducate the
patient on the importance of taking his or her asthma medications as directed, performing the
inhalers and nebulizers correctly, and continuing to use environmental controls in the home. Be
sure that the patient’s comorbidities are still being appropriately treated.

Inhaled Corticosteroids

Inhaled corticosteroids are maintenance or “controller” medications used for the treatment of
moderate to severe asthma. These medications work by reducing the inflammation in the patient’s
bronchioles and preventing future asthma exacerbations. Inhaled corticosteroids block the release
of inflammatory proteins that trigger asthma symptoms such as airway inflammation,
bronchoconstriction, and mucus production. Inhaled corticosteroids reduce airway swelling and
prevent late-phase inflammatory responses. The use of these steroids is considered a localized
treatment as the patient inhales the medication directly into the lungs. This action reduces the risk
of bothersome and potentially severe side effects such as those associated with systemic
corticosteroids. The dose of inhaled corticosteroids is also much smaller compared to the dosage of
systemic corticosteroids, which also reduces the risk of serious complications and side effects.

- 67 -
Copyright © Mometrix Media. You have been licensed one copy of this document for personal use only. Any other reproduction or redistribution is strictly prohibited. All rights reserved.

Licensed to elliot mattison (2146499335, i.astraltears@gmail.com) of 9801 royal ln, dallas, Texas 75231. If this product is distributed to others,
elliot mattison agrees to pay Mometrix the full retail price for each unlicensed recipient. Please play fair and respect the work of our authors.
These medications are associated with better asthma control and are considered a first-line
treatment in the prevention of future asthma exacerbations.

Oral Corticosteroids

Oral corticosteroids, also known as systemic steroids, are long-acting corticosteroids used for
reduction of inflammation in the airway. This medication is taken via tablet or intravenous (IV)
form. Oral corticosteroids are considered systemic because, rather than being a localized
medication inhaled directly into the lungs, it is taken by mouth or IV, causing the medication to
course through the body systems. Therefore, more side effects are observed in this therapeutic
intervention. It is often used as a last-resort therapy after all other drug classifications have been
attempted with no adequate symptoms relief. This is a common treatment choice in the last step of
the stepwise approach to asthma management chart, where severe and moderate exacerbations
occur. These medications are used in short bursts, and the patient is weaned off of them toward the
end of the regimen.

Methylxanthines

The methylxanthines classification of drug is a long-acting medication that has bronchodilation


effects on the airways. This type of medication is not a strong bronchodilator and is not used for
quick relief. It is considered a maintenance medication but is not commonly used today. Many
pharmacies do not carry this drug, and patients often don’t tolerate its use due to troublesome and
potentially severe side effects. Short-acting bronchodilators are more commonly used as the
minimal side effects and quick response time are more helpful in the treatment of acute asthma.
Often methylxanthines, also known as Theophylline, are used instead of inhaled corticosteroids as
an alternative therapy; however, this is not a common substitution. Careful and consistent
monitoring of Theophylline levels in the blood is critical as toxicity can occur at high levels.

Mast Cell Stabilizers

Mast cell stabilizers are a category of asthma medication that blocks chloride channels and release
mast cell mediators. This action reduces bronchospasm and relieves asthma symptoms. This is
considered a maintenance medication but can also be used for pre-medication prior to exercise,
activity, or the expected unavoidable exposure to asthma triggers. These medications are not a first-
line therapy for asthma but are instead considered an alternative in patients with more mild
asthma symptoms and less-frequent asthma episodes. Some common names of mast cell stabilizers
are Cromolyn sodium and Nedocromil. These medications can be used four times a day (QID), twice
a day (BID), or even as needed (PRN) for pre-medicating purposes. There are few side effects
associated with these drugs, and they are well tolerated by most patients. However, mast cell
stabilizers are not a commonly used drug and may be difficult to find at local pharmacies.

Long-Acting Bronchodilators

Long-acting bronchodilators work on the smooth muscle of the airways, relaxing the bronchioles
and reducing bronchoconstriction. These medications are not to be used alone. Long-acting
bronchodilators are used in conjunction with inhaled corticosteroids in a maintenance medication
regimen. These medications are effective for up to 12 hours and are often ordered on a twice per
day (BID) schedule. Long-acting bronchodilators are used in patients with moderate to severe
asthma symptoms, often in Step 3 or higher on the stepwise approach to asthma management
chart. The most troublesome side effect for these medications is the risk of severe and life-
threatening rebound bronchospasm; however, this risk is minimal as it is not common, and the
benefit of this medication’s use outweighs the risks. Other common side effects include jittery

- 68 -
Copyright © Mometrix Media. You have been licensed one copy of this document for personal use only. Any other reproduction or redistribution is strictly prohibited. All rights reserved.

Licensed to elliot mattison (2146499335, i.astraltears@gmail.com) of 9801 royal ln, dallas, Texas 75231. If this product is distributed to others,
elliot mattison agrees to pay Mometrix the full retail price for each unlicensed recipient. Please play fair and respect the work of our authors.
feelings, increased heart rate, and tremors. These side effects are often mild, and some patients
have no side effects at all.

Combination Therapy

The term combination therapy refers to the use of several drugs in conjunction to treat moderate to
severe asthma. The typical combination therapy is comprised of an inhaled corticosteroid plus long-
acting bronchodilator. Combination therapy is commonly administered via inhaler and is
considered one of the most effective therapeutic interventions for the treatment and control of
asthma symptoms. The combinations can include fluticasone with salmeterol (known as Advair)
and budesonide with formoterol (known as Symbicort). These inhalers come in multiple doses for
optimal asthma management and dose titration. Advair comes in three strengths, including 100/50,
250/50, and 500/50. The labels for the different doses correspond to the traffic light motif of the
asthma action plan, with red, yellow, and green fonts. Symbicort comes in two doses, including
80/4.5 and 160/4.5. The side effects associated with these medications include thrush of the
mouth, so rinsing and spitting is required after their use.

Leukotriene Modifiers

Leukotriene modifiers interrupt the allergic cascade by blocking CysLT1 receptors and limiting or
blocking the release of leukotrienes. This action prevents the release of inflammatory cells, which
cause mucus and inflammation in the bronchioles and sinuses. This is an allergic response from
exposure to an allergen. Leukotriene modifiers are only effective in patients with allergies or
allergy-triggered asthma. Allergy testing may be helpful for patients with suspected allergies prior
to the administration of this drug classification; however, this action is not required. These
medications are taken by mouth in tablet form and are approved for adults and children. These are
not considered first-line medications for the treatment of asthma because not all patients have an
allergic component to their disease process. Instead, leukotriene modifiers are considered an
alternative therapy beginning in Step 2 of the stepwise approach to asthma management chart.
These are maintenance medications taken daily for the treatment of asthma.

Immunomodulating Biological Medications

Immunomodulating biological medications are used in specific subtypes of the asthma population.
Some common names of immunomodulating biological medications are Omalizumab (Xolair),
Reslizumab (Cinqair), Mepolizumab (Nucala), and Benralizumab (Fasenra). These medications
work by changing the way the patient’s immune system responds to allergic triggers.
Immunomodulating biological medications prevent allergic reactions by reducing the release of
antibodies and inflammatory cells in response to an allergen. These medications are not widely
used as certain criteria must be met before their implementation. Allergy testing must be
performed, including immunoglobulin E (IgE) levels to determine if these medications will be
effective in treating the patient’s allergic asthma symptoms. A major risk factor with these
medications is the possibility of anaphylaxis after the injection of the drug. Immunomodulating
biological are given via injection and are not safe for all asthma patients. Any patients with history
of tumor development should not use these medications.

Short-Acting Bronchodilators

Short-acting bronchodilators, also known as beta agonists, work by reducing bronchoconstriction


and relaxing the smooth muscle of the bronchioles. These medications are known as “rescue” drugs
as they work within minutes to relieve bronchospasm and bronchoconstriction. Most patients feel
an improvement within 10 to 15 minutes of using their inhalers or nebulizers. This classification of

- 69 -
Copyright © Mometrix Media. You have been licensed one copy of this document for personal use only. Any other reproduction or redistribution is strictly prohibited. All rights reserved.

Licensed to elliot mattison (2146499335, i.astraltears@gmail.com) of 9801 royal ln, dallas, Texas 75231. If this product is distributed to others,
elliot mattison agrees to pay Mometrix the full retail price for each unlicensed recipient. Please play fair and respect the work of our authors.
medication comes in both nebulizer and inhaler form. Some common names of short-acting
bronchodilators include albuterol, Ventolin, ProAir, levalbuterol, and Xopenex. Bronchodilators are
helpful for the treatment of acute asthma symptoms but can also be used for pre-medication prior
to exercise, exertion, and other physical activity. They can also be used for pre-medication prior to
expected and unavoidable trigger exposures. Common side effects associated with these
medications include jittery feelings, increased heart rate, and tremors. These side effects are often
mild, and some patients have no side effects at all.

Muscarinic Antagonists

Muscarinic antagonists, also known as “anticholinergics,” are medications that work on the small
muscles of the airway, reducing airway constriction by impeding the binding of acetylcholine to the
receptors within the bronchioles. Subsequently the airway musculature relaxes, reducing
bronchoconstriction and improving a patient’s shortness of breath. These medications are used in
conjunction with short-acting bronchodilators. Anticholinergics can be used alone; however, that
practice is more common in patients with chronic obstructive pulmonary disease (COPD) than in
patients with asthma. These medications are not commonly used in mild intermittent asthma but
are more often incorporated into the asthma action plans of asthma patients with moderate to
severe symptoms. In rare cases, this medication can take the place of a short-acting bronchodilator
in the event that a patient does not tolerate traditional rescue medications. Some common names of
anticholinergic asthma medications include ipratropium bromide and Atrovent.

Controller Medications

When educating an asthma patient on the medication regimen, it is important for the educator to
explain the difference between rescue and controller medications. Some patients expect quick relief
from controller medications and don’t adhere to the prescribed medication regimen due to the
medication “not working.” Therefore, the educator must explain the difference between these
medications using clear, simple language. When educating on the purpose of controller
medications, the educator might say, “Controller medicines take several days or weeks to work.
They do not work fast. You have to take them every day until they build up in your system. Then,
when you have enough in your lungs, they start to heal and protect your airways so that your
asthma is better controlled. It is important to continue taking these medications even if you feel
better because they are designed for long-term use with very few side effects.”

Asthma controller medications include inhaled corticosteroids, leukotriene modifiers, and long-
acting bronchodilators. These medications are designed for long-term use with minimal side effects.
Many drugs, such as inhaled corticosteroids, can take several days to several weeks before the
patient notices any benefit. At that point, the patient must continue to take the drug; otherwise, the
patient’s asthma symptoms will return. Some commonly used controller medications include
budesonide and fluticasone (both are inhaled corticosteroids), Montelukast (a leukotriene
modifier), salmeterol, and formoterol (both are long-acting bronchodilators). Some of these
medications are also found in combination inhalers such as Advair, where both an inhaled
corticosteroid and long-acting bronchodilator are used in tandem for long-term relief of moderate
to severe, persistent asthma symptoms. Combination therapy is a first-line treatment for moderate
to severe asthma, although alternatives are available for certain patient populations.

Rescue Medications

When educating an asthma patient on the medication regimen, it is important for the educator to
explain the difference between rescue and controller medications. Some patients overuse their

- 70 -
Copyright © Mometrix Media. You have been licensed one copy of this document for personal use only. Any other reproduction or redistribution is strictly prohibited. All rights reserved.

Licensed to elliot mattison (2146499335, i.astraltears@gmail.com) of 9801 royal ln, dallas, Texas 75231. If this product is distributed to others,
elliot mattison agrees to pay Mometrix the full retail price for each unlicensed recipient. Please play fair and respect the work of our authors.
rescue medications and don’t understand how they are supposed to be used. Therefore, the
educator must explain the difference between rescue and controller medications using clear, simple
language. When educating on the purpose of rescue medications, the educator might say, “Rescue
medicines work quickly to help you breathe easier. This is the inhaler you should reach for if you’re
having an asthma episode. It takes just a few minutes for it to work. But it is important to take them
exactly as your doctor says. If you need more medication that what’s written in your asthma action
plan, then you need to contact your doctor and make some changes to your plan. It is dangerous to
take too much medication even if it is just an inhaler.”

Rescue medications are the most well-known asthma medications on the market today. All patients
with asthma should have a prescribed rescue inhaler for quick relief of acute asthma symptoms.
These medications are available in both nebulizer and inhaler form. Rescue medications, also
known as short-acting bronchodilators, work within minutes to relax the smooth muscle of the
airway and relieve bronchoconstriction and dyspnea. Some common names of rescue medications
for asthma patients include albuterol (ProAir) and levalbuterol (Xopenex). These medications have
minimal side effects, although some patients find the tremors associated with these medications
bothersome. The benefits outweigh the risks, however, and these medications are the gold standard
in asthma care. Every patient with asthma, regardless of age, needs to have an appropriately dosed
rescue medication. Rescue medications are present in each step of the stepwise approach to asthma
management chart.

Determining Which Medications Patient Uses at Home

Although a patient’s chart will include a list of home medications, it is important for the asthma
educator to ask the patient directly about medication use. Some patients may not be aware of what
their medications are used for, and some may not even know the names of the drugs. Often patients
will refer to drugs as “the purple circle inhaler” (Advair) or “the red inhaler” (ProAir). The asthma
educator should ask what medications the patient uses and what they are for. The asthma educator
can ask questions such as this: “Do you take any rescue medications for your asthma? These are the
ones that work fast.” This helps the patient understand exactly what type of medication to which
the educator is referring. Offering some brand names such as Ventolin, Xopenex, and ProAir is also
helpful. The generic names of these medications are albuterol and levalbuterol.

Dangers Associated with Short-Acting Bronchodilator Overuse

Short-acting bronchodilators, also called beta agonists, are used for quick relief of asthma
symptoms. These medications are safe and effective for the treatment of acute asthma
exacerbations but should not be used excessively. Short-acting bronchodilators are typically
prescribed for use every 4 to 6 hours as needed. Some physicians will allow a patient to use an
inhaler up to every 3 hours as needed for short-term relief of an acute exacerbation or illness. The
stepwise approach for asthma management chart allows for three back-to-back treatments given
20 minutes apart for the treatment of a severe exacerbation in the adult population; however, if the
patient requires more medication that this schedule provides, he or she must be assessed by a
clinician. Overuse of short-acting bronchodilators is associated with asthma-related death due to
poorly controlled symptoms that can result in respiratory failure.

Inhaled and Systemic Corticosteroids

When educating on the differences between inhaled and systemic corticosteroids, the asthma
educator must use simple, clear language that the patient will understand. The asthma educator
might say, “There are two different kinds of steroids that we use for treating asthma: inhaled

- 71 -
Copyright © Mometrix Media. You have been licensed one copy of this document for personal use only. Any other reproduction or redistribution is strictly prohibited. All rights reserved.

Licensed to elliot mattison (2146499335, i.astraltears@gmail.com) of 9801 royal ln, dallas, Texas 75231. If this product is distributed to others,
elliot mattison agrees to pay Mometrix the full retail price for each unlicensed recipient. Please play fair and respect the work of our authors.
steroids and steroids taken by mouth. The inhaled steroids are the ones that you must take every
day without missing a dose. These go straight to the lungs, so there are few side effects. The most
common side effect is thrush, but we can avoid that by rinsing your mouth and gargling after you
use your steroid inhaler. Inhaled steroids keep your lungs healthy and prevent asthma episodes
from starting. The steroids taken by mouth are the ones that have more side effects. This
medication is taken only for a short time to treat an asthma episode. It is important that you take it
exactly as directed and wean off the medication gradually.”

Side Effects of Short-Acting Bronchodilators

Short-acting bronchodilators are safe and effective medications for the treatment of acute asthma
symptoms. They work within minutes to relieve bronchoconstriction and bronchospasm. As with
all medications, the use of short-acting bronchodilators is associated with some side effects.
Common side effects associated with the use of short-acting bronchodilators include tremors,
increased heart rate, and jittery feelings. Most patients find these side effects tolerable, but some
patients find these effects particularly bothersome. For this more sensitive patient population, the
asthma educator can suggest that the patient rinse his or her mouth after the use of any short-
acting bronchodilators. Some studies indicate that removing residue from the mucus membranes of
the mouth can interrupt the continued absorption of the drug into the patient’s bloodstream and
therefore minimize side effects. Other less common side effects include headache, stomach pain,
and sleeplessness.

Side Effects of Long-Acting Bronchodilators

Long-acting bronchodilators are maintenance medications used for the treatment of chronic
asthma symptoms. The use of long-acting bronchodilators is associated with several potential side
effects. Common side effects associated with the use of long-acting bronchodilators are similar to
those associated with short-term bronchodilators. These side effects include tremors, increased
heart rate, and jittery feelings. Patients may also experience sleeplessness, headache, or stomach
pain. Some of these side effects can be minimized by instructing the patient to rinse his or her
mouth after use, which stops the medication from absorbing into the mucus membranes of the
mouth. Long-acting bronchodilators, when used without an inhaled corticosteroid, are under a
black box warning for increased risk of asthma-related death. The asthma educator must be aware
of this risk and possibly suggest a medication change if the patient is utilizing this medication alone,
with no inhaled corticosteroid.

Black Box Warning for Long-Acting Bronchodilators

Long-acting bronchodilators were under a black box warning by the Food and Drug Administration
for potentially severe and life-threatening side effects associated with their use. It was previously
thought that these medications increased the incidence of asthma-related deaths, and as a result,
the medications were under a black box warning, which stated that patients who utilized these
medications were at increased risk of asthma-related death. Clinical trials were conducted for
several years, measuring the morbidity of asthma as well as the incidence of severe and life-
threatening asthma symptoms. After several years of clinical trials were completed, it was
determined that these medications, used in conjunction with inhaled corticosteroids, are not
associated with asthma-related deaths, and the black box warning was removed in 2017. However,
these medications are still under a black box warning when used alone, so these medications
should always be ordered in combination with inhaled corticosteroids to ensure patient safety.

- 72 -
Copyright © Mometrix Media. You have been licensed one copy of this document for personal use only. Any other reproduction or redistribution is strictly prohibited. All rights reserved.

Licensed to elliot mattison (2146499335, i.astraltears@gmail.com) of 9801 royal ln, dallas, Texas 75231. If this product is distributed to others,
elliot mattison agrees to pay Mometrix the full retail price for each unlicensed recipient. Please play fair and respect the work of our authors.
Differences Between Short-Acting and Long-Acting Bronchodilators

When educating a patient on the differences between short- and long-acting bronchodilators, the
asthma educator must use clear, simple terminology that the patient can easily understand. The
asthma educator might say, “There are two kinds of bronchodilators: short acting and long acting.
Bronchodilators open up the air passages in your lungs so that you can breathe easier. Some work
quickly, whereas others take a little longer to work. Short-acting bronchodilators are called ‘rescue’
medications. These medications work quickly to stop an asthma attack and help you breathe easier.
These are the inhalers to reach for in an asthma emergency or when you experience asthma
symptoms. Long-acting bronchodilators are different. These are considered controller medicines.
They do not work quickly. They are used every day to help control your asthma. These will not help
you in an asthma episode but can help keep you healthy if you use them every day as the doctor
recommends.”

Side Effects of Medications

Inhaled Corticosteroids
Inhaled corticosteroids are a safe and effective form of controller medication for the treatment of
moderate to severe, persistent asthma. Some patients are concerned about the use of these
medications because the term steroids has negative connotations. Therefore, the asthma educator
must be well informed on the possible side effects as well as ways to treat and avoid them. The
main side effect associated with the use of inhaled corticosteroids is a fungal infection of the mouth,
commonly known as “thrush.” To mitigate these effects, the patient should be instructed to rinse
and spit after their use. Another common side effect is hoarseness, which is especially common in
dry powder inhalers due to the powder content. Patients should be instructed to rinse and gargle
after their use, which can help with the hoarseness. If the hoarseness is particularly bothersome,
the clinician may choose to change the patient to a metered-dose (spray) inhaler form of the drug.

Systemic Corticosteroids
Systemic corticosteroids are steroids taken by mouth or intravenously. These steroids have a
systemic effect on the body due to the fact that the medication is not given in a localized fashion like
the inhaled corticosteroids. Because the systemic corticosteroids have to be ingested or infused,
they travel throughout the systems of the body and can cause potentially severe and bothersome
side effects. This is why these medications are used mainly for acute exacerbations. Some common
side effects associated with systemic corticosteroids are changes in appetite (increased hunger) and
subsequent weight gain, personality changes such as anger and frustration, headache, stomach
upset, fluid retention, blood sugar changes, and insomnia. Systemic corticosteroids are used safely
for short periods of time but are associated with serious side effects if taken long term. Some
common long-term side effects of systemic corticosteroids include Cushing’s disease, hypothalamic
pituitary adrenal axis suppression, osteoporosis or osteopenia, glaucoma, and cataracts.

Leukotriene Modifiers
Leukotriene modifiers are asthma controller medications taken by mouth for the treatment of
allergic asthma and allergies. These medications are well tolerated by many patients and are
associated with few side effects. Some of the more common side effects associated with the use of
leukotriene modifiers include sleeplessness, headache, stomach upset, and fatigue. Rarely, patients
have reported personality changes and suicidal thoughts. It is important for the clinician to weigh
the risks and benefits of adding any drug to a patient’s regimen. If the side effects are too
bothersome for the patient, the educator or clinician can suggest alternative treatments as outlined
in the stepwise approach to asthma management chart. However, leukotriene modifiers are well

- 73 -
Copyright © Mometrix Media. You have been licensed one copy of this document for personal use only. Any other reproduction or redistribution is strictly prohibited. All rights reserved.

Licensed to elliot mattison (2146499335, i.astraltears@gmail.com) of 9801 royal ln, dallas, Texas 75231. If this product is distributed to others,
elliot mattison agrees to pay Mometrix the full retail price for each unlicensed recipient. Please play fair and respect the work of our authors.
tolerated in most patients and are effective in the treatment of allergic triggered asthma as well as
environmental and seasonal allergies.

Muscarinic Antagonists
Muscarinic antagonists, also known as anticholinergics, are medications that have a
bronchodilation effect on the airways by impeding the binding of acetylcholine to the receptors
within the bronchioles. Common side effects associated with the use of muscarinic antagonists
include cough, wheeze, chest pain, chest tightness or general discomfort, and shortness of breath.
These medications are often given in conjunction with traditional bronchodilators such as albuterol
(Ventolin or ProAir) or levalbuterol (Xopenex); therefore, a patient taking a treatment or inhaler
with both an anticholinergic and bronchodilator will likely experience side effects more from the
bronchodilator portion of the intervention. These very common side effects include tremors, jittery
feelings, headache, and stomachache. Most patients tolerate these medications well, with limited
side effects; however, this category of medication is considered an alternative on the stepwise
approach to asthma management chart and is therefore not commonly used in the treatment of
asthma.

Immunomodulating Biologicals
Immunomodulating biologicals are drugs that change the patient’s immune system by blocking the
allergic response to triggers. These drugs are used mainly for the treatment of severe, refractory,
allergic-triggered asthma that does not respond to traditional treatments. These medications are
administered via injection. Common side effects associated with the use of immunomodulating
biologicals include redness, swelling, and pain at the injection site as well as cold symptoms,
malaise, and fatigue. Patients using immunomodulating biologicals must not have a history of
cancer as tumor development was a significant side effect observed in some patients. Less common
side effects include immediate or late-phase reaction anaphylaxis, fever, inflammation of the blood
vessels, and parasitic infection. Patients who take immunomodulating biologicals must be carefully
assessed prior to the initiation of treatment for any preexisting conditions that may result in these
severe side effects. In addition, patients using these medications must be carefully monitored
throughout the treatment process for any potentially serious side effects.

Possible Drug Interactions Between Beta Blockers and Short-Acting Bronchodilators

Short-acting bronchodilators work by relaxing the smooth muscle of the bronchioles while dilating
the patient’s airways and relieving shortness of breath. These medications, when taken in
conjunction with certain cardiovascular medications and antidepressants, can cause potentially
serious drug interactions. The combination of tricyclic antidepressants, such as monoamine oxidase
inhibitors (MAOIs) and bronchodilators, can cause an increase in heart rate and blood pressure and
can cause palpitations. The use of short-acting bronchodilators in combination with beta blockers
can cause bronchospasm. The beta blockers impede the bronchodilation effect of the short-acting
bronchodilators (also called beta agonists), essentially canceling each other out. In addition,
patients who take diuretics for the treatment of cardiovascular conditions should not use
anticholinergic medications (also called muscarinic antagonists) because they can cause
hypokalemia in some patients, which is exacerbated by the use of diuretics.

Dose, Frequency, and Administration Route for Medications

Inhaled Corticosteroids
Inhaled corticosteroids are controller medications used daily for treatment of moderate to severe
persistent asthma. They come in three different administration routes, including dry powder

- 74 -
Copyright © Mometrix Media. You have been licensed one copy of this document for personal use only. Any other reproduction or redistribution is strictly prohibited. All rights reserved.

Licensed to elliot mattison (2146499335, i.astraltears@gmail.com) of 9801 royal ln, dallas, Texas 75231. If this product is distributed to others,
elliot mattison agrees to pay Mometrix the full retail price for each unlicensed recipient. Please play fair and respect the work of our authors.
inhaler (DPI), metered dose inhaler (MDI), and suspension for nebulizer treatment. Some common
inhaled corticosteroids available in DPI form are budesonide (Pulmicort), mometasone (Asmanex),
and fluticasone (Flovent). Budesonide also comes in suspension for nebulizer form, and fluticasone
also comes in MDI form. Inhaled corticosteroids available in MDI form include beclomethasone
(Qvar) and triamcinolone (Azmacort).

The route of administration, dose, and frequency for these drugs are listed in the table below.

Drug Name Administration Dose Frequency


Route
Budesonide DPI Dry Powder Inhaler 90 mcg, 180 mcg up to two puffs twice per day
Budesonide Suspension for 0.25 mg, 0.5 mg one ampule nebulized once
Suspension Nebulizer or twice per day
Fluticasone DPI Dry Powder Inhaler 50 mcg up to two puffs twice per day
Fluticasone MDI Metered Dose 44mcg, 110mcg, up to two puffs twice per day
Inhaler 220 mcg
Mometasone Dry Powder Inhaler 110 mcg, 220 up to two puffs twice per day
mcg
Beclomethasone Metered Dose 40 mcg, 80 mcg up to two puffs twice per day
Inhaler
Triamcinolone Metered Dose 150 mcg two puffs three to four times
Inhaler per day

Budesonide with Formoterol Combination Inhalers


Combination therapy, inhalers that contain both an inhaled corticosteroid and long-acting
bronchodilator, are used for the treatment and long-term management of moderate to severe,
persistent asthma. These drugs are controller medications that are taken daily for long-term
asthma control. Two common names of combination inhalers include fluticasone with salmeterol
(Advair) and budesonide with formoterol (Symbicort). Budesonide with formoterol is available
only in metered dose inhaler form. This is a 12-hour medication that is taken twice daily. The
patient will take two inhalations twice per day, every day, for the management of asthma
symptoms. Budesonide with formoterol inhalers are available in two doses, including 80/4.5 and
160/4.5 strengths. In these ratios, the top number refers to the dosage of inhaled corticosteroid,
whereas the bottom number refers to the dosage of long-acting bronchodilator.

Fluticasone with Salmeterol Combination Inhalers


Combination therapy inhalers that contain both an inhaled corticosteroid and long-acting
bronchodilator are used for the treatment and long-term management of moderate to severe
persistent asthma. These drugs are controller medications that are taken daily for long-term
asthma control. Two common names of combination inhalers include fluticasone with salmeterol
(Advair) and budesonide with formoterol (Symbicort). Fluticasone with salmeterol is available in
both dry powder inhaler and metered dose inhaler forms. This medication is available in three
strengths for each type of inhaler. The dry powder inhalers come in 100/50, 250/50, and 500/50
strengths, whereas the metered dose inhaler forms come in 45/21, 115/21, and 230/21 strengths.
In these ratios, the top number refers to the dosage of corticosteroid, whereas the bottom number
refers to the dosage of the long-acting bronchodilator. A patient will take one inhalation of the dry
powder inhaler twice per day as this is a medication with 12-hour duration. For the metered dose
inhaler, a patient will take two inhalations twice per day.

- 75 -
Copyright © Mometrix Media. You have been licensed one copy of this document for personal use only. Any other reproduction or redistribution is strictly prohibited. All rights reserved.

Licensed to elliot mattison (2146499335, i.astraltears@gmail.com) of 9801 royal ln, dallas, Texas 75231. If this product is distributed to others,
elliot mattison agrees to pay Mometrix the full retail price for each unlicensed recipient. Please play fair and respect the work of our authors.
Short-Acting Bronchodilators
Short-acting bronchodilators are used for the acute treatment of asthma symptoms. These
medications are available in both suspension for nebulizer and metered dose inhaler form. Some
common names of short-acting bronchodilators include albuterol (ProAir) and levalbuterol
(Xopenex). Albuterol for nebulization comes in two strengths, 2.5 mg and 5 mg; however, some
physicians will order a 1.25 mg dose for small children and infants. These treatments are taken up
to every 3 hours as needed for relief of asthma symptoms. They may also be given as a continuous
nebulizer in the hospital setting, with several doses given over 1 hour’s time. Levalbuterol by
nebulization comes in two doses, including 0.63 mg and 1.25 mg. These treatments are also given
up to every 3 hours as needed and can sometimes be given more frequently under a doctor’s
supervision. A ProAir metered dose inhaler contains 8 g of drug. A patient will commonly take two
puffs up to every 3 hours as needed for asthma symptoms.

Long-Acting Bronchodilators
Long-acting bronchodilators are controller medications used for the long-term treatment of
moderate to severe asthma. Some common names of long-acting bronchodilators include
salmeterol (Serevent) and formoterol (Foradil). Salmeterol and formoterol are both dry powder
inhalers; however, the mechanism of action of these inhalers is different. For example, salmeterol is
a traditional dry powder inhaler in which the inhalations are premeasured and loaded into the
inhaler, ready for inhalation with the click of a button. Formoterol has a different process
associated with its use. The patient must insert a powder-filled capsule into the inhaler, close the
chamber, and puncture the capsule with the press of a button. Then the patient may inhale the drug.
These inhalers each come in one strength. Salmeterol is a 50 mcg dose, and formoterol is a 12 mcg
dose. Patients take one inhalation twice per day, every day.

Leukotriene Modifiers
Leukotriene modifiers are controller medications used to treat allergic asthma and allergies. These
medications are used daily for the long-term treatment of allergic asthma. Some common names of
leukotriene modifiers include zafirlukast (Accolate), Montelukast (Singulair), and zileuton (Zyflo).
These medications are given by mouth in either tablet form for adults or chewable form for
children. Zafirlukast comes in two doses, including 20 mg twice per day for adults and children over
12 years old and 10 mg twice daily for children 5 to 11 years of age. Montelukast comes in two
doses, including 10 mg once daily for adults 15 years old and older, and 5 mg for children ages 6 to
14 years old. Last, Zileuton comes in only one dose. This medication comes in a 600 mg dose given
four times a day for adults and children ages 12 years old and older.

Systemic Corticosteroids
Systemic corticosteroids are used in short-term bursts for the treatment of acute, severe asthma
exacerbations. These medications are commonly taken by mouth in tablet or syrup form. Dosage of
this medication varies widely depending on a patient’s level of impairment, severity and duration of
symptoms, and other contributing factors. Systemic corticosteroids are given in three doses: high,
medium, and low. Generally speaking, a high dose is between 40 and 60 mg, a medium dose is
approximately 20 to 30 mg, and a low dose of systemic corticosteroids is under 20 mg. These doses
can be given all at once each day, or the physician can recommend spreading out the doses. In the
latter case, the patient would take only a small number of tablets multiple times per day rather than
taking a larger number of tablets all at once. In addition, it is common for a physician to order
systemic corticosteroids on a taper, meaning that the amount of drug taken is reduced daily until
the patient is finished with the medication. This is also called “weaning.”

- 76 -
Copyright © Mometrix Media. You have been licensed one copy of this document for personal use only. Any other reproduction or redistribution is strictly prohibited. All rights reserved.

Licensed to elliot mattison (2146499335, i.astraltears@gmail.com) of 9801 royal ln, dallas, Texas 75231. If this product is distributed to others,
elliot mattison agrees to pay Mometrix the full retail price for each unlicensed recipient. Please play fair and respect the work of our authors.
Muscarinic Antagonists
Muscarinic antagonists are short-acting bronchodilators that are used for the treatment of asthma
as an alternative choice in the stepwise approach to asthma management chart. This is not a
commonly used controller medication for asthma but is sometimes used in a hospital setting during
acute exacerbations or for home use when other medications are not relieving a patient’s
symptoms. The most commonly used muscarinic antagonist is ipratropium bromide (Atrovent).
Ipratropium bromide comes in two forms for the treatment of asthma, including metered dose
inhaler form and solution for nebulization. The dosage for the metered dose inhaler form is 17 mcg.
The patient would take two puffs four times per day. This is not a rescue medication. For the
nebulized version of the drug, the dosage is 0.5 mg four times per day. A physician may choose to
alter these schedules depending on the patient’s severity of symptoms during a hospital stay, but
this is the commonly prescribed frequency for home use.

Immunomodulating Biologicals
Immunomodulating biologicals are drugs that change the patient’s immune system by blocking the
allergic response to triggers and subsequently treating severe allergic asthma. These drugs are used
mainly for the treatment of refractory, allergic-triggered asthma that does not respond to
traditional treatments. These medications are administered via injection. Some common names of
immunomodulating biologicals are Omalizumab (Xolair), Reslizumab (Cinqair), Mepolizumab
(Nucala), and Benralizumab (Fasenra). Omalizumab is approved for use in adults and children 6
years of age and older. Dosing varies depending on a patient’s weight and pretreatment
immunoglobulin E (IgE) serum levels. Omalizumab can be administered once or twice per month.
Reslizumab is an intravenous medication approved for use in patients 18 and older and is given
once per month. Dosing is weight dependent. Mepolizumab is approved for the use of patients 12
years old and older, and the 100 mg injections are given once per month. Benralizumab is given to
those 12 years old and older. The 30 mg injections are given once per month for three doses then
once every 2 months.

Inhaled Corticosteroids and Anabolic Steroids

When educating a patient on the use of inhaled corticosteroids, it is important that the asthma
educator address any misconceptions about asthma educations. Inhaled corticosteroids are the
most commonly misunderstood medications for the treatment of asthma. Patients hear the term
steroid and imagine anabolic steroids. When educating on the use of inhaled corticosteroids, the
asthma educator might say, “This medication is called an inhaled steroid. But don’t worry, this isn’t
like the “body-building” steroids you might have heard of. These are tiny doses of a different kind of
steroid that heals and conditions your lungs and prevents asthma episodes. The medication is given
with an inhaler, which means that the medicine goes directly to your lungs where it’s needed. The
good news is that because it goes directly to the lungs, it has very few side effects. The most
common one is something called thrush, which is a fungal infection of the mouth. But that is easily
avoided by rinsing your mouth after using your inhaler.”

Dry Powder Inhalers and Metered Dose Inhalers

When educating on the use of asthma medications, the asthma educator must be able to compare
and contrast medication delivery systems so the patient understands that all inhalers are not the
same. For example, the asthma educator might say, “There are two kinds of inhalers that we use for
treating asthma: dry powder inhalers and metered dose inhalers. The metered dose inhalers are the
traditional inhalers that you’ve probably seen many times. They are small canisters that squirt
medicine out when you press them. But the dry powder inhalers don’t work like that. These are a

- 77 -
Copyright © Mometrix Media. You have been licensed one copy of this document for personal use only. Any other reproduction or redistribution is strictly prohibited. All rights reserved.

Licensed to elliot mattison (2146499335, i.astraltears@gmail.com) of 9801 royal ln, dallas, Texas 75231. If this product is distributed to others,
elliot mattison agrees to pay Mometrix the full retail price for each unlicensed recipient. Please play fair and respect the work of our authors.
newer type of inhaler that doesn’t spray at all! Instead, when you push the button, a small amount
of powder is loaded and ready for you to breathe in. It’s waiting for you until you’re ready to
inhale.”

MDIs

Use of Spacers
When using a metered dose inhaler (MDI), the patient must also use a spacer (valved holding
chamber) with the inhaler. If the patient does not use a spacer, much of the medicine delivered is
lost to the patient’s mouth due to the force of the spray and particle size of the drug. A spacer
captures the puff of medication and holds it, suspending the particles long enough for the patient to
inhale slowly and deeply. The patient then has greater drug deposition in the lungs, and the
technique itself is slower and easier rather than the patient rushing to catch the puff in his or her
mouth. Spacers vary as there are many on the market today. Some offer features such as anti-static
chambers, whistles that warn of inhalations that are too fast or too hard, and collapsible chambers
for easy storage. Every patient who takes MDIs must be given a spacer.

Proper Technique
When educating on metered dose inhaler (MDI) technique, an asthma educator must ensure that a
patient is using as spacer (valved holding chamber). If the patient does not have one, a spacer must
be provided. During the education session, the asthma educator might say, “This is called a metered
dose inhaler. When you push the canister, a puff of medicine comes out. You inhale this medicine
into your lungs to help control your asthma. See how fast the medication comes out when you press
it? It’s almost impossible for anyone to catch that puff, so we recommend using a spacer to help
hold the puff and slow it down so you can inhale it all the way into your lungs without wasting any
medicine. Here’s how it works. Remove the cap from the inhaler. Put the inhaler into the spacer.
Now, exhale completely, and put the spacer mouthpiece into your mouth. Press the canister and
breathe in slowly and deeply. Hold your breath for up to 10 seconds, and then let it out slowly. Then
do the same thing again for the second puff.”

Spacer Mask Attachments

Spacers are not only available as a mouthpiece. Some spacers also come with mask attachments
that can be used for adults who are unable to follow instructions, or have trouble coordinating their
inhalation technique, or for children who are too young to properly use their spacer. Spacer mask
attachments come in sizes from infant up to adult. When utilizing a spacer with a mask in a
cooperative adult patient, the process is similar to using a spacer with a mouthpiece. The patient
will prepare the spacer for use by removing the inhaler cap and inserting it into the spacer. The
patient exhales completely, places the mask over his or her nose and mouth, presses the inhaler
canister to release the medication, and then takes a slow, deep breath in. Instruct the patient to hold
his or her breath for up to 10 seconds and then exhale slowly. The process is then repeated for
additional doses. Spacer masks are helpful for patients who have trouble inhaling through a
mouthpiece.

When treating an unconscious adult patient or young child, spacers with metered dose inhalers can
still be utilized. Rather than using a mouthpiece spacer, a spacer with mask attachment can be used.
Spacer mask attachments come in sizes from infant up to adult. When utilizing a spacer with a mask
in an unconscious adult patient or young child, the process is different than the traditional spacer
procedure. In this case, the caregiver will prepare the inhaler for use by removing its cap and
inserting it into the spacer. The caregiver will firmly place the mask over the patient’s nose and
mouth, ensuring a tight seal. Then the caregiver will press the inhaler’s canister, releasing a puff.

- 78 -
Copyright © Mometrix Media. You have been licensed one copy of this document for personal use only. Any other reproduction or redistribution is strictly prohibited. All rights reserved.

Licensed to elliot mattison (2146499335, i.astraltears@gmail.com) of 9801 royal ln, dallas, Texas 75231. If this product is distributed to others,
elliot mattison agrees to pay Mometrix the full retail price for each unlicensed recipient. Please play fair and respect the work of our authors.
The caregiver should then observe the patient’s chest rise for at least six breaths to ensure the
medication was delivered. The process is then repeated for additional doses.

Formoterol

Formoterol is a long-acting bronchodilator that is used for the treatment of moderate to severe
asthma. Formoterol is a dry powder inhaler but is administered differently than other dry powder
inhalers. When educating a patient on formoterol technique, the educator might say, “This
medication is a controller medicine that opens up your lungs and helps you breathe easier for a
long period of time. It does not work fast but has to be taken every day on schedule for you to feel
better. It’s called a dry powder inhaler. This is the inhaler part, and these small capsules hold the
powdered medicine. You place a capsule into the chamber, close it up, and squeeze the triggers.
Now the capsule has been punctured, and you can breathe in the medicine. Exhale all the way. Now
place the mouthpiece in your mouth and inhale deeply and fast. Hold your breath for up to 10
seconds. Now you’re done! Just throw away the used capsule.”

Proper Storage of Dry Powder Inhalers

Dry powder inhalers require special storage instructions to help preserve their efficacy. When
educating on storage and introducing a patient to dry powder inhalers for the first time, the
educator might say, “These are called dry powder inhalers. They have tiny amounts of premeasured
powder inside that is loaded into the chamber for you to inhale. The powder is such a tiny amount;
you won’t feel it at all. But you might notice a sweet taste after taking the medicine. The company
added that flavor so that the patients know they got the medicine. It is important to not keep this
medicine in a hot or steamy place like the bathroom because the humidity can cause the powder to
clump up, and you won’t be able to inhale it. It’s also important to always keep the inhaler closed
until you’re ready to use it and keep it level once it’s open so the medicine doesn’t spill out through
the mouthpiece.”

Dry Powder Diskus Inhalers

Dry powder inhalers each have a slightly different design, which requires careful instruction by the
asthma educator so that the patient uses it properly. Diskus inhalers, such as Advair Diskus,
Serevent Diskus, and Flovent Diskus are all round, flat inhalers that look somewhat like a small
Frisbee. When educating on their use, the asthma educator might say, “This is called a dry powder
inhaler diskus. Here’s how it works. First, slide open the inhaler. Next, slide back the trigger. Now
the medicine is loaded and ready for you to inhale. Exhale all the way and place the mouthpiece in
your mouth. Be careful to keep the inhaler level so the medicine doesn’t spill out. Inhale fast and
deep. Now hold your breath for up to 10 seconds and let it out slowly. You’re done! Now rinse and
spit so that the powder doesn’t cause irritation in your mouth. That can happen with dry powder
inhalers that are steroids.”

Pulmicort Flexhaler

Pulmicort Flexhaler has a different procedure associated with its use as compared to other dry
powder inhalers. When educating on its use, the asthma educator might say, “This is a dry powder
inhaler that you use every day to help control your asthma. It does not work quickly. It has to build
up in your system over time before it can start to work. Here’s how you use it. The first time you use
it, you have to prime it. That means that you have to get it ready for use. You do that by removing
the cap and then twisting the bottom to one side and back again until you hear a click. Now the
inhaler is primed and ready to use. To use it, twist the bottom to one side and back again, just like
when you were priming it. Now exhale completely and place the mouthpiece in your mouth. Now

- 79 -
Copyright © Mometrix Media. You have been licensed one copy of this document for personal use only. Any other reproduction or redistribution is strictly prohibited. All rights reserved.

Licensed to elliot mattison (2146499335, i.astraltears@gmail.com) of 9801 royal ln, dallas, Texas 75231. If this product is distributed to others,
elliot mattison agrees to pay Mometrix the full retail price for each unlicensed recipient. Please play fair and respect the work of our authors.
inhale deeply and fast, hold your breath for up to 10 seconds, and let it out slowly. Now rinse your
mouth with water so that the powder doesn’t cause any mouth irritation.”

Asmanex Twisthaler

The Asmanex Twisthaler has a different procedure associated with its use as compared to other dry
powder inhalers. When educating on its use, the asthma educator might say, “This is a dry powder
inhaler that is used every day to help control your asthma. It will not work quickly. It has to build
up in your system over time for you to feel better. Here’s how you use it. First, twist the cap off the
inhaler. The cap is what releases the medication. Now the powder is in the chamber and ready for
you to inhale it. To get ready to use the inhaler, you should first exhale all the way. Now, place the
mouthpiece in your mouth. Next, inhale deeply and fast. Hold your breath for 10 seconds and let it
out slowly. You’re done! Now just replace the cap by twisting it back on, and you’re ready for the
next dose. If your doctor wants you to take a second dose, just do the same process one more time.”

Dose Counting Process

Dry Powder Inhalers


When educating on the use of metered dose inhalers and dry powder inhalers, it is of the utmost
importance for the patient to understand how to determine how many doses are left in the inhaler.
Dose counting is much more difficult in metered dose inhalers than in dry powder inhalers. All dry
powder inhalers have built-in dose counters that show exactly how many doses are left in the
inhaler. The dose counter counts down by one dose every time the patient takes a dose of the
inhaler. Some dry powder inhalers, such as the fluticasone with salmeterol (Advair) diskus, display
the dose count number in red once it hits five doses or less. That way, patients are alerted to the
fact that they are almost out of medication, and they are aware that it is time to refill their
prescription.

Metered Dose Inhalers


With metered dose inhalers, it is complicated to determine how many doses are left in an inhaler.
Few metered dose inhalers, such as budesonide with formoterol (Symbicort), have built-in dose
counters that display the number of remaining doses. With most metered dose inhalers, the patient
must estimate use of the inhaler and calculate the number of puffs left. For example, if a metered
dose inhaler contains 120 doses and the patient uses two puffs four times per day, then the patient
must divide 120 doses by eight daily puffs. With this calculation, the inhaler will last 15 days. If the
patient uses the inhaler on an as-needed basis, such as with rescue medications, then the
calculation becomes more difficult. The patient must then record the doses on a piece of paper or a
smartphone application. This ensures that the patient does not inadvertently run out of medication
due to an estimation error.

Soft-Mist Inhaler

When educating on the use of a soft-mist inhaler, such as ipratropium bromide and albuterol
(Combivent Respimat), the educator might say, “This is called a soft-mist inhaler. The first time you
use the inhaler, you have to prime it. That means that you have to get it ready for use. To do that,
you have to first pull off the base and write the discard date on the label. That’s 3 months from
today. Next, we push the cartridge all the way into the inhaler until it clicks. Then we put the base
back on, twisting until it clicks. Now turn the base in the direction of the arrows, and then open the
cap. Hold the inhaler away from you and press the button to release the spray. Then put the cap
back on. We will do this three more times, and then the inhaler is ready for use. Now to use the
inhaler, turn the base toward the arrows until it clicks. Then open the cap, and exhale completely.

- 80 -
Copyright © Mometrix Media. You have been licensed one copy of this document for personal use only. Any other reproduction or redistribution is strictly prohibited. All rights reserved.

Licensed to elliot mattison (2146499335, i.astraltears@gmail.com) of 9801 royal ln, dallas, Texas 75231. If this product is distributed to others,
elliot mattison agrees to pay Mometrix the full retail price for each unlicensed recipient. Please play fair and respect the work of our authors.
Now place the mouthpiece in your mouth, and take a slow, deep breath while you press the button.
Hold your breath for up to 10 seconds, and then replace cap.”

Home Nebulizer

When using a nebulizer for home use, the patient must be educated on the proper procedure for
using the machine. Nebulizers vary according to manufacturers and models, so the asthma educator
should refer to the machine’s instruction booklet prior to educating the patient on its use. Generally
speaking, all nebulizers have similar components: a gas source (air or oxygen), tubing that goes
from the gas source to the nebulizer cup, a nebulizer cup, and a mouthpiece or mask. The asthma
educator must ensure that the patient has the correct size mouthpiece or mask depending on the
patient’s age and comfort. A mouthpiece is appropriate for adults and older children, especially
those who might be claustrophobic wearing a mask. Masks come in various sizes depending on the
patient’s age and facial structure. The mask should sit on the bridge of the nose and cover the front
part of the patient’s chin. It should not hang below the chin or rise above the bridge of the nose.
Take care not to allow the nebulizer vapor to go in the patient’s eyes during treatment. It is
recommended that small children use masks as the mask is not technique dependent.

Correct Spacer Technique

Valved holding chambers, also known as “spacers,” are tools used with metered dose inhalers that
collect and suspend the puff of medication, allowing the patient to breathe the medication in slowly
without losing medication. Due to the quickly propelled puff of medication, most patients cannot
time their inhalation perfectly to have good drug deposition in the lungs. Instead, the patient loses
much of the medication to the mouth. A spacer allows the process to slow down and gives greater
drug deposition to the patient’s lungs. All patients who use metered dose inhalers must have
spacers. When using a spacer, the patient removes the cap from the inhaler and inserts it into the
spacer. Then the patient exhales completely and puts the spacer mouthpiece into his or her mouth.
The patient then presses the canister and breathes in slowly and deeply. Then, the patient will hold
his or her breath for up to 10 seconds and then exhale slowly. The process is repeated for
subsequent doses.

Cleaning and Disinfecting Nebulizers

Patients who use nebulizers at home must care for the equipment according to the manufacturer’s
instructions. Although cleaning techniques vary among models and manufacturers, the patient
should refer first to the instruction booklet that comes with the nebulizer. Generally speaking,
nebulizer cups and masks or mouthpieces should be cleaned daily and disinfected once per week.
To clean the nebulizer, the patient disassembles the nebulizer cup and places the parts in warm,
soapy water. The mask or mouthpiece should also be cleaned with warm, soapy water. Then the
components are rinsed with water and left to air dry. When disinfecting nebulizer components, the
nebulizer cup must be disassembled, and then the patient can soak the components and mask or
mouthpiece in either 70 percent isopropyl alcohol for 5 minutes or 3 percent hydrogen peroxide for
30 minutes. The components are then rinsed with sterile water and left to air dry.

Cleaning Spacers

Patients who use spacers (valved holding chambers) with their metered dose inhalers must care for
the spacer according to the manufacturer’s instructions. Although cleaning techniques vary among
models and manufacturers, the patient should refer first to the instruction booklet that comes with
the spacer. Generally speaking, spacers can be disassembled and soaked in warm, soapy water for
15 minutes. Then the spacers should air dry vertically so that water is able to drain out of the

- 81 -
Copyright © Mometrix Media. You have been licensed one copy of this document for personal use only. Any other reproduction or redistribution is strictly prohibited. All rights reserved.

Licensed to elliot mattison (2146499335, i.astraltears@gmail.com) of 9801 royal ln, dallas, Texas 75231. If this product is distributed to others,
elliot mattison agrees to pay Mometrix the full retail price for each unlicensed recipient. Please play fair and respect the work of our authors.
spacer. Bottle brushes should not be used on spacers. In addition, if a spacer is dishwasher safe
according to the manufacturer, then the spacer can be cleaned in the top rack portion of the
dishwasher. Follow all manufacturer instructions on the care and cleaning of the spacer.

Replacing Spacers

When determining when to replace a spacer (valved holding chamber), the patient should refer to
the manufacturer’s instruction booklet that comes with the spacer. Guidelines vary among
manufacturers and models. Generally speaking, if a spacer is in good working order, is not cracked
or otherwise damaged, and the one-way valve in intact, then the spacer is still usable. Valved
holding chambers should be replaced if there is any sign of damage or wear and tear. The spacer
should also be replaced if it is several years old or if the flexible plastic and rubber pieces are
becoming stiff or cracked. If the patient cleans the spacer regularly according to the manufacturer’s
instructions, the spacer should be usable for up to several years. Patients can purchase replacement
spacers at a local pharmacy.

Replacing Nebulizer Cups and Masks or Mouthpieces

When determining when to replace nebulizer components, the patient should refer to the
manufacturer’s instruction booklet that comes with the nebulizer. Guidelines vary among
manufacturers and models. Generally speaking, nebulizer components should be replaced if they
show signs of wear and tear or other forms of damage. If the patient has disposable nebulizer
accessories, such as a mask from a previous hospital stay, those items should be thrown away after
a few uses. Most non-disposable nebulizer components are expected to last 6 months to 1 year or
longer. The life span of the components is greater when the patient follows the manufacturer’s
instructions for cleaning and care of the nebulizer parts. Patients should have an extra set of
components available in case their equipment becomes damaged or no longer works properly.
These parts are purchased through a patient’s durable medical equipment company and/or their
doctor’s office. The patient requires a doctor’s order for any medical equipment purchases.

Troubleshooting Home Nebulizer Machines

Home nebulizers, like all medical equipment, can sometimes malfunction or stop working. If a
nebulizer has stopped working, the patient or clinician should first ensure that the unit is plugged
in and any battery backup systems have fresh batteries. If the machine is running, but mist won’t
come out of nebulizer, the patient or clinician should check the equipment for kinks in the tubing,
disconnections, or other signs of damage. The patient or clinician can also check any filters that
might be part of the nebulizer and clean or change them according to the manufacturer’s
instructions. If the machine is still not functioning properly, the patient should then contact the
durable medical equipment company for instructions as it may be time to replace the nebulizer or
its components. It is important to note that regular cleaning, disinfection, and inspection of the
patient’s home nebulizer will help ensure the machine continues to work effectively.

Troubleshooting Spacers

Valved holding chambers (spacers) do not have many components, and therefore, it is fairly easy to
troubleshoot any problems that might arise with their use. Before using the spacer each time, the
patient should visually inspect the spacer for any signs of damage, dirt, dust, or foreign objects. Any
debris must be removed prior to use. In addition, if the spacer seems clogged or if the patient can’t
inhale through the spacer, the patient and clinician should visually inspect the spacer to see if it has
a foreign body inside it or if a flap and/or valve is dislodged or damaged. Most problems associated

- 82 -
Copyright © Mometrix Media. You have been licensed one copy of this document for personal use only. Any other reproduction or redistribution is strictly prohibited. All rights reserved.

Licensed to elliot mattison (2146499335, i.astraltears@gmail.com) of 9801 royal ln, dallas, Texas 75231. If this product is distributed to others,
elliot mattison agrees to pay Mometrix the full retail price for each unlicensed recipient. Please play fair and respect the work of our authors.
.

with spacer use are either due to foreign bodies and or obstructions or damaged and or misaligned
one-way valves. If the spacer is still not functioning, it might be time to replace it.

Educating Patients on Use of Nebulizers

When educating a patient on nebulizer use, it is important to refer to the manufacturer’s


instructions that came with the equipment. Each machine is different in assembly, cleaning, and
use. Generally speaking, the educator can say, “This is a breathing treatment machine for you to use
at home. It is important to keep all the parts clean and check them each time you use them to make
sure there’s no damage to the equipment. To use it, first open the vial of medication and squeeze it
into the cup. Then place the mask or mouthpiece on top and turn on the machine. The treatment
will take 15 to 20 minutes to finish. When it starts to sputter a little toward the end, give the cup a
shake or a tap to get the rest of the medication down into the cup.”

Procedures Associated with Rescue and Controller Nebulizer Treatments

When educating on the different processes associated with rescue and controller nebulizer
treatments, the educator might say, “You can use this nebulizer with both rescue and controller
medications. For your inhaled steroids, which are the controller medications, make sure you rinse
your mouth out after you take your treatment. You should also wipe your face with a wet cloth or
wash your face if you use a mask with your treatment. Don’t let the medicine go in your eyes
through the mask holes. If the mask is so big that it is allowing smoke to get in your eyes, we can get
you a smaller mask that fits correctly. If you used this nebulizer with your rescue medicine, you
don’t have to do anything special afterward. But if you feel very jittery after a treatment, you can
rinse your mouth out and see if that helps. You may also notice that your controller medications
leave a sticky residue in your nebulizer cup. If that happens, follow the manufacturer’s cleaning
directions each time you use your controller medicine nebulizer treatment.”

Importance of Demonstration

Comprehensive asthma education, especially for medical equipment, is an important component of


successful asthma management. Simply telling a patient how to use a piece of equipment is often
not enough to ensure comprehension of the task. Asthma educators should bring demo versions of
common asthma-related medical equipment to asthma education sessions. This can include `
metered dose inhalers, dry powder inhalers, peak flow meters, spacers with mask attachments, and
mouthpiece spacers. Demonstrating them to the patient often results in better comprehension than
just giving verbal directions. It is also important to instruct the patient on the task and then have
the patient demonstrate the skill. That way, the asthma educator is sure that the patient not only
understands how to use the equipment but uses it correctly and safely. This is especially important
with medication delivery devices as inadequate technique results in poor drug deposition.

Importance of Refilling Asthma Medications Prior to Running Out

All asthma patients must be educated on the importance of refilling asthma medications prior to
them running out. The patient must not wait until the inhaler is empty before refilling it. Sometimes
there are drug shortages, which can delay access to a patient’s lifesaving medications. Other
patients might use a mail order pharmacy that can take 1 week or longer to receive medications.
Therefore, a good rule of thumb is to refill a prescription at least 1 week prior to it running out,
depending on the patient’s insurance plan restrictions and requirements for prescription refills. For
dry powder inhalers and metered dose inhalers with dose counters, the patient can see exactly how
many puffs are left and divide that number by how many puffs he or she takes per day to figure out
how many days the inhaler will last. For metered dose inhalers without dose counters, the patient

- 83 -
Copyright © Mometrix Media. You have been licensed one copy of this document for personal use only. Any other reproduction or redistribution is strictly prohibited. All rights reserved.

Licensed to elliot mattison (2146499335, i.astraltears@gmail.com) of 9801 royal ln, dallas, Texas 75231. If this product is distributed to others,
elliot mattison agrees to pay Mometrix the full retail price for each unlicensed recipient. Please play fair and respect the work of our authors.
should calculate how many puffs are in the canister, divided by how many puffs he or she takes per
day to determine how many days it will last. Then the patient can mark the calendar at least one
week prior to that date to refill the medication.

Criteria by Which Medical Equipment Must Be Replaced

All medical equipment must be cleaned and cared for in accordance with the manufacturer’s
directions. When medical equipment, including spacers, peak flow meters, and nebulizers, are
properly maintained, they can last for 6 months or longer. Before each use, the patient must
carefully inspect all equipment for signs of damage. If any component of the equipment is visibly
damaged, the patient should refrain from using it and seek a replacement. For peak flow meters and
spacers, they should contact their physician for a new prescription for these tools. They can be
ordered via mail order or filled at a local pharmacy. For nebulizers, the patient should contact the
durable medical equipment supplier and inquire about a replacement or service call with an
equipment repair technician. Damaged medical equipment can be dangerous, and patients should
seek a replacement or repair immediately upon noticing damage.

Troubleshooting Peak Flow Meters

Peak flow meters rarely have a need for troubleshooting; however, sometimes the equipment can
malfunction due to damage or obstruction. If the patient is blowing into the peak flow meter, and it
is not registering a reading, the patient should visually inspect the peak flow meter for signs of
damage such as cracks or missing pieces. The patient should also look for any obstructions
including foreign bodies in the mouthpiece. If none of these problems are present, the patient
should reassess his or her peak flow meter technique. Ensure a good, tight seal with the mouth
around the mouthpiece, and be sure that all of the exhaled air is going through the patient’s mouth
into the mouthpiece rather than being exhaled through the patient’s nose. If none of these actions
resolve the issue, the peak flow meter should be replaced. The patient can purchase a peak flow
meter at a local pharmacy.

Modifying Medical Delivery for Patients with Special Needs

Patients with special needs often need minor adjustments to their medication delivery devices to
optimize medication delivery. For example, in patients who are unconscious or unable to follow
commands, spacers with masks can help a patient successfully use a metered dose inhaler. The
masks allow the patient to breathe normally rather than taking long, deep inhalations on command.
Dry powder inhalers are also generally easier to use for patients who have cognitive or physical
restrictions as they require less coordination and hand strength than metered dose inhalers. For
patients with reduced hand strength or arthritis in their hands, simply turning the metered dose
inhaler with spacer upside down can help the patient use the inhaler successfully. In this case, the
patient would push up on the canister with the thumbs rather than pushing down on the canister
(in the upright position) with the index finger. If the patients are able to, they can also push down
on the canister with a thumb in the upright position; however, this hand positioning is a bit more
awkward. If inhalers are too difficult for the patient to use, nebulizers can be used instead.

Asthma-Related Risks Associated with Hot Yoga Classes

It is important to incorporate the patient’s fitness activities into an asthma education plan as
physical wellness and weight loss can help reduce the incidence of severe asthma exacerbations in
some patients. For exercises such as yoga or tai chi, the slow, fluid movements are not likely to
exacerbate a patient’s asthma. Certain positions, such as lying flat on the back or folding into more
restrictive yoga poses may cause some shortness of breath or a feeling of restricted breathing. A

- 84 -
Copyright © Mometrix Media. You have been licensed one copy of this document for personal use only. Any other reproduction or redistribution is strictly prohibited. All rights reserved.

Licensed to elliot mattison (2146499335, i.astraltears@gmail.com) of 9801 royal ln, dallas, Texas 75231. If this product is distributed to others,
elliot mattison agrees to pay Mometrix the full retail price for each unlicensed recipient. Please play fair and respect the work of our authors.
simple change of position or modified pose may help improve the patient’s breathing while
mitigating any feelings of dyspnea. Most yoga activities are acceptable and won’t cause harm to a
patient with asthma. The exception to this statement is hot yoga, or bikram yoga, where indoor
temperatures are greater than 100 degrees and indoor humidity is at increased levels. This hot,
steamy environment is likely to trigger an asthma exacerbation, especially in combination with
restrictive yoga poses that might be used. Hot yoga is not recommended for asthmatic patients or
those with other health conditions.

Dangers Associated with Over-the-Counter Asthma Medications

Some patients utilize over-the-counter asthma medications for several reasons. Sometimes the cost
of prescriptions or doctor visits are too much for the patient to pay, or sometimes the patient
simply doesn’t understand the dangers of using these treatments. Over-the-counter medications,
including sprays, pills, and inhalers, are not acceptable for the treatment of asthma. These
medications, most of which contain ephedrine and or epinephrine, can cause severe and life-
threatening side effects. Epinephrine and ephedrine can cause serious cardiac side effects including
arrhythmias, cardiac arrest, and death. In addition, these drugs do not successfully treat the asthma
exacerbation as they treat the symptoms and not the underlying disease process. These
medications relax the smooth muscle of the bronchioles for short periods of time. Therefore, the
patient may take increased amounts of these medications, resulting in cardiac events and rebound
bronchospasm.

Potential Dangers Associated with Herbal Supplements and Nutritional Supplements

Herbal and nutritional supplements are popular today due to the “natural” source of the
ingredients; however, these products are not regulated or approved by the U.S. Food and Drug
Administration. There is no way to tell if these products contain the ingredients they advertise, and
some supplements have hidden additives that might be harmful. For example, some herbal
supplements are associated with severe allergic reactions and allergic asthma exacerbations. All
herbal supplements are derived from plants and flowers. If a patient is allergic to any of those
components, the patient is likely to have an allergic reaction or allergic asthma exacerbation when
using those supplements. Some patients are not aware of any seasonal allergies or food allergies
until they begin taking supplements. Omega-3 supplements are often derived from seafood, which
can cause allergic reactions in sensitive patients. In addition, sulfites from shellfish can cause
asthma exacerbations, and this substance can be present in omega-3 products. Asthma educators
must ask the patient about any natural, herbal, or nutritional supplements at each asthma
education session.

Adherence to Prescribed Medication Regimen When Alternative Therapy is Available

It is important for the asthma educator to incorporate alternative therapies per the patient’s
request, as long as these interventions are harmless and do not interfere or interact with any of the
conventional asthma treatments ordered by the patient’s physician. The asthma educator must
stress the importance of prescribed medication regimen adherence as these therapies have been
studied extensively and have been proven to be safe and effective. The asthma educator might say,
“It is important to take your prescribed medication exactly as the doctor prescribed. Missing a dose
can cause your asthma to become much worse and can cause asthma emergencies. We can
incorporate some of your home remedies as long as you continue to take your prescribed
medication. You should not take any over-the-counter asthma medicines as these are not safe and
can be bad for your heart. The medications your doctor ordered are safe and effective for asthma
patients and will help improve your asthma symptoms.”

- 85 -
Copyright © Mometrix Media. You have been licensed one copy of this document for personal use only. Any other reproduction or redistribution is strictly prohibited. All rights reserved.

Licensed to elliot mattison (2146499335, i.astraltears@gmail.com) of 9801 royal ln, dallas, Texas 75231. If this product is distributed to others,
elliot mattison agrees to pay Mometrix the full retail price for each unlicensed recipient. Please play fair and respect the work of our authors.
Treating Atopic Diseases

Atopic diseases, such as seasonal allergies and eczema, have a direct relationship to the severity
and frequency of asthma symptoms. Allergic reactions characterized by an itchy nose, nasal
congestion, runny nose, headache, itchy eyes, and/or postnasal drip can trigger asthma symptoms
and cause severe asthma exacerbations. The increased mucus production, inflammation, and cough
associated with allergies can all trigger bronchospasm and bronchoconstriction in an asthmatic
patient. Therefore, these allergic disorders must be carefully monitored and treated to avoid
exacerbating a patient’s asthma. The use of intranasal steroids, which reduce inflammation and
tissue reactivity in the nose, can help treat hay fever and allergic rhinitis, which will subsequently
help reduce the severity and frequency of asthma symptoms. Immunomodulating biologicals, such
as Omalizumab, are especially effective in some patient populations with specific immunoglobulin E
(IgE) levels. These medications are costly, with potentially severe side effects, and are used
sparingly. Last, immunotherapy can be helpful for patients with seasonal allergies as the chronic
inflammation and respiratory symptoms associated with an allergic reaction can trigger asthma
episodes.

Yearly Influenza Vaccines

Influenza infection is a serious health issue for patients with asthma. Influenza is especially
dangerous for patients with asthma because these patients are more susceptible to infections and
pneumonia due to their reactive airways. These patients are more likely to have serious respiratory
symptoms associated with the flu and subsequent severe asthma exacerbations. Patients with
asthma are more likely to be hospitalized and die of the flu than their otherwise healthy
counterparts. Children with asthma are especially vulnerable to serious complications associated
with influenza contraction, including hospitalization and death. All asthmatic patients age 6 months
old and older are strongly encouraged to get a yearly flu shot. Often these shots are available at
physician offices in September and October but can be found earlier in some locations such as
pharmacies. Not all locations offer pediatric flu shots, so patients should check with their doctors
for information on flu shots for children.

Treating Comorbid Conditions

Patients with comorbidities such as acid reflux disease, obesity, and obstructive sleep apnea all
have an increased risk of poor asthma control. These underlying diseases cause changes in the body
that directly exacerbate a patient’s asthma. For example, in acid reflux disease (gastroesophageal
acid reflux disease [GERD]), rising stomach acids through the esophagus cause airway inflammation
and bronchial provocation, which results in increased severity and frequency of asthma symptoms.
Obesity can cause acid reflux disease and can also cause a reduction in general lung function due to
excess weight on the chest wall. This extra weight can also result in obstructive sleep apnea, which
is another comorbidity that must be successfully treated in asthmatic patients. All of these
comorbidities affect a patient’s level of asthma control and can cause frequent, poorly controlled
asthma exacerbations. Ensuring the successful treatment of comorbidities is an important part of
the asthma education session, and asthma educators should be prepared to ask questions about any
possible comorbidities and refer the patient to specialists if needed.

Smoking Cessation Medications

Asthmatic patients who smoke have a high risk of chronic, uncontrollable asthma symptoms and
exacerbations. Quitting smoking is the intervention that has the highest impact on asthma
frequency and severity. All patients with asthma who smoke should be strongly encouraged to quit.

- 86 -
Copyright © Mometrix Media. You have been licensed one copy of this document for personal use only. Any other reproduction or redistribution is strictly prohibited. All rights reserved.

Licensed to elliot mattison (2146499335, i.astraltears@gmail.com) of 9801 royal ln, dallas, Texas 75231. If this product is distributed to others,
elliot mattison agrees to pay Mometrix the full retail price for each unlicensed recipient. Please play fair and respect the work of our authors.
The asthma educator can refer the patient to health-care professionals who can help. Educating the
patient on some possible methods is helpful as the patient will then have information to refer to.
Several methods of smoking cessation can be used, including quitting “cold turkey” and utilizing
medications. Smoking cessation medications come in several forms, including gums and tablets.
Some medications contain nicotine replacement ingredients, whereas others do not. Personal
preference, medical history, and a physician’s assessment can help in the decision-making process
when trying to determine which drug is right for the patient. The asthma educator can provide
support and referrals for patients who need additional help with their smoking cessation.

Exercise-Induced Asthma

Exercise-induced asthma is a common form of asthma that can either stand alone or coexist with a
general asthma diagnosis. In other words, exercise can be a patient’s only asthma trigger, or it can
be one of many asthma triggers. Exercise-induced asthma must be prevented when possible and
treated appropriately during episodes. A patient’s activity level and physical fitness must be
maintained as obesity can contribute to asthma exacerbations. Exercise-induced asthma is easily
prevented by pre-medicating with a short-acting bronchodilator 10 to 15 minutes prior to exercise.
Patients can keep their inhalers in their school lockers, pants pockets, or gym bags and use the
inhalers before they exercise to prevent asthma symptoms. Some patients prefer to pre-medicate
with Cromolyn sodium before exercise, which is also an effective intervention. If exercise-induced
asthma symptoms occur, activity should be temporarily stopped, and the patient should take a
short-acting bronchodilator. Once the patient’s symptoms subside, activity can continue. Many
patients also utilize warm-up and cooldown periods to help ease into exercise and reduce the
likelihood of exercise-induced asthma symptoms from occurring.

Combating Psychosocial Factors

Psychological factors, including stress, depression, anxiety, and psychological disorders can all
contribute to asthma exacerbations. Increased stress and anxiety cause physical symptoms in many
patients, such as changes in breathing patterns and tidal volumes (the amount of air inhaled during
normal breathing) as well as sleeping changes and weight gain. These physical changes can
subsequently trigger an inflammatory response in the lungs. Patients with psychological ailments
must be treated promptly. Asthma educators can suggest some relaxing and deep-breathing
techniques to help with stress relief, and the educator can also refer the patient to external sources
that might be helpful. For example, if a patient is battling depression, anxiety, or a serious
psychological disorder, the asthma educator can encourage that patient to either see his or her
primary care physician or refer the patient to a mental health professional for assessment and
treatment.

Addressing Economic Issues

Patients with asthma utilize many different components of the health-care system, including
general physician visits, educational visits, prescription medications, and sometimes diagnostic
imaging and lung function testing. These interventions can be costly, and for patients with financial
problems, these costs become a burden. Asthma educators can refer the patient to social workers
who can help find low- or no-cost alternatives to many asthma medications and therapies,
physician or clinic visits, and diagnostic testing. Lack of health-care resources can also be a problem
with patients in low-income neighborhoods, and these patients should be referred to clinics and
resources that are available in their area. The asthma educator can also supply the patient with
samples (if available) of peak flow meters, spacers, and other asthma management tools. The

- 87 -
Copyright © Mometrix Media. You have been licensed one copy of this document for personal use only. Any other reproduction or redistribution is strictly prohibited. All rights reserved.

Licensed to elliot mattison (2146499335, i.astraltears@gmail.com) of 9801 royal ln, dallas, Texas 75231. If this product is distributed to others,
elliot mattison agrees to pay Mometrix the full retail price for each unlicensed recipient. Please play fair and respect the work of our authors.
patient’s primary physician may also be able to provide the patient with medications samples,
including metered dose inhalers and dry powder inhalers.

Effects of Drug Abuse

Patients who use illegal drugs such as heroin and cocaine, and legal drugs such as alcohol and
marijuana, are at a much higher risk for severe, life-threatening asthma exacerbations as compared
to patients who do not abuse drugs. Patients with asthma who also abuse drugs have more
frequent, severe asthma exacerbations and are more likely to die or be hospitalized due to their
asthma. Illegal drugs can cause damaging effects to the pulmonary system and cause long-term
damage to the lung tissues. That underlying damage results in more severe and frequent
exacerbations. If an asthma educator knows that a patient abuses drugs, the educator should
consult with the health-care team to determine the best course of action for that patient. The
patient may need to be referred to a facility or health-care provider that can treat drug addiction.

Importance of Avoiding Asthma Triggers

Trigger avoidance is one of the most important components of asthma management as this
preventative measure can stop an asthma episode before it starts. When educating a patient about
trigger avoidance, the asthma educator might say, “The things that bother your asthma are called
triggers. Asthma triggers are different for each person, so we need to find out what your possible
triggers are and decide how best to avoid them. We want to make sure you can still do all the
activities you love while avoiding those things that might bother you. We will work together to
make a plan on how to avoid triggers, and we will make sure that plan fits into your lifestyle. If we
don’t avoid your asthma triggers, we can’t improve your symptoms. Triggers irritate your lungs and
cause asthma episodes. If we don’t avoid those, your lungs will not be able to heal, and you’ll have
more frequent asthma episodes.”

Elimination of Secondhand Smoke in Home Environment

Patients with asthma are at an increased risk of severe, life-threatening asthma exacerbations when
exposed to secondhand smoke. Patients who smoke should be encouraged to quit and given
smoking cessation tools to help them. If the patient is exposed to secondhand smoke from family
and friends, the patient must ensure no one smokes in the home. The family member will have to
smoke outside until he or she is able to quit. The family member will also have to change clothes
and bathe to remove thirdhand smoke residue from the clothing and skin. Even this trace amount of
smoke residue can cause severe asthma symptoms. Family members must not smoke in the family
car as this closed environment (even with open windows) is dangerous for a patient with asthma.
Air fresheners and smoke-reducing sprays are not helpful for these patients as these chemicals
often cover up the smell of smoke with a different fragrance. This can be an asthma trigger in itself
as scents and fragrances are common inhaled irritants that contribute to asthma exacerbations.

Avoidance of Wood Smoke and Leaf Smoke

Tobacco smoke is not the only smoke that is harmful to patients with asthma. Wood smoke and leaf
smoke are both carcinogenic as well as generally irritating to the lining of the bronchioles. Wood-
burning stoves and fireplaces must be well ventilated and well maintained, with proper yearly
cleaning and inspections performed. If smoke enters the home, the patient must quickly exit the
home, while others ventilate the home to remove the smoke from the environment. Leaf smoke is a
particularly dangerous type of smoke for asthmatics as it contains several carcinogenic compounds.
Patients should avoid going outside on days when neighbors are burning leaves. Windows and
doors on the home should remain closed for the duration of the burning. These types of smoke can

- 88 -
Copyright © Mometrix Media. You have been licensed one copy of this document for personal use only. Any other reproduction or redistribution is strictly prohibited. All rights reserved.

Licensed to elliot mattison (2146499335, i.astraltears@gmail.com) of 9801 royal ln, dallas, Texas 75231. If this product is distributed to others,
elliot mattison agrees to pay Mometrix the full retail price for each unlicensed recipient. Please play fair and respect the work of our authors.
cause more frequent and severe asthma exacerbations and can cause more general respiratory
symptoms such as itchy, irritated eyes, watery eyes, runny nose, and sinus pressure.

Avoidance of Air Fresheners and Cleansers

Inhaled irritants, such as air fresheners and cleansers, can irritate a patient’s airways and trigger
asthma exacerbations. Any strong scent can trigger a patient’s asthma, regardless of its source.
Even “natural” essential oils can cause severe asthma exacerbations, especially if these oils are
derived from plants to which the patient is allergic. Patients with asthma should avoid the use of all
scents and fragrances, including scented candles, detergents, fabric softeners, plug-in air
fresheners, spray air fresheners, perfumes, and cologne. Unscented laundry detergents are found in
every major retailer, and many of the well-known detergent companies offer unscented varieties.
For cleansers, simple vinegar and water solutions or minimally scented cleaning products are
helpful. Bleach and other heavily scented or chemically based cleansers are not recommended for
patients with asthma due to their strong scents, which irritate reactive asthmatic airways.

Preventing Weather-Related Asthma Exacerbations

Patients with asthma often find that weather plays a role in the frequency and severity of their
exacerbations. Hot, humid conditions or cold and dry air can both cause asthma symptoms in
patients with reactive airways. Sometimes a change in weather, such as a change in temperature
and humidity levels after a thunderstorm, can be a serious asthma trigger. Although it is impossible
to completely avoid weather-related asthma episodes, patients can take simple steps to help
prevent these episodes. Avoiding outside activity during hot and humid conditions can help prevent
asthma exacerbations in the summer. In the winter, patients should cover their mouths and noses
with scarves to help warm and humidify the cold, dry air. This can help prevent bronchospasm due
to the inhalation of cold air. Patients should keep their air-conditioning on in the summer to help
filter out ozone, smog, and other particulates that are common on hot, humid days. This can also
help patients who are allergic to mold because mold spores are often stirred up during rainstorms.

Travel-Related Issues

When patients with asthma travel, they must plan carefully for their trips. The patients must find
out what the expected weather conditions will be at their destinations and plan accordingly. If the
patient is traveling to a different climate than his or her hometown, he or she can expect to have
some asthma symptoms upon arrival due to the quick weather change between locations, especially
if flying to the destination. A short plane ride is not enough time for a patient’s hyperreactive lungs
to adjust to a new climate. The patient may need to pre-medicate with controller medications a few
weeks prior to the trip and continue those medications throughout the travel time so that his or her
lungs are not as reactive to the weather conditions at the destination. Patients must also carry on all
asthma medications and equipment. Absolutely no asthma medications should be placed in checked
baggage. All medications should be refilled prior to the trip, and the patient should ensure he or she
has extra medication in case there are any unexpected delays or layovers.

Avoidance of Animal Dander

Indoor allergens are a major asthma trigger for most patients. Patients spend most of their time in
their homes and especially in their bedrooms. The patient’s home should be as asthma friendly as
possible. Therefore, indoor allergens must be managed and eliminated as much as possible. For
animal dander, patients should avoid getting furry pets. If there is already a furry pet in the home
that can’t be removed, the patient must keep the pet out of his or her bedroom at all times. Even
this action isn’t completely effective as dander can travel through the ductwork of a home and

- 89 -
Copyright © Mometrix Media. You have been licensed one copy of this document for personal use only. Any other reproduction or redistribution is strictly prohibited. All rights reserved.

Licensed to elliot mattison (2146499335, i.astraltears@gmail.com) of 9801 royal ln, dallas, Texas 75231. If this product is distributed to others,
elliot mattison agrees to pay Mometrix the full retail price for each unlicensed recipient. Please play fair and respect the work of our authors.
spread throughout the house. Regular vacuuming is necessary to remove some of the dander from
the home. There is no such thing as an allergy-free pet. “Hypoallergenic” pets refer to animals that
have lower dander counts and may be a better choice for patients with allergies. It is important to
note that if a patient is allergic to dogs, any dog (regardless of how “hypoallergenic” it is) will cause
asthma exacerbations and allergic reactions. Complete avoidance is necessary to successfully
manage a patient’s asthma and underlying allergies.

Avoidance of Dust Mites

Dust mites are a common asthma trigger for many patients. Dust mites are found in most soft
surfaces of the home, including upholstered furniture, rugs, curtains, stuffed toys, and even clothing
that has been in storage. To control dust mites in the home, the patient can eliminate some of the
sources of the problem by removing carpet and placing hard flooring in the home, removing
curtains and using hard blinds for window covers, and limiting plush toys in the bedroom. Patients
can also minimize dust mites by vacuuming rugs weekly with a HEPA-filtered vacuum cleaner and
dusting the home regularly utilizing a damp cloth. For plush toys, the patient can wash these
weekly, place them in the freezer overnight, or run them through one cycle of the clothes dryer on
the high heat setting. Patients should note, however, that not all stuffed toys are machine washable
and dryable. For dust mites in the bedroom, the patient should use mattress and pillow encasement
covers and wash all bedding weekly in hot water.

Avoidance of Mold, Moisture, and Pests in Home

Moisture in the home causes several asthma-related triggers, including mold and mildew growth
and cockroach infestation. To prevent these triggers, the patient must control the moisture level in
the home. The patient should use dehumidifiers, especially in basement areas. Indoor humidity
levels should be kept at less than 60 percent. All water leaks or water seepage should be dried out
within 24 hours. It only takes 1 to 2 days of wetness for mold growth to begin. If mold and mildew
has begun growing, the area must be cleaned utilizing detergent and water and dried thoroughly.
Mold is a naturally occurring organism in nature, and it is impossible to remove all spores from the
home; however, visible mold growth is a health hazard that must be carefully cleaned and removed.
Patients should use personal protective equipment during mold cleanup, including face masks.
Pests, like cockroaches, are attracted to moisture and food, so all sources of moisture in the home
should be removed. In addition, food should be stored in sealed canisters, and all food-related spills
must be cleaned promptly.

Avoidance of Outdoor Allergens

Outdoor allergens, such as trees, grasses, and weeds, are all common asthma triggers for many
asthma patients. Treating outdoor allergens is not a simple task as staying indoors indefinitely just
isn’t possible. Keeping windows closed and using an air-conditioner during the summer are some
effective practices for the prevention of outdoor allergens passing into the home. Patients should
try to limit their outdoor activities when allergen counts are high and bathe after outdoor activity
when allergen counts are high. Using an allergen-reporting tool, such as pollen counters, can be
helpful for patients who are planning outdoor activities and events. Generally speaking, springtime
is when trees and grasses are prevalent, and fall is when weeds are prevalent. Molds occur
throughout the year but are worse after rainstorms and during wet days. Pollen counts are often
higher on dry, windy days. Patients should take their allergy-controlling medications as directed
and should begin using them a few weeks before allergy season begins so that the medication is in
full effect by the time the allergy season hits. The same is true for the patient’s asthma-controller

- 90 -
Copyright © Mometrix Media. You have been licensed one copy of this document for personal use only. Any other reproduction or redistribution is strictly prohibited. All rights reserved.

Licensed to elliot mattison (2146499335, i.astraltears@gmail.com) of 9801 royal ln, dallas, Texas 75231. If this product is distributed to others,
elliot mattison agrees to pay Mometrix the full retail price for each unlicensed recipient. Please play fair and respect the work of our authors.
medications. These medications can be used seasonally for patients with allergic asthma who have
exacerbations only during allergy season.

Protecting Against Respiratory Infections

Respiratory infections are one of the most common triggers for patients with asthma and other
pulmonary diseases. Infection avoidance measures are simple and effective. Patients should have
good hand washing technique, including washing their hands for a full 20 seconds with soap and
water, washing before eating, and after using the restroom. Patients should be conscious about
touching their faces, and refrain from touching their eyes, noses, and mouths without washing their
hands first. Patients with asthma should avoid contact with sick people as much as possible and
stay home when they are sick. Flu shots and pneumonia shots are especially helpful for asthmatic
patients. All patients 6 months of age and older should get a seasonal flu shot. This is one of the
most important actions a patient can take to preserve their health. Asthmatic patients are at an
increased risk of complications from influenza, including respiratory distress and potential death.

Avoidance of Chemical Exposures

Chemical exposure can be an asthma trigger for some patients, especially those who work with
chemicals. Patients should take steps to avoid chemical inhalation, including the use of masks and
other personal protective equipment. Chemical exposures are also common in schools, with the use
of polishes, varnishes, and strong cleansers. Patients often don’t anticipate these exposures and
cannot plan around them. Therefore, the patient must always carry rescue medications for the
treatment of possible unexpected chemical inhalation. Patients who work in the gardening industry,
or those who have gardens as hobbies, may also be exposed to chemicals. Pesticides, fertilizers, and
other gardening supplies often contain chemicals that can trigger a patient’s asthma. Using masks
when working with these chemicals can be helpful. The patient might also look for some alternative
choices for these chemicals in their homes. Humidifiers and allergen covers were covered on
another card.

Environmental Control Tools for Use in Patient’s Home

Patients can utilize several different tools to help control their home environments and avoid
asthma triggers. The use of air cleaners is especially helpful for patients with indoor allergies. These
machines take in ambient air through a filter and circulate the filtered air back into the home. If a
patient is considering an air cleaner, the most important place to have one is in the bedroom. It
should be running constantly, and the patient’s windows should remain closed to minimize
circulation of allergens into the air. There are no specific recommendations for air cleaner models;
however, patients should be careful not to purchase one that releases ozone into the air as this is an
asthma trigger also. These air cleaners usually do not have filters and often have an ionizer feature.
Other asthma trigger mitigation tools for home use include vacuum cleaners and dehumidifiers.
Vacuums with HEPA filters are helpful but not necessary. Patients should be careful when emptying
their vacuums as emptying the canister contents into the trash receptacle can release the dust back
into the air. Performing this task outdoors or in a garage is helpful.

Home Visits

Home visits, although helpful for the control of home-based asthma triggers, are a rare offering for
most asthma education programs. Liability issues often prevent these visits from taking place, but
they have an important role in asthma management. Asthma educators can visit a patient’s home
and visually assess the patient’s environment to identify possible environmental asthma triggers in
the home and suggest ways to eliminate or mitigate them. For example, an asthma educator might

- 91 -
Copyright © Mometrix Media. You have been licensed one copy of this document for personal use only. Any other reproduction or redistribution is strictly prohibited. All rights reserved.

Licensed to elliot mattison (2146499335, i.astraltears@gmail.com) of 9801 royal ln, dallas, Texas 75231. If this product is distributed to others,
elliot mattison agrees to pay Mometrix the full retail price for each unlicensed recipient. Please play fair and respect the work of our authors.
enter a child’s bedroom and notice that the patient has fluffy comforters on the bed, plush
carpeting, heavy window curtains, and a hanging net holding dozens of stuffed toys on display.
These dust catchers are potential hazards for asthmatic children as dust mites are a common
asthma trigger. The asthma educator in this case can suggest either rotating stuffed animals (with
some cleaned and stored in plastic while others are on display), washing them weekly, or putting
them through a dryer cycle on the hot setting. For the fluffy bedding and window treatments, the
asthma educator can suggest washing them all weekly or removing the curtains and replacing them
with blinds. The educator can also suggest weekly vacuuming of the carpet.

Encouraging Patients to Communicate with Health-Care Team

Some patients are intimidated by the health-care team as well as the health-care system as a whole.
Many patients, especially those who are elderly, believe the “doctor knows best” and won’t ask
questions. These patients often think that if something is important, the physician or team will
inform them, and there is no need for questions. It is important for the asthma educator to
empower the patient, explaining that he or she is not only a member of the health-care team but the
most important member of the team. The patient’s goals are the ones that matter, and the health-
care team has to tailor an asthma management plan that meets those goals as well as any other
goals the team sets for the patient (such as reduction of symptoms, improved lung capacities, and
other medical assessments). The asthma educator should explain that the patient is in charge of his
or her health care, and if they need further assistance, the asthma educator or a professional patient
advocate can help communicate with the team.

Assessing Patient’s Judgment Regarding Asthma Management

Part of an asthma educator’s responsibility is assessing the patient’s judgment regarding asthma
management. The patient must be able to make decisions on how and when to use asthma
medications, when to seek emergency help, when to see a physician, and how to identify worsening
asthma symptoms. The asthma educator should ask open-ended questions based on the patient’s
written action plan to determine the patient’s abilities. For example, the asthma educator might
ask: “Tell me what you feel like when your asthma gets worse.” “What would you do in the yellow
zone of your asthma action plan?” The asthma educator might also ask questions such as these:
“When would you call for an ambulance?” “How do you know it’s time to visit the doctor for your
asthma?” These questions can help the asthma educator assess the patient’s decision-making skills
as well as his or her adherence to the asthma action plan.

Importance of Self-Management and Routine Follow-Up Care

Part of an asthma education session is preparing the patient to take an active role in his or her own
asthma management. It is important to explain to the patient that the asthma management plan is
designed especially for him or her, and he or she is the goal and decision maker for the care plan.
Carefully assessing a patient’s readiness to self-manage is an important part of the asthma
education process. The asthma educator must ensure that the patient understands all of the topics
presented in the asthma education session, especially those that relate to self-management, such as
the asthma action plan and medication regimen. Asking questions and reviewing the educational
topics during each asthma education session can help the educator ensure the patient is properly
prepared for self-management. Routine follow-up visits with the asthma educator as well as the
patient’s primary care physician or pulmonologist are essential to the successful management of
asthma. This type of continuous assessment can identify adherence issues, changes in symptoms,
and changes in circumstance that affect an asthma patient’s disease management.

- 92 -
Copyright © Mometrix Media. You have been licensed one copy of this document for personal use only. Any other reproduction or redistribution is strictly prohibited. All rights reserved.

Licensed to elliot mattison (2146499335, i.astraltears@gmail.com) of 9801 royal ln, dallas, Texas 75231. If this product is distributed to others,
elliot mattison agrees to pay Mometrix the full retail price for each unlicensed recipient. Please play fair and respect the work of our authors.
Tracking Patient’s Progress and Self-Management

Part of an asthma educator’s responsibility is to create detailed documentation for each asthma
education session. This documentation includes a physical examination, a technique assessment,
and any notations the educator wants to include regarding a patient’s self-management. Depending
on the facility, the asthma educator might have specific documentation tools such as assessment
pages or progress notes in the patient’s electronic or paper chart. This documentation should
include a patient’s complete asthma history; current physical examination findings; any changes in
home, work, or school environments; changes in the patient’s social history; and a review of the
patient’s peak flow monitoring and asthma action plan. Documenting these pieces of information
can help the asthma educator look back over the previous education session documentation and
identify patterns and changes in the patient’s asthma history.

Asthma Action Plan

An asthma action plan is a written document that outlines the actions a patient must take to treat
his or her asthma, depending on a variety of factors. The asthma action plan includes assessing the
patient’s severity and frequency of symptoms, medication regimen, self-monitoring plan,
environmental controls, and when to seek medical care. This plan is designed by the patient along
with the patient’s physician and asthma educator. Once developed, the action plan is shared among
the care team, and a copy is kept in the patient’s record. The patient keeps a copy of the action plan
at home, school, and/or work. This serves as a constant reminder of exactly what the patient should
do at each disease severity level and is an important tool for emergency situations. The action plan
is tailored to the individual; no two asthma action plans are exactly alike. The asthma action plan
should be reviewed each time the patient follows up with the physician and/or asthma educator,
and the asthma educator should reeducate on the action plan during each follow-up visit. Some
reeducation sessions might be brief, whereas others are extensive. The time spent reassessing the
asthma action plan mainly depends on the patient’s progress, severity and frequency of symptoms,
and general level of asthma control and impairment.

Reevaluation
During each follow-up visit with the asthma educator, the patient’s written asthma action plan must
be reassessed. Changes in circumstance, such as a change in environment or introduction of a pet
into the home, can affect a patient’s management level and require an adjustment to the patient’s
action plan. First, the asthma educator must check the patient’s level of control. The asthma
educator should ask open-ended questions such as this: “How has your asthma been since we last
talked?” This will allow the patient to assess his or her own asthma management. The asthma
educator can then ask more specific questions about severity and frequency of symptoms and use
of short-acting bronchodilators. The educator should also reassess adherence to the action plan and
can ask questions such as these: “How has this action plan been working out for you?” “Do we need
to make adjustments to it so it’s easier for you to follow?” The asthma educator should ask if the
patient’s goals for management have changed. If a patient has a sudden change in the frequency or
severity of symptoms, the patient might want to change the goals of treatment.

Monitoring Tools

Monitoring tools, such as peak flow meters and asthma diaries, are important quantitative tools for
the management of asthma. Some patients achieve better asthma management with the help of a
visual reference document and/or numeric values corresponding to asthma treatment. In other
words, some patients need clear measurements of symptoms to successfully manage their asthma.
One such tool is the peak flow meter. Some patients require daily peak flow measurements due to

- 93 -
Copyright © Mometrix Media. You have been licensed one copy of this document for personal use only. Any other reproduction or redistribution is strictly prohibited. All rights reserved.

Licensed to elliot mattison (2146499335, i.astraltears@gmail.com) of 9801 royal ln, dallas, Texas 75231. If this product is distributed to others,
elliot mattison agrees to pay Mometrix the full retail price for each unlicensed recipient. Please play fair and respect the work of our authors.
poor perception of symptoms, whereas others do not need to measure peak flows at all. All newly
diagnosed patients should use peak flows as a tool to help them perceive their symptom severity.
Asthma diaries can also be used to help the patient determine the level of asthma control. These
diaries are a daily record of the patient’s symptoms, general health, weather conditions,
environmental conditions, activity level, and activities performed. This is a tool that can help a
patient identify possible triggers. This is especially helpful when first diagnosed with asthma as
triggers vary widely among patients.

- 94 -
Copyright © Mometrix Media. You have been licensed one copy of this document for personal use only. Any other reproduction or redistribution is strictly prohibited. All rights reserved.

Licensed to elliot mattison (2146499335, i.astraltears@gmail.com) of 9801 royal ln, dallas, Texas 75231. If this product is distributed to others,
elliot mattison agrees to pay Mometrix the full retail price for each unlicensed recipient. Please play fair and respect the work of our authors.
Organizational Issues
Determining If Formal Asthma Education Program is Warranted

When determining whether a formal asthma education program is needed in your facility or
community, several measurements must be taken. First, the asthma educator needs to decide what
criteria he or she will measure to determine positive or negative outcomes. This may include
patient-reported events such as self-reported symptoms, frequency, or severity. Other indicators
include hospitalization rates and use of health-care resources. Second, the asthma educator needs
to assess the incidence and prevalence of asthma in his or her area or facility. Tracking historical
data is helpful in this pursuit. Outside organizations such as disease-based charitable organizations,
or professional asthma consortiums and memberships, can also be helpful. Once these pieces of
information are obtained, the asthma educator can then determine if there is a large enough patient
population for a formal asthma education program in the community or facility.

Assessing Availability of Resources for Development of Asthma Education Program

In the early stages of asthma education program development, several criteria must be assessed to
determine the resources available for program creation. Funding, either from grants and other
external sources or from in-facility budgets, is an important component of program development.
Inpatient asthma education sessions can be charged utilizing CPT codes per half hour, but
reimbursement varies. Although providers may not see any reimbursement revenue, they are able
to show productivity by “charging” for these services. This simple act can be the deciding factor for
executives who give program approval. The asthma educator must also determine what kind of
facility is used for asthma education sessions. Sessions can be one-on-one, inpatient sessions or
large-group community sessions. The asthma educator needs to secure a location for asthma
education visits, whether it is in the patient’s hospital room or in a community center event space.
Last, the asthma educator needs to decide who will be conducting the asthma education sessions. It
is recommended that any personnel providing asthma education should be certified by the National
Asthma Educator Certification Board so that the patient receives the most accurate information.

Deciding Style of Asthma Program

When designing an asthma education program, an asthma educator must decide on the style and
characteristics of the program based on the patient population served. For an inpatient asthma
education program, the asthma educator might design a program that includes both inpatient, one-
on-one education sessions as well as outpatient follow-up sessions and community outreach
lectures. An asthma educator based in a physician’s office or clinic might design a program that is
solely for the surrounding community, especially if the office is based in the inner city or other
areas where asthma is prevalent, and the incidence of severe exacerbations is higher. The asthma
educator can then determine what resources are needed for these programs, such as peak flow
meters, spacer samples, demo inhalers, written materials, and other tools. The asthma educator can
then research local nonprofit organizations to determine how to best source these tools, perhaps
applying for grants or other governmental assistance as needed.

Setting and Meeting Asthma Education Program Goals

Setting and meeting or exceeding goals is one component to a successful asthma education
program. The first step in the goal-setting process is to identify the needs of the patient population
the asthma educator is serving. For example, if the asthma educator is serving a high-risk patient

- 95 -
Copyright © Mometrix Media. You have been licensed one copy of this document for personal use only. Any other reproduction or redistribution is strictly prohibited. All rights reserved.

Licensed to elliot mattison (2146499335, i.astraltears@gmail.com) of 9801 royal ln, dallas, Texas 75231. If this product is distributed to others,
elliot mattison agrees to pay Mometrix the full retail price for each unlicensed recipient. Please play fair and respect the work of our authors.
population in the inner city, the asthma educator’s goals might include reduction of hospitalizations
and emergency department visits, decrease in frequency of exacerbations, and improved access to
health-care information. To meet these potential goals, the asthma educator could offer free asthma
education sessions that include no-cost samples of peak flow meters, spacers, and one-on-one or
group asthma education sessions. The asthma educator could also include support groups, follow-
up visits with individual patients and their families, and comprehensive family education. All of
these support measures help empower the asthmatic patient with accurate information about their
disease and a tailored approach to care. This improves quality of life and meets the asthma
educator’s goals of reducing the frequency of asthma exacerbation.

Selecting Materials and Deciding upon Teaching Methods

Asthma educators must determine the teaching methods they will use for education sessions. For
example, in community-based lectures, the asthma educator might choose to use a casual, group
discussion style of education that allows for free exchange of ideas and questions. This approach
helps engage the audience and limit boredom and disinterest. Asthma educators must carefully
select written materials for their educational programs. These tools must be easy to understand,
written a lower-grade reading level, and illustrated with demonstrative pictures. The asthma
educator will need to evaluate these materials for accuracy and credibility, so it is recommended
that the educator sources these materials from well-respected health-care organizations that
specialize in asthma. Asthma educators can also use materials from medical publishers, ensuring
that the information provided is in accordance with national asthma education guidelines. The
educator should ensure that the material’s references include credible sources free from bias or
inaccuracy.

Patient Privacy and Safety in Asthma Education Sessions

Respecting and maintaining a patient’s privacy is a federally mandated component of any health-
care related service. Asthma educators must ensure the patient’s privacy at all times and must
enforce the Heath Insurance Portability and Accountability Act (HIPAA). The asthma educator must
not share any information learned in the asthma education sessions with anyone other than the
patient’s care team. When conducting asthma education, the educator might ask the patient if he or
she wants the visiting family members to exit the room prior to conducting education. Many
patients want their family members to take part in education, which is acceptable if the patient
gives proper permission for this type of education to take place. The asthma educator must
document this information in the patient’s chart for future reference. Other safety measures can
include upholding Occupational Safety and Health Administration (OSHA) requirements for the
facility and instituting infection control measures. Asthma educators should refer to their facility’s
policies and procedures for upholding these requirements.

Training Staff and Maintaining Records of Asthma Education Participants

When providing asthma education to patients, the asthma educator must keep comprehensive
records of the education sessions for future reference. This includes documentation in the patient’s
chart (for inpatient participants) and participant assessment files (for outpatient participants). At a
minimum, asthma educators should have on file the patient’s contact information and emergency
contact information. This information is required of any participant, regardless of the educational
environment (group discussion, community outreach lecture, or one-on-one session). For private,
individual sessions, complete documentation must be performed. This includes information about a
patient’s physical exam, medical history, history of present illness, spacer use, peak flow use,
medication regimen, and other information.

- 96 -
Copyright © Mometrix Media. You have been licensed one copy of this document for personal use only. Any other reproduction or redistribution is strictly prohibited. All rights reserved.

Licensed to elliot mattison (2146499335, i.astraltears@gmail.com) of 9801 royal ln, dallas, Texas 75231. If this product is distributed to others,
elliot mattison agrees to pay Mometrix the full retail price for each unlicensed recipient. Please play fair and respect the work of our authors.
Evaluating Program Processes

Asthma educators must constantly reassess their patients’ comprehension of the asthma education
topics as well as reassessing the asthma education program’s processes and curriculum. In every
follow-up education session, the asthma educator must reassess the patient’s adherence to the
asthma action plan. If a patient has been consistently completing their asthma “homework,” such as
recording information in an asthma diary and recording peak flows, but then stops performing
these tasks, the asthma educator must address any barriers that are preventing the patient from
adhering to the prescribed action plan. Investigating sources of concern, such as medication side
effects, inability for the patient to carry a spacer daily, and annoyance with performing daily peak
flows all require additional counseling and suggestions for process or behavior modification. A lack
of adherence can be caused by many different factors, and all possibilities must be investigated to
have a successful outcome.

Assessing Patient Outcomes and Measuring Program Effectiveness

The measure of a successful asthma education program relies in part on the measurement of
patient outcomes. Outcomes can be measured in many different ways, utilizing many different
factors. For example, an outpatient community outreach asthma education program can measure a
successful outcome by program attendance (attendance may increase due to word of mouth among
community members), patient-reported reduction in symptom frequency or severity, and
community-wide statistics on the incidence and prevalence of asthma since the program was
initiated. Inpatient asthma education programs often have more reliable sources of outcome
information, such as patient readmission rates as well as medical records that include physical
examinations, asthma education assessments, peak flow readings, and other quantitative
information regarding the patient’s asthma exacerbations. A reduction in emergency department
visits and low asthma readmission rates all demonstrate the effectiveness of a hospital-based
asthma education program. If these criteria are not met, the program must be reevaluated to
determine weaknesses, and modifications can take place to improve outcomes and patient
satisfaction.

Expanding Asthma Education Programs to Include Multiple Groups and Providers

A successful asthma education program requires a partnership between the patient and asthma
educator; however, the asthma educator also needs a partnership between themselves and the
community. When offering an outreach asthma education program, the asthma educator needs to
assess the community and find groups that might benefit from incorporation into the program. For
example, the asthma educator can expand the outreach asthma education program to include
community groups, businesses, schools, churches, and health-care organizations. If the asthma
educator is practicing as part of a hospital “family” or large corporate hospital group, the asthma
educator can expand his or her offerings to include the other hospitals or clinics in the corporation.
If the asthma educator offers asthma education to local public schools, the area parochial or private
schools might also be interested in asthma education classes for staff. Expanding the asthma
educator’s reach is important for a sustained, successful asthma education program.

- 97 -
Copyright © Mometrix Media. You have been licensed one copy of this document for personal use only. Any other reproduction or redistribution is strictly prohibited. All rights reserved.

Licensed to elliot mattison (2146499335, i.astraltears@gmail.com) of 9801 royal ln, dallas, Texas 75231. If this product is distributed to others,
elliot mattison agrees to pay Mometrix the full retail price for each unlicensed recipient. Please play fair and respect the work of our authors.
NAECB Practice Exam
1. An asthma episode includes which of the following criteria?
a. Mucus production
b. Fever
c. Rhinitis
d. Gastroesophageal reflux disease (GERD)

2. Which of the following are considered inhaled irritants?


a. Pet dander and cleaning sprays
b. Perfumes and tree pollen
c. Leaf smoke and air fresheners
d. Cigarette smoke and dust mites

3. What is the mechanism of action for short-acting bronchodilators?


a. Prevention of the release of histamine
b. Prevention of the release of leukotrienes
c. Relaxation of smooth muscle in the airways
d. Relaxation of smooth muscle in the alveoli

4. The asthma educator is called to educate the parent of a six-year-old female who has asthma. She
has a history of nasal congestion and sneezing in the early morning hours, which wakes her up and
exacerbates her asthma during the daytime hours. Which of the following is the likely cause of her
morning symptoms?
a. Daytime allergies
b. Dust mite exposure
c. Inhaled irritants
d. Upper respiratory infection

5. The Advair (fluticasone with salmeterol) Diskus is available in what dosages?


a. 500/50, 250/50, 100/50
b. 500/50, 110/75, 220/75
c. 500/50, 250/50, 90/50
d. 500/50, 200/50, 100/50

6. A 45-year-old male presents to the clinic for his six-month checkup. The patient complains of
increasing nighttime symptoms, which occur up to four times per week. His daytime symptoms
have also increased, with cough and shortness of breath occurring several times per week during
waking hours. The patient currently uses levalbuterol two puffs q6h, and Advair 100/50 one puff
twice a day (BID). Which of the following changes should be made to the patient’s medication
regimen?
a. Increase the Advair dosage to 250/50 two puffs BID.
b. Discontinue levalbuterol and substitute albuterol.
c. Discontinue Advair and change to budesonide BID.
d. Add cromolyn sodium two puffs daily.

- 98 -
Copyright © Mometrix Media. You have been licensed one copy of this document for personal use only. Any other reproduction or redistribution is strictly prohibited. All rights reserved.

Licensed to elliot mattison (2146499335, i.astraltears@gmail.com) of 9801 royal ln, dallas, Texas 75231. If this product is distributed to others,
elliot mattison agrees to pay Mometrix the full retail price for each unlicensed recipient. Please play fair and respect the work of our authors.
7. When educating a patient on peak flow technique, the asthma educator asks the patient to
demonstrate use of her peak flow meter. The patient takes a deep breath in, blows fast and hard
into the peak flow meter, and records the value. Which of the following statements best describes
this patient’s peak flow meter technique?
a. The patient demonstrates proper peak flow meter technique.
b. The patient is exhaling too quickly into the peak flow meter.
c. The patient needs to repeat the maneuver three times.
d. The patient is blowing too forcefully into the peak flow meter.

8. A 37-year-old female presents to the clinic complaining that her albuterol inhaler is not relieving
her asthma symptoms. She has a history of exercise-induced asthma and uses the inhaler 10
minutes prior to activity. She states that for the past few weeks, she is having shortness of breath
during exercise despite using her inhaler. The clinician conducts a physical exam, and the findings
are normal. What should the clinician do first?
a. Add a low-dose inhaled corticosteroid to her medication regimen.
b. Change her albuterol to cromolyn sodium.
c. Ask her when she last refilled her inhaler.
d. Instruct her to avoid sports for two weeks.

9. A seven-year-old female presents to the emergency department on a Monday morning with


retractions, shortness of breath, and wheezing on inspiration and expiration. The patient’s mother
states that the patient began with a runny nose with clear mucus and persistent cough over the
weekend. The mother states that the patient’s breathing became progressively worse over the past
few days. The patient has a pet cat who sleeps on her bed. She went on a field trip to the pumpkin
farm three days prior where she went on a hayride and played in the leaves. The patient is also
allergic to nuts. The patient is afebrile with diffuse wheezing in all lung fields. What is the likely
cause of this patient’s asthma episode?
a. Cat dander exposure
b. Outdoor allergen exposure
c. Nut protein exposure
d. Infectious disease exposure

10. Which of the following is a contraindication for the use of Xolair?


a. History of tumors
b. History of prior intubation
c. History of refractory bronchospasm
d. History of allergies

11. An asthma educator is called to educate an adult inpatient prior to discharge. The patient is
eager to go home and is attempting to rush through the education session. What is the best way for
the asthma educator to engage this patient in the asthma education session?
a. Use a lecture-based approach to education.
b. Use written handouts exclusively.
c. Offer referral for a group session.
d. Ask open-ended questions.

- 99 -
Copyright © Mometrix Media. You have been licensed one copy of this document for personal use only. Any other reproduction or redistribution is strictly prohibited. All rights reserved.

Licensed to elliot mattison (2146499335, i.astraltears@gmail.com) of 9801 royal ln, dallas, Texas 75231. If this product is distributed to others,
elliot mattison agrees to pay Mometrix the full retail price for each unlicensed recipient. Please play fair and respect the work of our authors.
12. A three-year-old female presents to the clinic with coughing, retractions, and an SpO2 of 88%.
Diffuse wheezing is heard upon auscultation. The patient has a family history of asthma, but the
patient has not had respiratory symptoms prior to this episode. The patient’s symptoms began two
days before and have become progressively worse. Which of the following is the most likely
diagnosis for this patient?
a. Asthma
b. Reactive airway disease
c. Viral upper respiratory infection
d. Pneumonia

13. When asthma is severe and poorly controlled for long periods of time, what changes take place
within the patient’s respiratory system?
a. Permanent airway remodeling
b. Reversible inflammation
c. Hypercapnia due to hyperventilation
d. Cushing’s disease due to corticosteroid use

14. Which of the following criteria is NOT used for the categorization of a patient’s asthma?
a. Impairment
b. Risk
c. Control
d. Support

15. A 74-year-old male presents to the clinic for his routine six-month pulmonary function test. The
patient was diagnosed with asthma in 1980 and takes inhaled corticosteroids daily. Today he is
experiencing mild shortness of breath, which he attributes to working in his garden in the morning.
The patient’s respiratory rate is 22, and his breath sounds are clear to slightly diminished with a
faint expiratory wheeze. What pattern should the clinician expect to see on this patient’s flow
volume loop?
a. Normal lung function
b. Reduced flows and normal volumes
c. Reduced volumes and normal flows
d. A restrictive pattern

16. A 16-year-old male presents to the clinic for his yearly checkup and asthma education session.
The patient states that he was diagnosed with asthma six years ago. He states that his asthma
episodes are always mild, requiring one dose of his short-acting bronchodilator. He plays sports
regularly, with no impairment from his asthma. He states that his asthma is well controlled with
one exacerbation per year or less. The patient takes Advair 150/50 twice daily and levalbuterol two
puffs as needed. Which of the following actions should be taken at this time?
a. Discontinue inhaled corticosteroids.
b. Discontinue short-acting bronchodilators.
c. Change Advair to daily dosing.
d. Continue current medication regimen.

- 100 -
Copyright © Mometrix Media. You have been licensed one copy of this document for personal use only. Any other reproduction or redistribution is strictly prohibited. All rights reserved.

Licensed to elliot mattison (2146499335, i.astraltears@gmail.com) of 9801 royal ln, dallas, Texas 75231. If this product is distributed to others,
elliot mattison agrees to pay Mometrix the full retail price for each unlicensed recipient. Please play fair and respect the work of our authors.
17. A 40-year-old male presents to the clinic for his first asthma education session. The patient
states “I never have problems with my asthma. I had asthma in high school, but I outgrew it. I have
an inhaler for emergencies, but I never use it. I get a chest cold once or twice a year, but that’s all.”
Which of the following messages is most important to relay to this patient?
a. It is very common to outgrow asthma.
b. Despite good control, exacerbations can happen.
c. Asthma can be life threatening.
d. Daily medication is needed for asthma control.

18. All of the following are common signs and symptoms of asthma episodes EXCEPT:
a. Cough and wheeze
b. Throat clearing and orthopnea
c. Dizziness and chest tightness
d. Retractions and tracheal tugging

19. Which of the following statements is true regarding irritants and allergens?
a. Allergens can also be irritants.
b. Allergens can be avoided.
c. Irritants do not cause allergy symptoms.
d. Irritants are not natural substances.

20. Which of the following statements is true regarding asthma triggers?


a. Leaf and wood smoke is harmful to all patients with asthma.
b. Dust mites cause asthma symptoms in all asthma patients.
c. Inhaled irritants cause shortness of breath in all asthma patients.
d. Mold is harmful for some patients with asthma.

21. A seven-year-old male presents to the emergency room with an asthma exacerbation. The
patient has a history of poorly controlled asthma with frequent and severe exacerbations. The
patient is well known to the asthma educator because he has been admitted numerous times for his
asthma. The patient’s father is a smoker, and he smokes inside the home and car. The asthma
educator is called to educate the patient’s father on the dangers of smoking around a child with
asthma. Which of the following choices is the MOST important action for the patient’s father to
take?
a. The father must smoke outside until he is able to quit.
b. The father must change his clothes after smoking.
c. The father must not smoke in the car.
d. The father must not smoke in the garage.

- 101 -
Copyright © Mometrix Media. You have been licensed one copy of this document for personal use only. Any other reproduction or redistribution is strictly prohibited. All rights reserved.

Licensed to elliot mattison (2146499335, i.astraltears@gmail.com) of 9801 royal ln, dallas, Texas 75231. If this product is distributed to others,
elliot mattison agrees to pay Mometrix the full retail price for each unlicensed recipient. Please play fair and respect the work of our authors.
22. A 37-year-old male presents to the asthma educator’s office. He states that his asthma has
become worse since taking a promotion in the field of information technology. He states that his
new cubicle is positioned by the printer and he has noticed an increase in symptoms since he
moved his desk to the new location. He currently requires daily use of his rescue medication,
despite using his inhaled corticosteroids as directed. He states that he loves his job and feels
fulfilled in his new duties. He also states that he is afraid to approach his supervisor with his health
problems because he does not want to lose his new job. What is the likely reason for his increased
asthma symptoms?
a. Dust from the printer paper
b. Ozone from the printer
c. Perfume from his office mates
d. Stress from his new job

23. Which of the following statements is true regarding patients with asthma and allergies?
a. Asthma is included under the Americans with Disabilities Act.
b. Workplaces are not required to make concessions for patients with asthma.
c. Students have no right to request asthma trigger avoidance measures in the classroom.
d. Only food allergies are included under the Americans with Disabilities Act.

24. A 65-year-old male presents to the clinic complaining of increased severity and frequency of
asthma symptoms. He states that he started a regimen of nonsteroidal anti-inflammatory drugs
(NSAIDs) and cardioselective beta blockers due to an undisclosed heart condition four weeks ago.
The patient has two cats and one dog in his home, and he is an avid gardener. He is also a swimmer
and enjoys going to the senior center for a swim every morning. He says he has been a swimmer for
all his life. He says that his increased asthma symptoms began about a month ago, and his short-
acting bronchodilators aren’t providing him with enough relief. What is the likely cause of his
asthma symptoms?
a. Beta blockers
b. Cat and dog dander
c. NSAIDs
d. Exercise-induced asthma

25. The asthma educator is educating the family of a five-year-old female who has a history of
asthma. The patient’s bedroom is fully carpeted, with curtains on the windows and numerous
stuffed animals on the bed. The patient was recently tested for allergies and was subsequently
diagnosed with a dust mite allergy. She frequently wakes up with nasal allergy symptoms and mild
asthma symptoms. Which of the following choices is the BEST action to take to mitigate dust mite
exposure in the bedroom?
a. Remove all stuffed animals from the bedroom.
b. Replace carpeting with hardwood floors.
c. Enclose the patient’s mattress and pillows.
d. Vacuum the patient’s room twice weekly.

26. Which of the following statements is true regarding pet dander in the home?
a. Pet dander can be restricted to one area in the home.
b. Some pets are hypoallergenic and are safe for sensitive patients.
c. Pet dander is an inhaled irritant.
d. Pet saliva is a common allergen.

- 102 -
Copyright © Mometrix Media. You have been licensed one copy of this document for personal use only. Any other reproduction or redistribution is strictly prohibited. All rights reserved.

Licensed to elliot mattison (2146499335, i.astraltears@gmail.com) of 9801 royal ln, dallas, Texas 75231. If this product is distributed to others,
elliot mattison agrees to pay Mometrix the full retail price for each unlicensed recipient. Please play fair and respect the work of our authors.
27. Which of the following choices is NOT an inhaled irritant?
a. Scented candles
b. Dust mites
c. Bleach
d. Essential oils

28. A 17-year-old female presents to the emergency department complaining of shortness of breath
on exertion. She states that she recently joined the track team and is unable to keep up with her
peers due to shortness of breath. She states that she starts coughing and wheezing partway through
her running events and is unable to finish. The patient states that she uses her short-acting
bronchodilator when the symptoms begin, and they subside within 10 minutes. The patient does
not currently take any other medication. Which medication regimen change would be most helpful
for this patient?
a. Adding a low-dose inhaled corticosteroid
b. Premedicating with a long-acting bronchodilator
c. Premedicating with cromolyn sodium
d. Adding a leukotriene modifier

29. A 16-year-old female presents to the clinic complaining of an asthma exacerbation. The patient’s
respiratory rate is 25 with accessory muscle use. Expiratory wheeze is heard upon auscultation.
The patient has a barking cough and complains of throat irritation. The patient’s voice is raspy, and
she is having trouble speaking. The patient states that she has been using her rescue inhaler with no
relief, and pharmacy records indicate that her inhaled corticosteroids have been refilled regularly.
She states that she takes her inhalers exactly as directed. She is currently taking Fluticasone 110
mcg two puffs twice daily, albuterol two puffs twice daily, and montelukast 10 mg daily. A
continuous nebulizer was administered with no change to the patient’s condition. What should the
asthma educator suggest at this time?
a. Spirometry and visual inspection of the vocal cords
b. Intubation and continuous nebulization
c. Administration of racemic epinephrine
d. Administration of saline nebulizer treatment

30. Which of the following statements is true regarding pregnancy and asthma?
a. During pregnancy, a patient’s asthma may become worse.
b. During pregnancy, a patient may notice no change in asthma symptoms or severity.
c. During pregnancy, a patient’s asthma symptoms may improve.
d. All of the above.

31. All of the following conditions are comorbidities for asthma EXCEPT:
a. Allergic rhinitis
b. Gastroesophageal reflux disease (GERD)
c. Obstructive sleep apnea
d. Vocal cord dysfunction

- 103 -
Copyright © Mometrix Media. You have been licensed one copy of this document for personal use only. Any other reproduction or redistribution is strictly prohibited. All rights reserved.

Licensed to elliot mattison (2146499335, i.astraltears@gmail.com) of 9801 royal ln, dallas, Texas 75231. If this product is distributed to others,
elliot mattison agrees to pay Mometrix the full retail price for each unlicensed recipient. Please play fair and respect the work of our authors.
32. Which of the following statements is true regarding asthma and chronic obstructive pulmonary
disorder (COPD)?
a. COPD is reversible with treatment.
b. Asthma can turn into COPD over time.
c. COPD is associated with normal expiratory flow.
d. Asthma is associated with normal lung volumes.

33. A 32-year-old female presents to the clinic for her first asthma education session. The asthma
educator introduces herself to the patient and then contemplates what to say first. Which of the
following statements or questions should the asthma educator lead with?
a. Tell me about your asthma.
b. Does your asthma affect your home life?
c. Do you take your inhaler more than once per week?
d. Tell me about your home environment.

34. An 18-year-old female presents to the clinic for peak flow reevaluation. She started performing
peak flows six months ago when she was first diagnosed with asthma. The patient performs the
peak flow maneuver well, blowing forcefully into the peak flow meter and repeating the maneuver
three times. The patient records the highest number and compares it to her written asthma action
plan. The readings she is achieving are considerably higher than the values outlined in her asthma
action plan. Which of the following choices is the best explanation for this outcome?
a. Her asthma is improving.
b. She has grown since her asthma action plan was designed.
c. Her action plan was based on her predicted values.
d. She did not calculate her personal best correctly.

35. The asthma educator is called to educate a four-year-old female on her asthma. The patient is
eager to learn and asks appropriate questions. The asthma educator needs to determine the child’s
perception of her asthma symptoms. Which of the following questions and statements is the BEST
way to achieve this goal?
a. “Can you tell me what happens during an asthma attack?”
b. “Tell me about what your asthma feels like.”
c. “How often do you have asthma symptoms?”
d. “What medicine do you take for your asthma?”

36. Which of the following choices describes the best way to determine quality of life issues in an
elderly patient with asthma?
a. Ask the patient about her exercise tolerance.
b. Ask the patient how often they attend parties.
c. Ask the patient if she is able to go to the grocery store.
d. Ask the patient what hobbies or activities she enjoys.

37. When educating on the signs and symptoms of respiratory distress in a two-year-old patient,
the asthma educator should highlight which of the following symptoms:
a. Subcostal retractions
b. Accessory muscle use
c. Tripod sitting
d. Increased respiratory rate

- 104 -
Copyright © Mometrix Media. You have been licensed one copy of this document for personal use only. Any other reproduction or redistribution is strictly prohibited. All rights reserved.

Licensed to elliot mattison (2146499335, i.astraltears@gmail.com) of 9801 royal ln, dallas, Texas 75231. If this product is distributed to others,
elliot mattison agrees to pay Mometrix the full retail price for each unlicensed recipient. Please play fair and respect the work of our authors.
38. Which of the following choices is a sign of a severe asthma exacerbation in an adult patient?
a. Nasal flaring
b. Inability to speak in full sentences
c. Decreased respiratory rate
d. Disorientation

39. An adult patient has a history of multiple intubations during past asthma episodes. Which of the
following statements is true regarding this patient?
a. This patient is at high risk for asthma-related death.
b. This patient is at a low risk for ventilator dependency.
c. This patient is at a high risk for hypercapnia.
d. This patient is at a low risk for future hospitalizations due to asthma.

40. A patient states that she uses her inhaler every hour during asthma episodes. This patient uses
only bronchodilators for the treatment of her asthma. Which of the following statements is true
regarding this patient?
a. This patient has good asthma control.
b. This patient is using his medication as directed.
c. This patient is at risk for reduced bronchodilator tolerance.
d. This patient is at risk for a future cardiac event.

41. Frequent use of systemic corticosteroids is associated with all of the following conditions
EXCEPT:
a. Cushing’s disease
b. Mood swings
c. Increased appetite
d. Localized thrush infection

42. What is the definition of a poor perceiver?


a. A patient who has trouble measuring the frequency of their asthma symptoms
b. A patient who has trouble identifying their asthma triggers
c. A caregiver who has trouble measuring the severity of an asthma episode
d. A caregiver who is unable to identify an asthma-related emergency

43. All of the following choices contribute to a sudden loss of asthma control EXCEPT:
a. A new house
b. A new job
c. Weather changes
d. Weight loss

44. Describe the best way to determine a patient’s level of asthma control, severity, and risk.
a. Interview the patient.
b. Review past medical records.
c. Obtain pharmacy refill records.
d. Interview the patient’s family.

- 105 -
Copyright © Mometrix Media. You have been licensed one copy of this document for personal use only. Any other reproduction or redistribution is strictly prohibited. All rights reserved.

Licensed to elliot mattison (2146499335, i.astraltears@gmail.com) of 9801 royal ln, dallas, Texas 75231. If this product is distributed to others,
elliot mattison agrees to pay Mometrix the full retail price for each unlicensed recipient. Please play fair and respect the work of our authors.
.

45. All of the following are criteria for the referral of a patient to a pulmonologist EXCEPT:
a. Frequent hospitalizations
b. History of past intubations
c. Increasing medication dosage requirement
d. Inability to perform daily activities

46. A 17-year-old patient presents to the clinic for her six-month evaluation. The patient complains
of shortness of breath and cough during gym class. The patient is not an athlete and doesn’t enjoy
playing sports. She says that she can’t keep up with her classmates and coughs frequently during
exertion. Which of the following is the best course of action for this patient?
a. Allow the patient to be excused from gym class.
b. Premedicate with a short-acting bronchodilator.
c. Encourage the patient to attend gym class despite her symptoms.
d. Incorporate a low-dose inhaled corticosteroid for use after exertion.

47. Which of the following statements is true regarding exercise-induced asthma?


a. It can be present with any type of physical exertion.
b. It is commonly associated with athletes.
c. It is never a stand-alone asthma diagnosis.
d. It is only present in adult and adolescent patients.

48. A 21-year-old female presents to the clinic complaining of increasing frequency and severity of
asthma symptoms that coincide with her recent diagnosis of sinusitis. She complains of daily sinus
and throat pain and pain in her teeth. Which of the following statements is true regarding this
patient?
a. This patient has nasal polyps that are causing chronic sinusitis.
b. This patient has postnasal drip that is contributing to her lack of asthma control.
c. This patient should be referred to a dentist regarding her tooth pain.
d. This patient should be referred to an allergist for her chronic sinusitis.

49. Which of the following statements is true regarding gastroesophageal reflux (GERD) and
obesity?
`
a. Patients with GERD always experience symptoms while reclining.
b. Obesity does not cause asthma.
c. Patients with GERD always suffer from heartburn.
d. Obese patients are at a high risk for GERD-related asthma symptoms.

50. Which of the following choices correctly describes the effect that obstructive sleep apnea has on
asthma exacerbations?
a. Airway obstruction stimulates the vagal nerve, causing bronchoconstriction.
b. Hypoxia, secondary to airway obstruction, exacerbates asthma symptoms.
c. The tongue falls back into the oral cavity, causing increased airway resistance.
d. The airway is compressed due to increased body weight, causing inflammation.

- 106 -
Copyright © Mometrix Media. You have been licensed one copy of this document for personal use only. Any other reproduction or redistribution is strictly prohibited. All rights reserved.

Licensed to elliot mattison (2146499335, i.astraltears@gmail.com) of 9801 royal ln, dallas, Texas 75231. If this product is distributed to others,
elliot mattison agrees to pay Mometrix the full retail price for each unlicensed recipient. Please play fair and respect the work of our authors.
51. Which of the following statements is true regarding over-the-counter asthma medications?
a. Over-the-counter asthma medications do not interfere with prescription therapies.
b. Over-the-counter asthma medications are acceptable in inhaler form only.
c. The use of over-the-counter asthma medications is never recommended.
d. The use of over-the-counter asthma medications is an acceptable alternative to expensive
prescription therapies.

52. A 32-year-old female presents to the clinic for her yearly checkup. She states that she does not
take any prescription medications for her asthma and prefers natural and holistic remedies instead.
She states that she uses several different herbal supplements daily for the treatment of her asthma.
Which of the following choices is the BEST way for the asthma educator to address this situation?
a. The asthma educator should tell the patient to stop using the herbal remedies.
b. The asthma educator should tell the patient to continue her current regimen.
c. The asthma educator should warn the patient of potential side effects.
d. The asthma educator should make additional recommendations for natural therapies.

53. Which of the following substances is a food-based asthma trigger?


a. Sulfites
b. Preservatives
c. Artificial coloring
d. All of the above

54. All of the following are potential side effects of alternative medicine treatments for asthma
EXCEPT:
a. Bloodborne illness
b. Tissue damage
c. Bronchospasm
d. Piercing of internal organs

55. An asthma educator is called to examine a newly diagnosed 18-year-old male with exercise-
induced asthma. The asthma educator begins by asking the patient about his past medical history as
well as his family’s medical history. What is the clinical value of the family’s medical history?
a. There is no value in the family’s medical history; the asthma educator only needs the patient’s
medical history.
b. Asthma is a cyclical disease that can affect the entire family.
c. Asthma has a hereditary component that can predict the severity of a future exacerbation.
d. Obtaining the family’s medical history can predict the patient’s asthma risk.

56. A 43-year-old male presents to the clinic for his six-month checkup. He states that he is
experiencing increased asthma symptoms in the spring and fall, but he says that his asthma is well
controlled at other times of the year. He states that he has nasal congestion and itchy eyes in the
spring and sinus congestion in the fall. He is interested in allergy testing to pinpoint his allergies
and possibly undergo immunotherapy. Which allergy testing should be offered to this patient?
a. Radioallergosorbent testing (RAST)
b. Skin prick testing
c. Both RAST and skin prick testing
d. Saliva allergy testing

- 107 -
Copyright © Mometrix Media. You have been licensed one copy of this document for personal use only. Any other reproduction or redistribution is strictly prohibited. All rights reserved.

Licensed to elliot mattison (2146499335, i.astraltears@gmail.com) of 9801 royal ln, dallas, Texas 75231. If this product is distributed to others,
elliot mattison agrees to pay Mometrix the full retail price for each unlicensed recipient. Please play fair and respect the work of our authors.
57. A 14-year-old female presents to the clinic for her yearly follow-up appointment. She states that
she feels good and has not had any asthma symptoms in the past several months. Which of the
following interventions should the patient expect to undergo at this appointment?
a. Pulmonary function testing
b. Baseline arterial blood gas
c. Chest x-ray
d. Pulse oximetry testing

58. Which of the following statements is true regarding pulmonary function testing for patients
with asthma?
a. Pulmonary function testing can predict the frequency of future exacerbations.
b. Pulmonary function testing can predict the severity of future exacerbations.
c. Pulmonary function testing can show the severity of past exacerbations.
d. Pulmonary function testing can only show changes associated with a current exacerbation.

59. Which of the following statements is true regarding acute asthma exacerbations?
a. Coughing can be a patient’s only asthma symptom.
b. All patients with asthma have wheezing during an exacerbation.
c. Some asthma patients with asthma have stridorous breath sounds.
d. Rhonchi is a common breath sound associated with asthma exacerbations.

60. Which of the following breath sounds is most dangerous in a patient who is experiencing an
asthma exacerbation?
a. Rhonchi
b. Wheezing
c. Rales
d. Silent chest

61. When should an asthma educator refer a patient to emergency services?


a. When the patient can’t speak in full sentences
b. When the patient has diffuse wheezing throughout all lung fields
c. When the patient is suspected to have pneumonia
d. When the patient has one or more comorbidities

62. An asthma educator is asked to recommend self-assessment tools for a 19-year-old male with
asthma. All of the following are appropriate options EXCEPT:
a. Peak flow monitoring
b. Environmental controls
c. The Asthma Control Test
d. Asthma diary
63. An asthma educator is called to the emergency room to set up a patient’s peak flow meter for
home use. The asthma educator uses the patient’s predicted peak flow value to calculate their peak
flow zones. Which of the following ranges is a correct peak flow zone?
a. 75%–100%
b. 50%–79%
c. 0%–45%
d. 60%–80%

- 108 -
Copyright © Mometrix Media. You have been licensed one copy of this document for personal use only. Any other reproduction or redistribution is strictly prohibited. All rights reserved.

Licensed to elliot mattison (2146499335, i.astraltears@gmail.com) of 9801 royal ln, dallas, Texas 75231. If this product is distributed to others,
elliot mattison agrees to pay Mometrix the full retail price for each unlicensed recipient. Please play fair and respect the work of our authors.
64. An asthma educator is interpreting a patient with asthma’s pre- and postbronchodilator
pulmonary function test results. After bronchodilation, the patient’s FEV1 increased by 11%, the
FEF25–75 increased by 25%, and the patient’s FVC increased by 10%. Which of the following
choices best describes a correct interpretation of these results?
a. The patient’s FEV1 percent change indicates an inadequate response to bronchodilation.
b. The patient’s FEF25–75 percent change indicates an adequate response to bronchodilation.
c. The patient’s FVC percent change indicates an adequate response to bronchodilation.
d. More testing is needed to confirm an adequate response to bronchodilation.

65. The asthma educator is called to educate a 69-year-old female. Upon entering the room, the
asthma educator sees that the patient’s pulse oximeter measures only 81%, with a heart rate of 77.
The patient is alert and oriented, but she states that she’s cold and asks for another blanket. The
asthma educator measures the patient’s pulse and notices that her hands are cold. The asthma
educator measures the patient’s pulse at 91 bpm. The patient is resting comfortably and states that
she is scheduled to go home later that day. Which of the following choices is correct regarding this
patient’s condition?
a. This patient needs supplemental oxygen.
b. The patient’s pulse oximeter isn’t correlating.
c. The asthma educator needs to call for help stat.
d. The asthma educator needs to administer a nebulizer treatment.

66. What is the role of exhaled nitric oxide in asthma management?


a. Nitric oxide is given via nebulizer to improve oxygenation in patients with severe
bronchospasm.
b. Nitric oxide is measured to determine the degree of airway inflammation.
c. Nitric oxide is given via nonrebreather mask to improve oxygenation in patients with severe
bronchospasm.
d. Nitric oxide is measured to determine the degree of airway reversibility.

67. All of the following are common risks associated with methacholine challenge testing in a
patient with asthma EXCEPT:
a. Refractory bronchospasm
b. Respiratory arrest
c. Hypoxia
d. Syncope

68. Which of the following statements is true regarding peak flow meter readings?
a. If a peak flow meter is cracked, the readings will be lower.
b. If a patient partially blocks the mouthpiece with their tongue, the readings will be lower.
c. If a patient spits into the mouthpiece, the readings will be lower.
d. None of the above.

- 109 -
Copyright © Mometrix Media. You have been licensed one copy of this document for personal use only. Any other reproduction or redistribution is strictly prohibited. All rights reserved.

Licensed to elliot mattison (2146499335, i.astraltears@gmail.com) of 9801 royal ln, dallas, Texas 75231. If this product is distributed to others,
elliot mattison agrees to pay Mometrix the full retail price for each unlicensed recipient. Please play fair and respect the work of our authors.
69. The asthma educator is called to the bedside of a six-year-old female who has asthma. The
physician has written discharge orders including orapred 6.5 ml daily for five days and albuterol
2.5 mg nebulizer q4h prn. The patient’s mother states that “she is too little to take those muscle
building steroids! There are too many side effects for such a small child.” What should the asthma
educator say to this parent?
a. “These steroids have very serious side effects, but she’s only taking them for a short time, so it
will not be a problem.”
b. “These steroids are not like the body building ones. These cut down the inflammation in her
lungs and have minimal side effects.”
c. “These steroids work on her lungs and are different than the body building ones. Let’s talk
about some of the potential side effects that you might notice.”
d. “These steroids aren’t required, and we can substitute something else that will work just as
well to make her asthma symptoms better.”

70. Which of the following summer asthma triggers is dangerous for all patients with asthma?
a. Grass pollen exposure
b. Poor air quality
c. High mold levels
d. Temperature changes

71. When conducting asthma education sessions with adult asthma patients, who is responsible for
setting the goals of therapy?
a. The asthma educator
b. The physician
c. The patient’s family
d. The patient

72. The asthma educator is called to educate a 22-year-old male with asthma. The asthma educator
is standing during the education session, while the patient is sitting on the edge of the bed. The
patient’s wife has left the bedside to go to the cafeteria. The asthma educator is standing with
crossed arms while talking with the patient and stops to take several lengthy notes during the
education session. The patient seems disengaged from the asthma education session and does not
offer many details about their asthma control. Despite asking many open-ended questions about
the patient’s job, home, interests, and general health, the asthma educator is not able to engage the
patient in conversation. What mistake did the asthma educator make?
a. The asthma educator is asking too many open-ended questions.
b. The asthma educator’s body language is aggressive.
c. The asthma educator’s questions are inappropriate.
d. The asthma educator is not including the patient’s visitor in the session.

73. When compared to other socioeconomic status (SES) groups, which of the following statements
is true regarding low-SES patients with asthma?
a. Low-SES patients have adequate healthcare access, often from walk-in clinics.
b. Low-SES patients are more likely to get annual flu shots.
c. Low-SES patients have a higher incidence of chronic diseases.
d. Low-SES patients are less likely to have frequent asthma symptoms.

- 110 -
Copyright © Mometrix Media. You have been licensed one copy of this document for personal use only. Any other reproduction or redistribution is strictly prohibited. All rights reserved.

Licensed to elliot mattison (2146499335, i.astraltears@gmail.com) of 9801 royal ln, dallas, Texas 75231. If this product is distributed to others,
elliot mattison agrees to pay Mometrix the full retail price for each unlicensed recipient. Please play fair and respect the work of our authors.
74. A 35-year-old female is admitted to the hospital due to an asthma exacerbation secondary to an
upper respiratory infection. The patient states that her asthma is worse during different times in
her menstrual cycle. She says that her husband has the same asthma triggers as she does, and they
take the same medication, but his asthma is much better controlled than hers is. What is the likely
explanation for this patient’s worsening asthma symptoms compared to her husband?
a. Men respond better to conventional asthma therapies.
b. Estrogen fluctuations are causing this patient’s symptoms.
c. Men are more likely to seek medical help early in an exacerbation.
d. A lack of environmental controls is causing this patient’s symptoms.

75. An asthma educator is called to educate a patient who speaks only Mandarin. Which of the
following choices is an appropriate way to comprehensively educate this patient on their asthma?
a. Give the patient handouts and pamphlets in their native language.
b. Ask the patient’s family to translate your education session.
c. Use a hospital-based translator.
d. Demonstrate the proper techniques for peak flow meters and inhaler use.

76. Which of the following statements is true regarding the Asthma Guideline Implementation Steps
& Tools (GIST) project’s Stepwise Approach to Asthma Management?
a. There are six steps that measure a patient’s impairment and risk.
b. There are two versions, each for a different age group.
c. Once a patient moves up a step, they will not move back down.
d. The plan must be followed exactly, regardless of clinical judgment.

77. A 16-year-old female presents to the clinic for her yearly checkup. She tells the asthma educator
that she has asthma symptoms twice yearly, usually because of a respiratory infection. She says she
is not woken up at night due to asthma symptoms, and she can exercise without impairment. Using
the Stepwise Approach to Asthma Management, which of the following choices best describes this
patient?
a. The patient is in step 1 of the chart and requires a daily inhaled corticosteroid as well as a
bronchodilator.
b. The patient is not considered a candidate for the chart due to her infrequent symptoms.
c. The patient is in step 1 of the chart and only requires a short-acting bronchodilator.
d. The patient is in step 2 of the chart and requires a bronchodilator and a leukotriene modifier.

78. A 27-year-old male presents to the clinic complaining of increasing asthma symptoms that are
affecting his daily activities. The patient currently uses Advair 250/50 one puff BID, albuterol two
puffs q4h prn, and montelukast 10 mg daily. Using the Stepwise Approach to Asthma Management,
which step is this patient on currently and what action should be taken next?
a. This patient is on step 3 and needs an oral corticosteroid.
b. This patient is on step 4 and needs a higher dose of Advair.
c. This patient is on step 5 and needs omalizumab.
d. This patient is on step 6 and needs theophylline.

- 111 -
Copyright © Mometrix Media. You have been licensed one copy of this document for personal use only. Any other reproduction or redistribution is strictly prohibited. All rights reserved.

Licensed to elliot mattison (2146499335, i.astraltears@gmail.com) of 9801 royal ln, dallas, Texas 75231. If this product is distributed to others,
elliot mattison agrees to pay Mometrix the full retail price for each unlicensed recipient. Please play fair and respect the work of our authors.
79. A 34-year-old female presents to the clinic for her six-month checkup. The patient is currently
taking levalbuterol 2 puffs q4h prn and Advair 500/50. She suspects that she has seasonal allergies
because her asthma symptoms worsen and become more frequent in the spring and fall. The
patient states that her asthma medication isn’t keeping her well controlled and she is unable to
participate in her usual activities during allergy season. Using the Stepwise Approach to Asthma
Management, which step is this patient on currently and what action should be taken next?
a. This patient is on step 2 and should start theophylline.
b. This patient is on step 4 and should start omalizumab.
c. This patient is on step 5 and should start allergy testing.
d. This patient is on step 3 and should start cromolyn sodium.

80. In step 1 of the Stepwise Approach to Asthma Management for children ages 0–4, which
medication regimen is correct for the treatment of an acute exacerbation?
a. Short-acting bronchodilator treatments every 20 minutes for up to three treatments
b. Short-acting bronchodilator treatments every 30 minutes for up to three treatments
c. Short-acting bronchodilator treatments every 6 hours for up to three treatments
d. Short-acting bronchodilator treatments every 4 hours for up to 24 hours

81. A 10-year-old female presents to the clinic for her yearly checkup. Her mother states that her
daughter’s asthma has been very well controlled on her current medication regimen. The patient
was diagnosed with asthma at age 7, when she was having shortness of breath and wheezing with
exertion. At that time, the patient had daily asthma symptoms and was unable to participate in
running or swimming. The patient was also diagnosed with seasonal allergies including trees and
pollen. The mother noticed an absence of asthma symptoms when allergy season passes. Currently,
the patient is swimming daily and runs during gym class and recess with no impairment. The
patient premedicates with levalbuterol two puffs prn before both gym class and recess. In addition
to the levalbuterol, the patient is currently taking budesonide 0.5 mg nebulizer BID. Which of the
following actions should be taken at this time?
a. This patient needs to step down treatment. Discontinue budesonide and restart one month
before allergy season begins.
b. This patient needs to step down treatment. Discontinue levalbuterol and change to cromolyn
sodium.
c. This patient is adequately controlled, and no change is needed.
d. This patient needs to step up treatment during allergy season. Add montelukast 0.5 mg.

82. Which of the following statements is true regarding stepping down therapy?
a. If a patient has been well controlled for at least three months, the patient can step down one
step.
b. If a patient has been well controlled for six months or more, the patient can step down two
steps.
c. If a patient has been well controlled for at least one year, the patient can step down one step.
d. Once a patient is at a step, they will not move back down the chart. Stepping down is never
recommended.

- 112 -
Copyright © Mometrix Media. You have been licensed one copy of this document for personal use only. Any other reproduction or redistribution is strictly prohibited. All rights reserved.

Licensed to elliot mattison (2146499335, i.astraltears@gmail.com) of 9801 royal ln, dallas, Texas 75231. If this product is distributed to others,
elliot mattison agrees to pay Mometrix the full retail price for each unlicensed recipient. Please play fair and respect the work of our authors.
83. An asthma educator is called to educate a patient on her medication regimen. The patient is
confused about which medications are daily controller medications and which are rescue
medications. The patient’s medication regimen currently includes levalbuterol, mometasone,
cromolyn sodium, and montelukast. Which of the following choices best describes this medication
regimen?
a. Mometasone and montelukast are controller medications.
b. Mometasone is the only controller medication.
c. Montelukast is the only controller medication.
d. Cromolyn sodium and levalbuterol are rescue medications.

84. Which of the following statements is true regarding long-acting bronchodilators?


a. Long-acting bronchodilators are under a black box warning and should not be used.
b. Long-acting bronchodilators can be used as a patient’s sole controller medication.
c. Long-acting bronchodilators must be used in combination with systemic corticosteroids.
d. Long-acting bronchodilators are indicated for moderate to severe asthma.

85. What is the role of mast cell stabilizers in asthma management?


a. Mast cell stabilizers prevent the allergic cascade and treat allergic asthma.
b. Mast cell stabilizers can be used to treat exercise-induced asthma.
c. Mast cell stabilizers are a first-line therapy for the treatment of mild asthma.
d. Mast cell stabilizers are never used on an as-needed basis.

86. A 30-year-old male presents to the clinic complaining of increasing frequency and severity of
asthma symptoms. The patient was diagnosed with allergies to ragweed, dust mites, and mold. He
works outdoors for the electric company and spends many hours per week outside. He states that
his allergies are affecting his job performance because he often misses work due to wheezing and
sinusitis. The patient states that his bedroom is often dusty because he doesn’t have much time to
clean. He currently uses albuterol two puffs q4h prn and fluticasone 220 mcg BID. What suggestions
should be made at this time?
a. Start the patient on a leukotriene modifier.
b. Start the patient on a mast cell stabilizer.
c. Suggest that the patient request a transfer indoors.
d. Suggest that the patient wear a mask during sleep.

87. All of the following statements regarding immunomodulating biologicals are true EXCEPT:
a. Allergy testing is required prior to administration.
b. Patients must have a recent immunoglobulin E (IgE) level measured prior to administration.
c. A history of cancer is an absolute contraindication to therapy.
d. This medication increases the number of inflammatory cells in an allergic reaction.

- 113 -
Copyright © Mometrix Media. You have been licensed one copy of this document for personal use only. Any other reproduction or redistribution is strictly prohibited. All rights reserved.

Licensed to elliot mattison (2146499335, i.astraltears@gmail.com) of 9801 royal ln, dallas, Texas 75231. If this product is distributed to others,
elliot mattison agrees to pay Mometrix the full retail price for each unlicensed recipient. Please play fair and respect the work of our authors.
88. A 20-year-old male presents to the clinic for an asthma education session. The patient states
that his short-acting bronchodilator is causing bothersome side effects and he wants to stop taking
it. He says that if he takes his medication during class, he becomes so jittery that is he unable to
write. He also says that the medication gives him headaches and makes him feel nervous. He
currently takes levalbuterol two puffs q4h prn. He demonstrates his inhaler technique to the
asthma educator by inserting the inhaler into his mouth, pressing the canister, and taking a slow
deep breath in. All of the following are possible ways to combat his troublesome side effects
EXCEPT:
a. Instruct the patient to use a valved holding chamber.
b. Instruct the patient to rinse his mouth after using his inhaler.
c. Instruct the patient to use one puff instead of two.
d. Instruct the patient to gargle after using his inhaler.

89. Which of the following statements is true regarding controller medications?


a. Controller medications must be used daily without missing a dose.
b. Controller medications must be used for at least three days before noticing a change.
c. Controller medications cause serious systemic side effects.
d. Controller medications can be stopped when symptoms subside.

90. The asthma educator is educating a patient on his newly diagnosed asthma. The patient states
that he currently “takes the red inhaler for emergencies” and the “orange medicine for daily
maintenance.” To which medications is he referring?
a. Xopenex and Flovent
b. ProAir and Flovent
c. ProAir and QvAR
d. Asmanex and QvAR

91. All of the following are common side effects associated with short bursts of systemic
corticosteroids EXCEPT:
a. Weight gain
b. Personality changes
c. Blood sugar changes
d. Jittery feeling

92. Which of the following statements is true regarding leukotriene modifiers?


a. These medications do not exacerbate mental illness.
b. These medications are used for the treatment of severe refractory asthma.
c. These medications work on mast cells to reduce the allergic response.
d. These medications can cause sleeplessness and fatigue.

93. Which of the following drugs interacts with short-acting bronchodilators?


a. Tricyclic antidepressants
b. Long-acting bronchodilators
c. Mast cell stabilizers
d. Anticholinergics

- 114 -
Copyright © Mometrix Media. You have been licensed one copy of this document for personal use only. Any other reproduction or redistribution is strictly prohibited. All rights reserved.

Licensed to elliot mattison (2146499335, i.astraltears@gmail.com) of 9801 royal ln, dallas, Texas 75231. If this product is distributed to others,
elliot mattison agrees to pay Mometrix the full retail price for each unlicensed recipient. Please play fair and respect the work of our authors.
94. An asthma patient tells the asthma educator that he takes an inhaled corticosteroid twice daily
for asthma maintenance. He states that although the medication is working well, he prefers dry
powder inhalers to metered dose inhalers because he finds them easier to use. The asthma
educator discovers that the patient’s metered dose inhaler is also available in dry powder form,
eliminating the need to change to a different inhaled corticosteroid. Which of the following choices
is an inhaled corticosteroid available in both forms of inhaler?
a. Fluticasone
b. Mometasone
c. Budesonide
d. Beclomethasone

95. A 44-year-old male presents to the clinic for an asthma education session. The patient states
that he takes fluticasone with salmeterol Diskus, one inhalation daily, albuterol metered dose
inhaler two puffs q4h prn, and zafirlukast daily. The patient states that he is compliant in his
medication regimen and is well controlled on this schedule. What action should the asthma
educator take next?
a. Ask the patient about his level of allergy control.
b. Instruct the patient to increase his fluticasone with salmeterol to BID.
c. Educate the patient on the importance of valved holding chamber use.
d. Instruct the patient to begin monitoring peak flows daily.

96. A 50-year-old male tells the asthma educator that he has been using his levalbuterol inhaler q1–
2h prn this week due to an asthma exacerbation. He states that he doesn’t feel that the medication
works well for him, and he is requesting a different inhaler to try. What should the asthma educator
do FIRST?
a. Explain that levalbuterol is not a rescue inhaler for quick relief.
b. Give the patient a valved holding chamber and instruct him on its use.
c. Explain the dangers of short-acting bronchodilator overuse.
d. Give the patient an albuterol metered dose inhaler and instruct him on its use.

97. Which of the following asthma-related drugs is administered via injection?


a. Zileuton
b. Benralizumab
c. Mometasone
d. Nedocromil

98. Which of the following statements is true regarding metered dose inhalers and dry powder
inhalers?
a. Most metered dose inhalers must be used with a valved holding chamber.
b. Dry powder inhalers require a slow, even inhalation.
c. Metered dose inhalers are all breath actuated.
d. Dry powder inhalers should be stored in the patient’s kitchen.

- 115 -
Copyright © Mometrix Media. You have been licensed one copy of this document for personal use only. Any other reproduction or redistribution is strictly prohibited. All rights reserved.

Licensed to elliot mattison (2146499335, i.astraltears@gmail.com) of 9801 royal ln, dallas, Texas 75231. If this product is distributed to others,
elliot mattison agrees to pay Mometrix the full retail price for each unlicensed recipient. Please play fair and respect the work of our authors.
99. The asthma educator is called to a patient’s room to instruct the patient’s family on how to use a
metered dose inhaler at home. The patient is an elderly adult who is confused and needs assistance
with medication administration. Which of the following choices would be best for this patient?
a. Change the medication to nebulizer form.
b. Educate the family on a valved holding chamber with mask.
c. Change the medication to dry powder inhaler form.
d. Educate the family on a valved holding chamber with mouthpiece.

100. A patient presents to the clinic for reinstruction on his Asmanex Twisthaler. The asthma
educator asks the patient to demonstrate its use. The patient twists off the cap, exhales completely,
places the mouthpiece in his mouth, and inhales quickly and forcefully. He holds his breath for 10
seconds before exhaling slowly. He then pushes the cap back on the inhaler. Which step was
performed incorrectly?
a. The patient should breathe in slowly and deeply.
b. The patient must twist the cap back on.
c. The patient didn’t hold his breath long enough.
d. The patient didn’t actuate the dosage of medication.

101. A patient presents to the clinic for reinstruction on her Pulmicort Flexhaler. The asthma
educator asks the patient to demonstrate its use. The patient removes the cap, exhales completely,
places the mouthpiece in her mouth, and inhales quickly and deeply. She then holds her breath for
15 seconds and exhales slowly. She twists the cap back on the inhaler. Which step was performed
incorrectly?
a. The patient didn’t actuate the dose.
b. The patient should have pushed the cap back on.
c. The patient held her breath too long.
d. The patient needs a spacer for this inhaler.

102. A patient presents to the clinic for a follow-up visit. The patient states he is having trouble with
his Advair Diskus. The patient states that he can’t tell when he’s running out of doses, and he’s
afraid he is using the inhaler but receiving no medicine. What should the asthma educator tell this
patient?
a. “There is no dose counter on this medication, so we need to calculate how many puffs you
use.”
b. “There is a dose counter on this medication that turns red when you are on your last five
doses.”
c. “There is a dose counter on the bottom on this inhaler that tells you how many doses are left.”
d. “You should refill your medication every month, regardless of how often you use it.”

103. The asthma educator is educating the parent of a three-year-old female on her home nebulizer.
It is time for the patient’s scheduled breathing treatment, so the asthma educator begins
demonstrating the use of the home nebulizer. The patient is sleeping currently, and the mother
requests a “blow-by” treatment. What should the asthma educator say?
a. “A blow-by treatment is fine for at night, but it should not be used during the day.”
b. “Blow-by treatments are ineffective and should not be used at any time.”
c. “A blow-by treatment is acceptable only for infants.”
d. “Blow-by treatments are only given if the doctor orders it specifically.”

- 116 -
Copyright © Mometrix Media. You have been licensed one copy of this document for personal use only. Any other reproduction or redistribution is strictly prohibited. All rights reserved.

Licensed to elliot mattison (2146499335, i.astraltears@gmail.com) of 9801 royal ln, dallas, Texas 75231. If this product is distributed to others,
elliot mattison agrees to pay Mometrix the full retail price for each unlicensed recipient. Please play fair and respect the work of our authors.
104. All of the following choices are steps in the proper valved holding chamber technique EXCEPT:
a. Inhale until you hear a whistle.
b. Remove the spacer valve.
c. Hold your breath for 10 seconds.
d. Exhale slowly.

105. Which of the following statements is true regarding drug deposition when using inhalers and
valved holding chambers?
a. Most of the dose of a dry powder inhaler is lost to the oral cavity.
b. Valved holding chambers provide better drug deposition in the lungs.
c. Valved holding chambers capture the dose of medication for inhalation at a later time.
d. Some dry powder inhalers have antistatic technology.

106. During an asthma education session, a patient asks the asthma educator about the proper
technique for cleaning her home nebulizer machine. She uses the machine during exacerbations,
which take place once or twice yearly during flu season. What should the asthma educator tell her?
a. “Nebulizer parts are dishwasher safe and can be cleaned on the top rack.”
b. “Be sure to disinfect your nebulizer components at least once per week.”
c. “Soak your nebulizer parts in 70% isopropyl alcohol for 10 minutes.”
d. “Rinse your nebulizer parts in 3% hydrogen peroxide for 15 minutes.”

107. A patient presents to the clinic for an asthma education session. The patient prepares to
demonstrate his inhaler technique, but the asthma educator stops him. The asthma educator
inspects the patient’s spacer and finds it to be visibly soiled. The patient states that no one told him
he was supposed to clean it. The asthma educator begins to educate the patient on spacer cleaning
technique. Which of the following choices is the BEST option when the spacer instruction booklet is
not available?
a. Clean the spacer with a bottle brush to scrub away the dirt.
b. Clean the spacer in a dishwasher to disinfect the chamber.
c. Clean the spacer with warm, soapy water and dry vertically.
d. Clean the spacer with warm, soapy water and dry horizontally.

108. A patient has had the same valved holding chamber for over a year. It is in good working order,
with no cracks or visible damage. The patient asks how long he can use this spacer because he likes
the way it works and it is easy for him to use. What should the asthma educator say?
a. “Spacers can be used for several years as long as they are not damaged.”
b. “Spacers have a shelf life of one year.”
c. “Spacers should be replaced at least every two years.”
d. “Spacers can have hidden damage and should be replaced every three years.”

109. A patient is preparing for discharge from the hospital after a 24-hour stay. The patient has a
home nebulizer and asks if he can bring home the nebulizer mask and cup he used while in the
hospital. What should the asthma educator say?
a. “Yes, just be sure to clean it after each use and throw it away after a day or two.”
b. “Yes, this set should last for several months with proper cleaning.”
c. “No, disposable nebulizer components don’t usually work with home machines.”
d. “No, disposable nebulizer components shouldn’t be used more than twice.”

- 117 -
Copyright © Mometrix Media. You have been licensed one copy of this document for personal use only. Any other reproduction or redistribution is strictly prohibited. All rights reserved.

Licensed to elliot mattison (2146499335, i.astraltears@gmail.com) of 9801 royal ln, dallas, Texas 75231. If this product is distributed to others,
elliot mattison agrees to pay Mometrix the full retail price for each unlicensed recipient. Please play fair and respect the work of our authors.
110. A patient presents to the emergency room in respiratory distress. The physician has ordered a
nebulizer treatment with 2.5 mg albuterol. The asthma educator assembles the nebulizer and turns
the oxygen flow rate up to 8 liters. The educator notices that the nebulizer is sputtering and not
releasing a constant spray. What should the asthma educator do first?
a. Check all connections in the system.
b. Tap the nebulizer cup to release the condensation into the cup.
c. Increase the flow rate to 10 liters because 8 liters is inadequate.
d. Replace the nebulizer cup because it is not functioning correctly.

111. A patient asks the asthma educator to check his valved holding chamber. The patient says that
it doesn’t feel right when he uses it. He says that there is no resistance when he inhales, and he feels
like he is not getting the medication. All of the following choices are possible explanations EXCEPT:
a. There is a crack in the chamber.
b. The valve is displaced.
c. The mouthpiece is damaged.
d. There is a foreign body in the chamber.

112. All of the following statements are true regarding inhaled corticosteroids administered via
nebulizer treatment EXCEPT:
a. The patient should rinse their mouth and gargle after use.
b. The mist must not get into the patient’s eyes.
c. Inhaled corticosteroids leave a sticky residue in the nebulizer cup.
d. This medication cannot be mixed with other drugs in the cup.

113. An 18-year-old male presents to the clinic for an asthma education session. The patient states
that he uses his inhalers exactly as directed but he needs a refill on his albuterol inhaler. He tells the
asthma educator that he ran out last week but decided to wait until his appointment today to
request a new refill order. He states that he only uses it prior to gym class and he does not have
school this week. What should the asthma educator tell him?
a. “It is important to refill your medication about a week before it runs out.”
b. “Next time, call me as soon as you run out of your medication.”
c. “I will give you extra refills so that this doesn’t happen again.”
d. “Sometimes there are drug shortages, so you should refill your medication as soon as your
insurance allows.”

114. A 72-year-old female presents to the clinic complaining of trouble using her fluticasone
metered dose inhaler (MDI). The patient states that due to her arthritis, she is unable to actuate the
dose. All of the following are appropriate alternatives for this patient EXCEPT:
a. Change to budesonide via home nebulizer.
b. Change to a dry powder inhaler.
c. Instruct the patient to use her thumb to depress the canister.
d. Instruct the patient to turn the MDI upside down when depressing the canister.

- 118 -
Copyright © Mometrix Media. You have been licensed one copy of this document for personal use only. Any other reproduction or redistribution is strictly prohibited. All rights reserved.

Licensed to elliot mattison (2146499335, i.astraltears@gmail.com) of 9801 royal ln, dallas, Texas 75231. If this product is distributed to others,
elliot mattison agrees to pay Mometrix the full retail price for each unlicensed recipient. Please play fair and respect the work of our authors.
115. A patient presents to the clinic for a routine checkup. The patient tells the asthma educator
that she is currently practicing yoga. She states that her asthma is unaffected by her yoga classes,
and she is now considering Bikram yoga classes. What should the asthma educator tell this patient?
a. “Yoga of all kinds is beneficial for asthma, so this shouldn’t be a problem.”
b. “Yoga is not recommended for patients with asthma due to the rhythmic breathing
component.”
c. “Bikram yoga is not recommended due to the hot, humid conditions in the room.”
d. “Bikram yoga is more strenuous than other types of yoga, so premedicate prior to class.”

116. A 33-year-old female presents to the clinic for her six-month follow-up with the asthma
educator. The patient is allergic to ragweed, cats, and mold. She has one dog in the home and
exercises daily. She currently takes Advair 500/50 BID and albuterol two puffs prn. She states that
her asthma has been worse over the past month. She states that she started taking herbal
supplements six weeks ago because her sister said that they cured her asthma. She currently uses
echinacea and licorice root daily. She has also started acupuncture and visits a holistic practitioner
monthly for evaluation. Which of the following options is the likely cause of her increased asthma
symptoms?
a. Licorice root
b. Echinacea
c. Acupuncture
d. Dog dander

117. Which of the following statements is true regarding influenza vaccination for patients with
asthma?
a. Flu shots are required for all adults and children with asthma.
b. Flu shots are only recommended for patients with strong immune systems.
c. Flu shots are recommended for all asthma patients at least six months old.
d. Flu shots are required only if the patient has a prior history of influenza.

118. All of the following statements regarding comorbidities for asthma are true EXCEPT:
a. Obesity can cause restricted breathing due to excess body weight.
b. GERD causes bronchial provocation and inflammation.
c. Obstructive sleep apnea directly affects a patient’s asthma control level.
d. Asthma educators must refer patients to comorbidity specialists for treatment.

119. All of the following criteria are taken into consideration when deciding upon smoking
cessation medication EXCEPT:
a. Family history
b. Medical history
c. Personal preference
d. Physician’s assessment

120. The effects of stress can manifest into which of the following asthma-related symptoms?
a. Bronchospasm
b. Airway inflammation
c. Mucus production
d. Exercise-induced dyspnea

- 119 -
Copyright © Mometrix Media. You have been licensed one copy of this document for personal use only. Any other reproduction or redistribution is strictly prohibited. All rights reserved.

Licensed to elliot mattison (2146499335, i.astraltears@gmail.com) of 9801 royal ln, dallas, Texas 75231. If this product is distributed to others,
elliot mattison agrees to pay Mometrix the full retail price for each unlicensed recipient. Please play fair and respect the work of our authors.
121. A patient presents to the clinic for an asthma education session. The patient has suffered from
depression for the past several months after the death of her mother. She states that her asthma has
become worse lately and she is using her rescue inhaler more often. She recently lost her job due to
excessive absences and is in the middle of a divorce settlement. Which of the following
interventions should the asthma educator suggest?
a. Referral to a specialist for treatment.
b. Relaxation and deep breathing exercises.
c. Add antidepressants to the patient’s medication regimen.
d. Start the patient on inhaled corticosteroids.

122. A patient presents to the hospital for an asthma education session. The patient states that this
will be her last visit with the asthma educator because she has recently lost her job and is unable to
afford the visits. She states that she stopped taking her asthma medications because she can’t afford
them. She currently takes Advair 500/50 one puff BID and levalbuterol two puffs prn. All of the
following suggestions are possible ways to help the patient afford her medication EXCEPT:
a. Refer the patient to a social worker.
b. Change the patient’s Advair to separate fluticasone and salmeterol inhalers.
c. Offer the patient medication samples.
d. Continue seeing the patient, but don’t charge her.

123. All of the following statements are true regarding drug abuse in asthma patients EXCEPT:
a. Patients with asthma who use illegal drugs are at a much higher risk for severe asthma
exacerbations.
b. Patients with asthma who use illegal drugs have more frequent asthma exacerbations.
c. Patients with asthma who use illegal drugs are more likely to die of asthma-related
complications.
d. Patients with asthma who use illegal drugs are more likely to have severe comorbidities.

124. Which of the following interventions can help eliminate second- and third-hand smoke
exposure?
a. Room fragrance sprays
b. Odor-neutralizing products
c. Airing out clothes outdoors
d. Washing items or bathing

125. A patient presents to the clinic for her yearly follow-up visit. She tells the asthma educator that
she recently moved into an old home whose primary source of heat is a woodburning stove. The
patient is concerned that this stove will cause her asthma to worsen because smoke is one of her
triggers. What should the asthma educator tell this patient?
a. “The woodburning stove needs to be removed and replaced with a more traditional heat
source.”
b. “Wood smoke is particularly dangerous. See if you can use another fuel type in the stove.”
c. “If you have it serviced and inspected regularly, you can try the stove and see how it goes.”
d. “The stove is highly unlikely to cause smoke to enter the home, but be sure to clean it yearly.”

- 120 -
Copyright © Mometrix Media. You have been licensed one copy of this document for personal use only. Any other reproduction or redistribution is strictly prohibited. All rights reserved.

Licensed to elliot mattison (2146499335, i.astraltears@gmail.com) of 9801 royal ln, dallas, Texas 75231. If this product is distributed to others,
elliot mattison agrees to pay Mometrix the full retail price for each unlicensed recipient. Please play fair and respect the work of our authors.
126. A 56-year-old male and his wife present to the clinic for a follow-up asthma education session.
He states that his asthma has been worse lately since his wife became an essential oils salesperson.
The patient’s wife says that she diffuses oils frequently, and because they are natural products, they
can’t possibly cause harm. She says that many of them can be helpful for chest congestion, cough,
and nasal stuffiness. The patient is allergic to environmental allergens such as trees, pollen, grasses,
and flowers. He also states that room fragrance sprays are one of his triggers. What should the
asthma educator tell this couple?
a. “Essential oils are naturally derived but can cause asthma symptoms similar to those that are
triggered by synthetic fragrances.”
b. “Essential oils are likely not the cause of your asthma; I will refer you to an allergist for more
testing.”
c. “Essential oils are naturally derived but are diluted; therefore, they cannot cause asthma
symptoms like environmental allergens do.”
d. “Essential oils can cause allergic reactions and asthma symptoms, but only if the patient is
allergic to the substance that the oil is derived from.”

127. An asthma educator is called to the bedside of a six-year-old male who is admitted to the
hospital for the second time this month. The patient’s mother is at the bedside. The patient’s
mother states that the boy seems to get better while taking his discharge medications, but then
when the medications run out, he gets worse again. Which of the following options should the
asthma educator investigate FIRST to help determine the cause of the patient’s chronic asthma
symptoms?
a. The patient’s activity level at school
b. The patient’s home environment
c. The patient’s sleep habits
d. The patient’s family medical history

128. An asthma educator is called to evaluate the home environment of a seven-year-old female
with asthma. The asthma educator visits the patient’s home and notices a strong smell of bleach in
the kitchen. There is one pet in the home, which is a small poodle. The patient says that the dog
sleeps in the kitchen. The patient’s mother states that they have had the dog for 10 years and he is
hypoallergenic. The patient has several toys in her room and three stuffed animals on her bed. The
patient’s bedroom also has mini blinds in the windows and hardwood flooring. Which of the
following potential environmental asthma triggers needs to be addressed FIRST?
a. The pet
b. The mini blinds
c. The stuffed animals
d. The bleach

129. Weather-related asthma exacerbations can occur in all of the following situations EXCEPT:
a. Traveling from Chicago to Seattle on an airplane
b. Driving from Phoenix to Orlando
c. Staying indoors during the summer
d. Wearing a scarf over your face in winter

- 121 -
Copyright © Mometrix Media. You have been licensed one copy of this document for personal use only. Any other reproduction or redistribution is strictly prohibited. All rights reserved.

Licensed to elliot mattison (2146499335, i.astraltears@gmail.com) of 9801 royal ln, dallas, Texas 75231. If this product is distributed to others,
elliot mattison agrees to pay Mometrix the full retail price for each unlicensed recipient. Please play fair and respect the work of our authors.
130. When a patient with asthma is traveling on an airplane, all of the following interventions are
recommended EXCEPT:
a. Packing your rescue medication in your carry-on luggage
b. Packing your nebulizer machine in your checked bags
c. Bringing your asthma action plan and peak flow meter
d. Investigating the climate and weather at your destination

131. A 27-year-old female is attending her first asthma education visit. The patient recently had
allergy testing done and was found to be highly sensitive to dust mites. Which of the following dust
mite mitigation interventions is not helpful in reducing a patient’s asthma symptoms?
a. Duct cleaning
b. Removal of carpeting
c. Replacing curtains with blinds
d. Encasing mattress and pillows

132. Which of the following statements is true regarding mold growth and mitigation after a water
leak in the home?
a. Mold must be cleaned with specialized detergents.
b. Water leaks must be dried out within three days.
c. Every home in America has mold.
d. Indoor humidity levels should be less than 70%.

133. All of the following choices are ways to prevent cockroach and pest infestation in the home
EXCEPT:
a. Keep indoor humidity levels below 60%.
b. Store food in sealed containers.
c. Clean up water and food spills immediately.
d. Set traps for pests and insects.

134. An asthma educator is called to educate a seven-year-old female patient and her mother. The
patient is in the second grade and plays soccer in the springtime. She enjoys playing outside with
her friends, and her school attendance is good. She states that her asthma is “ok” most of the time.
Her mother mentions that the patient is highly allergic to pollen, which is making her soccer playing
more difficult. The patient premedicates with albuterol prior to playing and does not have any
exercise-induced asthma symptoms; however, the patient is having allergy symptoms despite
treatment with various allergy medications. The mother states that the patient is on “every
medicine available” and she’s not sure what to do next because the patient’s allergies are keeping
her awake at night. What should the asthma educator suggest?
a. Stay indoors in the springtime.
b. Play soccer in the morning hours.
c. Bathe after playing outside.
d. Wear a mask while playing soccer.

- 122 -
Copyright © Mometrix Media. You have been licensed one copy of this document for personal use only. Any other reproduction or redistribution is strictly prohibited. All rights reserved.

Licensed to elliot mattison (2146499335, i.astraltears@gmail.com) of 9801 royal ln, dallas, Texas 75231. If this product is distributed to others,
elliot mattison agrees to pay Mometrix the full retail price for each unlicensed recipient. Please play fair and respect the work of our authors.
135. A 57-year-old male presents to the clinic for an asthma education session. The patient states
that he has been sick numerous times over the past few months and he can’t seem to get well. He
states that he seems to “catch every cold that’s going around” and it is affecting his asthma control.
What is the most helpful intervention that the asthma educator can suggest for combating this
problem?
a. Practicing hand hygiene
b. Avoiding individuals who are ill
c. Taking vitamins
d. Getting a pneumonia shot

136. A 43-year-old male presents to the clinic with shortness of breath and cough. The patient
states that his asthma has become worse since starting his new job as a landscaper at a golf course.
The patient says that he has noticed increasing asthma symptoms while he was fertilizing the grass
on the green. What intervention should the asthma educator suggest to help the patient avoid this
asthma trigger?
a. Wear a mask at work.
b. Change to a natural fertilizer.
c. Use gloves when working with chemicals.
d. Bathe after using fertilizer.

137. An asthma educator is called to conduct a follow-up asthma education session on an inpatient.
The patient is admitted for pneumonia but has an underlying history of asthma. The patient states
that her asthma is not as well controlled as it used to be. The patient moved to a new home and had
increasing asthma symptoms since that time. Her new home is near wetlands, and she says she likes
to keep her windows open at night so she can hear the sounds of nature. The patient has several
environmental allergies and purchased an air cleaner for her bedroom; but she states that it is not
helping her asthma. The air cleaner has a high-efficiency particulate air (HEPA) filter, which the
patient changes monthly, and she leaves the air cleaner running continuously. What intervention
would help this patient to better control her environmental allergies?
a. Change the HEPA filter weekly.
b. Run the air purifier only at night.
c. Keep her windows closed at night.
d. Put a HEPA filter on her furnace.

138. An asthma educator is called to educate an inpatient on her asthma. The patient states that she
is highly allergic to dust and she has renovated her home to be more allergy friendly. The patient
states she removed the wall-to-wall carpeting in her home and replaced it with area rugs that she
vacuums weekly. She empties her canister vacuum into the kitchen garbage and then immediately
takes it outside for disposal. She uses mattress and pillow encasements to prevent dust mites in her
bedding and uses air cleaners throughout her home. What action would help this patient further
prevent dust mites in her home?
a. Change window draperies to blinds.
b. Vacuum twice weekly with a HEPA filter.
c. Dust twice weekly with a feather duster.
d. Empty the vacuum outside.

- 123 -
Copyright © Mometrix Media. You have been licensed one copy of this document for personal use only. Any other reproduction or redistribution is strictly prohibited. All rights reserved.

Licensed to elliot mattison (2146499335, i.astraltears@gmail.com) of 9801 royal ln, dallas, Texas 75231. If this product is distributed to others,
elliot mattison agrees to pay Mometrix the full retail price for each unlicensed recipient. Please play fair and respect the work of our authors.
139. Which of the following statements is true regarding home visits in asthma education
programs?
a. Home visits often aren’t needed because patients can assess their own environment.
b. Home visits are expensive and are therefore not recommended.
c. Home visits are no longer conducted due to safety issues.
d. Home visits are important for the assessment of environmental asthma triggers.

140. All of the following components are found in a written asthma action plan EXCEPT:
a. Medications
b. Symptoms
c. Peak flow readings
d. Allergies

141. Which of the following statements is true regarding asthma action plans?
a. Asthma action plans are all identical except for medications.
b. Only patients with severe asthma need a written action plan.
c. Asthma action plans are most helpful for good perceivers.
d. Asthma action plans tell a patient when to call for help.

142. How often should written asthma action plans be reassessed?


a. Yearly
b. Monthly
c. During every office visit
d. Only when the patient is poorly controlled

143. A patient’s follow-up visit with an asthma educator should last how long?
a. No more than 30 minutes.
b. At least one hour.
c. It depends on the Current Procedural Terminology (CPT) code’s requirements.
d. The patient’s needs dictate the time requirement.

144. A six-year-old patient has just completed her first asthma education session. The patient
formulated her asthma action plan along with her mother and asthma educator. Where should the
patient’s asthma action plan be stored?
a. At home
b. At school
c. At the doctor’s office
d. All of the above

145. Which of the following choices is not a component of a follow-up visit with an asthma
educator?
a. Assessing control
b. Assessing perception
c. Assessing finances
d. Assessing adherence

- 124 -
Copyright © Mometrix Media. You have been licensed one copy of this document for personal use only. Any other reproduction or redistribution is strictly prohibited. All rights reserved.

Licensed to elliot mattison (2146499335, i.astraltears@gmail.com) of 9801 royal ln, dallas, Texas 75231. If this product is distributed to others,
elliot mattison agrees to pay Mometrix the full retail price for each unlicensed recipient. Please play fair and respect the work of our authors.
146. Which of the following statements is true regarding peak flow meters for asthma management
at home?
a. All patients benefit from referring to numeric or graphic data for asthma management.
b. Peak flow monitoring is optional.
c. Patients younger than the age of six are physically unable to perform peak flows.
d. Peak flow monitoring is not necessary in the elderly population.

147. A 33-year-old female presents to the emergency department in respiratory distress. The
patient is treated in the emergency department and is transferred to the general medical floor. The
asthma educator is called to the patient’s bedside to conduct an asthma education session. The
patient states that she has had asthma all her life and it has always been unmanageable. She states
that she has daily asthma symptoms with weekly nighttime symptoms. She is unaware of anything
that makes her asthma better, aside from taking her rescue inhaler with valved holding chamber.
What action should the asthma educator suggest FIRST?
a. Start an asthma diary.
b. Begin peak flow measurements.
c. Add a leukotriene modifier.
d. Suggest environmental controls.

148. An asthma educator is attempting to design a community-based asthma education program.


Which of the following indicators can help determine the need for an outpatient asthma education
program in a community?
a. Patient-reported symptom severity and frequency
b. Asthma hospitalization rates
c. Incidence and prevalence of asthma
d. All of the above

149. Which of the following options would be a helpful partnership in the development of an
asthma education program?
a. Disease-based charitable organizations
b. Asthma drug corporations
c. Self-assessment tool developers
d. None of the above

150. All of the following options are appropriate sources of funding for an asthma education
program EXCEPT:
a. CPT code usage
b. Grants
c. Facility-approved budgets
d. Patient fees

151. An asthma educator is designing an asthma education program for inner-city patients. Funds
are limited for this endeavor, but the asthma educator has applied for several grants to help fund
the program. Which of the following program components is most important for the asthma
education program serving an inner-city patient population?
a. Offering samples of medication and tools
b. Offering outpatient follow-up sessions
c. Offering eighth-grade reading level materials
d. Offering inpatient education sessions

- 125 -
Copyright © Mometrix Media. You have been licensed one copy of this document for personal use only. Any other reproduction or redistribution is strictly prohibited. All rights reserved.

Licensed to elliot mattison (2146499335, i.astraltears@gmail.com) of 9801 royal ln, dallas, Texas 75231. If this product is distributed to others,
elliot mattison agrees to pay Mometrix the full retail price for each unlicensed recipient. Please play fair and respect the work of our authors.
152. What is the first step in setting asthma program goals when designing an asthma education
program for a facility or community?
a. Secure funds for the program’s implementation.
b. Identify the needs of the patient population served.
c. Set a budget for expenditures such as tools and written materials.
d. Obtain qualified staff members to teach the program.

153. An asthma educator is choosing materials for her asthma education program. The educator
implemented an inpatient asthma education program that is offered in every inpatient unit of the
hospital. The booklets are written at a fifth-grade reading level and feature numerous colored
photographs, charts, and graphs. The booklets were ordered from an asthma-related charitable
organization at no cost to the educator. The educator unpacks the boxes of booklets, starts stocking
the hospital units with the pamphlets, and adds spacers and peak flow meters to the equipment
carts. What did the asthma educator do wrong in this vignette?
a. The educator should not have ordered the books from a charitable organization because that
can cause bias.
b. The educator ordered booklets that were written at too low of a reading level.
c. The educator did not review the booklets before integrating them into the program.
d. The educator should not stock peak flow meters and spacers because they are costly tools.

154. Which of the following individuals should NOT be present for asthma education when
educating an inpatient?
a. The patient’s nurse
b. The patient’s physician
c. The patient’s social worker
d. The patient’s family

155. Which of the following types of asthma education sessions require comprehensive attendance
and topical records kept by the asthma educator?
a. Inpatient
b. Outpatient
c. Group sessions
d. All of the above

156. A 17-year-old female presents to the clinic for her six-month follow-up visit to the asthma
educator. The patient mentions that she will be attending the prom this spring and she is not sure
how she is going to be able to carry her valved holding chamber with her because her evening bag
is too small. She states this is a common problem for her and she sometimes just leaves her inhaler
and spacer at home for special occasions. All of the following choices are appropriate options to
combat this issue EXCEPT:
a. Consider changing to a breath-actuated metered dose inhaler.
b. Offer the patient a collapsible spacer.
c. Tell the patient to leave the spacer in her car at the event.
d. Tell the patient to carry her spacer in her date’s jacket pocket.

- 126 -
Copyright © Mometrix Media. You have been licensed one copy of this document for personal use only. Any other reproduction or redistribution is strictly prohibited. All rights reserved.

Licensed to elliot mattison (2146499335, i.astraltears@gmail.com) of 9801 royal ln, dallas, Texas 75231. If this product is distributed to others,
elliot mattison agrees to pay Mometrix the full retail price for each unlicensed recipient. Please play fair and respect the work of our authors.
157. All of the following are barriers to successful asthma control EXCEPT:
a. Lack of adherence
b. Severity of asthma exacerbations
c. Medication side effects
d. Skipping asthma diary entries

158. When measuring patient outcomes for a group outpatient asthma education program, all of the
following criteria must be considered EXCEPT:
a. Patient-perceived symptoms
b. Hospital admission rate
c. Program attendance
d. Emergency department visit incidence

159. An asthma educator practices education in an independent community hospital. The educator
wants to expand her current asthma education program. Which of the following choices would be
an appropriate partnership for this asthma educator in order to expand her program?
a. A local parochial school
b. A pharmaceutical representative
c. A valved holding chamber company
d. A large hospital corporation

160. Which of the following choices is not a part of normal lung anatomy?
a. Trachea
b. Alveoli
c. Bronchioles
d. Mainstem bronchus

161. What is the function of the alveoli during respiration?


a. Gas exchange
b. Perfusion
c. Air flow
d. All of the above

162. Which of the following choices is not an indicator of a patient’s asthma-related risk?
a. Frequency of exacerbations
b. Reduction of lung function
c. Unpleasant medication side effects
d. Inability to perform daily tasks

163. A patient presents to the clinic for his yearly pulmonary function test. The patient begins the
forced vital capacity maneuver by first inhaling completely, then exhaling forcefully until he is
unable to exhale any further. Finally, he inhales slowly and evenly. What part of this process was
performed incorrectly?
a. The patient should have inhaled quickly until he was at a maximum inhalation.
b. The patient should have exhaled completely prior to beginning the maneuver.
c. The patient should have exhaled slowly and evenly.
d. The patient should have inhaled for exactly three seconds at the end of the maneuver.

- 127 -
Copyright © Mometrix Media. You have been licensed one copy of this document for personal use only. Any other reproduction or redistribution is strictly prohibited. All rights reserved.

Licensed to elliot mattison (2146499335, i.astraltears@gmail.com) of 9801 royal ln, dallas, Texas 75231. If this product is distributed to others,
elliot mattison agrees to pay Mometrix the full retail price for each unlicensed recipient. Please play fair and respect the work of our authors.
164. Which of the following choices is not a common sign of respiratory distress in an infant or
small child?
a. Tracheal tugging
b. Intercostal retractions
c. Tripod sitting
d. Nasal flaring

165. Which of the following choices best describes an immunoglobulin E (IgE)-mediated allergic
reaction?
a. IgE bonds to an allergen and causes the release of histamine from mast cells.
b. Mast cells bond to an allergen and cause the release of histamine.
c. IgE is released into the bloodstream, starting the allergic cascade.
d. Leukotrienes bond to an allergen and cause the release of histamine.

166. Which of the following statements is true regarding asthma exacerbations due to allergen
exposure?
a. Only IgE-mediated allergic reactions cause allergy symptoms.
b. Allergens act as airway irritants causing asthma exacerbations.
c. Asthma exacerbations are rarely caused by allergen exposure.
d. Allergy exposure can cause delayed onset of asthma symptoms.

167. A 56-year-old male presents to the emergency department with shortness of breath after
eating lunch. The patient has a history of asthma that is generally well controlled. The patient states
that he has dined at this deli before and always orders the same Italian cold cut sandwich and glass
of red wine. He states that he has noticed shortness of breath after eating there before, but it was
never this severe. What is the likely cause of this patient’s shortness of breath?
a. Food allergy
b. Sulfite exposure
c. Tannin ingestion
d. Cross contamination

168. Which of the following statements is true regarding diet and asthma?
a. High levels of vitamin D are linked to the development of asthma.
b. A healthy diet can reduce a patient’s risk for developing asthma.
c. Obesity can cause more frequent and severe asthma exacerbations.
d. Food allergies are directly related to the development of asthma.

169. All of the following statements are true regarding asthma and respiratory infections EXCEPT:
a. The respiratory infection’s inflammatory response triggers an asthma exacerbation.
b. Patients with asthma are more susceptible to respiratory infections than are healthy patients.
c. Infections trigger a cyclical pattern of asthma exacerbations.
d. Patients with asthma require specialized infection prevention interventions.

170. Which of the following choices is not an asthma trigger associated with surgery?
a. Anesthesia gases
b. Preoperative spirometry
c. Conscious sedation
d. Intubation

- 128 -
Copyright © Mometrix Media. You have been licensed one copy of this document for personal use only. Any other reproduction or redistribution is strictly prohibited. All rights reserved.

Licensed to elliot mattison (2146499335, i.astraltears@gmail.com) of 9801 royal ln, dallas, Texas 75231. If this product is distributed to others,
elliot mattison agrees to pay Mometrix the full retail price for each unlicensed recipient. Please play fair and respect the work of our authors.
.

171. All of the following choices are signs of mold growth on a home’s drywall EXCEPT:
a. Wet spots
b. Chalky residue
c. Black specks
d. Water staining

172. A 13-year-old male with a history of asthma presents to the emergency room with shortness of
breath. His respiratory rate is 22, and breath sounds are slightly diminished. His oxygen saturation
is 97%. He says his chest feels tight and he can’t breathe. His mother states that she thinks he is
faking this exacerbation because he has an important math test at school this afternoon and that he
is trying to avoid taking the test today. What action should the clinician take FIRST?
a. Observe the patient for 10 minutes to determine his level of impairment.
b. Give the patient a saline nebulizer to see if his condition improves after treatment.
c. Give the patient an albuterol nebulizer to treat bronchospasm.
d. Send the patient home with an albuterol metered dose inhaler.

173. Which of the following choices is not a common classroom asthma trigger?
a. Chalk dust
b. Varnishes
c. Ozone
d. Pests

174. A 33-year-old female presents to the clinic with increasing severity of asthma symptoms
during working hours. She states that she has had comprehensive allergy testing done in the past,
and she was found to be allergic to cats, horses, mold, pollen, ragweed, and dust. She works as a dog
groomer, and she states that she noticed her asthma has become worse since starting her career.
Which of the following asthma triggers is NOT a possible cause of her increasing symptoms?
a. Dog dander
b. Fragrances
c. Outdoor allergens
d. Cleansers

175. Which of the following spirometry results is consistent with a diagnosis of vocal cord `

dysfunction?
a. Scooped out flow volume loop
b. Flattened flow volume loop
c. Increased FEV1
d. Decreased tidal volume

- 129 -
Copyright © Mometrix Media. You have been licensed one copy of this document for personal use only. Any other reproduction or redistribution is strictly prohibited. All rights reserved.

Licensed to elliot mattison (2146499335, i.astraltears@gmail.com) of 9801 royal ln, dallas, Texas 75231. If this product is distributed to others,
elliot mattison agrees to pay Mometrix the full retail price for each unlicensed recipient. Please play fair and respect the work of our authors.
Answers and Explanations
1. A: The three components of an asthma exacerbation are inflammation, bronchoconstriction, and
mucus production.

2. C: Although all the response choices are asthma triggers, leaf smoke and air fresheners are
inhaled irritants. The other choices are allergens.

3. C: Short-acting bronchodilators work on the smooth muscle lining of the airways by relaxing the
muscles and opening the bronchioles.

4. B: Early morning symptoms of nasal congestion, sneezing, and/or cough are often caused by
exposure to dust mites in the pillow and/or bedding. Patients are exposed to these allergens
throughout the night, causing early morning symptoms.

5. A: The Advair Diskus comes in three dosages: 500/50, 250/50, and 100/50.

6. A: The patient is currently taking the lowest possible Advair dosage. Increasing the dose to
250/50, which is the moderate dose, will likely improve his underlying inflammation and
bronchoconstriction. Substituting albuterol for levalbuterol will make no difference because these
medications are similar in composition. Changing from a combination inhaler such as Advair to a
single corticosteroid inhaler such as budesonide will likely make the patient’s symptoms worse.
Finally, cromolyn sodium is not indicated in this vignette.

7. C: When performing peak flows, the patient must take a maximum inhalation and blow hard and
fast into the peak flow meter. Then the patient repeats the steps three times and records the best
(highest) number of the three tries.

8. C: This patient has a normal physical exam and is only having asthma symptoms during exercise.
Because she states that her albuterol previously treated her exercise-induced asthma symptoms
successfully, the clinician should first ask the patient when she last refilled her inhaler. Patients can
shake an inhaler and assume that because they feel something moving inside, then it must be full.
However, the propellant is the last to leave the inhaler, and there may be no medication inside at all.

9. B: This patient is likely having an asthma episode secondary to outdoor allergen exposure. The
patient is afebrile, with clear nasal mucus, which indicates that an infection is not likely. These
symptoms may indicate an allergic exposure, but the source is likely not the patient’s cat because
these are new symptoms that just began after visiting the pumpkin farm. Playing in fallen leaves,
going on hayrides, and other outdoor activities can cause allergic rhinitis and subsequent asthma
exacerbation due to exposure to outdoor allergens such as mold.

10. A: Xolair is approved for the treatment of severe allergic asthma in some patients who do not
respond to traditional therapies. Patients with a history of tumors or cancer should not use this
drug because it can cause malignant tumors in some populations.

11. D: When conducting asthma education, a conversational approach is best. Asking open-ended
questions and talking with a patient in a less formal manner can help to build a relationship with
the patient and reveal clues as to the patient’s overall health, asthma triggers, and asthma control.
Although this patient is eager to go home, the educator should never just drop off handouts and
leave. This is unacceptable because comprehensive asthma education is characterized by the
exchange of information between the educator and the patient.

- 130 -
Copyright © Mometrix Media. You have been licensed one copy of this document for personal use only. Any other reproduction or redistribution is strictly prohibited. All rights reserved.

Licensed to elliot mattison (2146499335, i.astraltears@gmail.com) of 9801 royal ln, dallas, Texas 75231. If this product is distributed to others,
elliot mattison agrees to pay Mometrix the full retail price for each unlicensed recipient. Please play fair and respect the work of our authors.
12. C: Although the patient has a family history of asthma, she does not have a history of asthma
herself. In fact, this is the first episode of respiratory illness she has ever had. Although this may be
the first in many asthmatic episodes, there is no pattern of symptoms to make that determination at
this time. Reactive airway disease is also a cyclical respiratory disorder, which is similar to asthma.
Often, patients with cyclical respiratory symptoms are diagnosed with reactive airway disease
before age five, when an asthma diagnosis is commonly reached. Additionally, there are no chest
films or other diagnostic test results to suggest a diagnosis of pneumonia.

13. A: Permanent airway remodeling is the result of chronic, poorly controlled asthma. These
changes are irreversible and mimic chronic obstructive pulmonary disease (COPD).

14. D: A patient’s asthma severity is categorized by measuring his or her impairment, risk, and level
of control. Although support is a necessary component of successful asthma management, it is not
one of the criteria used to measure severity.

15. B: The clinician should expect this patient to have a “scooped out” flow volume loop pattern
with reduced flows and normal volumes. Patients with asthma often have air trapping due to a
reduced ability to exhale completely during an exacerbation. This patient is currently experiencing
asthma symptoms, so it is expected that his flow volume loop will look like an obstructive asthmatic
pattern.

16. D: Although this patient’s asthma is well controlled, it is likely well controlled due to his current
medication regimen. Reducing Advair to once daily is not recommended because this medication
has a 12-hour duration of efficacy. In addition, discontinuing short-acting bronchodilators is never
recommended because this is the patient’s rescue medication for emergencies.

17. B: Changes in environment, new allergies, and infections can all affect a patient’s asthma.
Although some patients have infrequent asthma exacerbations, it is important to reiterate that
these patients must always be prepared for the possibility of an exacerbation. Patients do not
outgrow asthma. Asthma changes over time, but the underlying disease process is always there.

18. C: All of the choices are common signs of an asthma episode except dizziness. Although some
patients can become dizzy during hyperventilation or while experiencing dyspnea, dizziness is not
a common asthma symptom.

19. B: Allergens can be avoided either by putting a barrier between you and the allergen or
completely removing the allergen from your environment. Allergens are not irritants. Irritants are
chemicals that cause airway inflammation and bronchoconstriction. Irritants can cause symptoms
that are similar to those of an allergic reaction, such as nasal congestion, watery eyes, and runny
nose. Irritants can also be natural substances such as leaf or wood smoke.

20. A: Triggers vary widely between patients with asthma. Some patients find that animal dander
triggers their asthma, whereas other patients have furry pets with no change in their frequency or
severity of symptoms. Dust mites are a common asthma symptom, but they do not affect all asthma
patients. Inhaled irritants are another common asthma trigger, but they do not affect all patients
with asthma. Option D is incorrect because mold is harmful for all patients with asthma, not some of
them. Finally, option A is the correct answer because leaf and wood smoke are carcinogenic and
harmful for ALL people, not just patients with asthma.

21. A: Although all of the choices are correct, the most important action to take is to smoke outside
the home until he is able to quit. The patient’s home should be free of asthma triggers, and
secondhand smoke is a common asthma trigger. The patient spends most of his hours in the home,

- 131 -
Copyright © Mometrix Media. You have been licensed one copy of this document for personal use only. Any other reproduction or redistribution is strictly prohibited. All rights reserved.

Licensed to elliot mattison (2146499335, i.astraltears@gmail.com) of 9801 royal ln, dallas, Texas 75231. If this product is distributed to others,
elliot mattison agrees to pay Mometrix the full retail price for each unlicensed recipient. Please play fair and respect the work of our authors.
especially at night when he is sleeping. Therefore, the interior of the home should be the first
location where trigger control measures take place.

22. B: The likely cause of the patient’s sudden increase in asthma symptoms is the ozone from the
printer. Printers often emit ozone and other fumes, which can trigger asthma and cause
exacerbations.

23. A: Asthma and allergies are protected under the Americans with Disabilities Act. Workplaces
and schools must make reasonable accommodations for patients with asthma.

24. C: This patient has been a swimmer for his entire life, so exercise-induced asthma is not a likely
cause of his asthma symptoms. In addition, there is no indication that the cats and dogs are new
additions to his home, so they are not the likely cause of his symptoms. He states that the asthma
symptoms became worse four weeks ago, which is when he began his new medication regimen.
Although beta blockers are a common asthma trigger, cardioselective ones are not. Therefore, the
likely cause of his asthma symptoms is the use of NSAIDs, which is a common asthma trigger.

25. C: Because this patient wakes up with allergy symptoms, she is likely breathing in dust mite
droppings from inside her bedding and pillow. Enclosing the mattress and pillow with allergen
encasements is an inexpensive and impactful way to combat dust mite exposure. Removing stuffed
animals from a child’s room is never recommended; these toys can be cleaned and/or rotated to
allow the child to keep their comfort items while limiting dust mite exposure. Replacing carpeting
with hardwood floors is a very expensive process and should not be a first-line defense for dust
mite exposure. And finally, although vacuuming the patient’s room will help with dust mite
exposure, enclosing the patient’s pillows is likely to have a more dramatic and impactful effect on
the patient’s asthma and allergy symptoms.

26. D: Pet saliva and dander are common allergens for patients who are sensitive to furry animals.
Pet dander flakes off and becomes airborne, traveling through ductwork in homes and spreading
throughout the building. Pet dander cannot be restricted to one area of the home. Hypoallergenic
pets are considered to be lower allergy risk pets, but they still produce dander and saliva that can
cause symptoms in sensitive patients. And finally, pet dander is an inhaled allergen, not an irritant.

27. B: Dust mites are allergens, not inhaled irritants. Any scented substance, natural or artificial, is
an inhaled irritant. Bleach and other cleansers are very irritating to the airways and can cause
severe asthma exacerbations.

28. C: The best choice out of the options given is to premedicate with cromolyn sodium. This is an
alternative to the more common recommendation of premedicating with a short-acting
bronchodilator, as stated in the stepwise approach to asthma management. Long-acting
bronchodilators are maintenance medications and should never be used as a rescue inhaler for
premedication prior to activity. Leukotriene modifiers are not helpful in this case because this
patient has no known allergic history. In addition, adding a low-dose inhaled corticosteroid is not
necessary if the patient is not having any symptoms outside of exercise and exertion.

29. A: This patient may have vocal cord dysfunction and should be assessed immediately. Vocal
cord dysfunction mimics a refractory asthma exacerbation that is unresponsive to traditional
therapy and features additional symptoms such as raspy voice and throat irritation. Option B is
incorrect because this patient is not in acute or impending respiratory failure and, therefore,
intubation is not necessary. Administering racemic epinephrine may be helpful if the patient was
diagnosed with croup, but there is no mention of stridor in this patient’s assessment. Finally,

- 132 -
Copyright © Mometrix Media. You have been licensed one copy of this document for personal use only. Any other reproduction or redistribution is strictly prohibited. All rights reserved.

Licensed to elliot mattison (2146499335, i.astraltears@gmail.com) of 9801 royal ln, dallas, Texas 75231. If this product is distributed to others,
elliot mattison agrees to pay Mometrix the full retail price for each unlicensed recipient. Please play fair and respect the work of our authors.
administering a saline nebulizer is never indicated because it can trigger life-threatening
bronchospasm in patients with asthma.

30. D: All of the above statements are true regarding pregnancy and asthma. Some pregnant
patients with asthma will notice their symptoms improving, whereas others will notice no change
at all. Still others will experience an increase in the severity and frequency of asthma symptoms.
Asthma in pregnancy is unpredictable.

31. D: Allergic rhinitis, obstructive sleep apnea, and GERD are all comorbidities associated with
asthma. Vocal cord dysfunction is a condition that mimics asthma, but it is unrelated.

32. D: Although asthma can cause irreversible damage if left untreated or poorly controlled over
time, it does not turn into COPD. COPD, like asthma, is associated with reduced expiratory flow and
normal or above-average lung volumes. COPD is irreversible.

33. A: Asking open-ended questions is the most important skill an asthma educator can use during
an education session. Leading with a broad, open-ended statement such as “Tell me about your
asthma” is a simple and effective way to engage a patient in the education session. Asking specific
questions about wheezing frequency and home environment can come later in the education
session after a baseline of information is established. When asked to talk about their asthma, the
patient will often discuss the issues of perceived importance first, which is what the asthma
educator should focus on. This is the first step in setting patient-driven goals for asthma education.

34. A: It is likely that this patient’s asthma symptoms are improving and that the reduction in
airway inflammation and bronchoconstriction is allowing her to achieve higher peak flow numbers.
It is unlikely that this adult patient has grown physically in the six months since her asthma action
plan was designed, although growth can cause increased peak flow meter values in growing
children. In addition, if this patient’s asthma action plan was based on her predicted peak flow
values, the asthma action plan’s numbers would be HIGHER than what she is able to achieve. This is
because predicted values are based on healthy individuals who do not have asthma.

35. B: Options C and D are not age-appropriate questions for a four-year-old. Option A would be a
correct choice if the language was different. Asthma exacerbations should NEVER be described as
an “attack” because this terminology instills fear in patients. The terms “episode” or “event” can be
used instead. Option B is the correct answer because the statement is open ended, age appropriate,
and will provide the asthma educator with information on the patient’s perception of symptoms.

36. D: To measure quality of life as it relates to asthma management, the asthma educator should
ask broad questions about the patient’s daily activities and hobbies. Asking the patient about
hobbies and activities can provide valuable information as to how often the patient participates in
these activities and how their asthma affects them. Options A and C are not correct because many
elderly patients may not participate in fitness activities or do their own shopping. Option B is
inappropriate because this question can contribute to feelings of loneliness or isolation, or it simply
might not be applicable to this patient.

37. A: Subcostal and intercostal retractions are a sign of respiratory distress in small children.
Accessory muscle use and tripod sitting are commonly seen in adults and teenagers because these
patients have the strength and dexterity to prop themselves up to change position when short of
breath. An increased respiratory rate can be a sign of respiratory distress, but children have a
higher respiratory rate than adults and breathe more rapidly at rest; therefore, an increased
respiratory rate is not always a clear sign of respiratory distress.

- 133 -
Copyright © Mometrix Media. You have been licensed one copy of this document for personal use only. Any other reproduction or redistribution is strictly prohibited. All rights reserved.

Licensed to elliot mattison (2146499335, i.astraltears@gmail.com) of 9801 royal ln, dallas, Texas 75231. If this product is distributed to others,
elliot mattison agrees to pay Mometrix the full retail price for each unlicensed recipient. Please play fair and respect the work of our authors.
38. B: The inability of speak in full sentences is a common sign of a severe asthma exacerbation in
an adult patient. Nasal flaring is commonly seen in infants and very young children in respiratory
distress. Decreased respiratory rate and disorientation are not commonly associated with an
asthma exacerbation.

39. A: Option A is correct because this patient is at a high risk for asthma-related death due to his
previous history of multiple asthma-related intubations. This history also puts him at a high risk for
future ventilator dependency and hospitalization. Hypercapnia is not a common condition
associated with asthma.

40. D: The overuse of bronchodilators is associated with an increased risk of a future cardiac event,
increased bronchodilator tolerance, and asthma-related death. This patient is not using her
medication as directed because an every-one-hour medication schedule is never prescribed due to
the dangers associated with this frequency of use.

41. D: A localized thrush infection is commonly associated with inhaled corticosteroids. Some
patients contract thrush infections in the mouth and throat due to inhaling the corticosteroids
directly into the lungs. Mood swings, Cushing’s disease, and increased appetite are all associated
with systemic corticosteroids, not inhaled corticosteroids.

42. B: A poor perceiver is a patient who is unable to accurately measure the severity or frequency of
their asthma symptoms.

43. D: A change in environment, such as with a new home or workplace, and weather-related
changes can contribute to a sudden loss of asthma control. Weight loss can help to increase a
patient’s level of asthma control because obesity is a common comorbidity for asthma.

44. A: Although all of the above choices can help an asthma educator to determine a patient’s level
of asthma control, the BEST option is to interview the patient. This allows the asthma educator to
obtain comprehensive information regarding past medical history, history of past exacerbations,
asthma triggers, environmental factors, and other important information. The patient interview is a
comprehensive source for asthma-related information.

45. C: Referral to a pulmonologist is warranted when a patient has a history of frequent


hospitalizations and intubations, inability to perform daily activities, and frequent asthma
symptoms. Increasing the therapeutic dosage of medication is not a reason to refer to a
pulmonologist because a simple change in dosage may increase a patient’s level of asthma control
and resolve their refractory asthma symptoms.

46. B: Premedicating with a short-acting bronchodilator is an appropriate course of action for a


patient with exercise-induced asthma symptoms. Removing a patient from gym class is not
recommended because a lack of exercise can cause weight gain and obesity, which is a common
asthma comorbidity. Patients should not continue with activity when having asthma symptoms, so
option C is not correct. Inhaled corticosteroids are not indicated for exercise-induced asthma and
are not designed for the quick relief of asthma symptoms.

47. A: Exercise-induced asthma can be present with any type of physical exertion and is not
associated exclusively with athletes or sports activities. Taking the stairs, carrying heavy objects,
and other sources of physical exertion are exercise-induced asthma triggers. Exercise-induced
asthma can be a stand-alone asthma diagnosis, in which the patient only has asthma symptoms
during exertion. It can be present in any patient population and is not limited to just adults and
adolescents.

- 134 -
Copyright © Mometrix Media. You have been licensed one copy of this document for personal use only. Any other reproduction or redistribution is strictly prohibited. All rights reserved.

Licensed to elliot mattison (2146499335, i.astraltears@gmail.com) of 9801 royal ln, dallas, Texas 75231. If this product is distributed to others,
elliot mattison agrees to pay Mometrix the full retail price for each unlicensed recipient. Please play fair and respect the work of our authors.
48. B: This patient is complaining of classic sinusitis symptoms such as sinus pain and pressure,
sore throat, and tooth pain. The patient does not need to further investigate the tooth pain with a
dentist because this is a classic sinusitis symptom. The postnasal drip associated with sinusitis
often causes throat pain as well as an increased frequency and severity of asthma symptoms
because the postnasal drip irritates the airways, causing inflammation and bronchoconstriction.
There is no evidence of nasal polyps in this patient, aside from postnasal drip, which is also a sign of
simple sinusitis, so option A is incorrect. Although allergies are a common trigger for sinusitis, there
is no evidence of allergies in this patient; therefore, option D is incorrect.

49. D: Obese patients are at a high risk for GERD-related asthma symptoms due to increased body
weight that puts pressure on the esophageal sphincter, allowing stomach acids to rise up and
irritate the airways. Obesity can cause asthma due to the inflammatory process associated with
increased adipose tissue in the body. Patients with GERD may be completely unaware of any
symptoms, and many do not experience heartburn. GERD is not position dependent, and it can
cause symptoms even while a patient is sitting upright or standing.

50. A: Obstructive sleep apnea is defined as a blockage of the airway during sleep that causes an
interruption in normal breathing pattern. The obstruction of the airway, either by the tongue falling
back in the oral cavity or compression due to excess body weight, stimulates the vagal nerve, which
causes bronchoconstriction.

51. C: Over-the-counter asthma medications are extremely dangerous and can have deadly
consequences for patients with asthma. The ingredients in these medications often include
ephedrine and other stimulants, which can have serious cardiac side effects, including death. Over-
the-counter asthma medications are never recommended in any form.

52. C: Herbal and holistic remedies can be used in tandem with traditional medications if the herbal
medications are not causing dangerous or unwanted side effects. Patients should always be warned
of any potential side effects including allergic reactions (all herbal remedies are derived from a
plant, flower, or other natural products that can be seasonal allergy triggers).

53. D: Food-based asthma triggers include sulfites, preservatives, additives, artificial coloring, and
other chemicals.

54. C: Alternative medicine therapies for the treatment of asthma can include cupping, coining,
acupuncture, and other interventions. These therapies put a patient at risk for bloodborne illness,
tissue damage, piercing of internal organs, pain, and bruising. Bronchospasm is not a common side
effect for these interventions.

55. D: Asthma is a hereditary disease that can run in families. Understanding the family’s history of
asthma can help to predict the patient’s level of asthma risk. Although this information does not
directly predict the severity or frequency of a patient’s symptoms, it can help the clinician to
anticipate the patient’s needs and the presence of different subtypes of asthma within the family.

56. C: Neither radioallergosorbent testing (RAST) nor skin prick testing is completely accurate.
Allergists recommend a combination of RAST and skin prick testing to confirm a patient’s allergies.
Saliva testing is an alternative allergy testing option that is not clinically proven to be effective or
accurate.

57. A: All patients with asthma should undergo pulmonary function testing at least once per year to
track any changes in the patient’s pulmonary status and to compare current trends to past results.
Arterial blood gas monitoring is an invasive and unnecessary test for a routine clinic appointment.

- 135 -
Copyright © Mometrix Media. You have been licensed one copy of this document for personal use only. Any other reproduction or redistribution is strictly prohibited. All rights reserved.

Licensed to elliot mattison (2146499335, i.astraltears@gmail.com) of 9801 royal ln, dallas, Texas 75231. If this product is distributed to others,
elliot mattison agrees to pay Mometrix the full retail price for each unlicensed recipient. Please play fair and respect the work of our authors.
A chest x-ray is also not indicated because this patient has no symptoms of pneumonia or other
respiratory disease. In addition, pulse oximetry testing is not indicated because this patient is
asymptomatic at this time.

58. D: Pulmonary function testing is a snapshot in time. It can only measure values based on how a
patient is feeling at that exact moment in time. It is not able to predict future exacerbations or show
any information associated with previous exacerbations.

59. A: Although many patients have wheezing during an asthma exacerbation, some patients have
completely normal breath sounds and only cough during asthma episodes. For some patients,
coughing is their only symptom; unfortunately, this can lead to misdiagnosis. Asthma patients do
not have stridorous breath sounds. Stridor is associated with upper airway obstructions such as
epiglottitis and croup. Rhonchi may be present in some asthma patients, but it is not a common
breath sound in asthma exacerbations.

60. D: A silent chest, where no breath sounds are heard, is the most dangerous type of breath
sounds associated with an asthma exacerbation. A silent chest refers to lack of air movement;
therefore, it is considered life threatening.

61. A: If a patient can’t speak in full sentences and must stop to breathe in between words, the
patient is in respiratory distress and the clinician should request emergency services immediately.
Diffuse wheezing is not considered a medical emergency on its own. Pneumonia is also not
necessarily a medical emergency because it can be treated at home in some patients. Comorbidities,
although a complication to asthma control, are not a medical emergency.

62. B: Environmental controls are an effective way to treat asthma, but it is not considered a self-
assessment tool. Self-assessment tools, such as peak flow monitoring, the Asthma Control Test, and
keeping an asthma diary, help the patient to measure their own asthma control as well as watch for
patterns of changes in their asthma symptoms.

63. B: Correct peak flow zones include the green zone (80%–100%), yellow zone (50%–79%), and
red zone (0%–49%).

64. B: To confirm adequate response to bronchodilation during a pulmonary function test, one of
the following three criteria must be met: the patient’s FEV1 or FVC needs to increase by at least
12% or the patient’s FEF25–75 percent change needs to increase by at least 25%.

65. B: This patient’s pulse oximeter is measuring her oxygen saturation at only 81%, and her heart
rate is 77. But when the asthma educator takes her pulse, the patient’s heart rate is 91. This
discrepancy indicates that the patient’s pulse oximeter is not correlating. This is likely due to the
patient’s cold hands, which results in poor perfusion. There is no need to call for help because this
patient is resting comfortably and is in no distress. Oxygen is also not indicated because this
patient’s oxygen saturation of 81% is not accurate. A nebulizer treatment would not be helpful in
this clinical situation.

66. B: Nitric oxide is measured on exhalation to determine the degree of airway inflammation in a
patient with asthma. This value has no relationship to airway reversibility. Answers A and C are
referring to heliox, not nitric oxide. Heliox is a combination of helium and oxygen that is
administered to some patients with severe asthma exacerbations. This is unrelated to nitric oxide,
which is measured as part of a pulmonary function test.

- 136 -
Copyright © Mometrix Media. You have been licensed one copy of this document for personal use only. Any other reproduction or redistribution is strictly prohibited. All rights reserved.

Licensed to elliot mattison (2146499335, i.astraltears@gmail.com) of 9801 royal ln, dallas, Texas 75231. If this product is distributed to others,
elliot mattison agrees to pay Mometrix the full retail price for each unlicensed recipient. Please play fair and respect the work of our authors.
67. D: Methacholine challenge testing is a diagnostic pulmonary function test in which a patient’s
airways are provoked into an asthma exacerbation. This test is used for patients who do not have a
clear diagnosis of asthma. This test is associated with several serious risks including refractory
bronchospasm, respiratory arrest, and hypoxia. Syncope is not a common risk associated with this
test.

68. A: If a peak flow meter is cracked, air will escape through the crack resulting in lower readings.
If a patient partially blocks the mouthpiece with their tongue or spits into the mouthpiece, the
readings will be higher. To achieve accurate peak flow results, the patient should ensure a clear
mouthpiece with no visible damage and breathe hard and fast into the peak flow meter.

69. C: Orapred is a different type of corticosteroid than the “body building” anabolic steroids. This
medication works on the inflammation of the lungs and is associated with some potentially serious
side effects that should be discussed with the patient and family. Option A is incorrect because
taking steroids for a short time does not necessarily mean that serious side effects will not take
place. Option B is incorrect because this medication does NOT have minimal side effects. The
corticosteroids that are associated with minimal side effects are the localized inhaled
corticosteroids. Option D is incorrect because an asthma educator should never substitute a
medication on their own without a physician’s approval and/or consent.

70. B: Although all of the above choices are common summertime asthma symptoms, not every
patient will experience symptoms with exposure to these substances. When the air quality is poor
(also known as ozone action days), air particulate levels are high, and the air temperature is hot and
humid. On these days, the air quality is poor, which is dangerous for all patients with asthma as well
as patients who are elderly or very young.

71. D: The patient sets the goal of therapy for their asthma education sessions. Although patients
often work together with the healthcare team, including the physician and asthma educator, the
goals of therapy are ultimately for the patient to decide. One of the asthma educator’s
responsibilities is to assist the patient in assessing their environment, triggers, activity level, and
other factors to help them set appropriate goals.

72. B: The asthma educator’s body language in this case is very aggressive. Standing above the
patient is often thought of as looking down upon the person. Folded arms are not an acceptable
stance for asthma education because this is a closed off and defensive pose. The asthma educator
should be in a neutral position, preferably seated on the same level as the patient. The asthma
educator is asking appropriate open-ended questions, which is the recommended approach for
asthma education. Including the patient’s visitor in the education session is not necessary,
especially if privacy is to be maintained.

73. C: Low socioeconomic status (SES) groups have a higher incidence of chronic diseases such as
asthma. These patients may have reduced access to adequate medical care due to both financial
constraints as well as lack of medical facilities in their neighborhoods and surrounding areas. These
patients are less likely to get annual preventative care, such as flu shots, compared to higher SES
groups. These patients are also more likely to have frequent, severe asthma symptoms due to their
lack of adequate medical care.

74. B: Women are more likely to have severe asthma exacerbations than men, due in part to
estrogen fluctuations in the menstrual cycle that cause airway inflammation. Men don’t necessarily
respond better to conventional asthma therapies compared to women; in fact, the response to
medications varies from person to person regardless of sex. Men are also statistically less likely to

- 137 -
Copyright © Mometrix Media. You have been licensed one copy of this document for personal use only. Any other reproduction or redistribution is strictly prohibited. All rights reserved.

Licensed to elliot mattison (2146499335, i.astraltears@gmail.com) of 9801 royal ln, dallas, Texas 75231. If this product is distributed to others,
elliot mattison agrees to pay Mometrix the full retail price for each unlicensed recipient. Please play fair and respect the work of our authors.
seek medical help for exacerbations. A lack of environmental controls is not likely this patient’s
cause of symptoms because she states that her triggers are the same as her husband’s and there is
no evidence that he is experiencing any environmentally triggered symptoms currently.

75. C: The only way to comprehensively educate a patient who does not speak the same language as
the educator is to use a professional translator such as those provided by hospitals. Although giving
the patient pamphlets in their native language and demonstrating proper peak flow and inhaler
technique are important parts of the education session, they are not comprehensive. These are
additional tools that can help the educator teach the patient about their asthma. Option B is
incorrect because due to privacy laws, a patient’s family member should never be used as a
translator for sensitive healthcare-related information.

76. A: The stepwise approach to asthma management is a chart that measures a patient’s
impairment, risk, and asthma symptoms. There are six steps in each chart, and there are three
different charts, each for a different age group. Option C is incorrect because patients move up and
down the steps based on their symptoms, impairment, and risk. These plans are a good tool for
asthma management, but they do not negate the importance of clinical judgment.

77. C: This patient has mild, intermittent symptoms that occur infrequently. Because this patient is
well controlled, she only requires a short-acting bronchodilator for emergency use. Option A is
incorrect because inhaled corticosteroids are in step 2 of the chart. Option B is incorrect because all
asthma patients are represented in the chart, regardless of the infrequency or complete absence of
symptoms. Option D is incorrect because the patient is not a candidate for step 2 because she has
minimal asthma symptoms and virtually no impairment.

78. B: This patient is currently using a medium-dose inhaled corticosteroid with long-acting
bronchodilator, short-acting bronchodilator, and leukotriene modifier. These medications correlate
with step 4 of the Stepwise Approach to Asthma Management. Therefore, because the patient is not
well controlled on step 4, he needs to move to step 5, which includes the introduction of a higher
dose of inhaled corticosteroid. In this case, his Advair dosage would increase to 500/50.

79. C: This patient is currently using a short-acting bronchodilator as needed plus a high-dose
corticosteroid with long-acting bronchodilator. This criterion places her in step 5. At this point, the
patient should begin allergy testing to determine what her allergies are. If allergy treatment does
not help her asthma, then a treatment such as omalizumab may be initiated if she meets the clinical
criteria. Option B is incorrect because omalizumab is in step 5, not 4.

80. D: For children ages 0–4, the correct medication regimen for an acute exacerbation is short-
acting bronchodilator treatments every 4–6 hours for 24 hours. If the patient is not experiencing
relief, the physician must be notified and the patient may need to step up on the Stepwise Approach
to Asthma Management chart.

81. A: Because this patient has been well controlled over the past year, stepping down treatment
can be considered. It is never appropriate to discontinue a short-acting bronchodilator. This patient
has stopped experiencing exercise-induced asthma symptoms due to the addition of the
levalbuterol prior to exertion, and therefore it should not be changed. The mother states that her
daughter has worse symptoms during allergy season. For that reason, the patient can try stepping
down treatment by taking the budesonide seasonally, one month before the spring allergy season
begins. Patients should always be on the lowest possible dosage of medication; therefore, because
this patient is well controlled, it is time to consider stepping down therapy.

- 138 -
Copyright © Mometrix Media. You have been licensed one copy of this document for personal use only. Any other reproduction or redistribution is strictly prohibited. All rights reserved.

Licensed to elliot mattison (2146499335, i.astraltears@gmail.com) of 9801 royal ln, dallas, Texas 75231. If this product is distributed to others,
elliot mattison agrees to pay Mometrix the full retail price for each unlicensed recipient. Please play fair and respect the work of our authors.
82. A: If a patient has been well controlled for at least three months, they can move down one step
on the Stepwise Approach to Asthma Management. Then, if they are well controlled at that new
step, they can move down one more after three more months of good control. The goal of therapy is
to maintain the patient at the lowest dosage of medication possible.

83. A: Although cromolyn sodium is also a controller medication, option A is the best choice; this is
because both mometasone and montelukast are controller medications. Option D is incorrect
because only levalbuterol is a rescue medication. Cromolyn sodium is often used to premedicate
prior to exertion for patients with exercise-induced asthma.

84. D: Long-acting bronchodilators are used in moderate to severe asthma. Option A is incorrect
because although medications in this classification do have warnings regarding asthma-related
death and rebound bronchospasm, they are considered safe and effective for the treatment of
moderate to severe asthma when used in combination with INHALED corticosteroids. Therefore,
because option C mentions systemic steroids, this choice is incorrect. Option B is incorrect because
long-acting bronchodilators must never be used alone due to the risk of severe and life-threatening
side effects.

85. B: Mast cell stabilizers, such as cromolyn sodium, are often used to treat exercise-induced
asthma. These inhalers can be used prior to exercise to prevent asthma symptoms on exertion.
Option A is incorrect because this answer refers to leukotriene modifiers, not mast cell stabilizers.
Mast cell stabilizers are not a first-line treatment for asthma but can be used as an alternative for
mild asthma. They may be prescribed on a schedule or on an as-needed basis.

86. A: This patient is experiencing symptoms of allergic asthma. His allergies are affecting his level
of asthma control; therefore, he should be prescribed a leukotriene modifier to treat the allergies
and the allergic asthma. Option B is incorrect because mast cell stabilizers do not treat allergies.
Option C is incorrect because the patient should be able to work without symptoms in any
environment. This option may also not be possible due to the nature of his job. Option D is incorrect
because although wearing a mask would be helpful for his outdoor work, it is not recommended for
sleeping. Other dust control measures should be used in the bedroom instead.

87. D: Immunomodulating biologicals work by reducing the number of inflammatory cells and
antibodies released during an allergic reaction. These drugs are used to treat allergic asthma;
therefore, allergy testing must be performed as well as taking an IgE measurement prior to
beginning therapy.

88. C: All of the above choices are correct except option C. An asthma educator should not suggest a
reduction in dosage when other options haven’t been attempted. First, the asthma educator should
encourage the patient to use a valved holding chamber to help reduce the medication deposition in
the mouth. Next, the asthma educator can suggest rinsing and gargling to prevent the constant
reabsorption of the medication into the patient’s bloodstream through the mucus membranes of the
mouth and throat.

89. A: Controller medications, such as inhaled corticosteroids, must be used daily without missing a
dose. These medications take several days to several weeks before the patient will notice any relief
of symptoms. Once the symptoms subside, the medication must be continued in order to keep the
patient at that level of control. Controller medications often cause minimal, localized side effects
such as thrush infections of the mouth and hoarseness.

90. B: The patient is referring to ProAir (albuterol), which is a red metered dose inhaler, and
Flovent (fluticasone), which is an inhaled corticosteroid. QvAR is an inhaled corticosteroid metered

- 139 -
Copyright © Mometrix Media. You have been licensed one copy of this document for personal use only. Any other reproduction or redistribution is strictly prohibited. All rights reserved.

Licensed to elliot mattison (2146499335, i.astraltears@gmail.com) of 9801 royal ln, dallas, Texas 75231. If this product is distributed to others,
elliot mattison agrees to pay Mometrix the full retail price for each unlicensed recipient. Please play fair and respect the work of our authors.
dose inhaler that is brown or rose colored. Asmanex is a pink and white, dry powder, upright
Twisthaler inhaler.

91. D: Having the jitters is not commonly associated with systemic corticosteroids. Common side
effects associated with short-term use of oral or intravenous corticosteroids include but are not
limited to weight gain, personality changes, blood sugar changes, stomach upset, and insomnia.

92. D: Leukotrienes can cause side effects such as sleeplessness and fatigue, stomach upset, and
headache. They also are found to cause personality changes and suicidal thoughts in some patients.
The clinician must weigh the benefits and risks of this use of this medication, and care should be
used with patients who have a history of mental illness or depression. These medications are not
mast cell stabilizers; therefore, option C is incorrect. Leukotriene modifiers are used for the
treatment of allergic asthma, so option B is incorrect.

93. A: Tricyclic antidepressants cause an increase in heart rate and blood pressure when taken in
conjunction with short-acting bronchodilators, resulting in palpitations and other cardiac effects.
Options B through D are common asthma medications that do not interact with short-acting
bronchodilators.

94. A: Fluticasone is an inhaled corticosteroid that is available in metered dose inhaler and dry
powder inhaler forms. Mometasone and budesonide are only available as dry powder inhalers.
Beclomethasone is only available as a metered dose inhaler.

95. B: Fluticasone with salmeterol Diskus is ordered on a twice-daily dosing schedule; this is
because the salmeterol component has a 12-hour duration. So this patient is only taking his
medication once per day and is leaving his asthma untreated 50% of the time. The patient states
that he is well controlled, so perhaps the clinician can consider stepping down therapy to only
fluticasone, but the clinician would need more information to make that decision.

96. C: Using a rescue inhaler every one to two hours is considered short-acting bronchodilator
overuse. This can lead to serious cardiac side effects and asthma-related death. If a patient is using a
rescue inhaler more than every three to four hours, the physician must be notified immediately.
This patient is self-medicating, which is very dangerous. This patient likely needs a controller
medication added to his therapy after his current exacerbation resolves.

97. B: Benralizumab is an immunomodulating biological medication used for the treatment of


severe allergic asthma that is not well controlled on other therapies. These medications are given
via injection. Zileuton is a leukotriene modifier that is administered in tablet form. Mometasone and
nedocromil are both inhalers.

98. A: Although some metered dose inhalers such as Maxair are breath actuated, others require a
valved holding chamber to maximize the dose delivery to the patient’s airway. Most inhalers squirt
at a high rate, which leaves the medication to deposit in the mouth rather than in the lungs. Valved
holding chambers hold that puff of medication and allow the patient to inhale slowly to achieve
optimal drug deposition in the airway. Option B is incorrect because dry powder inhalers do not
squirt anything; instead, the patient must inhale the medication powder. This requires a fast
inhalation. Option D is incorrect because dry powder inhalers should not be stored anywhere that
steam or moisture is present because it will cause the powder to clump.

99. B: Valved holding chambers with masks are not just for small children. Mask attachments for
valved holding chambers come in many sizes and can be used in the adult population. The family
member would hold the mask over the patient’s nose and mouth, depress the inhaler, and watch

- 140 -
Copyright © Mometrix Media. You have been licensed one copy of this document for personal use only. Any other reproduction or redistribution is strictly prohibited. All rights reserved.

Licensed to elliot mattison (2146499335, i.astraltears@gmail.com) of 9801 royal ln, dallas, Texas 75231. If this product is distributed to others,
elliot mattison agrees to pay Mometrix the full retail price for each unlicensed recipient. Please play fair and respect the work of our authors.
the patient breathe in and out for at least six breaths per puff. Option A is incorrect because a home
nebulizer is much costlier for a patient than using an inhaler. Inhalers and nebulizers are equally
effective when used correctly, except in certain instances of severe bronchospasm when a nebulizer
treatment is preferred. Options C and D are incorrect because these choices require high levels of
patient cooperation that is not possible with this patient.

100. B: The Asmanex Twisthaler’s cap actuates the dosage of medication delivered. The twisting
motion of the removal of the cap sets the medication in the chamber. The patient then inhales the
medication using a fast, deep breath (this is a dry powder inhaler) and then replaces the cap by
twisting it back on.

101. A: The Pulmicort Flexhaler requires a twisting motion of the base of the inhaler to actuate the
dose. The cap is not the dose actuator. The patient must first remove the cap and then twist the
base of the inhaler to set the dose. The patient then inhales the medication. Option C is incorrect
because patients are instructed to hold their breath for 10–15 seconds when using an inhaler.

102. B: There is a dose counter window on the top of the inhaler. The number displayed turns red
when there are five doses left in the inhaler.

103. B: The term “blow-by treatment” refers to a nebulizer treatment in which the corrugated
tubing at the end of the mouthpiece is placed near the patient’s face so that the mist blows by the
patient’s nose and mouth. Blow-by treatments are ineffective and should not be used. The mist is
lost to the ambient air because it does not have a reservoir. Treatments given via mask or
mouthpiece have reservoirs (the mask holds the mist itself for mask-based treatments, and the
corrugated tubing on the opposite end of the mouthpiece holds the mist for mouthpiece-based
treatments). If a clinician does not want to wake a patient, they can hold a mask up to the patient’s
face without strapping it on.

104. A: Some brands of spacers, such as AeroChambers, have indicator whistles that make noise
when a patient inhales too quickly. If a patient hears a whistle when using a valved holding
chamber, the patient has inhaled too quickly or too forcefully. The correct spacer technique
includes removing the cap, exhaling completely, pressing the inhaler, inhaling slowly and deeply,
holding the breath for 10 seconds, and exhaling slowly.

105. B: Valved holding chambers, also known as spacers, capture the dose of medication from a
metered dose inhaler and suspend it, allowing the patient to inhale slowly and deeply rather than
rushing to inhale the puff of medication. This allows for better drug deposition in the lungs. Without
a valved holding chamber, most of the medication in a metered dose inhaler’s puff is lost to the oral
cavity. Option C is incorrect because the spacer can only hold the puff for a short time. It is not
designed to hold a puff for later. The patient must inhale the puff immediately after actuating the
inhaler. Finally, D is incorrect because antistatic technology is found in some valved holding
chambers such as the AeroChamber Z STAT.

106. B: Nebulizer components should be cleaned daily with warm, soapy water, and they should be
disinfected weekly. Nebulizer cleaning technique varies between machines and manufacturers, so
patients should refer to their instruction manual to determine the proper cleaning technique for
their machine. Generally speaking, patients may soak the components in 70% isopropyl alcohol for
5 minutes or 3% hydrogen peroxide for 30 minutes. Nebulizer components are generally not
dishwasher safe.

107. C: Generally speaking, spacers should be cleaned with warm, soapy water and left to dry
vertically so that the dirt and particles exit the chamber. Not all spacers are dishwasher safe, so this

- 141 -
Copyright © Mometrix Media. You have been licensed one copy of this document for personal use only. Any other reproduction or redistribution is strictly prohibited. All rights reserved.

Licensed to elliot mattison (2146499335, i.astraltears@gmail.com) of 9801 royal ln, dallas, Texas 75231. If this product is distributed to others,
elliot mattison agrees to pay Mometrix the full retail price for each unlicensed recipient. Please play fair and respect the work of our authors.
choice is incorrect because there is no instruction booklet available for the spacer. Finally, a bottle
brush should never be used to clean a spacer. This is especially true of antistatic valved holding
chambers because these have specialized coatings and materials that can become scratched and
subsequently lose their antistatic effects.

108. A: Spacers can last indefinitely if they are not damaged and the valve is intact. If there is any
sign of wear and tear, the spacer should be replaced.

109. A: Disposable nebulizer components can be brought home and used with a home nebulizer if
the components are cleaned properly and thrown away after a few uses. Disposable components
are not designed to be used for long periods of time.

110. A: The first step in troubleshooting a nebulizer is to ensure that all connections are tight and
secure. A lack of gas flow due to a loose oxygen tube connection is a common problem. Option B is
incorrect because this treatment hasn’t even started yet, so there will be no condensation buildup
yet in the cup. Option C is incorrect because 8 liters is the correct gas flow rate for a nebulizer
treatment. Option D is a possible solution to the problem, but only AFTER all connections have been
checked. If all the connections are tight and the nebulizer still doesn’t function, then it should be
replaced.

111. D: Foreign bodies in valved holding chambers often don’t cause any effects at all; however, if a
foreign body was big enough to be noticed, it would have the opposite effect on the patient. A large
foreign body would increase resistance because the patient would be fighting against the object to
inhale.

112. D: Inhaled corticosteroids can cause thrush infections regardless of the delivery device;
therefore, patients should rinse and gargle after their nebulized inhaled corticosteroid treatment.
The mist must not get into the patient’s eyes because it can be irritating and may cause infections.
Inhaled corticosteroids often leave a sticky residue in the nebulizer cup; therefore, the nebulizer
components may require more frequent cleaning. Finally, inhaled corticosteroid solutions can be
mixed with other medications, such as albuterol. There is no adverse event associated with this
practice, except for the fact that the treatment will take twice as long with two medications in the
cup.

113. A: Patients must refill their prescriptions prior to running out of them. This patient was
without his lifesaving rescue medication for more than a week, which could have had dire
consequences. It is a good practice to refill prescriptions at least one week prior to running out of
them because sometimes there are drug shortages and other issues that can delay the medication
delivery. Option C is incorrect because it doesn’t solve the problem of the patient waiting until he is
out of medication before he refills it. Option D is incorrect because a patient may not use their
rescue inhaler very often and would therefore be paying for a medication that perhaps they would
not use before it expired.

114. A: Switching from an inhaler to a nebulizer is a very costly and unnecessary change because
many inhalers are available in different forms that require less dexterity to use. For patients with
arthritis, several different modifications can be made to help them use their current metered dose
inhaler (MDI) with less difficulty. The patient can use their thumb, which is a stronger finger, to
depress the canister, or they can turn the MDI upside down and press upward to actuate the dose.
Finally, the patient can switch to a dry powder form of fluticasone, which requires less hand
strength and coordination.

- 142 -
Copyright © Mometrix Media. You have been licensed one copy of this document for personal use only. Any other reproduction or redistribution is strictly prohibited. All rights reserved.

Licensed to elliot mattison (2146499335, i.astraltears@gmail.com) of 9801 royal ln, dallas, Texas 75231. If this product is distributed to others,
elliot mattison agrees to pay Mometrix the full retail price for each unlicensed recipient. Please play fair and respect the work of our authors.
115. C: Although yoga is safe exercise for many patient populations, Bikram yoga is not
recommended for patients with asthma. This is due to the conditions associated with these types of
yoga classes. Bikram yoga is also known as hot yoga: The ambient temperatures are greater than
100 degrees, and the humidity is kept at a high level. This can make breathing very difficult for
patients with asthma. Option D is incorrect because there is no indication in the question that this
patient has exercise-induced asthma, nor does it state any information on a medication regimen.

116. B: Echinacea is related to ragweed, to which this patient is allergic. It is likely that consuming
echinacea is causing an allergic response that is adversely affecting her asthma.

117. C: Yearly influenza vaccines are recommended for all patients with asthma who are six months
old and up. Patients with asthma are especially susceptible to the flu and are more likely to have
serious, life-threatening side effects. All patients should be vaccinated regardless of prior history of
influenza infection.

118. D: Asthma educators can refer patients to specialists if needed; however, many patients can be
managed successfully by their primary care physicians. Often, referrals are only needed when
patients are unable to be successfully managed by their family physician.

119. A: Smoking cessation medication comes in many forms and types, including pills, patches,
gums, and other interventions. In order to choose a smoking cessation medication that is
appropriate for a patient, several criteria must be considered. Medical history, personal preference,
and physician’s assessment are all important pieces of information when making decisions on
adding new medications to a patient’s regimen. Family history does not factor into this decision
making.

120. B: Stress can cause a patient to have reduced tidal volumes and a higher respiratory rate.
Patients with high stress levels may also experience trouble sleeping and weight gain. All of these
effects can cause airway inflammation, which exacerbates a patient’s asthma.

121. A: This patient has several life changes that have made her clinically depressed. An asthma
educator is not trained or licensed to add antidepressants to her medication regimen. In addition,
some antidepressants can interact with asthma medications, so this may not be a good choice for
this patient who is already having increased asthma symptoms. Option B is incorrect because
although it is not a harmful intervention, it is not as helpful as getting professional help from a
specialist in depression. Option D is incorrect because there is not enough information to determine
what dosage or type of inhaled corticosteroid would be most helpful for this patient.

122. D: This patient is being seen at a hospital and is being charged for the asthma education
sessions. In this case, an asthma educator can’t necessarily stop charging a patient for education
sessions without prior approval. Failure to accurately charge a patient for services can result in
corrective action by the facility. Referral to a social worker is a good choice because the social
worker can help the patient to find local sources of healthcare that are more affordable and/or free.
Changing the patient’s Advair to separate fluticasone and salmeterol inhalers can be cheaper than
paying for the brand-name Advair inhaler. In addition, offering free samples of medications can be
helpful for a patient in a poor financial situation.

123. D: Patients with asthma who use illegal drugs are more likely to have frequent, severe
exacerbations. These episodes are more likely to end in asthma-related hospitalization or death.
Patients with asthma who abuse drugs are not more likely to have comorbidities such as GERD,
obstructive sleep apnea, or obesity.

- 143 -
Copyright © Mometrix Media. You have been licensed one copy of this document for personal use only. Any other reproduction or redistribution is strictly prohibited. All rights reserved.

Licensed to elliot mattison (2146499335, i.astraltears@gmail.com) of 9801 royal ln, dallas, Texas 75231. If this product is distributed to others,
elliot mattison agrees to pay Mometrix the full retail price for each unlicensed recipient. Please play fair and respect the work of our authors.
124. D: Smoke is dangerous in all forms, and even trace amounts of smoke residue can cause
asthma symptoms. Room fragrance sprays do not work to remove smoke residue, but instead they
cover up the scent with another fragrance. Fragrances are irritants and asthma triggers too, so
combining these chemicals with smoke residue can be particularly irritating to the airways. Odor-
neutralizing products also do not remove the smoke residue and will cover the scent with another,
lighter fragrance. Airing out the clothes outdoors will help to remove some of the smoke residue
but will not remove all of it. The only action that removes smoke residue is washing the items or
bathing.

125. C: Without more information regarding the condition of the stove, the best option is to instruct
the patient to have it serviced and inspected regularly and see how the patient feels while using it.
Telling a patient to remove and replace the stove is a costly and unnecessary intervention. If the
stove is in working order, it may not be a problem for this patient. Option B is incorrect because
instructing a patient to use another type of fuel in the stove could be very dangerous. Option D is
incorrect because without knowing the condition of the stove, it is impossible to predict whether
smoke will enter the home.

126. A: The use of essential oils is not recommended for patients with asthma. These oils are
naturally derived but can cause allergic reactions and asthma symptoms. If the patient is allergic to
the substance that the oil is derived from, a serious allergic reaction can take place. If the patient is
not allergic to the oil itself, the fragrance acts as an inhaled irritant and can cause asthma
exacerbations. Option C is incorrect not only because the oil fragrance can trigger asthma, but also
because essential oils are concentrated, not diluted.

127. B: When a patient’s asthma is poorly controlled, the educator should first look to the home
environment for possible asthma triggers that can be mitigated. A patient spends most of their time
in the home environment, and this area should be as asthma-friendly as possible. Options A and C
are incorrect because the asthma educator already knows that this patient is impaired. Getting
more information on these topics won’t help the educator determine the cause of symptoms. Option
D is incorrect because a family medical history will not help the educator determine the cause of the
current symptoms.

128. D: There is no indication that this patient has any animal dander allergies, so the pet is not
necessarily an asthma trigger. In addition, the pet sleeps outside of the patient’s bedroom, which is
a reasonable and recommended environmental control measure. The mini blinds are not a likely
asthma trigger because these can be wiped down frequently as opposed to heavy curtains that must
be laundered. Mini blinds are preferred over draperies for dust mite mitigation. The stuffed animals
on the patient’s bed are not excessive, but the asthma educator should speak with the mother about
washing them regularly as needed. But although the stuffed animals can be a POTENTIAL asthma
trigger, the bleach is a definite asthma trigger. Strong chemicals are dangerous for patients with
asthma because they release fumes that irritate the airways and cause bronchospasm. The asthma
educator should talk with the mother about possible alternatives such as unscented cleaners
and/or vinegar-based cleaning solutions.

129. D: Wearing a scarf over your nose and mouth is one of the best ways to avoid weather-related
asthma exacerbations in winter. The scarf warms and humidifies the ambient air, and it prevents
cold, dry air from provoking the patient’s airways into bronchospasm. Options A and B both
describe extreme regional weather differences that can cause asthma exacerbations during travel.
Option C is incorrect because there is no mention in this situation of a patient using air conditioning
or keeping the windows closed, both of which are necessary to prevent weather-related asthma
symptoms.

- 144 -
Copyright © Mometrix Media. You have been licensed one copy of this document for personal use only. Any other reproduction or redistribution is strictly prohibited. All rights reserved.

Licensed to elliot mattison (2146499335, i.astraltears@gmail.com) of 9801 royal ln, dallas, Texas 75231. If this product is distributed to others,
elliot mattison agrees to pay Mometrix the full retail price for each unlicensed recipient. Please play fair and respect the work of our authors.
130. B: Medication and lifesaving medical equipment should never be packed in a patient’s checked
bags. The Federal Aviation Administration allows nebulizers to be transported in a patient’s
checked bag but may require additional screening precautions at the security checkpoints.

131. A: Air ducts in the home can have a buildup of dust and debris from filtering the air in the
home; however, there is no definitive evidence that duct cleaning will help reduce the frequency
and severity of asthma symptoms. Sometimes, air duct cleaning has the opposite effect and will stir
up dust in the home. Adding a high-efficiency particulate air (HEPA) filter to the furnace can be a
helpful alternative to duct cleaning. The other interventions, including removal of carpeting,
replacing curtains with blinds, and encasing mattresses and pillows with allergy-proof bedding, are
all effective in the control of dust mites in the bedroom environment.

132. C: Mold is a naturally occurring substance, and every home has mold spores in it. When a
water leak occurs, there is an overgrowth of mold, which is hazardous to the health of all
individuals, especially those with asthma and other pulmonary diseases. Water leaks must be dried
out within 48 hours to prevent mold growth. If mold does grow, it can be cleaned with regular
detergent and water; no specialized cleaning agents are required. Finally, indoor humidity should
be kept at below 60% to prevent excessive moisture and increased mold growth.

133. D: Setting traps for pests and insects might be helpful once they have arrived in the home, but
it is not helpful for the prevention of infestation. To prevent infestation, indoor humidity levels
should be kept low, food should be stored in sealed containers, and any spills must be cleaned up
immediately.

134. C: Bathing and washing your hair after outdoor activity can wash away any pollen that
collected during activity and will help to reduce nighttime allergy symptoms. Staying indoors is not
possible for an active child, and it is not recommended due to the ongoing obesity epidemic in the
United States. Playing soccer in the morning hours is not a good choice because pollen counts are
highest in the morning hours. Finally, option D is incorrect because wearing a mask can obstruct the
patient’s breathing while running, which is especially dangerous for patients with asthma and
exercise-induced asthma specifically.

135. A: Proper hand hygiene is the single most helpful action a person can take to prevent illness.
Washing hands with soap and warm water for at least 15 seconds will help to reduce the possibility
of contracting illness. Avoiding individuals who are ill, taking vitamins, and getting a pneumonia
shot are all possible ways of preventing illness, but hand hygiene is the BEST and most helpful
choice for all patients.

136. A: Wearing a mask and other personal protective equipment is the best way to avoid chemical
exposure in the workplace. Option C is incorrect because although wearing gloves is helpful for
anyone who works with chemicals, it does not directly affect a person’s inhalation of chemicals and
subsequent asthma-related symptoms. Option B is incorrect because changing to a natural fertilizer
may not be possible because this is an employee and not a company owner. In addition, natural
fertilizers can act as inhaled airway irritants despite being naturally occurring. Option D is incorrect
because the patient states he started having increasing symptoms while working with the chemical.
If the patient was having symptoms later at night after work, then bathing would be an appropriate
way to reduce his asthma symptoms.

137. C: This patient is running her air cleaner continuously but is keeping her windows open. This
action is defeating the purpose of having an air cleaner. The air cleaner is filtering the ambient air,
but more allergens keep flooding in through the open window.

- 145 -
Copyright © Mometrix Media. You have been licensed one copy of this document for personal use only. Any other reproduction or redistribution is strictly prohibited. All rights reserved.

Licensed to elliot mattison (2146499335, i.astraltears@gmail.com) of 9801 royal ln, dallas, Texas 75231. If this product is distributed to others,
elliot mattison agrees to pay Mometrix the full retail price for each unlicensed recipient. Please play fair and respect the work of our authors.
138. D: This patient uses a canister-style bagless vacuum, which means that when she empties the
canister into the garbage, she is releasing plumes of dust as the debris hits the garbage can.
Emptying her vacuum outside or in the garage would help this patient to further mitigate her dust
mite exposure. Option A is incorrect because there is no information in this vignette to determine
what kind of window coverings this patient has in her home. Option B is incorrect because
vacuuming once per week is adequate, and although having a HEPA filter on a vacuum is helpful, it
is not always necessary. Option C is incorrect because dusting moves dust around and releases it
into the air, causing allergy symptoms for many patients.

139. D: Home visits are important because an asthma educator acts as an impartial third party who
can assess an environment without any bias. Then, the educator can make suggestions based on the
patient’s lifestyle and environment.

140. D: Allergies are not a component of an asthma action plan; however, they are an essential part
of an allergy action plan. Asthma action plans include the frequency and severity of symptoms, peak
flow readings, actions to take, and medication dosage and frequency.

141. D: Asthma action plans tell a patient when to call for help, when to take their medications, and
how to determine the severity of an exacerbation. Option A is incorrect because asthma action
plans vary widely among patients. The overall design may be similar, but the content is never
identical. Option B is incorrect because ALL patients with asthma require written asthma action
plans regardless of the severity and frequency of symptoms. Option C is incorrect because patients
who are poor perceivers are often the patients who need asthma action plans the most. They are
unable to determine the severity or frequency of their exacerbations, and they use the asthma
action plan as a guide for treatment.

142. C: A patient’s written asthma action plan is a fluid document that changes throughout a
patient’s lifetime based on their level of control. A written asthma action plan must be reevaluated
every time a patient follows up with the physician or asthma educator.

143. D: Although many facilities have time requirements and guidelines for asthma education, there
is no definitive timeframe in which an asthma education session should be completed. Some
patients require more instruction than others, and it is up to the patient’s needs to determine how
long the education session lasts. Option C is incorrect because although charging a patient for
asthma education is an important part of generating revenue for the facility, an asthma educator
must not limit the patient’s education based on financial issues.

144. D: A patient’s asthma action plan is an important medical document that must be kept
available for quick reference. One copy should be left in the patient’s file at the physician’s office.
Another copy should be kept at home, and a third copy should be kept at either the patient’s school
or workplace.

145. C: Although financial issues can contribute to poor asthma control due to lack of funds for
medication and medical visits, finances are never evaluated in a follow-up session. Instead, a
patient’s level of control and impairment, perception of symptoms, and adherence to the asthma
action plan are evaluated each time the patient follows up with the asthma educator.

146. B: Although peak flow monitoring is an important self-assessment tool in the management of
asthma, it is optional. Some patients do not show any variance in peak flow monitoring despite
severe symptoms. For these patients, it would be dangerous to rely on peak flow values to measure
severity and impairment. Option A is incorrect because although many patients benefit from
numeric and graphic data for asthma management, others prefer different ways of managing their

- 146 -
Copyright © Mometrix Media. You have been licensed one copy of this document for personal use only. Any other reproduction or redistribution is strictly prohibited. All rights reserved.

Licensed to elliot mattison (2146499335, i.astraltears@gmail.com) of 9801 royal ln, dallas, Texas 75231. If this product is distributed to others,
elliot mattison agrees to pay Mometrix the full retail price for each unlicensed recipient. Please play fair and respect the work of our authors.
asthma, which is encouraged especially if they are good perceivers. Option C is incorrect because
many patients younger than age six can physically perform peak flows. These patients may not have
perfect technique, and peak flows should not be relied upon in this population until proper
technique is perfected. Option D is incorrect because peak flows are not associated with any age
limitations aside from very young patients.

147. A: When an asthma educator can’t determine a cause of symptoms based on the information
given by a patient, the asthma educator should instruct the patient to keep an asthma diary. This
will help the patient to pinpoint the things that make her asthma better or worse and can reveal
important patterns associated with the patient’s asthma symptoms and exacerbations. Option B is
incorrect because although the patient is able to perceive that her asthma is worsening, she just
can’t determine the cause. So, although peak flows are helpful for many patients, keeping an asthma
diary is a better choice for this particular patient. Option C is incorrect because there is no evidence
currently that this patient has allergies. Option D is incorrect because although environmental
controls would be helpful, the asthma educator has no information regarding possible
environmental triggers at this time, and an asthma diary would be a better choice. This would
pinpoint some areas of concern that could then be addressed with environmental controls.

148. D: To determine the need for an outpatient asthma education program, an asthma educator
must take several pieces of information into consideration. Patient-reported symptom severity and
frequency, asthma hospitalization rates, and the incidence and prevalence of asthma in the
surrounding community are all criteria that must be measured to help determine the need for an
asthma education program.

149. A: A partnership between an asthma educator and a disease-based charitable organization or


professional consortium can be valuable for developing asthma education programs. Asthma drug
corporations and self-assessment tool developers are not helpful partnerships for an asthma
educator because an educator’s program must be free from bias. Endorsing a particular brand or
tool can cause a conflict of interest when performing asthma education because patients have the
right to choose the tools and medications that are right for them without any bias from outside
sources.

150. D: Charging a patient fee for education should be a last resort when providing asthma
education. CPT code usage, where an employee of a clinic charges a patient for service that is then
submitted to the patient’s insurance, is an appropriate way to raise funds for education.
Reimbursement amounts will vary, but the act of charging a patient shows productivity even if
reimbursement is not possible. Grants and facility-approved budgets are other possible ways to
raising funds for asthma education services.

151. A: The inner-city population is underserved and often has financial instability that directly
affects patients’ levels of asthma control. Offering free or reduced-cost tools, such as valved holding
chambers or peak flow meters, and free medication samples can help improve patient outcomes in
the inner-city population. Options B and D are incorrect because the location of asthma education is
irrelevant if the patient can’t afford the tools required to keep their asthma well controlled. Option
C is incorrect because written materials should be written at a sixth-grade reading level or lower.

152. B: The patient-focused goals and the financial goals of an asthma education program cannot be
set without first identifying the needs of the patient population being served. Once that is
determined, then the asthma educator can design a program with specific financial and patient-
focused goals in mind.

- 147 -
Copyright © Mometrix Media. You have been licensed one copy of this document for personal use only. Any other reproduction or redistribution is strictly prohibited. All rights reserved.

Licensed to elliot mattison (2146499335, i.astraltears@gmail.com) of 9801 royal ln, dallas, Texas 75231. If this product is distributed to others,
elliot mattison agrees to pay Mometrix the full retail price for each unlicensed recipient. Please play fair and respect the work of our authors.
153. C: This educator ordered booklets at an appropriate reading level and obtained them free of
charge from a reputable charitable organization but failed to review the books prior to stocking the
hospital with them. There is no way to know if these booklets contain accurate and\or appropriate
information. Stocking peak flow meters and spacers in the hospital units is an appropriate action
because the asthma educator can charge for these devices when she provides them to the patient.
Simply stocking them in a particular area has no bearing on the reimbursement for the cost of these
items.

154. D: To protect a patient’s privacy, and to stay in accordance with Health Insurance Portability
and Accountability Act of 1996 (HIPAA) regulations, only members of the patient’s healthcare team
should have access to the patient’s personal medical information. This includes asthma education
because the educator is discussing sensitive medical information, as well as other issues such as the
patient’s environment, stressors, and family dynamics. The patient’s family can be present ONLY
when requested by the patient. Any visitors and family members should be asked to leave the room
prior to education unless the patient gives permission for them to stay.

155. D: All forms of asthma education sessions require that comprehensive records are kept by the
asthma educator. Although a one-on-one inpatient session requires more individualized detail than
group sessions, all sessions must be properly documented for future reference.

156. C: The patient’s spacer and medication need to be close to her at all times. Changing to a
breath-actuated metered dose inhaler such as Maxair can help the patient to carry her medication
with her because that particular inhaler does not require the use of a spacer. Changing to a
collapsible spacer is also an option because these spacers are often much smaller than a traditional
valved holding chamber. The patient’s date may also carry the spacer for her if necessary. Option C
is incorrect because if the patient has an asthma exacerbation at the event, she would then need to
leave the venue and walk to the car while short of breath to retrieve the spacer. If someone else
went to get the spacer for her, she is still wasting valuable time during a potentially life-threatening
asthma exacerbation. Patients should carry their spacers and rescue medications in a purse, bag,
pocket, or other vessel.

157. B: Severity of asthma exacerbations is not a barrier to successful asthma control because all
levels of asthma severity can be controlled with interventions such as medications, environmental
controls, detailed asthma action plans, and keeping an asthma diary. Although some patients may
have a higher level of severity and are therefore more difficult to control, severity alone does not
predict the success of an asthma treatment plan.

158. A: Patient-perceived symptoms are an important factor in measuring patient outcomes for
inpatient and one-on-one asthma education programs. For community-based education, in which
education is conducted in a group setting, more generalized information is used such as the hospital
admission rate in the community, emergency department visit incidence in the surrounding area,
and program attendance trends. Patient-perceived symptoms are often unknown in this population
because there is often little to no individualized attention in group education, so the asthma
educator would not be aware of this information for each attendee.

159. A: In order to expand an asthma education program, the asthma educator needs to grow her
patient population by partnering with the community. Some possible partnerships include local
schools, community centers and support groups, churches, and disease-based charitable
organizations. Options B and C are incorrect because these are outside corporations that can cause
the educator to have bias. All asthma education should be free from bias. Option D would be correct
if the asthma educator’s hospital was part of a larger hospital corporation, but because it is a local

- 148 -
Copyright © Mometrix Media. You have been licensed one copy of this document for personal use only. Any other reproduction or redistribution is strictly prohibited. All rights reserved.

Licensed to elliot mattison (2146499335, i.astraltears@gmail.com) of 9801 royal ln, dallas, Texas 75231. If this product is distributed to others,
elliot mattison agrees to pay Mometrix the full retail price for each unlicensed recipient. Please play fair and respect the work of our authors.
independent community hospital, partnering with an outside corporation would be inappropriate
and a potential conflict of interest.

160. A: Although the trachea is part of the pulmonary system, it is not part of the lung anatomy. The
lungs are comprised of the mainstem bronchus, bronchioles, and alveoli.

161. A: The alveoli are tiny air sacs at the end of the bronchioles. This is where oxygen transfers
from the lungs into the bloodstream. Option B is incorrect because perfusion refers to the
movement of blood throughout the body, rather than oxygenation. Option C is incorrect because the
alveoli do not provide airflow to the lungs; instead, the alveoli are the last stop for the inhaled air.
After reaching the alveoli, the oxygen content of the air is transferred into the blood.

162. D: Risk is measured by determining the likelihood of a patient’s reduction of lung function,
possible unpleasant medication side effects, and future severity and frequency of asthma
symptoms. Each one of these criteria helps to predict the patient’s future risk of severe or life-
threatening asthma exacerbations. Option D is incorrect because an inability to perform daily tasks
is a criterion for the measurement of current impairment, not future risk.

163. A: The correct procedure for a forced vital capacity is to instruct the patient to take a
maximum inhalation, exhale quickly and forcefully until he is not able to exhale any further, then
instruct the patient to inhale quickly until he is at a maximum inhalation.

164. C: Tripod sitting refers to a patient position in which the patient is seated and leaning forward
on their forearms. Patients in this position are bracing themselves to enlist the help of accessory
muscles for respiration. This is not a common sign of respiratory distress in children because their
accessory muscle use presents as tracheal tugging, subcostal or intercostal retractions, and nasal
flaring.

165. A: In an IgE-mediated allergic reaction, the body releases IgE in response to allergen exposure.
The IgE then bonds to the allergen and causes the mast cells to release histamine. This in turn
causes the allergic and asthmatic symptoms that many allergy sufferers experience after exposure
to an allergen.

166. D: Allergy exposure can cause a delayed onset of asthma symptoms. Allergic reactions can be
long lasting, with early and late-phase components to the reaction. Asthma symptoms can be part of
a late-phase reaction to an allergen exposure. Allergen exposure can take place hours or even days
before the asthma exacerbation onset. Options A and B are incorrect because true allergic reactions
have an IgE component, but allergy-type symptoms can occur from allergen exposure in nonallergic
patients. These allergens can act as airway irritants. Option C is incorrect because allergen exposure
is a common asthma trigger.

167. B: Cured meats and red wine contain sulfites that can be an asthma trigger for some patients.
Option A is incorrect because the only symptom noted is shortness of breath, so an allergic reaction
is unlikely. Allergic reactions often include other symptoms such as hives, stomach pain, vomiting,
and loss of consciousness. Option C is incorrect because although red wine contains tannins, they
are unlikely to cause shortness of breath in a patient with asthma. Option D is incorrect because
there is no mention of food allergies, so cross contamination with an allergic substance is not an
issue in this vignette.

168. C: Being overweight or obese can cause more frequent and more severe asthma exacerbations.
Option A is incorrect because low levels of vitamin D cause more severe asthma symptoms. Option
B is incorrect because although a healthy diet can reduce a patient’s risk for obesity and more

- 149 -
Copyright © Mometrix Media. You have been licensed one copy of this document for personal use only. Any other reproduction or redistribution is strictly prohibited. All rights reserved.

Licensed to elliot mattison (2146499335, i.astraltears@gmail.com) of 9801 royal ln, dallas, Texas 75231. If this product is distributed to others,
elliot mattison agrees to pay Mometrix the full retail price for each unlicensed recipient. Please play fair and respect the work of our authors.
severe asthma symptoms, it does not affect the development of asthma. Option D is also incorrect
because food allergies can be present in addition to asthma but are not directly related to the
development of asthma.

169. D: Patients with asthma do not require special infection prevention interventions. Simple hand
washing, avoiding people who are sick, and getting a yearly influenza vaccine are adequate
infection control measures.

170. B: Spirometry is a common component of preoperative testing, in which the patient undergoes
testing for a variety of medical problems prior to surgery. Spirometry is not an asthma trigger
because it is designed to measure the volumes and flows associated with a patient’s breathing and
identify any underlying pulmonary problems. Anesthesia gases, conscious sedation, and intubation
are all asthma triggers. The anesthesia gases can cause bronchospasm due to the gases acting as an
inhaled irritant. Conscious sedation is an asthma trigger because during this type of anesthesia, the
patient is partially aware of the procedure. This can cause anxiety and pain, which can trigger an
asthma exacerbation. Finally, intubation is an asthma trigger because the act of placing an airway
into the trachea can provoke the bronchioles into bronchospasm.

171. A: Common signs of mold growth on drywall include the presence of a chalky residue, black or
green specks, and water staining. A damp area is not a sign of mold growth, but it can begin to grow
mold if it isn’t dried out within 48 hours.

172. C: This patient states that he can’t breathe and his chest is tight. Some patients with asthma do
not present with wheezing, and breath sounds can be mostly clear even during an exacerbation. His
oxygen saturation is slightly decreased. The asthma educator should treat the patient as if he were
having an asthma exacerbation and then observe the patient to see if there is any improvement.
Giving the patient a medication-free nebulizer treatment (saline only) is not only ethically wrong,
but dangerous. Saline can trigger severe, refractory bronchospasm in patients with asthma, which
can be life threatening. Option A is incorrect because observing a patient for 10 minutes can delay
lifesaving treatment and cause the patient’s bronchospasm to worsen. Option D is incorrect because
discharging a patient without treatment is not acceptable, especially when the patient has a history
of asthma and is complaining of shortness of breath.

173. D: Pests are not a common classroom asthma trigger because many schools have pest control
measures in place. Options A through C are common asthma triggers that are found in the
classroom environment. Chalk dust and varnishes are both inhaled irritants that can cause asthma
symptoms. Ozone is released by printers in office buildings and classrooms, which can cause severe
asthma symptoms in sensitive individuals.

174. A: This patient states that she has had comprehensive allergy testing performed, and she is not
allergic to dogs. Therefore, dog dander is not a possible cause of her symptoms. Fragrances are
commonly used in dog grooming facilities in the shampoos and conditioners as well as the
perfumes used after the dog has been washed and dried. Outdoor allergens, such as pollen,
ragweed, and mold, are a possible trigger for this patient because dogs often roam outdoors and
carry environmental allergens on their coats. Because she is washing these animals, it is very likely
that she is being exposed to outdoor allergens on these dogs. Cleansers are a common asthma
trigger in dog grooming facilities because strong cleaning agents are used as disinfectants.

175. B: During vocal cord dysfunction, a patient’s vocal cords close prematurely during inspiration.
This causes a flattened flow volume loop during spirometry due to the interruption of flow through
the cords on inspiration.

- 150 -
Copyright © Mometrix Media. You have been licensed one copy of this document for personal use only. Any other reproduction or redistribution is strictly prohibited. All rights reserved.

Licensed to elliot mattison (2146499335, i.astraltears@gmail.com) of 9801 royal ln, dallas, Texas 75231. If this product is distributed to others,
elliot mattison agrees to pay Mometrix the full retail price for each unlicensed recipient. Please play fair and respect the work of our authors.
How to Overcome Test Anxiety
Just the thought of taking a test is enough to make most people a little nervous. A test is an
important event that can have a long-term impact on your future, so it’s important to take it
seriously and it’s natural to feel anxious about performing well. But just because anxiety is normal,
that doesn’t mean that it’s helpful in test taking, or that you should simply accept it as part of your
life. Anxiety can have a variety of effects. These effects can be mild, like making you feel slightly
nervous, or severe, like blocking your ability to focus or remember even a simple detail.

If you experience test anxiety—whether severe or mild—it’s important to know how to beat it. To
discover this, first you need to understand what causes test anxiety.

Causes of Test Anxiety


While we often think of anxiety as an uncontrollable emotional state, it can actually be caused by
simple, practical things. One of the most common causes of test anxiety is that a person does not
feel adequately prepared for their test. This feeling can be the result of many different issues such
as poor study habits or lack of organization, but the most common culprit is time management.
Starting to study too late, failing to organize your study time to cover all of the material, or being
distracted while you study will mean that you’re not well prepared for the test. This may lead to
cramming the night before, which will cause you to be physically and mentally exhausted for the
test. Poor time management also contributes to feelings of stress, fear, and hopelessness as you
realize you are not well prepared but don’t know what to do about it.

Other times, test anxiety is not related to your preparation for the test but comes from unresolved
fear. This may be a past failure on a test, or poor performance on tests in general. It may come from
comparing yourself to others who seem to be performing better or from the stress of living up to
expectations. Anxiety may be driven by fears of the future—how failure on this test would affect
your educational and career goals. These fears are often completely irrational, but they can still
negatively impact your test performance.

Review Video: 3 Reasons You Have Test Anxiety


Visit mometrix.com/academy and enter code: 428468

- 151 -
Copyright © Mometrix Media. You have been licensed one copy of this document for personal use only. Any other reproduction or redistribution is strictly prohibited. All rights reserved.

Licensed to elliot mattison (2146499335, i.astraltears@gmail.com) of 9801 royal ln, dallas, Texas 75231. If this product is distributed to others,
elliot mattison agrees to pay Mometrix the full retail price for each unlicensed recipient. Please play fair and respect the work of our authors.
.

Elements of Test Anxiety


As mentioned earlier, test anxiety is considered to be an emotional state, but it has physical and
mental components as well. Sometimes you may not even realize that you are suffering from test
anxiety until you notice the physical symptoms. These can include trembling hands, rapid
heartbeat, sweating, nausea, and tense muscles. Extreme anxiety may lead to fainting or vomiting.
Obviously, any of these symptoms can have a negative impact on testing. It is important to
recognize them as soon as they begin to occur so that you can address the problem before it
damages your performance.

Review Video: 3 Ways to Tell You Have Test Anxiety


Visit mometrix.com/academy and enter code: 927847

The mental components of test anxiety include trouble focusing and inability to remember learned
information. During a test, your mind is on high alert, which can help you recall information and
stay focused for an extended period of time. However, anxiety interferes with your mind’s natural
processes, causing you to blank out, even on the questions you know well. The strain of testing
during anxiety makes it difficult to stay focused, especially on a test that may take several hours.
Extreme anxiety can take a huge mental toll, making it difficult not only to recall test information
but even to understand the test questions or pull your thoughts together.

Review Video: How Test Anxiety Affects Memory


Visit mometrix.com/academy and enter code: 609003

Effects of Test Anxiety


Test anxiety is like a disease—if left untreated, it will get progressively worse. Anxiety leads to poor
performance, and this reinforces the feelings of fear and failure, which in turn lead to poor
performances on subsequent tests. It can grow from a mild nervousness to a crippling condition. If
allowed to progress, test anxiety can have a big impact on your schooling, and consequently on your
future.

Test anxiety can spread to other parts of your life. Anxiety on tests can become anxiety in any `
stressful situation, and blanking on a test can turn into panicking in a job situation. But fortunately,
you don’t have to let anxiety rule your testing and determine your grades. There are a number of
relatively simple steps you can take to move past anxiety and function normally on a test and in the
rest of life.

Review Video: How Test Anxiety Impacts Your Grades


Visit mometrix.com/academy and enter code: 939819

- 152 -
Copyright © Mometrix Media. You have been licensed one copy of this document for personal use only. Any other reproduction or redistribution is strictly prohibited. All rights reserved.

Licensed to elliot mattison (2146499335, i.astraltears@gmail.com) of 9801 royal ln, dallas, Texas 75231. If this product is distributed to others,
elliot mattison agrees to pay Mometrix the full retail price for each unlicensed recipient. Please play fair and respect the work of our authors.
Physical Steps for Beating Test Anxiety
While test anxiety is a serious problem, the good news is that it can be overcome. It doesn’t have to
control your ability to think and remember information. While it may take time, you can begin
taking steps today to beat anxiety.

Just as your first hint that you may be struggling with anxiety comes from the physical symptoms,
the first step to treating it is also physical. Rest is crucial for having a clear, strong mind. If you are
tired, it is much easier to give in to anxiety. But if you establish good sleep habits, your body and
mind will be ready to perform optimally, without the strain of exhaustion. Additionally, sleeping
well helps you to retain information better, so you’re more likely to recall the answers when you
see the test questions.

Getting good sleep means more than going to bed on time. It’s important to allow your brain time to
relax. Take study breaks from time to time so it doesn’t get overworked, and don’t study right
before bed. Take time to rest your mind before trying to rest your body, or you may find it difficult
to fall asleep.

Review Video: The Importance of Sleep for Your Brain


Visit mometrix.com/academy and enter code: 319338

Along with sleep, other aspects of physical health are important in preparing for a test. Good
nutrition is vital for good brain function. Sugary foods and drinks may give a burst of energy but
this burst is followed by a crash, both physically and emotionally. Instead, fuel your body with
protein and vitamin-rich foods.

Also, drink plenty of water. Dehydration can lead to headaches and exhaustion, especially if your
brain is already under stress from the rigors of the test. Particularly if your test is a long one, drink
water during the breaks. And if possible, take an energy-boosting snack to eat between sections.

Review Video: How Diet Can Affect your Mood


Visit mometrix.com/academy and enter code: 624317

Along with sleep and diet, a third important part of physical health is exercise. Maintaining a steady
workout schedule is helpful, but even taking 5-minute study breaks to walk can help get your blood
pumping faster and clear your head. Exercise also releases endorphins, which contribute to a
positive feeling and can help combat test anxiety.

When you nurture your physical health, you are also contributing to your mental health. If your
body is healthy, your mind is much more likely to be healthy as well. So take time to rest, nourish
your body with healthy food and water, and get moving as much as possible. Taking these physical
steps will make you stronger and more able to take the mental steps necessary to overcome test
anxiety.

Review Video: How to Stay Healthy and Prevent Test Anxiety


Visit mometrix.com/academy and enter code: 877894

- 153 -
Copyright © Mometrix Media. You have been licensed one copy of this document for personal use only. Any other reproduction or redistribution is strictly prohibited. All rights reserved.

Licensed to elliot mattison (2146499335, i.astraltears@gmail.com) of 9801 royal ln, dallas, Texas 75231. If this product is distributed to others,
elliot mattison agrees to pay Mometrix the full retail price for each unlicensed recipient. Please play fair and respect the work of our authors.
Mental Steps for Beating Test Anxiety
Working on the mental side of test anxiety can be more challenging, but as with the physical side,
there are clear steps you can take to overcome it. As mentioned earlier, test anxiety often stems
from lack of preparation, so the obvious solution is to prepare for the test. Effective studying may
be the most important weapon you have for beating test anxiety, but you can and should employ
several other mental tools to combat fear.

First, boost your confidence by reminding yourself of past success—tests or projects that you aced.
If you’re putting as much effort into preparing for this test as you did for those, there’s no reason
you should expect to fail here. Work hard to prepare; then trust your preparation.

Second, surround yourself with encouraging people. It can be helpful to find a study group, but be
sure that the people you’re around will encourage a positive attitude. If you spend time with others
who are anxious or cynical, this will only contribute to your own anxiety. Look for others who are
motivated to study hard from a desire to succeed, not from a fear of failure.

Third, reward yourself. A test is physically and mentally tiring, even without anxiety, and it can be
helpful to have something to look forward to. Plan an activity following the test, regardless of the
outcome, such as going to a movie or getting ice cream.

When you are taking the test, if you find yourself beginning to feel anxious, remind yourself that
you know the material. Visualize successfully completing the test. Then take a few deep, relaxing
breaths and return to it. Work through the questions carefully but with confidence, knowing that
you are capable of succeeding.

Developing a healthy mental approach to test taking will also aid in other areas of life. Test anxiety
affects more than just the actual test—it can be damaging to your mental health and even
contribute to depression. It’s important to beat test anxiety before it becomes a problem for more
than testing.

Review Video: Test Anxiety and Depression


Visit mometrix.com/academy and enter code: 904704

- 154 -
Copyright © Mometrix Media. You have been licensed one copy of this document for personal use only. Any other reproduction or redistribution is strictly prohibited. All rights reserved.

Licensed to elliot mattison (2146499335, i.astraltears@gmail.com) of 9801 royal ln, dallas, Texas 75231. If this product is distributed to others,
elliot mattison agrees to pay Mometrix the full retail price for each unlicensed recipient. Please play fair and respect the work of our authors.
Study Strategy
Being prepared for the test is necessary to combat anxiety, but what does being prepared look like?
You may study for hours on end and still not feel prepared. What you need is a strategy for test
prep. The next few pages outline our recommended steps to help you plan out and conquer the
challenge of preparation.

Step 1: Scope Out the Test

Learn everything you can about the format (multiple choice, essay, etc.) and what will be on the
test. Gather any study materials, course outlines, or sample exams that may be available. Not only
will this help you to prepare, but knowing what to expect can help to alleviate test anxiety.

Step 2: Map Out the Material

Look through the textbook or study guide and make note of how many chapters or sections it has.
Then divide these over the time you have. For example, if a book has 15 chapters and you have five
days to study, you need to cover three chapters each day. Even better, if you have the time, leave an
extra day at the end for overall review after you have gone through the material in depth.

If time is limited, you may need to prioritize the material. Look through it and make note of which
sections you think you already have a good grasp on, and which need review. While you are
studying, skim quickly through the familiar sections and take more time on the challenging parts.
Write out your plan so you don’t get lost as you go. Having a written plan also helps you feel more
in control of the study, so anxiety is less likely to arise from feeling overwhelmed at the amount to
cover. A sample plan may look like this:

 Day 1: Skim chapters 1–4, study chapter 5 (especially pages 31–33)


 Day 2: Study chapters 6–7, skim chapters 8–9
 Day 3: Skim chapter 10, study chapters 11–12 (especially pages 87–90)
 Day 4: Study chapters 13–15
 Day 5: Overall review (focus most on chapters 5, 6, and 12), take practice test

Step 3: Gather Your Tools

Decide what study method works best for you. Do you prefer to highlight in the book as you study
and then go back over the highlighted portions? Or do you type out notes of the important
information? Or is it helpful to make flashcards that you can carry with you? Assemble the pens,
index cards, highlighters, post-it notes, and any other materials you may need so you won’t be
distracted by getting up to find things while you study.

If you’re having a hard time retaining the information or organizing your notes, experiment with
different methods. For example, try color-coding by subject with colored pens, highlighters, or post-
it notes. If you learn better by hearing, try recording yourself reading your notes so you can listen
while in the car, working out, or simply sitting at your desk. Ask a friend to quiz you from your
flashcards, or try teaching someone the material to solidify it in your mind.

Step 4: Create Your Environment

It’s important to avoid distractions while you study. This includes both the obvious distractions like
visitors and the subtle distractions like an uncomfortable chair (or a too-comfortable couch that
makes you want to fall asleep). Set up the best study environment possible: good lighting and a

- 155 -
Copyright © Mometrix Media. You have been licensed one copy of this document for personal use only. Any other reproduction or redistribution is strictly prohibited. All rights reserved.

Licensed to elliot mattison (2146499335, i.astraltears@gmail.com) of 9801 royal ln, dallas, Texas 75231. If this product is distributed to others,
elliot mattison agrees to pay Mometrix the full retail price for each unlicensed recipient. Please play fair and respect the work of our authors.
comfortable work area. If background music helps you focus, you may want to turn it on, but
otherwise keep the room quiet. If you are using a computer to take notes, be sure you don’t have
any other windows open, especially applications like social media, games, or anything else that
could distract you. Silence your phone and turn off notifications. Be sure to keep water close by so
you stay hydrated while you study (but avoid unhealthy drinks and snacks).

Also, take into account the best time of day to study. Are you freshest first thing in the morning? Try
to set aside some time then to work through the material. Is your mind clearer in the afternoon or
evening? Schedule your study session then. Another method is to study at the same time of day that
you will take the test, so that your brain gets used to working on the material at that time and will
be ready to focus at test time.

Step 5: Study!

Once you have done all the study preparation, it’s time to settle into the actual studying. Sit down,
take a few moments to settle your mind so you can focus, and begin to follow your study plan. Don’t
give in to distractions or let yourself procrastinate. This is your time to prepare so you’ll be ready to
fearlessly approach the test. Make the most of the time and stay focused.

Of course, you don’t want to burn out. If you study too long you may find that you’re not retaining
the information very well. Take regular study breaks. For example, taking five minutes out of every
hour to walk briskly, breathing deeply and swinging your arms, can help your mind stay fresh.

As you get to the end of each chapter or section, it’s a good idea to do a quick review. Remind
yourself of what you learned and work on any difficult parts. When you feel that you’ve mastered
the material, move on to the next part. At the end of your study session, briefly skim through your
notes again.

But while review is helpful, cramming last minute is NOT. If at all possible, work ahead so that you
won’t need to fit all your study into the last day. Cramming overloads your brain with more
information than it can process and retain, and your tired mind may struggle to recall even
previously learned information when it is overwhelmed with last-minute study. Also, the urgent
nature of cramming and the stress placed on your brain contribute to anxiety. You’ll be more likely
to go to the test feeling unprepared and having trouble thinking clearly.

So don’t cram, and don’t stay up late before the test, even just to review your notes at a leisurely
pace. Your brain needs rest more than it needs to go over the information again. In fact, plan to
finish your studies by noon or early afternoon the day before the test. Give your brain the rest of the
day to relax or focus on other things, and get a good night’s sleep. Then you will be fresh for the test
and better able to recall what you’ve studied.

Step 6: Take a practice test

Many courses offer sample tests, either online or in the study materials. This is an excellent
resource to check whether you have mastered the material, as well as to prepare for the test format
and environment.

Check the test format ahead of time: the number of questions, the type (multiple choice, free
response, etc.), and the time limit. Then create a plan for working through them. For example, if you
have 30 minutes to take a 60-question test, your limit is 30 seconds per question. Spend less time
on the questions you know well so that you can take more time on the difficult ones.

- 156 -
Copyright © Mometrix Media. You have been licensed one copy of this document for personal use only. Any other reproduction or redistribution is strictly prohibited. All rights reserved.

Licensed to elliot mattison (2146499335, i.astraltears@gmail.com) of 9801 royal ln, dallas, Texas 75231. If this product is distributed to others,
elliot mattison agrees to pay Mometrix the full retail price for each unlicensed recipient. Please play fair and respect the work of our authors.
If you have time to take several practice tests, take the first one open book, with no time limit. Work
through the questions at your own pace and make sure you fully understand them. Gradually work
up to taking a test under test conditions: sit at a desk with all study materials put away and set a
timer. Pace yourself to make sure you finish the test with time to spare and go back to check your
answers if you have time.

After each test, check your answers. On the questions you missed, be sure you understand why you
missed them. Did you misread the question (tests can use tricky wording)? Did you forget the
information? Or was it something you hadn’t learned? Go back and study any shaky areas that the
practice tests reveal.

Taking these tests not only helps with your grade, but also aids in combating test anxiety. If you’re
already used to the test conditions, you’re less likely to worry about it, and working through tests
until you’re scoring well gives you a confidence boost. Go through the practice tests until you feel
comfortable, and then you can go into the test knowing that you’re ready for it.

Test Tips
On test day, you should be confident, knowing that you’ve prepared well and are ready to answer
the questions. But aside from preparation, there are several test day strategies you can employ to
maximize your performance.

First, as stated before, get a good night’s sleep the night before the test (and for several nights
before that, if possible). Go into the test with a fresh, alert mind rather than staying up late to study.

Try not to change too much about your normal routine on the day of the test. It’s important to eat a
nutritious breakfast, but if you normally don’t eat breakfast at all, consider eating just a protein bar.
If you’re a coffee drinker, go ahead and have your normal coffee. Just make sure you time it so that
the caffeine doesn’t wear off right in the middle of your test. Avoid sugary beverages, and drink
enough water to stay hydrated but not so much that you need a restroom break 10 minutes into the
test. If your test isn’t first thing in the morning, consider going for a walk or doing a light workout
before the test to get your blood flowing.

Allow yourself enough time to get ready, and leave for the test with plenty of time to spare so you
won’t have the anxiety of scrambling to arrive in time. Another reason to be early is to select a good
seat. It’s helpful to sit away from doors and windows, which can be distracting. Find a good seat, get
out your supplies, and settle your mind before the test begins.

When the test begins, start by going over the instructions carefully, even if you already know what
to expect. Make sure you avoid any careless mistakes by following the directions.

Then begin working through the questions, pacing yourself as you’ve practiced. If you’re not sure
on an answer, don’t spend too much time on it, and don’t let it shake your confidence. Either skip it
and come back later, or eliminate as many wrong answers as possible and guess among the
remaining ones. Don’t dwell on these questions as you continue—put them out of your mind and
focus on what lies ahead.

Be sure to read all of the answer choices, even if you’re sure the first one is the right answer.
Sometimes you’ll find a better one if you keep reading. But don’t second-guess yourself if you do
immediately know the answer. Your gut instinct is usually right. Don’t let test anxiety rob you of the
information you know.

- 157 -
Copyright © Mometrix Media. You have been licensed one copy of this document for personal use only. Any other reproduction or redistribution is strictly prohibited. All rights reserved.

Licensed to elliot mattison (2146499335, i.astraltears@gmail.com) of 9801 royal ln, dallas, Texas 75231. If this product is distributed to others,
elliot mattison agrees to pay Mometrix the full retail price for each unlicensed recipient. Please play fair and respect the work of our authors.
If you have time at the end of the test (and if the test format allows), go back and review your
answers. Be cautious about changing any, since your first instinct tends to be correct, but make sure
you didn’t misread any of the questions or accidentally mark the wrong answer choice. Look over
any you skipped and make an educated guess.

At the end, leave the test feeling confident. You’ve done your best, so don’t waste time worrying
about your performance or wishing you could change anything. Instead, celebrate the successful
completion of this test. And finally, use this test to learn how to deal with anxiety even better next
time.

Review Video: 5 Tips to Beat Test Anxiety


Visit mometrix.com/academy and enter code: 570656

Important Qualification
Not all anxiety is created equal. If your test anxiety is causing major issues in your life beyond the
classroom or testing center, or if you are experiencing troubling physical symptoms related to your
anxiety, it may be a sign of a serious physiological or psychological condition. If this sounds like
your situation, we strongly encourage you to seek professional help.

- 158 -
Copyright © Mometrix Media. You have been licensed one copy of this document for personal use only. Any other reproduction or redistribution is strictly prohibited. All rights reserved.

Licensed to elliot mattison (2146499335, i.astraltears@gmail.com) of 9801 royal ln, dallas, Texas 75231. If this product is distributed to others,
elliot mattison agrees to pay Mometrix the full retail price for each unlicensed recipient. Please play fair and respect the work of our authors.
Thank You
We at Mometrix would like to extend our heartfelt thanks to you, our friend and patron, for
allowing us to play a part in your journey. It is a privilege to serve people from all walks of life who
are unified in their commitment to building the best future they can for themselves.

The preparation you devote to these important testing milestones may be the most valuable
educational opportunity you have for making a real difference in your life. We encourage you to put
your heart into it—that feeling of succeeding, overcoming, and yes, conquering will be well worth
the hours you’ve invested.

We want to hear your story, your struggles and your successes, and if you see any opportunities for
us to improve our materials so we can help others even more effectively in the future, please share
that with us as well. The team at Mometrix would be absolutely thrilled to hear from you! So
please, send us an email (support@mometrix.com) and let’s stay in touch.

If you’d like some additional help, check out these other resources we offer for your exam:

http://mometrixflashcards.com/NAECB

- 159 -
Copyright © Mometrix Media. You have been licensed one copy of this document for personal use only. Any other reproduction or redistribution is strictly prohibited. All rights reserved.

Licensed to elliot mattison (2146499335, i.astraltears@gmail.com) of 9801 royal ln, dallas, Texas 75231. If this product is distributed to others,
elliot mattison agrees to pay Mometrix the full retail price for each unlicensed recipient. Please play fair and respect the work of our authors.
Additional Bonus Material
Due to our efforts to try to keep this book to a manageable length, we’ve created a link that will give
you access to all of your additional bonus material.

Please visit https://www.mometrix.com/bonus948/naecb to access the information.

- 160 -
Copyright © Mometrix Media. You have been licensed one copy of this document for personal use only. Any other reproduction or redistribution is strictly prohibited. All rights reserved.

Licensed to elliot mattison (2146499335, i.astraltears@gmail.com) of 9801 royal ln, dallas, Texas 75231. If this product is distributed to others,
elliot mattison agrees to pay Mometrix the full retail price for each unlicensed recipient. Please play fair and respect the work of our authors.
Licensed to elliot mattison (2146499335, i.astraltears@gmail.com) of 9801 royal ln, dallas, Texas 75231. If this product is distributed to others,
elliot mattison agrees to pay Mometrix the full retail price for each unlicensed recipient. Please play fair and respect the work of our authors.
Licensed to elliot mattison (2146499335, i.astraltears@gmail.com) of 9801 royal ln, dallas, Texas 75231. If this product is distributed to others,
elliot mattison agrees to pay Mometrix the full retail price for each unlicensed recipient. Please play fair and respect the work of our authors.
Licensed to elliot mattison (2146499335, i.astraltears@gmail.com) of 9801 royal ln, dallas, Texas 75231. If this product is distributed to others,
elliot mattison agrees to pay Mometrix the full retail price for each unlicensed recipient. Please play fair and respect the work of our authors.
GUARANTEED TO IMPROVE YOUR SCORE
“Effective, Affordable, Guaranteed”

At Mometrix, we think differently about tests. We believe you can perform better on your
exam by implementing a few critical strategies and focusing your study time on what’s most
important. With so many demands on your time, you probably don’t have months to spend
preparing for an exam that holds the key to your future. Our team of testing experts devote
hours upon hours to painstakingly review piles of content and boil it all down to the critical
concepts that are most likely to be on your exam. We do a lot of work cutting through the
fluff to give you what you need the most to perform well on the exam. But you don’t have
to take our word for it; here is what some of our customers have to say:

“I’ve just retaken my test and I scored way better than my previous score. I had
this program for only 3 days and I just want to say that I can't believe how well
it worked.” - Mandy C.
“Just wanted to say thank you. Due to your product I was able to ace my exam
with very little effort. Your tricks did the trick. Thanks again, and I would
recommend this product to anyone.” - Erich L.
“Just dropping you a note to let you know that I am completely satisfied with the
product. I had already taken the test once and landed in the 75 th percentile of
those taking it with me. I took the test a second time and used some of your tips
and raised my score to the 97th percentile. Thanks for my much improved score.”
- Denise W.
“I just wanted to tell you I had ordered your study guide, and I finally aced the
test after taking it numerous times. I tried tutors and all sorts of study guides and
nothing helped. Your guide did the job and got me the score I needed! Thank
you!” - Nicholas R.

We offer study materials for over 1000 different standardized exams, including:
College Entrance Exams Teacher Certification Exams
Medical/Nursing Exams Insurance Exams
Financial Exams Dental Exams
Military Exams Graduate & Professional School Exams

Licensed to elliot mattison (2146499335, i.astraltears@gmail.com) of 9801 royal ln, dallas, Texas 75231. If this product is distributed to others,
elliot mattison agrees to pay Mometrix the full retail price for each unlicensed recipient. Please play fair and respect the work of our authors.

You might also like